OB final cramming guide

अब Quizwiz के साथ अपने होमवर्क और परीक्षाओं को एस करें!

The healthy infant must accomplish both behavioral and biologic tasks to develop normally. Behavioral characteristics form the basis of the social capabilities of the infant. Newborns pass through a hierarchy of developmental challenges as they adapt to their environment and caregivers. This progression in behavior is the basis for the Brazelton Neonatal Behavioral Assessment (NBAS). Match the cluster of neonatal behaviors with the correct level on the NBAS scale. a.Habituation b.Orientation c.Range of state d.Autonomic stability e.Regulation of state 1. Signs of stress related to homeostatic adjustment 2. Ability to respond to discrete stimuli while asleep 3. Measure of general arousability 4. How the infant responds when aroused 5. Ability to attend to visual and auditory stimuli while alert

1. ANS: D DIF: Cognitive Level: Apply REF: p. 544 TOP: Nursing Process: Assessment MSC: Client Needs: Physiologic Integrity 2. ANS: A DIF: Cognitive Level: Apply REF: p. 544 TOP: Nursing Process: Assessment MSC: Client Needs: Physiologic Integrity 3. ANS: C DIF: Cognitive Level: Apply REF: p. 544 TOP: Nursing Process: Assessment MSC: Client Needs: Physiologic Integrity 4. ANS: E DIF: Cognitive Level: Apply REF: p. 544 TOP: Nursing Process: Assessment MSC: Client Needs: Physiologic Integrity 5. ANS: B DIF: Cognitive Level: Apply REF: p. 544 TOP: Nursing Process: Assessment MSC: Client Needs: Physiologic Integrity

List the time span in lunar months, calendar months, weeks, and days that indicates the appropriate length for a normal pregnancy. ____________________

10 lunar months, 9 calendar months, 40 weeks, 280 days

Matching 27. Taking-in 28. Letting-go 29. Taking-hold a. Passive, dependent b. Begins to see self as a mother c. Autonomous, seeking information

27. a 28. b 29. c

A woman is 6 weeks pregnant. She has had a previous spontaneous abortion at 14 weeks of gestation and a pregnancy that ended at 38 weeks with the birth of a stillborn girl. GTPAL system?

3-1-0-1-0

Before the physician performs an external version, the nurse should expect an order for a: a. Tocolytic drug. c. Local anesthetic. b. Contraction stress test (CST). d. Foley catheter.

A

Immediately after the forceps-assisted birth of an infant, the nurse should: a. Assess the infant for signs of trauma. b. Give the infant prophylactic antibiotics. c. Apply a cold pack to the infant's scalp. d. Measure the circumference of the infant's head.

A

In evaluating the effectiveness of oxytocin induction, the nurse would expect: a. Contractions lasting 40 to 90 seconds, 2 to 3 minutes apart. b. The intensity of contractions to be at least 110 to 130 mm Hg. c. Labor to progress at least 2 cm/hr dilation. d. At least 30 mU/min of oxytocin will be needed to achieve cervical dilation.

A

The exact cause of preterm labor is unknown and believed to be multifactorial. Infection is thought to be a major factor in many preterm labors. Select the type of infection that has not been linked to preterm births. a. Viral c. Cervical b. Periodontal d. Urinary tract

A

Which patient status is an acceptable indication for serial oxytocin induction of labor? a. Past 42 weeks' gestation c. Polyhydramnios b. Multiple fetuses d. History of long labors

A

With regard to the process of augmentation of labor, the nurse should be aware that it: a. Is part of the active management of labor that is instituted when the labor process is unsatisfactory. b. Relies on more invasive methods when oxytocin and amniotomy have failed. c. Is a modern management term to cover up the negative connotations of forceps-assisted birth. d. Uses vacuum cups.

A

The abuse of which of the following substances during pregnancy is the leading cause of cognitive impairment in the United States? a. Alcohol b. Tobacco c. Marijuana d. Heroin

A (Alcohol abuse during pregnancy is recognized as one of the leading causes of cognitive impairment in the United States.)

A woman at 37 weeks of gestation is admitted with a placental abruption after a motor vehicle accident. Which assessment data are most indicative of her condition worsening? A. Pulse (P) 112, respiration (R) 32, blood pressure (BP) 108/60; fetal heart rate (FHR) 166--178 B. P 98, R 22, BP 110/74; FHR 150--162 C. P 88, R 20, BP 114/70; FHR 140--158 D. P 80, R 18, BP 120/78; FHR 138--150

A (Bleeding is the most dangerous problem, which impacts the mother's well-being as well as that of her fetus. The decreasing blood volume would cause increases in pulse and respirations and a decrease in blood pressure. The fetus often responds to decreased oxygenation as a result of bleeding, causing a decrease in perfusion. This causes the fetus' heart rate to increase above the normal range of 120--160 beats per minute. The other options have measurements that are in the "normal" range and would not reflect a deterioration of the patient's physical status. Test-Taking Tip: Do not worry if you select the same numbered answer repeatedly, because there usually is no pattern to the answers.)

A pregnant woman wants to breastfeed her infant; however, her husband is not convinced that there are any scientific reasons to do so. The nurse can give the couple printed information comparing breastfeeding and bottle-feeding. Which statement is most accurate? Bottle-feeding using commercially prepared infant formulas: a. Increases the risk that the infant will develop allergies. b. Helps the infant sleep through the night. c. Ensures that the infant is getting iron in a form that is easily absorbed. d. Requires that multivitamin supplements be given to the infant.

A (Exposure to cow's milk poses a risk of developing allergies, eczema, and asthma. "Bottle-feeding using commercially prepared infant formulas helps the infant sleep through the night" is a false statement. Iron is better absorbed from breast milk than from formula. Commercial formulas are designed to meet the nutritional needs of the infant and resemble breast milk.)

An infant with severe meconium aspiration syndrome (MAS) is not responding to conventional treatment. Which highly technical method of treatment may be necessary for an infant who does not respond to conventional treatment? a. Extracorporeal membrane oxygenation b. Respiratory support with a ventilator c. Insertion of a laryngoscope and suctioning of the trachea d. Insertion of an endotracheal tube

A (Extracorporeal membrane oxygenation is a highly technical method that oxygenates the blood while bypassing the lungs, thus allowing the infant's lungs to rest and recover. The infant is likely to have been first connected to a ventilator. Laryngoscope insertion and tracheal suctioning are performed after birth before the infant takes the first breath. An endotracheal tube will be in place to facilitate deep tracheal suctioning and ventilation.)

A woman diagnosed with marginal placenta previa gave birth vaginally 15 minutes ago. At the present time, she is at the greatest risk for: A. hemorrhage. B. infection. C. urinary retention. D. thrombophlebitis.

A (Hemorrhage is the most immediate risk because the lower uterine segment has limited ability to contract to reduce blood loss. Infection is a risk because of the location of the placental attachment site; however, it is not a priority concern at this time. Placenta previa poses no greater risk for urinary retention than with a normally implanted placenta. There is no greater risk for thrombophlebitis than with a normally implanted placenta.)

A patient with gestational hypertension is prescribed labetalol hydrochloride (Normodyne) therapy, which is continued after giving birth. What does the nurse instruct the patient about breastfeeding? A. "You may breastfeed the infant if you desire." B. "Breastfeeding may cause convulsions in the infant." C. "Breastfeed only once a day and use infant formulas." D. "There may be high levels of the drug in the breast milk."

A (Labetalol hydrochloride (Normodyne) has a low concentration in breast milk, so the patient can breastfeed the infant. Breastfeeding is safe and will not cause convulsions or any side effects in the infant. Infant formulas are used only if the mother is unable to breastfeed the infant or if the mother does not desire to breastfeed.)

After being rehydrated in the emergency department, a 24 year-old primipara in her 18th week of pregnancy is at home and is to rest at home for the next two days and take in small but frequent fluids and food as possible. Discharge teaching at the hospital by the nurse has been effective if the patient makes which statement? "A. I'm going to eat five to six small servings per day, which contain such foods and fluids as tea, crackers, or a few bites of baked potato." B. "A strip of bacon and a fried egg will really taste good as long as I eat them slowly." C. "As long as I eat small amounts and allow enough time for digestion, I can eat almost anything, like barbequed chicken or spaghetti." D. "I'm going to stay only on clear fluids for the next 24 hours and then add dairy products like eggs and milk."

A (Once the vomiting has stopped, feedings are started in small amounts at frequent intervals. In the beginning, limited amounts of oral fluids and bland foods such as crackers, toast, or baked chicken are offered. Clear fluids alone do not contain enough calories and contain no protein. Most women are able to take nourishment by mouth after several days of treatment. They should be encouraged to eat small, frequent meals and foods that sound appealing (e.g., nongreasy, dry, sweet, and salty foods). Test-Taking Tip: Many times the correct answer is the longest alternative given, but do not count on it. NCLEX item writers (those who write the questions) are also aware of this and attempt to avoid offering you such "helpful hints.")

Which condition is seen in a pregnant patient if uterine artery Doppler measurements in the second trimester of pregnancy are abnormal? A. Preeclampsia B. HELLP syndrome C. Molar pregnancy D. Gestational hypertension

A (Preeclampsia is a condition in which patients develop hypertension and proteinuria after 20 weeks' gestation. It can be diagnosed if uterine artery Doppler measurements in the second trimester of pregnancy are abnormal. HELLP syndrome is characterized by hemolysis (H), elevated liver enzymes (EL), and low platelet count (LP) in a patient with preeclampsia. Molar pregnancy refers to the growth of the placental trophoblast due to abnormal fertilization. Gestational hypertension is a condition in which hypertension develops in a patient after 20 weeks' gestation.)

The nurse is using the Ballard scale to determine the gestational age of a newborn. Which assessment finding is consistent with a gestational age of 40 weeks? a. Flexed posture b. Abundant lanugo c. Smooth, pink skin with visible veins d. Faint red marks on the soles of the feet

A (Term infants typically have a flexed posture. Abundant lanugo usually is seen on preterm infants. Smooth, pink skin with visible veins is seen on preterm infants. Faint red marks usually are seen on preterm infants.)

When preparing to administer a hepatitis B vaccine to a newborn, the nurse should: a. Obtain a syringe with a 25-gauge, 5/8-inch needle. b. Confirm that the newborn's mother has been infected with the hepatitis B virus. c. Assess the dorsogluteal muscle as the preferred site for injection. d. Confirm that the newborn is at least 24 hours old.

A (The hepatitis B vaccine should be administered with a 25-gauge, 5/8-inch needle. Hepatitis B vaccination is recommended for all infants. If the infant is born to an infected mother who is a chronic carrier, hepatitis vaccine and hepatitis B immune globulin should be administered within 12 hours of birth. Hepatitis B vaccine should be given in the vastus lateralis muscle. Hepatitis B vaccine can be given at birth.)

Which instructions does the nurse give to a patient who is prescribed methotrexate therapy for dissolving the tubal pregnancy? A. "Discontinue folic acid supplements." B. "Get adequate exposure to sunlight." C. "Take stronger analgesics for severe pain." D. "Vaginal intercourse is safe during the therapy."

A (The nurse advises the patient to discontinue folic acid supplements as they interact with methotrexate and may exacerbate ectopic rupture in the patient. Exposure to sunlight is avoided as the therapy makes the patient photosensitive. Analgesics stronger than acetaminophen are avoided, because they may mask symptoms of tubal rupture. Vaginal intercourse is avoided until the pregnancy is dissolved completely.)

The laboratory results for a postpartum woman are as follows: blood type, A; Rh status, positive; rubella titer, 1:8 (EIA 0.8); hematocrit, 30%. How would the nurse best interpret these data? a. Rubella vaccine should be given. b. A blood transfusion is necessary. c. Rh immune globulin is necessary within 72 hours of birth. d. A Kleihauer-Betke test should be performed.

A (This client's rubella titer indicates that she is not immune and that she needs to receive a vaccine. These data do not indicate that the client needs a blood transfusion. Rh immune globulin is indicated only if the client has a negative Rh status and the infant has a positive Rh status. A Kleihauer-Betke test should be performed if a large fetomaternal transfusion is suspected, especially if the mother is Rh negative. The data do not provide any indication for performing this test.)

The most important nursing action in preventing neonatal infection is: a. Good handwashing. b. Isolation of infected infants. c. Separate gown protocol d. Standard Precautions.

A (Virtually all controlled clinical trials have demonstrated that effective handwashing is responsible for the prevention of nosocomial infection in nursery units. Measures to be taken include Standard Precautions, careful and thorough cleaning, frequent replacement of used equipment, and disposal of excrement and linens in an appropriate manner. Overcrowding must be avoided in nurseries. However, the most important nursing action for preventing neonatal infection is effective handwashing.)

A woman with severe preeclampsia has been receiving magnesium sulfate by IV infusion for 8 hours. The nurse assesses the woman and documents the following findings: temperature 37.1° C, pulse rate 96 beats/min, respiratory rate 24 breaths/min, blood pressure 155/112 mm Hg, 3+ deep tendon reflexes, and no ankle clonus. The nurse calls the physician, anticipating an order for: A. hydralazine. B. magnesium sulfate bolus . C. diazepam. D. calcium gluconate.

A (hydralazine. Hydralazine is an antihypertensive commonly used to treat hypertension in severe preeclampsia. An additional bolus of magnesium sulfate may be ordered for increasing signs of central nervous system irritability related to severe preeclampsia (e.g., clonus) or if eclampsia develops. Diazepam sometimes is used to stop or shorten eclamptic seizures. Calcium gluconate is used as the antidote for magnesium sulfate toxicity. The client is not currently displaying any signs or symptoms of magnesium toxicity.)

A newborn is rooming-in with his teenage mother, who is watching TV. The nurse notes that the baby is awake and quiet. The best nursing action is to: a. Pick the baby up and point out his alert behaviors to the mother. b. Tell the mother to pick up her baby and talk with him while he is awake. c. Focus care on the mother, rather than the infant so she can recuperate. d. Encourage the mother to feed the infant before he begins crying.

A Modeling behavior by the nurse is an excellent way to teach infant care. The inexperienced teenage mother can observe the proper skills and then the nurse can encourage her to try those skills.

B (By age 3 or 4, children like to be told the story of their own beginning and accept its comparison with the present pregnancy. They like to listen to the fetal heartbeat and feel the baby move. Sometimes they worry about how the baby is being fed and what it wears. School-age children take a more clinical interest in their mother's pregnancy and may want to know, "How did the baby get in there?" and "How will it get out?" Whether her mother will die does not tend to be the focus of a child's questions about the impending birth of a sibling. The baby's eye color does not tend to be the focus of children's questions about the impending birth of a sibling.)

A 3-year-old girl's mother is 6 months pregnant. What concern is this child likely to verbalize? A. How the baby will "get out" B. What the baby will eat C. Whether her mother will die D. What color eyes the baby has

The CDC-recommended medication for the treatment of chlamydia would be: A) doxycycline. B) podofilox. C) acyclovir. D) penicillin.

A) doxycycline. Doxycycline is effective for treating chlamydia, but it should be avoided if the woman is pregnant. Podofilox is a recommended treatment for nonpregnant women diagnosed with human papilloma virus infection. Acyclovir is recommended for genital herpes simplex virus infection. Penicillin is not a CDC-recommended medication for chlamydia; it is the preferred medication for syphilis.

32. A woman is in for a routine prenatal checkup. You are assessing her urine for proteinuria. Which findings are considered normal (Select all that apply)? a. Dipstick assessment of trace to +1 b. <300 mg/24 hours c. Dipstick assessment of +2 d. >300 mg/24 hours

A, B

Which clinical reports does the nurse evaluate to identify ectopic pregnancy in a patient? Select all that apply. A. Quantitative human chorionic gonadotropin (β-hCG) levels B. Transvaginal ultrasound C. Progesterone level D. Thyroid test reports E. Kleihauer-Betke (KB) test

A, B, C (An ectopic pregnancy is indicated when β-hCG levels are >1500 milli-international units/mL but no intrauterine pregnancy is seen on the transvaginal ultrasound. A transvaginal ultrasound is repeated to verify if the pregnancy is inside the uterus. A progesterone level <5 ng/mL indicates ectopic pregnancy. Thyroid test reports need to be evaluated in case the patient has hyperemesis gravidarum, as hyperthyroidism is associated with this disorder. The KB test is used to determine transplacental hemorrhage.)

A pregnant patient in the first trimester reports spotting of blood with the cervical os closed and mild uterine cramping. What does the nurse need to assess? Select all that apply. A. Progesterone levels B. Transvaginal ultrasounds C. Human chorionic gonadotropin (hCG) measurement D. Blood pressure E. Kleihauer-Betke (KB) test reports

A, B, C (The spotting of blood with the cervical os closed and mild uterine cramping in the first trimester indicates a threatened miscarriage. Therefore the nurse needs to assess progesterone levels, transvaginal ultrasounds, and measurement of hCG to determine whether the fetus is alive and within the uterus. Blood pressure measurements do not help determine the fetal status. KB assay is prescribed to identify fetal-to-maternal bleeding, usually after a trauma.)

Examples of appropriate techniques to wake a sleepy infant for breastfeeding include (Select all that apply): a. Unwrapping the infant. b. Changing the diaper. c. Talking to the infant. d. Slapping the infant's hands and feet. e. Applying a cold towel to the infant's abdomen.

A, B, C (Unwrapping the infant, changing the diaper, and talking to the infant are appropriate techniques to use when trying to wake a sleepy infant. Slapping the infant's hand and feet and applying a cold towel to the infant's abdomen are not appropriate. The parent can rub the infant's hands or feet to wake the infant. Applying a cold towel to the infant's abdomen may lead to cold stress in the infant. The parent may want to apply a cool cloth to the infant's face to wake the infant.)

45. There is little consensus on the management of premenstrual dysphoric disorder (PMDD). However, nurses can advise women on several self-help modalities that often improve symptoms. The nurse knows that health teaching has been effective when the client reports that she has adopted a number of lifestyle changes, including (Select all that apply): a. Regular exercise. b. Improved nutrition. c. A daily glass of wine. d. Smoking cessation. e. Oil of evening primrose.

A, B, D, E

What does the nurse assess to detect the presence of a hypertensive disorder in a pregnant patient? Select all that apply. A. Proteinuria B. Epigastric pain C. Placenta previa D. Presence of edema E. Blood pressure (BP)

A, B, D, E (Proteinuria indicates hypertension in a pregnant patient. Proteinuria is concentration ≥300 mg/24 hours in a 24-hour urine collection. The nurse needs to assess the patient for epigastric pain because it indicates severe preeclampsia. Hypertension is likely to cause edema or swollen ankles as a result of greater hydrostatic pressure in the lower parts of the body. Therefore the nurse needs to assess the patient for the presence of edema. Accurate measurement of BP will help detect the presence of any hypertensive disorder. A systolic BP greater than 140 mm Hg or a diastolic BP greater than 90 mm Hg will indicate hypertension. Placenta previa is a condition wherein the placenta is implanted in the lower uterine segment covering the cervix, which causes bleeding when the cervix dilates.)

Which physiologic factors are reliable indicators of impending shock from postpartum hemorrhage? (Select all that apply.) a. Respirations b. Skin condition c. Blood pressure d. Level of consciousness e. Urinary output

A, B, D, E Blood pressure is not a reliable indicator; several more sensitive signs are available. Blood pressure does not drop until 30% to 40% of blood volume is lost. Respirations, pulse, skin condition, urinary output, and level of consciousness are more sensitive means of identifying hypovolemic shock.

31. The diagnosis of pregnancy is based on which positive signs of pregnancy (Select all that apply)? a. Identification of fetal heartbeat b. Palpation of fetal outline c. Visualization of the fetus d. Verification of fetal movement e. Positive hCG test

A, C, D

The nurse is caring for a woman who is at 24 weeks of gestation with suspected severe preeclampsia. Which signs and symptoms would the nurse expect to observe? (Select all that apply.) A. Decreased urinary output and irritability B. Transient headache and +1 proteinuria C. Ankle clonus and epigastric pain D. Platelet count of less than 100,000/mm3 and visual problems E. Seizure activity and hypotension

A, C, D (Decreased urinary output and irritability are signs of severe eclampsia. Ankle clonus and epigastric pain are signs of severe eclampsia. Platelet count of less than 100,000/mm3 and visual problems are signs of severe preeclampsia. A transient headache and +1 proteinuria are signs of preeclampsia and should be monitored. Seizure activity and hyperreflexia are signs of eclampsia.)

A maternity nurse should be aware of which fact about the amniotic fluid? A. It serves as a source of oral fluid and as a repository for waste from the fetus. B. The volume remains about the same throughout the term of a healthy pregnancy. C. A volume of less than 300 mL is associated with gastrointestinal malformations. D. A volume of more than 2 L is associated with fetal renal abnormalities.

A. It serves as a source of oral fluid and as a repository for waste from the fetus. Amniotic fluid also cushions the fetus and helps maintain a constant body temperature. The volume of amniotic fluid changes constantly. Too little amniotic fluid (oligohydramnios) is associated with renal abnormalities. Too much amniotic fluid (hydramnios) is associated with gastrointestinal and other abnormalities.

To promote bonding during the first hour after birth, the nurse can do which of the following? (Select all that apply). a. Delay procedures if appropriate. b. Allow the father to hold the newborn. c. Demonstrate proper bottle feeding techniques. d. Allow as much contact with the newborn as possible. e. Use the time to do parent teaching on newborn characteristics.

ABD

Nursing measures to promote bonding and attachment include which of the following? (Select all that apply). Assist the parents in unwrapping the baby to inspect. Point out that the infant grasping the mother's or father's finger is a natural reflex. Explain the physical changes in the newborn, such as molding, as being normal. Encourage the mother to let the infant stay in the nursery as much as possible so the mother can rest. Position the infant in a face to face position with the mother.

AE

A new mother states that her infant must be cold because the baby's hands and feet are blue. This common and temporary condition is called what? a.Acrocyanosis b.Erythema toxicum neonatorum c.Harlequin sign d.Vernix caseosa

ANS: A Acrocyanosis, or the appearance of slightly cyanotic hands and feet, is caused by vasomotor instability, capillary stasis, and a high hemoglobin level. Acrocyanosis is normal and intermittently appears over the first 7 to 10 days after childbirth. Erythema toxicum neonatorum (also called erythema neonatorum) is a transient newborn rash that resembles flea bites. The harlequin sign is a benign, transient color change in newborns. One half of the body is pale, and the other one half is ruddy or bluish-red with a line of demarcation. Vernix caseosa is a cheeselike, whitish substance that serves as a protective covering for the newborn.

17. Rho immune globulin will be ordered postpartum if which situation occurs? a. Mother Rh, baby Rh+ b. Mother Rh, baby Rh c. Mother Rh+, baby Rh+ d. Mother Rh+, baby Rh

ANS: A An Rh mother delivering an Rh+ baby may develop antibodies to fetal cells that entered her bloodstream when the placenta separated. The Rho immune globulin works to destroy the fetal cells in the maternal circulation before sensitization occurs. If mother and baby are both Rh+ or Rh the blood types are alike, so no antibody formation would be anticipated. If the Rh+ blood of the mother comes in contact with the Rh blood of the infant, no antibodies would develop because the antigens are in the mothers blood, not in the infants.

Which intervention can nurses use to prevent evaporative heat loss in the newborn? a.Drying the baby after birth, and wrapping the baby in a dry blanket b.Keeping the baby out of drafts and away from air conditioners c.Placing the baby away from the outside walls and windows d.Warming the stethoscope and the nurse's hands before touching the baby

ANS: A Because the infant is wet with amniotic fluid and blood, heat loss by evaporation quickly occurs. Heat loss by convection occurs when drafts come from open doors and air currents created by people moving around. If the heat loss is caused by placing the baby near cold surfaces or equipment, it is referred to as a radiation heat loss. Conduction heat loss occurs when the baby comes in contact with cold surfaces.

Of these psychosocial factors, which has the least negative effect on the health of the mother and/or fetus? a. Moderate coffee consumption b. Moderate alcohol consumption c. Cigarette smoke d. Emotional distress

ANS: A Birth defects in humans have not been related to caffeine consumption. Pregnant women who consume more than 300 mg of caffeine daily may be at increased risk for miscarriage or intrauterine growth restriction (IUGR).

The nurse expects which maternal cardiovascular finding during labor? a.Increased cardiac output b.Decreased pulse rate c.Decreased white blood cell (WBC) count d.Decreased blood pressure

ANS: A During each contraction, 400 ml of blood is emptied from the uterus into the maternal vascular system, which increases cardiac output by approximately 10% to 15% during the first stage of labor and by approximately 30% to 50% in the second stage of labor. The heart rate increases slightly during labor. The WBC count can increase during labor. During the first stage of labor, uterine contractions cause systolic readings to increase by approximately 10 mm Hg. During the second stage, contractions may cause systolic pressures to increase by 30 mm Hg and diastolic readings to increase by 25 mm Hg.

A first-time father is changing the diaper of his 1-day-old daughter. He asks the nurse, "What is this black, sticky stuff in her diaper?" What is the nurse's best response? a."That's meconium, which is your baby's first stool. It's normal." b."That's transitional stool." c."That means your baby is bleeding internally." d."Oh, don't worry about that. It's okay."

ANS: A Explaining what meconium is and that it is normal is an accurate statement and the most appropriate response. Transitional stool is greenish-brown to yellowish-brown and usually appears by the third day after the initiation of feeding. Telling the father that the baby is internally bleeding is not an accurate statement. Telling the father not to worry is not appropriate. Such responses are belittling to the father and do not teach him about the normal stool patterns of his daughter.

Which description of the four stages of labor is correct for both the definition and the duration? a.First stage: onset of regular uterine contractions to full dilation; less than 1 hour to 20 hours b.Second stage: full effacement to 4 to 5 cm; visible presenting part; 1 to 2 hours c.Third stage: active pushing to birth; 20 minutes (multiparous woman), 50 minutes (nulliparous woman) d.Fourth stage: delivery of the placenta to recovery; 30 minutes to 1 hour

ANS: A Full dilation may occur in less than 1 hour, but in first-time pregnancies full dilation can take up to 20 hours. The second stage of labor extends from full dilation to birth and takes an average of 20 to 50 minutes, although 2 hours is still considered normal. The third stage of labor extends from birth to the expulsion of the placenta and usually takes a few minutes. The fourth stage begins after the expulsion of the placenta and lasts until homeostasis is reestablished (approximately 2 hours).

Part of the health assessment of a newborn is observing the infant's breathing pattern. What is the predominate pattern of newborn's breathing? a.Abdominal with synchronous chest movements b.Chest breathing with nasal flaring c.Diaphragmatic with chest retraction d.Deep with a regular rhythm

ANS: A In a normal infant respiration, the chest and abdomen synchronously rise and infant breaths are shallow and irregular. Breathing with nasal flaring is a sign of respiratory distress. Diaphragmatic breathing with chest retraction is also a sign of respiratory distress.

Which newborn reflex is elicited by stroking the lateral sole of the infant's foot from the heel to the ball of the foot? a.Babinski b.Tonic neck c.Stepping d.Plantar grasp

ANS: A The Babinski reflex causes the toes to flare outward and the big toe to dorsiflex. The tonic neck reflex (also called the fencing reflex) refers to the posture assumed by newborns when in a supine position. The stepping reflex occurs when infants are held upright with their heel touching a solid surface and the infant appears to be walking. Plantar grasp reflex is similar to the palmar grasp reflex; when the area below the toes is touched, the infant's toes curl over the nurse's finger.

Which stage of labor varies the most in length? a.First b.Second c.Third d.Fourth

ANS: A The first stage of labor is considered to last from the onset of regular uterine contractions to the full dilation of the cervix. The first stage is significantly longer than the second and third stages combined. In a first-time pregnancy, the first stage of labor can take up to 20 hours. The second stage of labor lasts from the time the cervix is fully dilated to the birth of the fetus. The average length is 20 minutes for a multiparous woman and 50 minutes for a nulliparous woman. The third stage of labor lasts from the birth of the fetus until the placenta is delivered. This stage may be as short as 3 minutes or as long as 1 hour. The fourth stage of labor, recovery, lasts approximately 2 hours after the delivery of the placenta.

What is the nurse's understanding of the appropriate role of primary and secondary powers? a.Primary powers are responsible for the effacement and dilation of the cervix. b.Effacement is generally well ahead of dilation in women giving birth for the first time; they are closer together in subsequent pregnancies. c.Scarring of the cervix caused by a previous infection or surgery may make the delivery a bit more painful, but it should not slow or inhibit dilation. d.Pushing in the second stage of labor is more effective if the woman can breathe deeply and control some of her involuntary needs to push, as the nurse directs.

ANS: A The primary powers are responsible for dilation and effacement; secondary powers are concerned with expulsion of the fetus. Effacement is generally well ahead of dilation in first-time pregnancies; they are closer together in subsequent pregnancies. Scarring of the cervix may slow dilation. Pushing is more effective and less fatiguing when the woman begins to push only after she has the urge to do so.

What is the correct term for the cheeselike, white substance that fuses with the epidermis and serves as a protective coating? a.Vernix caseosa b.Surfactant c.Caput succedaneum d.Acrocyanosis

ANS: A The protection provided by vernix caseosa is needed because the infant's skin is so thin. Surfactant is a protein that lines the alveoli of the infant's lungs. Caput succedaneum is the swelling of the tissue over the presenting part of the fetal head. Acrocyanosis is cyanosis of the hands and feet, resulting in a blue coloring.

Which component of the sensory system is the least mature at birth? a.Vision b.Hearing c.Smell d.Taste

ANS: A The visual system continues to develop for the first 6 months after childbirth. As soon as the amniotic fluid drains from the ear (in minutes), the infant's hearing is similar to that of an adult. Newborns have a highly developed sense of smell and can distinguish and react to various tastes.

2. The laboratory results for a postpartum woman are as follows: blood type, A; Rh status, positive; rubella titer, 1:8 (enzyme immunoassay [EIA] 0.8); hematocrit, 30%. How should the nurse best interpret these data? a.Rubella vaccine should be administered. b.Blood transfusion is necessary. c.Rh immune globulin is necessary within 72 hours of childbirth. d. Kleihauer-Betke test should be performed.

ANS: A This clients rubella titer indicates that she is not immune and needs to receive a vaccine. These data do not indicate that the client needs a blood transfusion. Rh immune globulin is indicated only if the client has an Rh-negative status and the infant has an Rh-positive status. A Kleihauer-Betke test should be performed if a large fetomaternal transfusion is suspected, especially if the mother is Rh negative. However, the data provided do not indicate a need for performing this test.

During which phase of maternal adjustment will the mother relinquish the baby of her fantasies and accept the real baby? a. Letting-go b. Taking-in c. Taking-on d. Taking-hold

ANS: A Accepting the real infant and relinquishing the fantasy infant occurs during the letting-go phase of maternal adjustment. In the taking-in phase, the mother is primarily focused on her own needs. There is no taking-on phase of maternal adjustment. During the taking-hold phase, the mother assumes responsibility for her own care and shifts her attention to the infant.

Which occurrence is associated with cervical dilation and effacement? a. Bloody show b. False labor c. Lightening d. Bladder distention

ANS: A As the cervix begins to soften, dilate, and efface, expulsion of the mucous plug that sealed the cervix during pregnancy occurs. This causes rupture of small cervical capillaries. Cervical dilation and effacement do not occur with false labor. Lightening is the descent of the fetus toward the pelvic inlet before labor. Bladder distention occurs when the bladder is not emptied frequently. It may slow down the descent of the fetus during labor.

Marfan syndrome is an autosomal dominant genetic disorder that displays as weakness of the connective tissue, joint deformities, ocular dislocation, and weakness to the aortic wall and root. While providing care to a client with Marfan syndrome during labor, which intervention should the nurse complete first? a. Antibiotic prophylaxis b. b-Blockers c. Surgery d. Regional anesthesia

ANS: A Because of the potential for cardiac involvement during the third trimester and after birth, treatment with prophylactic antibiotics is highly recommended. b-Blockers and restricted activity are recommended as treatment modalities earlier in the pregnancy. Regional anesthesia is well tolerated by clients with Marfan syndrome; however, it is not essential to care. Adequate labor support may be all that is necessary if an epidural is not part of the womans birth plan. Surgery for cardiovascular changes such as mitral valve prolapse, aortic regurgitation, root dilation, or dissection may be necessary. Mortality rates may be as high as 50% in women who have severe cardiac disease. PTS: 1 DIF: Cognitive Level: Analysis REF: 287 OBJ: Nursing Process: Implementation MSC: Client Needs: Physiologic Integrity

As related to the care of the patient with anemia, the nurse should be aware that: a. It is the most common medical disorder of pregnancy. b. It can trigger reflex brachycardia. c. The most common form of anemia is caused by folate deficiency. d. Thalassemia is a European version of sickle cell anemia.

ANS: A Combined with any other complication, anemia can result in congestive heart failure. Reflex bradycardia is a slowing of the heart in response to the blood flow increases immediately after birth. The most common form of anemia is iron deficiency anemia. Both thalassemia and sickle cell hemoglobinopathy are hereditary but not directly related or confined to geographic areas. PTS: 1 DIF: Cognitive Level: Knowledge REF: 290 OBJ: Nursing Process: Planning MSC: Client Needs: Physiologic Integrity

When the pregnant diabetic woman experiences hypoglycemia while hospitalized, the nurse should intervene by having the patient: a. Eat six saltine crackers. b. Drink 8 oz of orange juice with 2 tsp of sugar added. c. Drink 4 oz of orange juice followed by 8 oz of milk. d. Eat hard candy or commercial glucose wafers.

ANS: A Crackers provide carbohydrates in the form of polysaccharides. Orange juice and sugar will increase the blood sugar but not provide a slow-burning carbohydrate to sustain the blood sugar. Milk is a disaccharide and orange juice is a monosaccharide. They will provide an increase in blood sugar but will not sustain the level. Hard candy or commercial glucose wafers provide only monosaccharides. PTS: 1 DIF: Cognitive Level: Application REF: 274 OBJ: Nursing Process: Implementation MSC: Client Needs: Physiologic Integrity

The nurse would expect which maternal cardiovascular finding during labor? a. Increased cardiac output b. Decreased pulse rate c. Decreased white blood cell (WBC) count d. Decreased blood pressure

ANS: A During each contraction, 400 mL of blood is emptied from the uterus into the maternal vascular system. This increases cardiac output by about 51% above baseline pregnancy values at term. The heart rate increases slightly during labor. The WBC count can increase during labor. During the first stage of labor, uterine contractions cause systolic readings to increase by about 10 mm Hg. During the second stage, contractions may cause systolic pressures to increase by 30 mm Hg and diastolic readings to increase by 25 mm Hg.

Which description of the four stages of labor is correct for both definition and duration? a. First stage: onset of regular uterine contractions to full dilation; less than 1 hour to 20 hours b. Second stage: full effacement to 4 to 5 cm; visible presenting part; 1 to 2 hours c. Third state: active pushing to birth; 20 minutes (multiparous women), 50 minutes (first-timer) d. Fourth stage: delivery of the placenta to recovery; 30 minutes to 1 hour

ANS: A Full dilation may occur in less than 1 hour, but in first-time pregnancies it can take up to 20 hours. The second stage extends from full dilation to birth and takes an average of 20 to 50 minutes, although 2 hours is still considered normal. The third stage extends from birth to expulsion of the placenta and usually takes a few minutes. The fourth stage begins after expulsion of the placenta and lasts until homeostasis is reestablished (about 2 hours).

Which presentation is described accurately in terms of both presenting part and frequency of occurrence? a. Cephalic: occiput; at least 95% b. Breech: sacrum; 10% to 15% c. Shoulder: scapula; 10% to 15% d. Cephalic: cranial; 80% to 85%

ANS: A In cephalic presentations (head first), the presenting part is the occiput; this occurs in 96% of births. In a breech birth, the sacrum emerges first; this occurs in about 3% of births. In shoulder presentations, the scapula emerges first; this occurs in only 1% of births.

In assessing the knowledge of a pregestational woman with type 1 diabetes concerning changing insulin needs during pregnancy, the nurse recognizes that further teaching is warranted when the client states: a. I will need to increase my insulin dosage during the first 3 months of pregnancy. b. Insulin dosage will likely need to be increased during the second and third trimesters. c. Episodes of hypoglycemia are more likely to occur during the first 3 months. d. Insulin needs should return to normal within 7 to 10 days after birth if I am bottle-feeding.

ANS: A Insulin needs are reduced in the first trimester because of increased insulin production by the pancreas and increased peripheral sensitivity to insulin. Insulin dosage will likely need to be increased during the second and third trimesters, Episodes of hypoglycemia are more likely to occur during the first 3 months, and Insulin needs should return to normal within 7 to 10 days after birth if I am bottle-feeding are accurate statements and signify that the woman has understood the teachings regarding control of her diabetes during pregnancy. PTS: 1 DIF: Cognitive Level: Application REF: 269 OBJ: Nursing Process: Evaluation MSC: Client Needs: Physiologic Integrity

With one exception, the safest pregnancy is one in which the woman is drug and alcohol free. For women addicted to opioids, ________________________ treatment is the current standard of care during pregnancy. a. Methadone maintenance b. Detoxification c. Smoking cessation d. 4 Ps Plus

ANS: A Methadone maintenance treatment (MMT) is currently considered the standard of care for pregnant women who are dependent on heroin or other narcotics. Buprenorphine is another medication approved for opioid addiction treatment that is increasingly being used during pregnancy. Opioid replacement therapy has been shown to decrease opioid and other drug use, reduce criminal activity, improve individual functioning, and decrease rates of infections such as hepatitis B and C, HIV, and other sexually transmitted infections. Detoxification is the treatment used for alcohol addiction. Pregnant women requiring withdrawal from alcohol should be admitted for inpatient management. Women are more likely to stop smoking during pregnancy than at any other time in their lives. A smoking cessation program can assist in achieving this goal. The 4 Ps Plus is a screening tool designed specifically to identify pregnant women who need in-depth assessment related to substance abuse. PTS: 1 DIF: Cognitive Level: Application REF: 298 OBJ: Nursing Process: Planning MSC: Client Needs: Psychosocial Integrity

Screening at 24 weeks of gestation reveals that a pregnant woman has gestational diabetes mellitus (GDM). In planning her care, the nurse and the woman mutually agree that an expected outcome is to prevent injury to the fetus as a result of GDM. The nurse identifies that the fetus is at greatest risk for: a. Macrosomia. b. Congenital anomalies of the central nervous system. c. Preterm birth. d. Low birth weight.

ANS: A Poor glycemic control later in pregnancy increases the rate of fetal macrosomia. Poor glycemic control during the preconception time frame and into the early weeks of the pregnancy is associated with congenital anomalies. Preterm labor or birth is more likely to occur with severe diabetes and is the greatest risk in women with pregestational diabetes. Increased weight, or macrosomia, is the greatest risk factor for this woman. PTS: 1 DIF: Cognitive Level: Comprehension REF: 270 OBJ: Nursing Process: Planning, Implementation MSC: Client Needs: Physiologic Integrity

With regard to the association of maternal diabetes and other risk situations affecting mother and fetus, nurses should be aware that: a. Diabetic ketoacidosis (DKA) can lead to fetal death at any time during pregnancy. b. Hydramnios occurs approximately twice as often in diabetic pregnancies. c. Infections occur about as often and are considered about as serious in diabetic and nondiabetic pregnancies. d. Even mild to moderate hypoglycemic episodes can have significant effects on fetal well-being.

ANS: A Prompt treatment of DKA is necessary to save the fetus and the mother. Hydramnios occurs 10 times more often in diabetic pregnancies. Infections are more common and more serious in pregnant women with diabetes. Mild to moderate hypoglycemic episodes do not appear to have significant effects on fetal well-being. PTS: 1 DIF: Cognitive Level: Comprehension REF: 270 OBJ: Nursing Process: Planning MSC: Client Needs: Physiologic Integrity

Prophylaxis of subacute bacterial endocarditis is given before and after birth when a pregnant woman has: a. Valvular disease. b. Congestive heart disease. c. Arrhythmias. d. Postmyocardial infarction.

ANS: A Prophylaxis for intrapartum endocarditis and pulmonary infection may be provided for women who have mitral valve stenosis. Prophylaxis for intrapartum endocarditis is not indicated for congestive heart disease, arrhythmias, or after myocardial infarction. PTS: 1 DIF: Cognitive Level: Comprehension REF: 285 OBJ: Nursing Process: Implementation MSC: Client Needs: Physiologic Integrity

To care for a laboring woman adequately, the nurse understands that the __________ stage of labor varies the most in length? a. First b. Second c. Third d. Fourth

ANS: A The first stage of labor is considered to last from the onset of regular uterine contractions to full dilation of the cervix. The first stage is much longer than the second and third stages combined. In a first-time pregnancy the first stage of labor can take up to 20 hours. The second stage of labor lasts from the time the cervix is fully dilated to the birth of the fetus. The average length is 20 minutes for a multiparous woman and 50 minutes for a nulliparous woman. The third stage of labor lasts from the birth of the fetus until the placenta is delivered. This stage may be as short as 3 minutes or as long as 1 hour. The fourth stage of labor, recovery, lasts about 2 hours after delivery of the placenta.

Which major neonatal complication is carefully monitored after the birth of the infant of a diabetic mother? a. Hypoglycemia b. Hypercalcemia c. Hypobilirubinemia d. Hypoinsulinemia

ANS: A The neonate is at highest risk for hypoglycemia because fetal insulin production is accelerated during pregnancy to metabolize excessive glucose from the mother. At birth, the maternal glucose supply stops and the neonatal insulin exceeds the available glucose, thus leading to hypoglycemia. Hypocalcemia is associated with preterm birth, birth trauma, and asphyxia, all common problems of the infant of a diabetic mother. Excess erythrocytes are broken down after birth and release large amounts of bilirubin into the neonates circulation, with resulting hyperbilirubinemia. Because fetal insulin production is accelerated during pregnancy, the neonate presents with hyperinsulinemia. PTS: 1 DIF: Cognitive Level: Comprehension REF: 272 OBJ: Nursing Process: Planning MSC: Client Needs: Health Promotion and Maintenance

In relation to primary and secondary powers, the maternity nurse comprehends that: a. Primary powers are responsible for effacement and dilation of the cervix. b. Effacement generally is well ahead of dilation in women giving birth for the first time; they are closer together in subsequent pregnancies. c. Scarring of the cervix caused by a previous infection or surgery may make the delivery a bit more painful, but it should not slow or inhibit dilation. d. Pushing in the second stage of labor is more effective if the woman can breathe deeply and control some of her involuntary needs to push, as the nurse directs.

ANS: A The primary powers are responsible for dilation and effacement; secondary powers are concerned with expulsion of the fetus. Effacement generally is well ahead of dilation in first-timers; they are closer together in subsequent pregnancies. Scarring of the cervix may slow dilation. Pushing is more effective and less fatiguing when the woman begins to push only after she has the urge to do so.

Which statements regarding physiologic jaundice are accurate? (Select all that apply.) a.Neonatal jaundice is common; however, kernicterus is rare. b.Appearance of jaundice during the first 24 hours or beyond day 7 indicates a pathologic process. c.Because jaundice may not appear before discharge, parents need instruction on how to assess for jaundice and when to call for medical help. d.Jaundice is caused by reduced levels of serum bilirubin. e.Breastfed babies have a lower incidence of jaundice

ANS: A, B, C Neonatal jaundice occurs in 60% of term newborns and in 80% of preterm infants. The complication called kernicterus is rare. Jaundice in the first 24 hours or that persists past day 7 is cause for medical concern. Parents need to be taught how to evaluate their infant for signs of jaundice. Jaundice is caused by elevated levels of serum bilirubin. Breastfeeding is associated with an increased incidence of jaundice.

Congenital anomalies can occur with the use of antiepileptic drugs (AEDs), including (Select all that apply): a. Cleft lip. b. Congenital heart disease. c. Neural tube defects. d. Gastroschisis. e. Diaphragmatic hernia.

ANS: A, B, C Congenital anomalies that can occur with AEDs include cleft lip or palate, congenital heart disease, urogenital defects, and neural tube defects. Gastroschisis and diaphragmatic hernia are not associated with the use of AEDs. PTS: 1 DIF: Cognitive Level: Comprehension REF: 294 OBJ: Nursing Process: Planning MSC: Client Needs: Physiologic Integrity

4. If a woman is at risk for thrombus and is not ready to ambulate, which nursing intervention would the nurse use? (Select all that apply.) a. Putting her in antiembolic stockings (thromboembolic deterrent [TED] hose) and/or sequential compression device (SCD) boots b. Having her flex, extend, and rotate her feet, ankles, and legs c. Having her sit in a chair d. Immediately notifying the physician if a positive Homans sign occurs e. Promoting bed rest

ANS: A, B, D Sitting immobile in a chair does not help; bed exercise and prophylactic footwear might. TED hose and SCD boots are recommended. The client should be encouraged to ambulate with assistance, not remain in bed. Bed exercises are useful. A positive Homans sign (calf muscle pain or warmth, redness, tenderness) requires the physicians immediate attention.

Because of its size and rigidity, the fetal head has a major effect on the birth process. Which bones comprise the structure of the fetal skull? (Select all that apply.) a.Parietal b.Temporal c.Fontanel d.Occipital e.Femoral

ANS: A, B, D The fetal skull has two parietal bones, two temporal bones, an occipital bone, and a frontal bone. The fontanels are membrane-filled spaces.

Which factors influence cervical dilation (Select all that apply) ? a. Strong uterine contractions b. The force of the presenting fetal part against the cervix c. The size of the female pelvis d. The pressure applied by the amniotic sac e. Scarring of the cervix

ANS: A, B, D, E Dilation of the cervix occurs by the drawing upward of the musculofibrous components of the cervix, which is caused by strong uterine contractions. Pressure exerted by the amniotic fluid while the membranes are intact or by the force applied by the presenting part also can promote cervical dilation. Scarring of the cervix as a result of a previous infection or surgery may slow cervical dilation. Pelvic size does not affect cervical dilation.

Which changes take place in the woman's reproductive system, days or even weeks before the commencement of labor? (Select all that apply.) a.Lightening b.Exhaustion c.Bloody show d.Rupture of membranes e.Decreased fetal movement

ANS: A, C, D Signs that precede labor may include lightening, urinary frequency, backache, weight loss, surge of energy, bloody show, and rupture of membranes. Many women experience a burst of energy before labor. A decrease in fetal movement is an ominous sign that does not always correlate with labor.

19. When caring for a newly delivered woman, what is the best measure to prevent abdominal distention after a cesarean birth? a. Rectal suppositories b. Early and frequent ambulation c. Tightening and relaxing abdominal muscles d. Carbonated beverages

ANS: B Activity will aid the movement of accumulated gas in the gastrointestinal tract. Rectal suppositories can be helpful after distention occurs; however, they do not prevent it. Ambulation is the best prevention. Carbonated beverages may increase distention.

What is the rationale for evaluating the plantar crease within a few hours of birth? a.Newborn has to be footprinted. b.As the skin dries, the creases will become more prominent. c.Heel sticks may be required. d.Creases will be less prominent after 24 hours.

ANS: B As the infant's skin begins to dry, the creases will appear more prominent, and the infant's gestation could be misinterpreted. Footprinting nor heel sticks will not interfere with the creases. The creases will appear more prominent after 24 hours.

Which statement best describes the transition period between intrauterine and extrauterine existence for the newborn? a.Consists of four phases, two reactive and two of decreased responses b.Lasts from birth to day 28 of life c.Applies to full-term births only d.Varies by socioeconomic status and the mother's age

ANS: B Changes begin immediately after birth; the cutoff time when the transition is considered over (although the baby keeps changing) is 28 days. This transition period has three phases: first reactivity, decreased response, and second reactivity. All newborns experience this transition period, regardless of age or type of birth. Although stress can cause variations in the phases, the mother's age and wealth do not disturb the pattern.

The process during which bilirubin is changed from a fat-soluble product to a water-soluble product is known as what? a.Enterohepatic circuit b.Conjugation of bilirubin c.Unconjugated bilirubin d.Albumin binding

ANS: B Conjugation of bilirubin is the process of changing the bilirubin from a fat-soluble to a water-soluble product and is the route by which part of the bile produced by the liver enters the intestine, is reabsorbed by the liver, and is then recycled into the intestine. Unconjugated bilirubin is a fat-soluble product. Albumin binding is the process during which something attaches to a protein molecule.

The nurse is circulating during a cesarean birth of a preterm infant. The obstetrician requests that cord clamping be delayed. What is the rationale for this directive? a.To reduce the risk for jaundice b.To reduce the risk of intraventricular hemorrhage c.To decrease total blood volume d.To improve the ability to fight infection

ANS: B Delayed cord clamping provides the greatest benefits to the preterm infant. These benefits include a significant reduction in intraventricular hemorrhage, a reduced need for a blood transfusion, and improved blood cell volume. The risk of jaundice can increase, requiring phototherapy. Although no difference in the newborn's infection fighting ability occurs, iron status is improved, which can provide benefits for 6 months.

Which statement is the best rationale for assessing the maternal vital signs between uterine contractions? a.During a contraction, assessing the fetal heart rate is the priority. b.Maternal circulating blood volume temporarily increases during contractions. c.Maternal blood flow to the heart is reduced during contractions. d.Vital signs taken during contractions are not accurate.

ANS: B During uterine contractions, blood flow to the placenta temporarily stops, causing a relative increase in the mother's blood volume, which, in turn, temporarily increases blood pressure and slows the pulse. Monitoring fetal responses to the contractions is important; however, this question concerns the maternal vital signs. Maternal blood flow is increased during a contraction. Vital signs are altered by contractions but are considered accurate for that period.

What is the primary difference between the labor of a nullipara and that of a multipara? a.Amount of cervical dilation b.Total duration of labor c.Level of pain experienced d.Sequence of labor mechanisms

ANS: B In a first-time pregnancy, the descent is usually slow but steady; in subsequent pregnancies, the descent is more rapid, resulting in a shorter duration of labor. Cervical dilation is the same for all labors. The level of pain is individual to the woman, not to the number of labors she has experienced. The sequence of labor mechanisms is the same with all labors.

What is the correct term describing the slight overlapping of cranial bones or shaping of the fetal head during labor? a.Lightening b.Molding c.Ferguson reflex d.Valsalva maneuver

ANS: B Molding also permits adaptation to various diameters of the maternal pelvis. Lightening is the mother's sensation of decreased abdominal distention, which usually occurs the week before labor. The Ferguson reflex is the contraction urge of the uterus after the stimulation of the cervix. The Valsalva maneuver describes conscious pushing during the second stage of labor.

The condition during which infants are at an increased risk for subgaleal hemorrhage is called what? a.Infection b.Jaundice c.Caput succedaneum d.Erythema toxicum neonatorum

ANS: B Subgaleal hemorrhage is bleeding into the subgaleal compartment and is the result of the transition from a forceps or vacuum application. Because of the breakdown of the red blood cells within a hematoma, infants are at greater risk for jaundice. Subgaleal hemorrhage does not increase the risk for infections. Caput succedaneum is an edematous area on the head caused by pressure against the cervix. Erythema toxicum neonatorum is a benign rash of unknown cause that consists of blotchy red areas.

Which statement related to fetal positioning during labor is correct and important for the nurse to understand? a.Position is a measure of the degree of descent of the presenting part of the fetus through the birth canal. b.Birth is imminent when the presenting part is at +4 to +5 cm below the spine. c.The largest transverse diameter of the presenting part is the suboccipitobregmatic diameter. d.Engagement is the term used to describe the beginning of labor.

ANS: B The station of the presenting part should be noted at the beginning of labor to determine the rate of descent. Position is the relationship of the presenting part of the fetus to the four quadrants of the mother's pelvis; station is the measure of degree of descent. The largest diameter is usually the biparietal diameter. The suboccipitobregmatic diameter is the smallest, although one of the most critical. Engagement often occurs in the weeks just before labor in nulliparous women and before or during labor in multiparous women.

Which cardiovascular changes cause the foramen ovale to close at birth? a.Increased pressure in the right atrium b.Increased pressure in the left atrium c.Decreased blood flow to the left ventricle d.Changes in the hepatic blood flow

ANS: B With the increase in the blood flow to the left atrium from the lungs, the pressure is increased, and the foramen ovale is functionally closed. The pressure in the right atrium decreases at birth and is higher during fetal life. Blood flow increases to the left ventricle after birth. The hepatic blood flow changes but is not the reason for the closure of the foramen ovale.

A woman has a history of drug use and is screened for hepatitis B during the first trimester. What is an appropriate action? a. Provide a low-protein diet. b. Offer the vaccine. c. Discuss the recommendation to bottle-feed her baby. d. Practice respiratory isolation.

ANS: B A person who has a history of high risk behaviors should be offered the hepatitis B vaccine. Care is supportive and includes bed rest and a high-protein, low-fat diet. The first trimester is too early to discuss feeding methods with a woman in the high risk category. Hepatitis B is transmitted through blood. PTS: 1 DIF: Cognitive Level: Application REF: 298 OBJ: Nursing Process: Planning MSC: Client Needs: Health Promotion and Maintenance

While providing care in an obstetric setting, the nurse should understand that postpartum care of the woman with cardiac disease: a. Is the same as that for any pregnant woman. b. Includes rest, stool softeners, and monitoring of the effect of activity. c. Includes ambulating frequently, alternating with active range of motion. d. Includes limiting visits with the infant to once per day.

ANS: B Bed rest may be ordered, with or without bathroom privileges. Bowel movements without stress or strain for the woman are promoted with stool softeners, diet, and fluid. Care of the woman with cardiac disease in the postpartum period is tailored to the womans functional capacity. The woman will be on bed rest to conserve energy and reduce the strain on the heart. Although the woman may need help caring for the infant, breastfeeding and infant visits are not contraindicated. PTS: 1 DIF: Cognitive Level: Comprehension REF: 290 OBJ: Nursing Process: Planning MSC: Client Needs: Physiologic Integrity

An 18-year-old client who has reached 16 weeks of gestation was recently diagnosed with pregestational diabetes. She attends her centering appointment accompanied by one of her girlfriends. This young woman appears more concerned about how her pregnancy will affect her social life than about her recent diagnosis of diabetes. Several nursing diagnoses are applicable to assist in planning adequate care. The most appropriate diagnosis at this time is: a. Risk for injury to the fetus related to birth trauma. b. Noncompliance related to lack of understanding of diabetes and pregnancy and requirements of the treatment plan. c. Deficient knowledge related to insulin administration. d. Risk for injury to the mother related to hypoglycemia or hyperglycemia.

ANS: B Before a treatment plan is developed or goals for the outcome of care are outlined, this client must come to an understanding of diabetes and the potential effects on her pregnancy. She appears to have greater concern for changes to her social life than adoption of a new self-care regimen. Risk for injury to the fetus related to either placental insufficiency or birth trauma may come much later in the pregnancy. At this time the client is having difficulty acknowledging the adjustments that she needs to make to her lifestyle to care for herself during pregnancy. The client may not yet be on insulin. Insulin requirements increase with gestation. The importance of glycemic control must be part of health teaching for this client. However, she has not yet acknowledged that changes to her lifestyle need to be made, and she may not participate in the plan of care until understanding takes place. PTS: 1 DIF: Cognitive Level: Analysis REF: 269 OBJ: Nursing Process: Diagnosis MSC: Client Needs: Psychosocial Integrity

Nurses caring for antepartum women with cardiac conditions should be aware that: a. Stress on the heart is greatest in the first trimester and the last 2 weeks before labor. b. Women with class II cardiac disease should avoid heavy exertion and any activity that causes even minor symptoms. c. Women with class III cardiac disease should have 8 to 10 hours of sleep every day and limit housework, shopping, and exercise. d. Women with class I cardiac disease need bed rest through most of the pregnancy and face the possibility of hospitalization near term.

ANS: B Class II cardiac disease is symptomatic with ordinary activity. Women in this category need to avoid heavy exertion and limit regular activities as symptoms dictate. Stress is greatest between weeks 28 and 32, when homodynamic changes reach their maximum. Class III cardiac disease is symptomatic with less than ordinary activity. These women need bed rest most of the day and face the possibility of hospitalization near term. Class I cardiac disease is asymptomatic at normal levels of activity. These women can carry on limited normal activities with discretion, although they still need a good amount of sleep. PTS: 1 DIF: Cognitive Level: Comprehension REF: 284 OBJ: Nursing Process: Planning MSC: Client Needs: Physiologic Integrity

Which statement is the best rationale for assessing maternal vital signs between contractions? a. During a contraction, assessing fetal heart rates is the priority. b. Maternal circulating blood volume increases temporarily during contractions. c. Maternal blood flow to the heart is reduced during contractions. d. Vital signs taken during contractions are not accurate.

ANS: B During uterine contractions, blood flow to the placenta temporarily stops, causing a relative increase in the mother's blood volume, which in turn temporarily increases blood pressure and slows pulse. It is important to monitor fetal response to contractions; however, this question is concerned with the maternal vital signs. Maternal blood flow is increased during a contraction. Vital signs are altered by contractions but are considered accurate for that period of time.

The slight overlapping of cranial bones or shaping of the fetal head during labor is called: a. Lightening. b. Molding. c. Ferguson reflex. d. Valsalva maneuver.

ANS: B Fetal head formation is called molding. Molding also permits adaptation to various diameters of the maternal pelvis. Lightening is the mother's sensation of decreased abdominal distention, which usually occurs the week before labor. The Ferguson reflex is the contraction urge of the uterus after stimulation of the cervix. The Valsalva maneuver describes conscious pushing during the second stage of labor.

The primary difference between the labor of a nullipara and that of a multipara is the: a. Amount of cervical dilation. c. Level of pain experienced. b. Total duration of labor. d. Sequence of labor mechanisms.

ANS: B In a first-time pregnancy, descent is usually slow but steady; in subsequent pregnancies, descent is more rapid, resulting in a shorter duration of labor. Cervical dilation is the same for all labors. Level of pain is individual to the woman, not to the number of labors she has experienced. The sequence of labor mechanisms is the same with all labors.

The use of methamphetamine (meth) has been described as a significant drug problem in the United States. In order to provide adequate nursing care to this client population the nurse must be cognizant that methamphetamine: a. Is similar to opiates. b. Is a stimulant with vasoconstrictive characteristics. c. Should not be discontinued during pregnancy. d. Is associated with a low rate of relapse.

ANS: B Methamphetamines are stimulants with vasoconstrictive characteristics similar to cocaine and are used similarly. As is the case with cocaine users, methamphetamine users are urged to immediately stop all use during pregnancy. Unfortunately, because methamphetamine users are extremely psychologically addicted, the rate of relapse is very high. PTS: 1 DIF: Cognitive Level: Comprehension REF: 299 OBJ: Nursing Process: Assessment MSC: Client Needs: Psychosocial Integrity

A new mother with which of these thyroid disorders would be strongly discouraged from breastfeeding? a. Hyperthyroidism b. Phenylketonuria (PKU) c. Hypothyroidism d. Thyroid storm

ANS: B PKU is a cause of mental retardation in infants; mothers with PKU pass on phenylalanine. A woman with hyperthyroidism or hypothyroidism would have no particular reason not to breastfeed. A thyroid storm is a complication of hyperthyroidism. PTS: 1 DIF: Cognitive Level: Comprehension REF: 283 OBJ: Nursing Process: Planning MSC: Client Needs: Physiologic Integrity

To teach patients about the process of labor adequately, the nurse knows that which event is the best indicator of true labor? a. Bloody show b. Cervical dilation and effacement c. Fetal descent into the pelvic inlet d. Uterine contractions every 7 minutes

ANS: B The conclusive distinction between true and false labor is that contractions of true labor cause progressive change in the cervix. Bloody show can occur before true labor. Fetal descent can occur before true labor. False labor may have contractions that occur this frequently; however, this is usually inconsistent.

As relates to fetal positioning during labor, nurses should be aware that: a. Position is a measure of the degree of descent of the presenting part of the fetus through the birth canal. b. Birth is imminent when the presenting part is at +4 to +5 cm below the spine. c. The largest transverse diameter of the presenting part is the suboccipitobregmatic diameter. d. Engagement is the term used to describe the beginning of labor.

ANS: B The station of the presenting part should be noted at the beginning of labor so that the rate of descent can be determined. Position is the relation of the presenting part of the fetus to the four quadrants of the mother's pelvis;station is the measure of degree of descent. The largest diameter usually is the biparietal diameter. The suboccipitobregmatic diameter is the smallest, although one of the most critical. Engagement often occurs in the weeks just before labor in nulliparas and before or during labor in multiparas.

A 26-year-old primigravida has come to the clinic for her regular prenatal visit at 12 weeks. She appears thin and somewhat nervous. She reports that she eats a well-balanced diet, although her weight is 5 pounds less than it was at her last visit. The results of laboratory studies confirm that she has a hyperthyroid condition. Based on the available data, the nurse formulates a plan of care. What nursing diagnosis is most appropriate for the woman at this time? a. Deficient fluid volume b. Imbalanced nutrition: less than body requirements c. Imbalanced nutrition: more than body requirements d. Disturbed sleep pattern

ANS: B This clients clinical cues include weight loss, which would support the nursing diagnosis of Imbalanced nutrition: less than body requirements. No clinical signs or symptoms support the nursing diagnosis of Deficient fluid volume. This client reports weight loss, not weight gain. Imbalanced nutrition: more than body requirements is not an appropriate nursing diagnosis. Although the client reports nervousness, based on the clients other clinical symptoms the most appropriate nursing diagnosis would be Imbalanced nutrition: less than body requirements. PTS: 1 DIF: Cognitive Level: Analysis REF: 282 OBJ: Nursing Process: Diagnosis MSC: Client Needs: Physiologic Integrity

In terms of the incidence and classification of diabetes, maternity nurses should know that: a. Type 1 diabetes is most common. b. Type 2 diabetes often goes undiagnosed. c. Gestational diabetes mellitus (GDM) means that the woman will be receiving insulin treatment until 6 weeks after birth. d. Type 1 diabetes may become type 2 during pregnancy.

ANS: B Type 2 diabetes often goes undiagnosed because hyperglycemia develops gradually and often is not severe. Type 2 diabetes, sometimes called adult onset diabetes, is the most common. GDM refers to any degree of glucose intolerance first recognized during pregnancy. Insulin may or may not be needed. People do not go back and forth between types 1 and 2 diabetes. PTS: 1 DIF: Cognitive Level: Knowledge REF: 268 OBJ: Nursing Process: Assessment MSC: Client Needs: Physiologic Integrity

What are the various modes of heat loss in the newborn? (Select all that apply.) a.Perspiration b.Convection c.Radiation d.Conduction e.Urination

ANS: B, C, D Convection, radiation, evaporation, and conduction are the four modes of heat loss in the newborn.

Autoimmune disorders often occur during pregnancy because a large percentage of women with an autoimmune disorder are of childbearing age. Identify all disorders that fall into the category of collagen vascular disease. a. Multiple sclerosis b. Systemic lupus erythematosus c. Antiphospholipid syndrome d. Rheumatoid arthritis e. Myasthenia gravis

ANS: B, C, D, E Multiple sclerosis is not an autoimmune disorder. This patchy demyelinization of the spinal cord may be a viral disorder. Autoimmune disorders (collagen vascular disease) make up a large group of conditions that disrupt the function of the immune system of the body. They include those listed, as well as systemic sclerosis. PTS: 1 DIF: Cognitive Level: Comprehension REF: 296 OBJ: Nursing Process: Assessment MSC: Client Needs: Health Promotion and Maintenance COMPLETION

5. Postpartum fatigue (PPF) is more than just feeling tired. It is a complex phenomenon affected by physiologic, psychologic, and situational variables. Which factors contribute to this phenomenon? (Select all that apply.) a. Precipitous labor b. Hospital routines c. Bottle feeding d. Anemia e. Excitement

ANS: B, D, E Physical fatigue and exhaustion are often associated with a long labor or cesarean birth, hospital routines, breastfeeding, and infant care. PPF is also attributed to anemia, infection, or thyroid dysfunction. The excitement and exhilaration of delivering a new infant along with well-intentioned visitors may make rest difficult.

1. The breast-feeding mother should be taught to expect which changes to the condition of the breasts? (Select all that apply.) a. Breast tenderness is likely to persist for approximately 1 week after the start of lactation. b. As lactation is established, a mass may form that can be distinguished from cancer by its positional shift from day to day. c. In nonlactating mothers, colostrum is present for the first few days after childbirth. d. If suckling is never begun or is discontinued, then lactation ceases within a few days to a week. e. Little change occurs to the breasts in the first 48 hours.

ANS: B,C,D

The nurse is performing an initial assessment of a client in labor. What is the appropriate terminology for the relationship of the fetal body parts to one another? a.Lie b.Presentation c.Attitude d.Position

ANS: C Attitude is the relationship of the fetal body parts to one another. Lie is the relationship of the long axis (spine) of the fetus to the long axis (spine) of the mother. Presentation refers to the part of the fetus that enters the pelvic inlet first and leads through the birth canal during labor at term. Position is the relationship of the presenting part of the fetus to the four quadrants of the mother's pelvis.

Which nursing assessment indicates that a woman who is in second-stage labor is almost ready to give birth? a.Fetal head is felt at 0 station during the vaginal examination. b.Bloody mucous discharge increases. c.Vulva bulges and encircles the fetal head. d.Membranes rupture during a contraction.

ANS: C During the active pushing (descent) phase, the woman has strong urges to bear down as the presenting part of the fetus descends and presses on the stretch receptors of the pelvic floor. The vulva stretches and begins to bulge, encircling the fetal head. Birth of the head occurs when the station is +4. A 0 station indicates engagement. Bloody show occurs throughout the labor process and is not an indication of an imminent birth. Rupture of membranes can occur at any time during the labor process and does not indicate an imminent birth.

While assessing the integument of a 24-hour-old newborn, the nurse notes a pink papular rash with vesicles superimposed on the thorax, back, and abdomen. What action is the highest priority for the nurse to take at this time? a.Immediately notify the physician. b.Move the newborn to an isolation nursery. c.Document the finding as erythema toxicum neonatorum. d.Take the newborn's temperature, and obtain a culture of one of the vesicles.

ANS: C Erythema toxicum neonatorum (or erythema neonatorum) is a newborn rash that resembles flea bites. Notification of the physician, isolation of the newborn, or additional interventions are not necessary when erythema toxicum neonatorum is present.

A woman's position is an important component of the labor progress. Which guidance is important for the nurse to provide to the laboring client? a.The supine position, which is commonly used in the United States, increases blood flow. b.The laboring client positioned on her hands and knees ("all fours" position) is hard on the woman's back. c.Frequent changes in position help relieve fatigue and increase the comfort of the laboring client. d.In a sitting or squatting position, abdominal muscles of the laboring client will have to work harder.

ANS: C Frequent position changes relieve fatigue, increase comfort, and improve circulation. Blood flow can be compromised in the supine position; any upright position benefits cardiac output. The "all fours" position is used to relieve backache in certain situations. In a sitting or squatting position, the abdominal muscles work in greater harmony with uterine contractions.

16. Postpartum overdistention of the bladder and urinary retention can lead to which complications? a. Postpartum hemorrhage and eclampsia b. Fever and increased blood pressure c. Postpartum hemorrhage and urinary tract infection d. Urinary tract infection and uterine rupture

ANS: C Incomplete emptying and overdistention of the bladder can lead to urinary tract infection. Overdistention of the bladder displaces the uterus and prevents contraction of the uterine muscle, thus leading to postpartum hemorrhage. No correlation exists between bladder distention and high blood pressure or eclampsia. The risk of uterine rupture decreases after the birth of the infant.

How would the nurse differentiate a meconium stool from a transitional stool in the healthy newborn? a.Observed at age 3 days b.Is residue of a milk curd c.Passes in the first 12 hours of life d.Is lighter in color and looser in consistency

ANS: C Meconium stool is usually passed in the first 12 hours of life, and 99% of newborns have their first stool within 48 hours. If meconium is not passed by 48 hours, then obstruction is suspected. Meconium stool is the first stool of the newborn and is made up of matter remaining in the intestines during intrauterine life. Meconium is dark and sticky.

What marks on a baby's skin may indicate an underlying problem that requires notification of a physician? a.Mongolian spots on the back b.Telangiectatic nevi on the nose or nape of the neck c.Petechiae scattered over the infant's body d.Erythema toxicum neonatorum anywhere on the body

ANS: C Petechiae (bruises) scattered over the infant's body should be reported to the pediatrician because they may indicate underlying problems. Mongolian spots are bluish-black spots that resemble bruises but gradually fade over months and have no clinical significance. Telangiectatic nevi (stork bites, angel kisses) fade by the second year and have no clinical significance. Erythema toxicum neonatorum is an appalling-looking rash; however, it has no clinical significance and requires no treatment.

A first-time dad is concerned that his 3-day-old daughter's skin looks "yellow." In the nurse's explanation of physiologic jaundice, what fact should be included? a.Physiologic jaundice occurs during the first 24 hours of life. b.Physiologic jaundice is caused by blood incompatibilities between the mother and the infant blood types. c.Physiologic jaundice becomes visible when serum bilirubin levels peak between the second and fourth days of life. d.Physiologic jaundice is also known as breast milk jaundice.

ANS: C Physiologic jaundice becomes visible when the serum bilirubin reaches a level of 5 mg/dl or higher when the baby is approximately 3 days old. This finding is within normal limits for the newborn. Pathologic jaundice, not physiologic jaundice, occurs during the first 24 hours of life and is caused by blood incompatibilities that result in excessive destruction of erythrocytes; this condition must be investigated. Breast milk jaundice occurs in one third of breastfed infants at 2 weeks and is caused by an insufficient intake of fluids.

Which statement by the client would lead the nurse to believe that labor has been established? a."I passed some thick, pink mucus when I urinated this morning." b."My bag of waters just broke." c."The contractions in my uterus are getting stronger and closer together." d."My baby dropped, and I have to urinate more frequently now."

ANS: C Regular, strong contractions with the presence of cervical change indicate that the woman is experiencing true labor. Although the loss of the mucous plug (operculum) often occurs during the first stage of labor or before the onset of labor, it is not the indicator of true labor. Spontaneous rupture of membranes often occurs during the first stage of labor; however, it is not an indicator of true labor. The presenting part of the fetus typically becomes engaged in the pelvis at the onset of labor but is not the indicator of true labor.

While examining a newborn, the nurse notes uneven skinfolds on the buttocks and a clunk when performing the Ortolani maneuver. These findings are likely indicative of what? a.Polydactyly b.Clubfoot c.Hip dysplasia d.Webbing

ANS: C The Ortolani maneuver is used to detect the presence of hip dysplasia. Polydactyly is the presence of extra digits. Clubfoot (talipes equinovarus) is a deformity in which the foot turns inward and is fixed in a plantar-flexion position. Webbing, or syndactyly, is a fusing of the fingers or toes.

The nurse is assessing a full term, quiet, and alert newborn. What is the average expected apical pulse range (in beats per minute)? a.80 to 100 b.100 to 120 c.120 to 160 d.150 to 180

ANS: C The average infant heart rate while awake is 120 to 160 beats per minute. The newborn's heart rate may be approximately 85 to 100 beats per minute while sleeping and typically a little higher than 100 to 120 beats per minute when alert but quiet. A heart rate of 150 to 180 beats per minute is typical when the infant cries.

Nurses should be cognizant of what regarding the mechanism of labor? a.Seven critical movements must progress in a more or less orderly sequence. b.Asynclitism is sometimes achieved by means of the Leopold's maneuver. c.Effects of the forces determining descent are modified by the shape of the woman's pelvis and the size of the fetal head. d.At birth, the baby is said to achieve "restitution"; that is, a return to the C-shape of the womb.

ANS: C The size of the maternal pelvis and the ability of the fetal head to mold also affect the process. The seven identifiable movements of the mechanism of labor simultaneously occur in combinations, not in precise sequences. Asynclitism is the deflection of the baby's head; the Leopold's maneuver is a means of judging descent by palpating the mother's abdomen. Restitution is the rotation of the baby's head after the infant is born.

7. Under the Newborns and Mothers Health Protection Act, all health plans are required to allow new mothers and newborns to remain in the hospital for a minimum of _____ hours after a normal vaginal birth and for _____ hours after a cesarean birth. What is the correct interpretation of this legislation? a. 24; 72 b. 24; 96 c. 48; 96 d. 48; 120

ANS: C The specified stays are 48 hours (2 days) for a vaginal birth and 96 hours (4 days) for a cesarean birth. The attending provider and the mother together can decide on an earlier discharge. A client may be discharged either 24 hours after a vaginal birth or 72 hours after a cesarean birth if she is stable and her provider is in agreement. A client is unlikely to remain in the hospital for 120 hours after a cesarean birth unless complications have developed.

Which adaptation of the maternal-fetal exchange of oxygen occurs in response to uterine contraction? a.The maternal-fetal exchange of oxygen and waste products continues except when placental functions are reduced. b.This maternal-fetal exchange increases as the blood pressure decreases. c.It diminishes as the spiral arteries are compressed. d.This exchange of oxygen and waste products is not significantly affected by contractions.

ANS: C Uterine contractions during labor tend to decrease circulation through the spiral electrodes and subsequent perfusion through the intervillous space. The maternal blood supply to the placenta gradually stops with contractions. The exchange of oxygen and waste products decreases. The exchange of oxygen and waste products is affected by contractions.

The nurse providing care for a woman with gestational diabetes understands that a laboratory test for glycosylated hemoglobin Alc: a. Is now done for all pregnant women, not just those with or likely to have diabetes. b. Is a snapshot of glucose control at the moment. c. Would be considered evidence of good diabetes control with a result of 5% to 6%. d. Is done on the patients urine, not her blood.

ANS: C A score of 5% to 6% indicates good control. This is an extra test for diabetic women, not one done for all pregnant women. This test defines glycemic control over the previous 4 to 6 weeks. Glycosylated hemoglobin level tests are done on the blood. PTS: 1 DIF: Cognitive Level: Comprehension REF: 273 OBJ: Nursing Process: Evaluation MSC: Client Needs: Health Promotion and Maintenance

Which nursing assessment indicates that a woman who is in second-stage labor is almost ready to give birth? a. The fetal head is felt at 0 station during vaginal examination. b. Bloody mucus discharge increases. c. The vulva bulges and encircles the fetal head. d. The membranes rupture during a contraction.

ANS: C During the active pushing (descent) phase, the woman has strong urges to bear down as the presenting part of the fetus descends and presses on the stretch receptors of the pelvic floor. The vulva stretches and begins to bulge encircling the fetal head. Birth of the head occurs when the station is +4. A 0 station indicates engagement. Bloody show occurs throughout the labor process and is not an indication of an imminent birth. Rupture of membranes can occur at any time during the labor process and does not indicate an imminent birth.

While providing care to a patient in active labor, the nurse should instruct the woman that: a. The supine position commonly used in the United States increases blood flow. b. The "all fours" position, on her hands and knees, is hard on her back. c. Frequent changes in position will help relieve her fatigue and increase her comfort. d. In a sitting or squatting position, her abdominal muscles will have to work harder.

ANS: C Frequent position changes relieve fatigue, increase comfort, and improve circulation. Blood flow can be compromised in the supine position; any upright position benefits cardiac output. The "all fours" position is used to relieve backache in certain situations. In a sitting or squatting position, the abdominal muscles work in greater harmony with uterine contractions.

To manage her diabetes appropriately and ensure a good fetal outcome, the pregnant woman with diabetes will need to alter her diet by: a. Eating six small equal meals per day. b. Reducing carbohydrates in her diet. c. Eating her meals and snacks on a fixed schedule. d. Increasing her consumption of protein.

ANS: C Having a fixed meal schedule will provide the woman and the fetus with a steadier blood sugar level, provide better balance with insulin administration, and help prevent complications. It is more important to have a fixed meal schedule than equal division of food intake. Approximately 45% of the food eaten should be in the form of carbohydrates. PTS: 1 DIF: Cognitive Level: Comprehension REF: 274 OBJ: Nursing Process: Planning MSC: Client Needs: Health Promotion and Maintenance

Nursing intervention for the pregnant diabetic patient is based on the knowledge that the need for insulin: a. Increases throughout pregnancy and the postpartum period. b. Decreases throughout pregnancy and the postpartum period. c. Varies depending on the stage of gestation. d. Should not change because the fetus produces its own insulin.

ANS: C Insulin needs decrease during the first trimester, when nausea, vomiting, and anorexia are a factor. They increase during the second and third trimesters, when the hormones of pregnancy create insulin resistance in maternal cells. Insulin needs increase during the second and third trimesters, when the hormones of pregnancy create insulin resistance in maternal cells. The insulin needs change throughout the different stages of pregnancy. PTS: 1 DIF: Cognitive Level: Comprehension REF: 269 OBJ: Nursing Process: Implementation MSC: Client Needs: Physiologic Integrity

Which heart condition is not a contraindication for pregnancy? a. Peripartum cardiomyopathy b. Eisenmenger syndrome c. Heart transplant d. All of these contraindicate pregnancy.

ANS: C Pregnancy is contraindicated for peripartum cardiomyopathy and Eisenmenger syndrome. Women who have had heart transplants are successfully having babies. However, conception should be postponed for at least 1 year after transplantation. PTS: 1 DIF: Cognitive Level: Comprehension REF: 287 OBJ: Nursing Process: Assessment MSC: Client Needs: Health Promotion and Maintenance

Metabolic changes throughout pregnancy that affect glucose and insulin in the mother and the fetus are complicated but important to understand. Nurses should understand that: a. Insulin crosses the placenta to the fetus only in the first trimester, after which the fetus secretes its own. b. Women with insulin-dependent diabetes are prone to hyperglycemia during the first trimester because they are consuming more sugar. c. During the second and third trimesters, pregnancy exerts a diabetogenic effect that ensures an abundant supply of glucose for the fetus. d. Maternal insulin requirements steadily decline during pregnancy.

ANS: C Pregnant women develop increased insulin resistance during the second and third trimesters. Insulin never crosses the placenta; the fetus starts making its own insulin around the tenth week. As a result of normal metabolic changes during pregnancy, insulin-dependent women are prone to hypoglycemia (low levels). Maternal insulin requirements may double or quadruple by the end of pregnancy. PTS: 1 DIF: Cognitive Level: Comprehension REF: 270 OBJ: Nursing Process: Assessment MSC: Client Needs: Physiologic Integrity

Which factor is known to increase the risk of gestational diabetes mellitus? a. Underweight before pregnancy b. Maternal age younger than 25 years c. Previous birth of large infant d. Previous diagnosis of type 2 diabetes mellitus

ANS: C Previous birth of a large infant suggests gestational diabetes mellitus. Obesity (BMI of 30 or greater) creates a higher risk for gestational diabetes. A woman younger than 25 years generally is not at risk for gestational diabetes mellitus. The person with type 2 diabetes mellitus already has diabetes and will continue to have it after pregnancy. Insulin may be required during pregnancy because oral hypoglycemia drugs are contraindicated during pregnancy. PTS: 1 DIF: Cognitive Level: Comprehension REF: 279 OBJ: Nursing Process: Assessment MSC: Client Needs: Health Promotion and Maintenance

A woman with asthma is experiencing a postpartum hemorrhage. Which drug would not be used to treat her bleeding because it may exacerbate her asthma? a. Pitocin b. Nonsteroidal antiinflammatory drugs (NSAIDs) c. Hemabate d. Fentanyl

ANS: C Prostaglandin derivatives should not be used to treat women with asthma, because they may exacerbate symptoms. Pitocin would be the drug of choice to treat this womans bleeding because it would not exacerbate her asthma. NSAIDs are not used to treat bleeding. Fentanyl is used to treat pain, not bleeding. PTS: 1 DIF: Cognitive Level: Analysis REF: 292 OBJ: Nursing Process: Planning MSC: Client Needs: Physiologic Integrity

The most common neurologic disorder accompanying pregnancy is: a. Eclampsia. b. Bells palsy. c. Epilepsy. d. Multiple sclerosis.

ANS: C The effects of pregnancy on epilepsy are unpredictable. Eclampsia sometimes may be confused with epilepsy, which is the most common neurologic disorder accompanying pregnancy. Bells palsy is a form of facial paralysis. Multiple sclerosis is a patchy demyelinization of the spinal cord that does not affect the normal course of pregnancy or birth. PTS: 1 DIF: Cognitive Level: Knowledge REF: 294 OBJ: Nursing Process: Planning MSC: Client Needs: Physiologic Integrity

With regard to factors that affect how the fetus moves through the birth canal, nurses should be aware that: a. The fetal attitude describes the angle at which the fetus exits the uterus. b. Of the two primary fetal lies, the horizontal lie is that in which the long axis of the fetus is parallel to the long axis of the mother. c. The normal attitude of the fetus is called general flexion. d. The transverse lie is preferred for vaginal birth.

ANS: C The normal attitude of the fetus is general flexion. The fetal attitude is the relation of fetal body parts to one another. The horizontal lie is perpendicular to the mother; in the longitudinal (or vertical) lie the long axes of the fetus and the mother are parallel. Vaginal birth cannot occur if the fetus stays in a transverse lie.

The maternity nurse understands that as the uterus contracts during labor, maternal-fetal exchange of oxygen and waste products: a. Continues except when placental functions are reduced. b. Increases as blood pressure decreases. c. Diminishes as the spiral arteries are compressed. d. Is not significantly affected.

ANS: C Uterine contractions during labor tend to decrease circulation through the spiral electrodes and subsequent perfusion through the intervillous space. The maternal blood supply to the placenta gradually stops with contractions. The exchange of oxygen and waste products decreases. The exchange of oxygen and waste products is affected by contractions.

An African-American woman noticed some bruises on her newborn daughter's buttocks. The client asks the nurse what causes these. How would the nurse best explain this integumentary finding to the client? a.Lanugo b.Vascular nevus c.Nevus flammeus d.Mongolian spot

ANS: D A Mongolian spot is a bluish-black area of pigmentation that may appear over any part of the exterior surface of the infant's body and is more commonly noted on the back and buttocks and most frequently observed on infants whose ethnic origins are Mediterranean, Latin American, Asian, or African. Lanugo is the fine, downy hair observed on a term newborn. A vascular nevus, commonly called a strawberry mark, is a type of capillary hemangioma. A nevus flammeus, commonly called a port wine stain, is most frequently found on the face.

Which basic type of pelvis includes the correct description and percentage of occurrence in women? a.Gynecoid: classic female pelvis; heart shaped; 75% b.Android: resembling the male pelvis; wide oval; 15% c.Anthropoid: resembling the pelvis of the ape; narrow; 10% d.Platypelloid: flattened, wide, and shallow pelvis; 3%

ANS: D A platypelloid pelvis is flattened, wide, and shallow; approximately 3% of women have this shape. The gynecoid pelvis is the classic female shape, slightly ovoid and rounded; approximately 50% of women have this shape. An android or malelike pelvis is heart shaped; approximately 23% of women have this shape. An anthropoid or apelike pelvis is oval and wide; approximately 24% of women have this shape.

12. Parents who have not already done so need to make time for newborn follow-up of the discharge. According to the American Academy of Pediatrics (AAP), when should a breastfeeding infant first need to be seen for a follow-up examination? a. 2 weeks of age b. 7 to 10 days after childbirth c. 4 to 5 days after hospital discharge d. 48 to 72 hours after hospital discharge

ANS: D Breastfeeding infants are routinely seen by the pediatric health care provider clinic within 3 to 5 days after birth or 48 to 72 hours after hospital discharge and again at 2 weeks of age. Formula-feeding infants may be seen for the first time at 2 weeks of age.

Certain changes stimulate chemoreceptors in the aorta and carotid bodies to prepare the fetus for initiating respirations immediately after birth. Which change in fetal physiologic activity is not part of this process? a.Fetal lung fluid is cleared from the air passages during labor and vaginal birth. b.Fetal partial pressure of oxygen (PO2) decreases. c.Fetal partial pressure of carbon dioxide in arterial blood (PaCO2) increases. d.Fetal respiratory movements increase during labor.

ANS: D Fetal respiratory movements actually decrease during labor. Fetal lung fluid is cleared from the air passages during labor and vaginal birth. Fetal PO2 decreases, and fetal PaCO2 increases.

9. Because a full bladder prevents the uterus from contracting normally, nurses intervene to help the woman spontaneously empty her bladder as soon as possible. If all else fails, what tactic might the nurse use? a. Pouring water from a squeeze bottle over the womans perineum b. Placing oil of peppermint in a bedpan under the woman c. Asking the physician to prescribe analgesic agents d. Inserting a sterile catheter

ANS: D Invasive procedures are usually the last to be tried, especially with so many other simple and easy methods available (e.g., water, peppermint vapors, pain pills). Pouring water over the perineum may stimulate voiding. It is easy, noninvasive, and should be tried first. The oil of peppermint releases vapors that may relax the necessary muscles. It, too, is easy, noninvasive, and should be tried early on. If the woman is anticipating pain from voiding, then pain medications may be helpful. Other nonmedical means should be tried first, but medications still come before the insertion of a catheter.

With regard to maternal, fetal, and neonatal health problems, nurses should be aware that: a. Infection has replaced pulmonary embolism as one of the three top causes of maternal death attributable to pregnancy b. The leading cause of death in the neonatal period is disorders related to short gestation and low birth weight c. Factors related to the maternal death rate include age and marital status but not race d. Antepartum fetal deaths can best be prevented by better recognizing and responding to abnormalities of pregnancy and labor

ANS: D Medical teams need to be alert to signs of trouble. Race is a factor. African-American maternal mortality rates are more than three times higher than those for Caucasian women. Infection used to be an important cause of maternal death; it has been replaced by pulmonary embolism. The leading cause of death in the neonatal period is congenital anomalies. Race is a factor. African-American maternal mortality rates are more than three times higher than those for Caucasian women.

A nursing student is helping the nursery nurses with morning vital signs. A baby born 10 hours ago by cesarean section is found to have moist lung sounds. What is the best interpretation of these data? a.The nurse should immediately notify the pediatrician for this emergency situation. b.The neonate must have aspirated surfactant. c.If this baby was born vaginally, then a pneumothorax could be indicated. d.The lungs of a baby delivered by cesarean section may sound moist during the first 24 hours after childbirth.

ANS: D Moist lung sounds will resolve within a few hours. A surfactant acts to keep the expanded alveoli partially open between respirations for this common condition of newborns. In a vaginal birth, absorption of the remaining lung fluid is accelerated by the process of labor and delivery. The remaining lung fluid will move into interstitial spaces and be absorbed by the circulatory and lymphatic systems. Moist lung sounds are particularly common in infants delivered by cesarean section. The surfactant is produced by the lungs; therefore, aspiration is not a concern.

6. The trend in the United States is for women to remain hospitalized no longer than 1 or 2 days after giving birth. Which scenario is not a contributor to this model of care? a. Wellness orientation model of care rather than a sick-care model b. Desire to reduce health care costs c. Consumer demand for fewer medical interventions and more family-focused experiences d. Less need for nursing time as a result of more medical and technologic advances and devices available at home that can provide information

ANS: D Nursing time and care are in demand as much as ever; the nurse simply has to do things more quickly. A wellness orientation model of care seems to focus on getting clients out the door sooner. In most cases, less hospitalization results in lower costs. People believe that the family gives more nurturing care than the institution.

While evaluating the reflexes of a newborn, the nurse notes that with a loud noise the newborn symmetrically abducts and extends his arms, his fingers fan out and form a C with the thumb and forefinger, and he has a slight tremor. The nurse would document this finding as a positive _____ reflex. a.tonic neck b.glabellar (Myerson) c.Babinski d.Moro

ANS: D The characteristics displayed by the infant are associated with a positive Moro reflex. The tonic neck reflex occurs when the infant extends the leg on the side to which the infant's head simultaneously turns. The glabellar (Myerson) reflex is elicited by tapping on the infant's head while the eyes are open. A characteristic response is blinking for the first few taps. The Babinski reflex occurs when the sole of the foot is stroked upward along the lateral aspect of the sole and then across the ball of the foot. A positive response occurs when all the toes hyperextend, with dorsiflexion of the big toe.

Which statement regarding the care of a client in labor is correct and important to the nurse as he or she formulates the plan of care? a.The woman's blood pressure will increase during contractions and fall back to prelabor normal levels between contractions. b.The use of the Valsalva maneuver is encouraged during the second stage of labor to relieve fetal hypoxia. c.Having the woman point her toes will reduce leg cramps. d.Endogenous endorphins released during labor will raise the woman's pain threshold and produce sedation.

ANS: D The endogenous endorphins released during labor will raise the woman's pain threshold and produce sedation. In addition, physiologic anesthesia of the perineal tissues, caused by the pressure of the presenting part, decreases the mother's perception of pain. Blood pressure levels increase during contractions but remain somewhat elevated between them. The use of the Valsalva maneuver is discouraged during the second stage labor because of a number of unhealthy outcomes, including fetal hypoxia. Pointing the toes can cause leg cramps, as can the process of labor itself.

A new mother asks the nurse when the "soft spot" on her son's head will go away. What is the nurse's best response, based upon her understanding of when the anterior frontal closes? a.2 months b.8 months c.12 months d.18 months

ANS: D The larger of the two fontanels, the anterior fontanel, closes by 18 months after birth. The posterior fontanel closes at 6 to 8 weeks. The remaining three options are too early for the anterior fontanel to close.

13. On examining a woman who gave birth 5 hours ago, the nurse finds that the woman has completely saturated a perineal pad within 15 minutes. What is the nurses highest priority at this time? a. Beginning an intravenous (IV) infusion of Ringers lactate solution b. Assessing the womans vital signs c. Calling the womans primary health care provider d. Massaging the womans fundus

ANS: D The nurse should first assess the uterus for atony by massaging the womans fundus. Uterine tone must be established to prevent excessive blood loss. The nurse may begin an IV infusion to restore circulatory volume, but this would not be the first action. Blood pressure is not a reliable indicator of impending shock from impending hemorrhage; assessing vital signs should not be the nurses first action. The physician would be notified after the nurse completes the assessment of the woman.

The brain is vulnerable to nutritional deficiencies and trauma in early infancy. What is the rationale for this physiologic adaptation in the newborn? a.Incompletely developed neuromuscular system b.Primitive reflex system c.Presence of various sleep-wake states d.Cerebellum growth spurt

ANS: D The vulnerability of the brain is likely due to the cerebellum growth spurt. By the end of the first year, the cerebellum ends its growth spurt that began at approximately 30 weeks of gestation. The neuromuscular system is almost completely developed at birth. The reflex system is not relevant to the cerebellum growth spurt. The various sleep-wake states are not relevant to the cerebellum growth spurt.

18. Which nursing action is most appropriate to correct a boggy uterus that is displaced above and to the right of the umbilicus? a. Notify the physician of an impending hemorrhage. b. Assess the blood pressure and pulse. c. Evaluate the lochia. d. Assist the client in emptying her bladder.

ANS: D Urinary retention may cause overdistention of the urinary bladder, which lifts and displaces the uterus. Nursing actions need to be implemented before notifying the physician. Evaluating blood pressure, pulse, and lochia is important if the bleeding continues; however, the focus at this point is to assist the client in emptying her bladder.

A pregnant woman is at 38 weeks of gestation. She wants to know whether there are any signs that "labor is getting close to starting." Which finding is an indication that labor may begin soon? a.Weight gain of 1.5 to 2 kg (3 to 4 lb) b.Increase in fundal height c.Urinary retention d.Surge of energy

ANS: D Women speak of having a burst of energy before labor. The woman may lose 0.5 to 1.5 kg, as a result of water loss caused by electrolyte shifts that, in turn, are caused by changes in the estrogen and progesterone levels. When the fetus descends into the true pelvis (called lightening), the fundal height may decrease. Urinary frequency may return before labor.

A woman with gestational diabetes has had little or no experience reading and interpreting glucose levels. She shows the nurse her readings for the past few days. Which one should the nurse tell her indicates a need for adjustment (insulin or sugar)? a. 75 mg/dL before lunch. This is low; better eat now. b. 115 mg/dL 1 hour after lunch. This is a little high; maybe eat a little less next time. c. 115 mg/dL 2 hours after lunch; This is too high; it is time for insulin. d. 60 mg/dL just after waking up from a nap. This is too low; maybe eat a snack before going to sleep.

ANS: D 60 mg/dL after waking from a nap is too low. During hours of sleep glucose levels should not be less than 70 mg/dL. Snacks before sleeping can be helpful. The premeal acceptable range is 65 to 95 mg/dL. The readings 1 hour after a meal should be less than 140 mg/dL. Two hours after eating, the readings should be less than 120 mg/dL. PTS: 1 DIF: Cognitive Level: Application REF: 277 OBJ: Nursing Process: Evaluation MSC: Client Needs: Health Promotion and Maintenance

Which basic type of pelvis includes the correct description and percentage of occurrence in women? a. Gynecoid: classic female; heart shaped; 75% b. Android: resembling the male; wider oval; 15% c. Anthropoid: resembling the ape; narrower; 10% d. Platypelloid: flattened, wide, shallow; 3%

ANS: D A platypelloid pelvis is flattened, wide, and shallow; about 3% of women have this shape. The gynecoid shape is the classical female shape, slightly ovoid and rounded; about 50% of women have this shape. An android, or malelike, pelvis is heart shaped; about 23% of women have this shape. An anthropoid, or apelike, pelvis is oval and wider; about 24% of women have this shape.

To promote bonding and attachment immediately after birth, which action should the nurse take? a. Assist the mother in feeding her baby. b. Allow the mother quiet time with her infant. c. Teach the mother about the concepts of bonding and attachment. d. Assist the mother in assuming an en face position with her newborn.

ANS: D Assisting the mother in assuming an en face position with her newborn will support the bonding process. After birth is a good time to initiate breastfeeding, but first the mother needs time to explore the new infant and begin the bonding process. The mother should be given as much privacy as possible; however, nursing assessments must still be continued during this critical time. The mother has just delivered and is more focused on the infant; she will not be receptive to teaching at this time.

Since the gene for cystic fibrosis was identified in 1989, data can be collected for the purposes of genetic counseling for couples regarding carrier status. According to statistics, how often does cystic fibrosis occur in Caucasian live births? a. 1 in 100 b. 1 in 1200 c. 1 in 2500 d. 1 in 3000

ANS: D Cystic fibrosis occurs in about 1 in 3000 Caucasian live births. PTS: 1 DIF: Cognitive Level: Comprehension REF: 293 OBJ: Nursing Process: Assessment MSC: Client Needs: Health Promotion and Maintenance

In order to care for obstetric patients adequately, the nurse understands that labor contractions facilitate cervical dilation by: a. Contracting the lower uterine segment. b. Enlarging the internal size of the uterus. c. Promoting blood flow to the cervix. d. Pulling the cervix over the fetus and amniotic sac.

ANS: D Effective uterine contractions pull the cervix upward at the same time that the fetus and amniotic sac are pushed downward. The contractions are stronger at the fundus. The internal size becomes smaller with the contractions; this helps to push the fetus down. Blood flow decreases to the uterus during a contraction.

What form of heart disease in women of childbearing years usually has a benign effect on pregnancy? a. Cardiomyopathy b. Rheumatic heart disease c. Congenital heart disease d. Mitral valve prolapse

ANS: D Mitral valve prolapse is a benign condition that is usually asymptomatic. Cardiomyopathy produces congestive heart failure during pregnancy. Rheumatic heart disease can lead to heart failure during pregnancy. Some congenital heart diseases produce pulmonary hypertension or endocarditis during pregnancy. PTS: 1 DIF: Cognitive Level: Knowledge REF: 284 OBJ: Nursing Process: Assessment MSC: Client Needs: Physiologic Integrity

Glucose metabolism is profoundly affected during pregnancy because: a. Pancreatic function in the islets of Langerhans is affected by pregnancy. b. The pregnant woman uses glucose at a more rapid rate than the nonpregnant woman. c. The pregnant woman increases her dietary intake significantly. d. Placental hormones are antagonistic to insulin, thus resulting in insulin resistance.

ANS: D Placental hormones, estrogen, progesterone, and human placental lactogen (HPL) create insulin resistance. Insulin also is broken down more quickly by the enzyme placental insulinase. Pancreatic functioning is not affected by pregnancy. The glucose requirements differ because of the growing fetus. The pregnant woman should increase her intake by 200 calories a day. PTS: 1 DIF: Cognitive Level: Comprehension REF: 279 OBJ: Nursing Process: Assessment MSC: Client Needs: Physiologic Integrity

A postpartum patient calls the clinic and reports to the nurse feelings of fatigue, tearfulness, and anxiety. What is the nurse's best response? a. "When did these symptoms begin?" b. "Sounds like normal postpartum depression." c. "Are you having trouble getting enough sleep?" d. "Are you able to get out of bed and provide care for your baby?"

ANS: D Postpartum blues must be distinguished from postpartum depression and postpartum psychosis, which are disabling conditions and require therapeutic management for full recovery. Nurses need to assess the depression to ascertain if she is unable to cope with daily life. Postpartum blues are self-limiting and frequently occur by the fifth postpartum day and resolve in 2 weeks. The response "Sounds like postpartum depression" does not offer the patient any help or encouragement through this challenging time. Asking if she is getting enough sleep does not add to the assessments already identified in the stem. Enough information exists to determine that she has the signs and symptoms of postpartum blues. The nurse must differentiate between postpartum blues and depression.

During a physical assessment of an at-risk client, the nurse notes generalized edema, crackles at the base of the lungs, and some pulse irregularity. These are most likely signs of: a. Euglycemia. b. Rheumatic fever. c. Pneumonia. d. Cardiac decompensation.

ANS: D Symptoms of cardiac decompensation may appear abruptly or gradually. Euglycemia is a condition of normal glucose levels. These symptoms indicate cardiac decompensation. Rheumatic fever can cause heart problems, but it does not manifest with these symptoms, which indicate cardiac decompensation. Pneumonia is an inflammation of the lungs and would not likely generate these symptoms, which indicate cardiac decompensation. PTS: 1 DIF: Cognitive Level: Analysis REF: 288 OBJ: Nursing Process: Assessment MSC: Client Needs: Physiologic Integrity

In order to evaluate the condition of the patient accurately during labor, the nurse should be aware that: a. The woman's blood pressure will increase during contractions and fall back to prelabor normal between contractions. b. Use of the Valsalva maneuver is encouraged during the second stage of labor to relieve fetal hypoxia. c. Having the woman point her toes will reduce leg cramps. d. The endogenous endorphins released during labor will raise the woman's pain threshold and produce sedation.

ANS: D The endogenous endorphins released during labor will raise the woman's pain threshold and produce sedation. In addition, physiologic anesthesia of the perineal tissues, caused by the pressure of the presenting part, decreases the mother's perception of pain. Blood pressure increases during contractions but remains somewhat elevated between them. Use of the Valsalva maneuver is discouraged during second stage labor because of a number of unhealthy outcomes, including fetal hypoxia. Pointing the toes can cause leg cramps, as can the process of labor itself.

The factors that affect the process of labor and birth, known commonly as the five Ps, include all except: a. Passenger. b. Passageway. c. Powers. d. Pressure.

ANS: D The five Ps are passenger (fetus and placenta), passageway (birth canal), powers (contractions), position of the mother, and psychologic response.

A primigravida at 39 weeks of gestation is observed for 2 hours in the intrapartum unit. The fetal heart rate has been normal. Contractions are 5 to 9 minutes apart, 20 to 30 seconds in duration, and of mild intensity. Cervical dilation is 1 to 2 cm and uneffaced (unchanged from admission). Membranes are intact. The nurse should expect the woman to be: a. Admitted and prepared for a cesarean birth. b. Admitted for extended observation. c. Discharged home with a sedative. d. Discharged home to await the onset of true labor.

ANS: D This situation describes a woman with normal assessments who is probably in false labor and will probably not deliver rapidly once true labor begins. These are all indications of false labor without fetal distress. There is no indication that further assessment or cesarean birth is indicated. The patient will likely be discharged; however, there is no indication that a sedative is needed.

In planning for the care of a 30-year-old woman with pregestational diabetes, the nurse recognizes that the most important factor affecting pregnancy outcome is the: a. Mothers age. b. Number of years since diabetes was diagnosed. c. Amount of insulin required prenatally. d. Degree of glycemic control during pregnancy.

ANS: D Women with excellent glucose control and no blood vessel disease should have good pregnancy outcomes. PTS: 1 DIF: Cognitive Level: Comprehension REF: 273 OBJ: Nursing Process: Planning MSC: Client Needs: Health Promotion and Maintenance

A (Little if any change)

As part of the postpartum assessment, the nurse examines the breasts of a primiparous breastfeeding woman who is 1 day postpartum. An expected finding is: a) Little if any change b) Leakage of milk at let-down c) Swollen, warm and tender on palpation d) A few blisters and a bruise on each areola

The goal of treatment of the infant with phenylketonuria (PKU) is to: a. Cure mental retardation. b. Prevent central nervous system (CNS) damage, which leads to mental retardation. c. Prevent gastrointestinal symptoms. d. Cure the urinary tract infection.

B (CNS damage can occur as a result of toxic levels of phenylalanine. No known cure exists for mental retardation. Digestive problems are a clinical manifestation of PKU. PKU does not involve any urinary problems.)

What action does the nurse take to relieve choking in a pregnant patient who is in the third trimester? A. Administering anesthesia B. Administering chest thrusts C. Placing a towel under the hips D. Positioning the patient onto one side

B (Choking is often relieved in patients by administering abdominal thrusts. However if the patient is in the third trimester of pregnancy, chest thrusts are administered to prevent injury to the uterus. Administering anesthesia or positioning the patient onto one side will not help dislodge the object and relieve choking. The nurse needs to place a towel under the hips to displace the uterus while administering cardiopulmonary resuscitation (CPR).)

The nurse observes that a pregnant patient with gestational hypertension who is on magnesium sulfate therapy is prescribed nifedipine (Adalat). What action does the nurse take? A. Evaluates the patient's renal function test B. Obtains a prescription for a change of drug C. Reduces the nifedipine (Adalat) dose by 50% D. Administers both medications simultaneously

B (Concurrent use of nifedipine (Adalat) and magnesium sulfate can result in skeletal muscle blockade in the patient. Therefore the nurse needs to report immediately to the primary health care provider (PHP) and obtain a prescription for a change of drug. The nurse assesses the patient's renal function to determine the risk for toxicity after administering any drug. However, it is not a priority in this case. Reducing the nifedipine (Adalat) dose is not likely to prevent the drug interaction in the patient. The nurse does not administer both drugs simultaneously because it may be harmful for the patient.)

To care adequately for infants at risk for neonatal bacterial infection, nurses should be aware that: a. Congenital infection progresses more slowly than does nosocomial infection. b. Nosocomial infection can be prevented by effective handwashing; early-onset infections cannot. c. Infections occur with about the same frequency in boy and girl infants, although female mortality is higher. d. The clinical sign of a rapid, high fever makes infection easier to diagnose.

B (Handwashing is an effective preventive measure for late-onset (nosocomial) infections because these infections come from the environment around the infant. Early-onset, or congenital, infections are caused by the normal flora at the maternal vaginal tract and progress more rapidly than do nosocomial (late-onset) infections. Infection occurs about twice as often in boys and results in higher mortality. Clinical signs of neonatal infection are nonspecific and are similar to those of noninfectious problems, thus making diagnosis difficult.)

Which is an important nursing intervention when a patient has an incomplete miscarriage with heavy bleeding? A. Initiate expectant management at once. B. Prepare the patient for dilation and curettage. C. Administer the prescribed oxytocin (Pitocin). D. Obtain a prescription for ergonovine (Methergine).

B (In the case of an incomplete miscarriage, sometimes there is heavy bleeding and excessive cramping and some part of fetal tissue remains in the uterus. Therefore the nurse needs to prepare the patient for dilation and curettage for the removal of the fetal tissue. Expectant management is initiated if the pregnancy continues after a threatened miscarriage. Oxytocin (Pitocin) is administered to prevent hemorrhage after evacuation of the uterus. Ergonovine (Methergine) is administered to contract the uterus.)

A woman with severe preeclampsia is being treated with an intravenous infusion of magnesium sulfate. This treatment is considered successful if: A. blood pressure is reduced to prepregnant baseline. B. seizures do not occur. C. deep tendon reflexes become hypotonic. D. diuresis reduces fluid retention

B (Magnesium sulfate is a central nervous system (CNS) depressant given primarily to prevent seizures . A temporary decrease in blood pressure can occur; however, this is not the purpose of administering this medication. Hypotonia is a sign of an excessive serum level of magnesium. It is critical that calcium gluconate be on hand to counteract the depressant effects of magnesium toxicity. Diuresis is not an expected outcome of magnesium sulfate administration.)

To provide optimal care of infants born to mothers who are substance abusers, nurses should be aware that: a. Infants born to addicted mothers are also addicted. b. Mothers who abuse one substance likely will use or abuse another, thus compounding the infant's difficulties. c. The NICU Network Neurobehavioral Scale (NNNS) is designed to assess the damage the mother has done to herself. d. No laboratory procedures are available that can identify the intrauterine drug exposure of the infant.

B (Multiple substance use (even just alcohol and tobacco) makes it difficult to assess the problems of the exposed infant, particularly with regard to withdrawal manifestations. Infants of substance-abusing mothers may have some of the physiologic signs but are not addicted in the behavioral sense. "Drug-exposed newborn" is a more accurate description than "addict." The NNNS is designed to assess the neurologic, behavioral, and stress/abstinence function of the neonate. Newborn urine, hair, or meconium sampling may be used to identify an infant's intrauterine drug exposure.)

The most prevalent clinical manifestation of abruptio placentae (as opposed to placenta previa) is: A. bleeding. B. intense abdominal pain. C. uterine activity. D. cramping

B (Pain is absent with placenta previa but may be agonizing with abruptio placentae. Bleeding may be present in varying degrees for both placental conditions. Uterine activity may be present with both placental conditions. Cramping is a form of uterine activity that may be present in both placental conditions.)

Which intervention does the nurse implement for a patient immediately after a severe abdominal trauma? A. Prep the patient for cesarean birth. B. Send the patient for pelvic computed tomography (CT) scanning. C. Provide fluids to the patient as part of the protocol for ultrasound examination. D. Prepare to administer Rho(D) immunoglobulin.

B (Pelvic CT scanning helps visualize extraperitoneal and retroperitoneal structures and the genitourinary tract. The nurse needs to prepare the patient for cesarean birth if there is no evidence of a maternal pulse. Ultrasound examination is not as effective as electronic fetal monitoring for determining placental abruption in the patient after the trauma. Therefore the nurse prepares the patient for a CT scan after a severe abdominal trauma. The nurse needs to administer Rho(D) immunoglobulin in an Rh-negative pregnant trauma patient. This helps protect the patient from isoimmunization.)

Necrotizing enterocolitis (NEC) is an inflammatory disease of the gastrointestinal mucosa. The signs of NEC are nonspecific. Some generalized signs include: a. Hypertonia, tachycardia, and metabolic alkalosis. b. Abdominal distention, temperature instability, and grossly bloody stools. c. Hypertension, absence of apnea, and ruddy skin color. d. Scaphoid abdomen, no residual with feedings, and increased urinary output.

B (Some generalized signs of NEC include decreased activity, hypotonia, pallor, recurrent apnea and bradycardia, decreased oxygen saturation values, respiratory distress, metabolic acidosis, oliguria, hypotension, decreased perfusion, temperature instability, cyanosis, abdominal distention, residual gastric aspirates, vomiting, grossly bloody stools, abdominal tenderness, and erythema of the abdominal wall. The infant may display hypotonia, bradycardia, and metabolic acidosis.)

A new mother recalls from prenatal class that she should try to feed her newborn daughter when she exhibits feeding readiness cues rather than waiting until her infant is crying frantically. On the basis of this information, this woman should feed her infant about every 2.5 to 3 hours when she: a. Waves her arms in the air. b. Makes sucking motions. c. Has hiccups. d. Stretches her legs out straight.

B (Sucking motions, rooting, mouthing, and hand-to-mouth motions are examples of feeding-readiness cues. Waving the arms in the air, hiccupping, and stretching the legs out straight are not typical feeding-readiness cues.)

A woman has determined that bottle-feeding is the best feeding method for her. Instructions the woman should receive regarding this feeding method should include which of the following? a. Provide the infant with supplemental vitamins along with the iron-fortified formula. b. Sterilize water by boiling, then cool and mix with formula powder or concentrate. c. Expect a 2-week-old newborn to drink approximately 30 to 60 mL of formula at each feeding. d. Microwave refrigerated formula for about 2 minutes before feeding the newborn.

B (Supplements are not required when using prepared formulas; a 2-week-old infant should consume approximately 90 to 150 mL of formula at each feeding; formula should never be heated in the microwave because it could be overheated or unevenly heated.)

A pregnant patient is at risk for cardiac arrest as a result of profound hypovolemia after a trauma. Which action does the nurse take? The nurse: A. Assesses airway, breathing, and pulse rate. B. Administers warmed crystalloid solutions. C. Administers calcium gluconate intravenously. D. Obtains a prescription for magnesium sulfate.

B (The nurse administers warmed crystalloid solutions for massive fluid resuscitation in the patient who has profound hypovolemia after a trauma. The nurse needs to assess the airway, breathing, and pulse in a patient after a convulsion so that prompt actions can be taken to stabilize the patient. The nurse administers calcium gluconate as an antidote to a patient who has magnesium toxicity. The nurse may administer magnesium sulfate for the treatment of eclamptic seizures in a patient with preeclampsia.)

A newborn was admitted to the neonatal intensive care unit after being delivered at 29 weeks of gestation to a 28-year-old multiparous, married, Caucasian woman whose pregnancy was uncomplicated until premature rupture of membranes and preterm birth. The newborn's parents arrive for their first visit after the birth. The parents walk toward the bedside but remain approximately 5 feet away from the bed. The nurse's most appropriate action would be to: a. Wait quietly at the newborn's bedside until the parents come closer. b. Go to the parents, introduce himself or herself, and gently encourage the parents to come meet their infant; explain the equipment first, and then focus on the newborn. c. Leave the parents at the bedside while they are visiting so they can have some privacy. d. Tell the parents only about the newborn's physical condition, and caution them to avoid touching their baby.

B (The nurse is instrumental in the initial interactions with the infant. The nurse can help the parents "see" the infant, rather than focus on the equipment. The importance and purpose of the apparatus that surrounds their infant also should be explained to them. Parents often need encouragement and recognition from the nurse to acknowledge the reality of the infant's condition. Parents need to see and touch their infant as soon as possible to acknowledge the reality of the birth and the infant's appearance and condition. Encouragement from the nurse is instrumental in this process. Telling the parents only about the newborn's physical condition and cautioning them to avoid touching their baby is an inappropriate action.)

During the complete physical examination 24 hours after birth: a. The parents are excused to reduce their normal anxiety. b. The nurse can gauge the neonate's maturity level by assessing the infant's general appearance. c. Once often neglected, blood pressure is now routinely checked. d. When the nurse listens to the heart, the S1 and S2 sounds can be heard; the first sound is somewhat higher in pitch and sharper than the second.

B (The nurse will be looking at skin color, alertness, cry, head size, and other features. The parents' presence actively involves them in child care and gives the nurse a chance to observe interactions. Blood pressure is not usually taken unless cardiac problems are suspected. The second sound is higher and sharper than the first.)

The process whereby parents awaken the infant to feed every 3 hours during the day and at least every 4 hours at night is: a. Known as demand feeding. b. Necessary during the first 24 to 48 hours after birth. c. Used to set up the supply-meets-demand system. d. A way to control cluster feeding.

B (The parents do this to make sure that the infant has at least eight feedings in 24 hours. Demand feeding is when the infant determines the frequency of feedings; this is appropriate once the infant is feeding well and gaining weight. The supply-meets-demand system is a milk production system that occurs naturally. Cluster feeding is not a problem if the baby has eight feedings in 24 hours.)

The new parents express concern that their 4-year-old son is jealous of the new baby. They are planning on going home tomorrow and are not sure how the preschooler will react when they bring the baby home. Which of the following suggestions by the nurse will be most helpful? a. Be aware that the child may regress to an earlier stage. b. Have the mother go into the house alone and spend time with the child before the father brings the baby in. c. Have the child stay with a grandparent until the parents adjust to the new baby. d. Tell the child that he is a "big boy" now and doesn't need his crib so the new baby will be using it for a while.

B The child needs to have the mother's love reaffirmed. By giving the child some private time with the mother, he will get the extra attention and reassurance he needs at this point.

A primiparous woman is in the taking-in stage of psychosocial recovery and adjustment following birth. The nurse, recognizing women's needs during this stage, should: a. Foster an active role in the baby's care. b. Provide time for the mother to reflect on the events of the childbirth. c. Recognize the woman's limited attention span by giving her written materials to read when she gets home rather than doing a teaching session now. d. Promote maternal independence by encouraging her to meet her own hygiene and comfort needs.

B The focus of the taking-in stage is nurturing the new mother by meeting her dependency needs for rest, comfort, hygiene, and nutrition. Once they are met, she is more able to take an active role, not only in her own care but also the care of the newborn. Women express a need to review their childbirth experience and evaluate their performance. Short teaching sessions and using written materials to reinforce the content presented are a more effective approach.

The nurse is monitoring a postpartum patient for signs of hemorrhage. Which observation would indicate excessive blood loss? A. A body temperature of 100.4º F B. An increase in pulse from 88 to 102 beats/min C. An increase in respiratory rate from 18 to 22 breaths/min D. A blood pressure change from 130/88 to 120/80 mm Hg

B (During the postpartum period, maternal blood pressure, pulse, and respiration should be checked every 15 minutes during the first hour. A rising pulse is an early sign of excessive blood loss because the heart pumps faster to increase the supply of blood. A body temperature of 100.4º F is a normal finding. A respiratory rate of 22 breaths/min indicates that the patient has no internal bleeding. A blood pressure of 120/80 mm Hg does not indicate that the patient has hemorrhage.)

Late in pregnancy, the woman's breasts should be assessed by the nurse to identify any potential concerns related to breastfeeding. Some nipple conditions make it necessary to provide intervention before birth. These include (Select all that apply): a. Everted nipples b. Flat nipples c. Inverted nipples d. Nipples that contract when compressed e. Cracked nipples

B, C, D (Flat nipples appear soft, like the areola, and do not stand erect unless stimulated by rolling them between the fingers. Inverted nipples are retracted into the breast tissue. These nipples appear normal; however, they will draw inward when the areola is compressed by the infant's mouth. Dome-shaped devices known as breast shells can be worn during the last weeks of pregnancy and between feedings after birth. The shells are placed inside the bra with the opening over the nipple. The shells exert slight pressure against the areola to help the nipples protrude. The helpfulness of breast shells is debated. A breast pump can be used to draw the nipples out before feedings after delivery. Everted nipples protrude and are normal. No intervention will be required. Cracked, blistered, and bleeding nipples occur after breastfeeding has been initiated and are the result of improper latch. The infant should be repositioned during feeding. Application of colostrum and breast milk after feedings will aid in healing.)

What are the possible causes of miscarriage during early pregnancy? Select all that apply. A. Premature dilation of cervix B. Chromosomal abnormalities C. Endocrine imbalance D. Hypothyroidism E. Antiphospholipid antibodies

B, C, D, E (Chromosomal abnormalities account for 50% of all early pregnancy losses. Endocrine imbalance is caused by luteal phase defects, hypothyroidism, and diabetes mellitus in pregnant patients and results in miscarriage. Antiphospholipid antibodies also increase the chances of miscarriage in pregnant patients. Premature dilation of the cervix may cause a second-trimester loss and is usually seen in patients between 12 and 20 weeks' gestation.)

What does the nurse include in the plan of care of a pregnant patient with mild preeclampsia? Select all that apply. A. Ensure prolonged bed rest. B. Provide diversionary activities. C. Encourage the intake of more fluids. D. Restrict sodium and zinc in the diet. E. Refer to Internet-based support group

B, C, E (Activity is restricted in patients with preeclampsia, so it is necessary to provide diversionary activities to such patients to prevent boredom. The nurse encourages the patient to increase fluid intake to enhance renal perfusion and bowel function. The nurse can suggest Internet-based support groups to reduce boredom and stress in the patient. Patients need to restrict activity, but complete bed rest is not advised because it may cause cardiovascular deconditioning, muscle atrophy, and psychological stress. The patient needs to include adequate zinc and sodium in the diet for proper fetal development.)

While discussing the societal impacts of breastfeeding, the nurse should be cognizant of the benefits and educate the patient accordingly. Which statement as part of this discussion would be incorrect? a. Breastfeeding requires fewer supplies and less cumbersome equipment. b. Breastfeeding saves families money. c. Breastfeeding costs employers in terms of time lost from work. d. Breastfeeding benefits the environment.

C (Actually less time is lost to work by breastfeeding mothers, in part because infants are healthier. Breastfeeding is convenient because it does not require cleaning or transporting bottles and other equipment. It saves families money because the cost of formula far exceeds the cost of extra food for the lactating mother. Breastfeeding uses a renewable resource; it does not need fossil fuels, advertising, shipping, or disposal.)

An infant boy was born just a few minutes ago. The nurse is conducting the initial assessment. Part of the assessment includes the Apgar score. The Apgar assessment is performed: a. Only if the newborn is in obvious distress. b. Once by the obstetrician, just after the birth. c. At least twice, 1 minute and 5 minutes after birth. d. Every 15 minutes during the newborn's first hour after birth.

C (Apgar scoring is performed at 1 minute and 5 minutes after birth. Scoring may continue at 5-minute intervals if the infant is in distress and requires resuscitation efforts.)

As the nurse assists a new mother with breastfeeding, the client asks, "If formula is prepared to meet the nutritional needs of the newborn, what is in breast milk that makes it better?" The nurse's best response is that it contains: a. More calories. b. Essential amino acids. c. Important immunoglobulins. d. More calcium.

C (Breast milk contains immunoglobulins that protect the newborn against infection. The calorie count of formula and breast milk is about the same. All the essential amino acids are in both formula and breast milk; however, the concentrations may differ. Calcium levels are higher in formula than in breast milk. This higher level can cause an excessively high renal solute load if the formula is not diluted properly.)

The nurse observes that maternal hypotension has decreased uterine and fetal perfusion in a pregnant patient. What does the nurse need to assess further to understand the maternal status? A. D-dimer blood test B. Kleihauer-Betke (KB) test C. Electronic fetal monitoring D. Electrocardiogram reading

C (Electronic fetal monitoring reflects fetal cardiac responses to hypoxia and hypoperfusion and helps to assess maternal status after a trauma. The D-dimer blood test is used to rule out the presence of a thrombus. The KB test is used to evaluate transplacental hemorrhage. Electrocardiogram reading is more useful to assess the cardiac functions in nonpregnant cardiac patients.)

An infant is to receive gastrostomy feedings. What intervention should the nurse institute to prevent bloating, gastrointestinal reflux into the esophagus, vomiting, and respiratory compromise? a. Rapid bolusing of the entire amount in 15 minutes b. Warm cloths to the abdomen for the first 10 minutes c. Slow, small, warm bolus feedings over 30 minutes d. Cold, medium bolus feedings over 20 minutes

C (Feedings by gravity are done slowly over 20- to 30-minute periods to prevent adverse reactions. Rapid bolusing of the entire amount in 15 minutes would most likely lead to the adverse reactions listed. Temperature stability in the newborn is critical. Warm cloths to the abdomen for the first 10 minutes would not be appropriate because it is not a thermoregulated environment. Additionally, abdominal warming is not indicated with feedings of any kind. Small feedings at room temperature are recommended to prevent adverse reactions.)

How many kilocalories per kilogram (kcal/kg) of body weight does a breastfed term infant require each day? a. 50 to 65 b. 75 to 90 c. 95 to 110 d. 150 to 200

C (For the first 3 months the infant needs 110 kcal/kg/day. At ages 3 to 6 months the requirement is 100 kcal/kg/day. This level decreases slightly to 95 kcal/kg/day from 6 to 9 months and increases again to 100 kcal/kg/day until the baby reaches 12 months.)

At 37 weeks of gestation, the patient is in a severe automobile crash where her abdomen was hit by the steering wheel and her seat belt. What actions would the emergency room nurse expect to perform upon the patient's arrival at the hospital? A. Stay with the patient, assure a patent airway is present, and keep the patient as calm as possible. B. Move the patient's skirt to determine if any vaginal bleeding is present, find out who to call, and monitor the level of consciousness. C. Assess the patient's vital signs, determine location and severity of pain, and establish continual fetal heart rate monitoring. D. Obtain arterial blood gases, obtain a hemoglobin and hematocrit, and oxygen saturation rate.

C (Full assessment of the patient and her fetus are essential and include vital signs, continual fetal heart rate monitoring, determining the location and severity of pain, whether any vaginal bleeding is dark red or bright red, and the status of the abdomen, which would be expected to be rigid or "board like." Staying with the patient, assuring a patent airway is present, and keeping the patient as calm as possible would be appropriate at the crash site before the arrival of emergency medical services (EMS). The current status of the patient and fetus are thepriority. The health care provider would prescribe the arterial blood gases and other laboratory work after the patient is assessed and stabilized.)

As part of Standard Precautions, nurses wear gloves when handling the newborn. The chief reason is: a. To protect the baby from infection. b. That it is part of the Apgar protocol. c. To protect the nurse from contamination by the newborn. d. the nurse has primary responsibility for the baby during the first 2 hours.

C (Gloves are worn to protect the nurse from infection until the blood and amniotic fluid are cleaned off the newborn.)

A new breastfeeding mother asks the nurse how to prevent nipple soreness. The nurse tells this woman that the key to preventing sore nipples would be which of the following? a. Limiting the length of breastfeeding to no more than 10 minutes on each breast until the milk comes in b. Applying lanolin to each nipple and areola after each feeding c. Using correct breastfeeding technique d. Using nipple shells to protect the nipples and areola between feeding

C (Limiting length of feeding does not protect the nipples and areola; b and d are correct actions but not the most important.)

A new mother wants to be sure that she is meeting her daughter's needs while feeding her commercially prepared infant formula. The nurse should evaluate the mother's knowledge about appropriate infant care. The mother meets her child's needs when she: a. Adds rice cereal to her formula at 2 weeks of age to ensure adequate nutrition. b. Warms the bottles using a microwave oven. c. Burps her infant during and after the feeding as needed. d. Refrigerates any leftover formula for the next feeding.

C (Most infants swallow air when fed from a bottle and should be given a chance to burp several times during a feeding and after the feeding. Solid food should not be introduced to the infant for at least 4 to 6 months after birth. A microwave should never be used to warm any food to be given to an infant. The heat is not distributed evenly, and this may pose a risk of burning the infant. Any formula left in the bottle after the feeding should be discarded because the infant's saliva has mixed with it.)

To initiate the milk ejection reflex (MER), the mother should be advised to: a. Wear a firm-fitting bra. b. Drink plenty of fluids. c. Place the infant to the breast. d. Apply cool packs to her breast.

C (Oxytocin, which causes the MER reflex, increases in response to nipple stimulation. A firm bra is important to support the breast; however, will not initiate the MER reflex. Drinking plenty of fluids is necessary for adequate milk production, but this alone will not initiate the MER reflex. Cool packs to the breast will decrease the MER reflex.)

A breastfeeding woman develops engorged breasts at 3 days' postpartum. What action would help this woman achieve her goal of reducing the engorgement? The woman: a. Skips feedings to let her sore breasts rest. b. Avoids using a breast pump. c. Breastfeeds her infant every 2 hours. d. Reduces her fluid intake for 24 hours.

C (The mother should be instructed to attempt feeding her infant every 2 hours while massaging the breasts as the infant is feeding. Skipping feedings may cause further swelling and discomfort. If the infant does not feed adequately and empty the breast, the mother may pump to extract the milk and relieve some of the discomfort. Dehydration further irritates swollen breast tissue.)

What is an advantage of external electronic fetal monitoring? A. The ultrasound transducer can accurately measure short-term variability and beat-to-beat changes in the FHR. B. The tocotransducer can measure and record the frequency, regularity, intensity, and approximate duration of UCs. C. The tocotransducer is especially valuable for measuring UA during the first stage of labor. D. Once correctly applied by the nurse, the transducer need not be repositioned even when the woman changes positions.

C A. Incorrect: These short-term changes cannot be measured with this technology. B. Incorrect: The tocotransducer cannot measure and record the intensity of UCs. C. Correct: This is especially true when the membranes are intact. D. Incorrect: The transducer must be repositioned when the woman or the fetus changes position. p. 500

The home care nurse is visiting a new mother who delivered 1 week ago. The mother complains about not being able to sleep and that she is tired and cries easily. The best response by the nurse would be: a. "Having a baby is difficult; it will be a long time before you get a good night's sleep." b. "Maybe your mother can come in and help you out." c. "It is normal for this to happen and should go away in 2 weeks. It must be very difficult for you to feel this way with a new baby." d. "The hospital nurses must not have taught you enough information about the changes you will experience during these first 6 weeks."

C Postpartum blues begins in the first week and usually last no longer than 2 weeks. The mother needs to be supported during this time and given accurate information about the process. Responses A and B belittle the mother and may make her feel inadequate. Response D places blame on someone else and does not deal with the problem.

33. During pregnancy, many changes occur as a direct result of the presence of the fetus. Which of these adaptations meet this criteria? a. Leukorrhea b. Development of the operculum c. Quickening d. Ballottement e. Lightening

C, D, E

A (CST has several contraindications. NST has a high rate of false-positive results, is less sensitive than the CST, and is relatively inexpensive.)

Compared with contraction stress test (CST), nonstress test (NST) for antepartum fetal assessment: A. Has no known contraindications. B. Has fewer false-positive results. C. Is more sensitive in detecting fetal compromise. D. Is slightly more expensive.

On day 3 of life, a newborn continues to require 100% oxygen by nasal cannula. The parents ask whether they can hold their infant during his next gavage feeding. Given that this newborn is physiologically stable, what response would the nurse give? a. "Parents are not allowed to hold infants who depend on oxygen." b. "You may hold only your baby's hand during the feeding." c. "Feedings cause more physiologic stress, so the baby must be closely monitored. Therefore, I don't think you should hold the baby." d. "You may hold your baby during the feeding."

D ("You may hold your baby during the feeding" is an accurate statement. Parental interaction via holding is encouraged during gavage feedings so that the infant will associate the feeding with positive interactions. Nasal cannula oxygen therapy allows for easier feedings and psychosocial interactions. The parent can swaddle the infant during gavage feedings to help the infant associate the feeding with positive interactions. Some parents like to do kangaroo care while gavage feeding their infant. Swaddling or kangaroo care during feedings provides positive interactions for the infant.)

Which statement concerning the benefits or limitations of breastfeeding is inaccurate? a. Breast milk changes over time to meet changing needs as infants grow. b. Long-term studies have shown that the benefits of breast milk continue after the infant is weaned. c. Breast milk/breastfeeding may enhance cognitive development. d. Breastfeeding increases the risk of childhood obesity.

D (Breastfeeding actually decreases the risk of childhood obesity. There are multiple benefits of breastfeeding. Breast milk changes over time to meet changing needs as infants grow. Long-term studies have shown that the benefits of breast milk continue after the infant is weaned. Breast milk/breastfeeding may enhance cognitive development.)

When teaching parents about mandatory newborn screening, it is important for the nurse to explain that the main purpose is to: a. Keep the state records updated. b. Allow accurate statistical information. c. Document the number of births. d. Recognize and treat newborn disorders early.

D (Early treatment of disorders will prevent morbidity associated with inborn errors of metabolism or other genetic conditions. Keeping records and reporting for statistical purposes are not the primary reason for the screening test. The number of births recorded is not reported from the newborn screening test.)

A patient reports excessive vomiting in the first trimester of the pregnancy, which has resulted in nutritional deficiency and weight loss. The urinalysis report of the patient indicates ketonuria. Which disorder does the patient have? A. Preeclampsia B. Hyperthyroid disorder C. Gestational hypertension D. Hyperemesis gravidarum

D (Hyperemesis gravidarum is characterized by excessive vomiting during pregnancy, which causes nutritional deficiency and weight loss. The presence of ketonuria is another indication of this disorder. Preeclampsia refers to hypertension and proteinuria in patients after 20 weeks' gestation. Hyperthyroid disorder may be one of the causes of hyperemesis gravidarum. Gestational hypertension also develops after 20 weeks' gestation.)

HIV may be perinatally transmitted: a. Only in the third trimester from the maternal circulation. b. From the use of unsterile instruments. c. Only through the ingestion of amniotic fluid. d. Through the ingestion of breast milk from an infected mother.

D (Postnatal transmission of HIV through breastfeeding may occur. Transmission of HIV from the mother to the infant may occur transplacentally at various gestational ages. This is highly unlikely because most health care facilities must meet sterility standards for all instrumentation. Transmission of HIV may occur during birth from blood or secretions.)

Human immunodeficiency virus (HIV) may be perinatally transmitted: a. Only in the third trimester from the maternal circulation. b. By a needlestick injury at birth from unsterile instruments. c. Only through the ingestion of amniotic fluid. d. Through the ingestion of breast milk from an infected mother.

D (Postnatal transmission of HIV through breastfeeding may occur. Transmission of HIV from the mother to the infant may occur transplacentally at various gestational ages. Transmission close to or at the time of birth is thought to account for 50% to 80% of cases.)

When assessing the preterm infant the nurse understands that compared with the term infant, the preterm infant has: a. Few blood vessels visible through the skin. b. More subcutaneous fat. c. Well-developed flexor muscles. d. Greater surface area in proportion to weight.

D (Preterm infants have greater surface area in proportion to their weight. More subcutaneous fat and well-developed muscles are indications of a more mature infant.)

As part of their teaching function at discharge, nurses should educate parents regarding safe sleep. Which statement is incorrect? a. Prevent exposure to people with upper respiratory tract infections. b. Keep the infant away from secondhand smoke. c. Avoid loose bedding, water beds, and beanbag chairs. d. Place the infant on his or her abdomen to sleep.

D (The infant should be laid down to sleep on his or her back for better breathing and to prevent sudden infant death syndrome. Infants are vulnerable to respiratory infections; infected people must be kept away. Secondhand smoke can damage lungs. Infants can suffocate in loose bedding, and furniture that can trap them. Per AAP guidelines, infants should always be placed "back to sleep" and allowed tummy time to play, to prevent plagiocephaly.)

Which is a priority nursing action when a pregnant patient with severe gestational hypertension is admitted to the health care facility? A. Prepare the patient for cesarean delivery. B. Administer intravenous (I.V.) and oral fluids. C. Provide diversionary activities during bed rest. D. Administer the prescribed magnesium sulfate.

D (The nurse administers the prescribed magnesium sulfate to the patient to prevent eclamptic seizures. I.V. oral fluids are indicated when there is severe dehydration in the patient. It is important to provide diversionary activities during bed rest, but it is secondary in this case. A patient who has experienced a multisystem trauma is prepared for cesarean delivery if there is no evidence of a maternal pulse, which increases the chance of maternal survival.)

Because of the premature infant's decreased immune functioning, what nursing diagnosis should the nurse include in a plan of care for a premature infant? a. Delayed growth and development b. Ineffective thermoregulation c. Ineffective infant feeding pattern d. Risk for infection

D (The nurse needs to understand that decreased immune functioning increases the risk for infection. Growth and development, thermoregulation, and feeding may be affected, although only indirectly.)

A woman at 39 weeks of gestation with a history of preeclampsia is admitted to the labor and birth unit. She suddenly experiences increased contraction frequency of every 1 to 2 minutes; dark red vaginal bleeding; and a tense, painful abdomen. The nurse suspects the onset of: A. eclamptic seizure. B. rupture of the uterus. C. placenta previa. D. placental abruption.

D (placental abruption. Eclamptic seizures are evidenced by the presence of generalized tonic-clonic convulsions. Uterine rupture presents as hypotonic uterine activity, signs of hypovolemia, and in many cases the absence of pain. Placenta previa presents with bright red, painless vaginal bleeding. Uterine tenderness in the presence of increasing tone may be the earliest finding of premature separation of the placenta (abruptio placentae or placental abruption). Women with hypertension are at increased risk for an abruption.)

Which nursing action is most appropriate to correct a boggy uterus that is displaced above and to the right of the umbilicus? a. Notify the physician of an impending hemorrhage. b. Assess the blood pressure and pulse. c. Evaluate the lochia. d. Assist the client in emptying her bladder.

D Urinary retention may cause overdistention of the urinary bladder, which lifts and displaces the uterus. Nursing actions need to be implemented before notifying the physician. Evaluating blood pressure, pulse, and lochia is important if the bleeding continues; however, the focus at this point is to assist the client in emptying her bladder.

Four hours after a difficult labor and birth, a primiparous woman refuses to feed her baby, stating that she is too tired and just wants to sleep. The nurse should: a. Tell the woman she can rest after she feeds her baby. b. Recognize this as a behavior of the taking-hold stage. c. Record the behavior as ineffective maternal-newborn attachment. d. Take the baby back to the nursery, reassuring the woman that her rest is a priority at this time.

D The behavior described is typical of this stage and not a reflection of ineffective attachment unless the behavior persists. Mothers need to reestablish their own well-being to care for their baby effectively.

A pregnant couple has formulated a birth plan and is reviewing it with the nurse at an expectant parent's class. Which aspect of their birth plan would be considered unrealistic and require further discussion with the nurse? A. "My husband and I have agreed that my sister will be my coach since he becomes anxious with regard to medical procedures and blood. He will be nearby and check on me every so often to make sure everything is OK." B. "We plan to use the techniques taught in the Lamaze classes to reduce the pain experienced during labor." C. "We want the labor and birth to take place in a birthing room. My husband will come in the minute the baby is born." D. "We do not want the fetal monitor used during labor since it will interfere with movement and doing effleurage."

D. "We do not want the fetal monitor used during labor since it will interfere with movement and doing effleurage." Since monitoring is essential to assess fetal well-being, it is not a factor that can be determined by the couple. The nurse should fully explain its importance. The option for intermittent electronic monitoring could be explored if this is a low-risk pregnancy and as long as labor is progressing normally.

Mexican

Eat only warm foods and hot drinks.

Korean or other South East Asian countries

Prefer not to give babies colostrum.

B (If a portion of the placenta is missing, the clinician can explore the uterus, locate the missing fragments, and remove the potential cause of late postpartum hemorrhage.)

The nurse knows that a measure for preventing late postpartum hemorrhage is to: a. administer broad-spectrum antibiotics. b. inspect the placenta after delivery. c. manually remove the placenta. d. pull on the umbilical cord to hasten the delivery of the placenta.

B (Rugae reappear within 3 to 4 weeks)

Which description of postpartum restoration or healing times is accurate? a) The cervix shortens, becomes firm, and returns to form within a month postpartum b) Rugae reappear within 3 to 4 weeks c) Most episiotomies healh within a week d) Hermorrhoids usually decrease in size within 2 weeks of childbirth

D (Venous congestion begins as soon as the woman stands up. The stockings should be applied before she rises from the bed in the morning.)

Which statement by a postpartal woman indicates that further teaching is not needed regarding thrombus formation? a. "I'll stay in bed for the first 3 days after my baby is born." b. "I'll keep my legs elevated with pillows." c. "I'll sit in my rocking chair most of the time." d. "I'll put my support stockings on every morning before rising."

Muslim countries

Will not eat pork or pork products

C (Extended contact is especially important for adolescents and low-income women because they are at risk for parenting inadequacies.)

With regard to parents' early and extended contact with their infant and the relationships built, nurses should be aware that: A. Immediate contact is essential for the parent-child relationship. B. Skin-to-skin contact is preferable to contact with the body totally wrapped in a blanket. C. Extended contact is especially important for adolescents and low-income women because they are at risk for parenting inadequacies. D. Mothers need to take precedence over their partners and other family matters.

1. A woman gave birth to an infant boy 10 hours ago. Where does the nurse expect to locate this woman's fundus? a. 1 centimeter above the umbilicus b. 2 centimeters below the umbilicus c. Midway between the umbilicus and the symphysis pubis d. Nonpalpable abdominally

a. 1 centimeter above the umbilicus

2. A woman at 10 weeks of gestation who is seen in the prenatal clinic with presumptive signs and symptoms of pregnancy likely will have: a. Amenorrhea. b. Positive pregnancy test. c. Chadwick's sign. d. Hegar's sign.

a. Amenorrhea.

With regard to prenatal genetic testing, nurses should be aware that: a. Maternal serum screening can determine whether a pregnant woman is at risk of carrying a fetus with Down syndrome. b. Carrier screening tests look for gene mutations of people already showing symptoms of a disease. c. Predisposition testing predicts with near certainty that symptoms will appear. d. Presymptomatic testing is used to predict the likelihood of breast cancer.

a. Maternal serum screening can determine whether a pregnant woman is at risk of carrying a fetus with Down syndrome. Maternal serum screening identifies the risk for the neural tube defect and the specific chromosome abnormality involved in Down syndrome. Carriers of some diseases such as sickle cell disease do not display symptoms. Predisposition testing determines susceptibility such as for breast cancer; presymptomatic testing indicates that, if the gene is present, symptoms are certain to appear.

19. The mucous plug that forms in the endocervical canal is called the: a. Operculum. b. Leukorrhea. c. Funic souffle. d. Ballottement.

a. Operculum.

24. To reassure and educate pregnant clients about the functioning of their kidneys in eliminating waste products, maternity nurses should be aware that: a. Increased urinary output makes pregnant women less susceptible to urinary infection. b. Increased bladder sensitivity and then compression of the bladder by the enlarging uterus results in the urge to urinate even if the bladder is almost empty. c. Renal (kidney) function is more efficient when the woman assumes a supine position. d. Using diuretics during pregnancy can help keep kidney function regular.

b. Increased bladder sensitivity and then compression of the bladder by the enlarging uterus results in the urge to urinate even if the bladder is almost empty.

23. While working with the pregnant woman in her first trimester, the nurse is aware that chorionic villus sampling (CVS) can be performed during pregnancy at: a. 4 weeks b. 8 weeks c. 10 weeks d. 14 weeks

c. 10 weeks

At approximately _____ weeks of gestation, lecithin is forming on the alveolar surfaces, the eyelids open, and the fetus measures approximately 27 cm crown to rump and weighs approximately 1110 g. a. 20 b. 24 c. 28 d. 30

c. 28 These are all milestones in human development that occur at approximately 28 weeks.

14. With regard to amniocentesis, nurses should be aware that: a. Because of new imaging techniques, amniocentesis is now possible in the first trimester. b. Despite the use of ultrasound, complications still occur in the mother or infant in 5% to 10% of cases. c. Administration RhoD immunoglobulin may be necessary. d. The presence of meconium in the amniotic fluid is always cause for concern.

c. Administration RhoD immunoglobulin may be necessary.

5. At 35 weeks of pregnancy a woman experiences preterm labor. Tocolytics are administered and she is placed on bed rest, but she continues to experience regular uterine contractions, and her cervix is beginning to dilate and efface. What would be an important test for fetal well-being at this time? a. Percutaneous umbilical blood sampling (PUBS) b. Ultrasound for fetal size c. Amniocentesis for fetal lung maturity d. Nonstress test (NST)

c. Amniocentesis for fetal lung maturity

6. Two days ago a woman gave birth to a full-term infant. Last night she awakened several times to urinate and noted that her gown and bedding were wet from profuse diaphoresis. Which physiologic alteration is the cause for the diaphoresis and diuresis that this client is experiencing? a. Elevated temperature caused by postpartum infection b. Increased basal metabolic rate after giving birth c. Loss of increased blood volume associated with pregnancy d. Increased venous pressure in the lower extremities

c. Loss of increased blood volume associated with pregnancy

12. A patient at 24 weeks of gestation contacts the nurse at her obstetric provider's office to complain that she has cravings for dirt and gravel. The nurse is aware that this condition is known as ________ and may indicate anemia. a. Ptyalism b. Pyrosis c. Pica d. Decreased peristalsis

c. Pica

5. Nafarelin is currently used as a treatment for mild-to-severe endometriosis. The nurse should tell a woman taking this medication that the drug: a. Stimulates the secretion of gonadotropin-releasing hormone (GnRH), thereby stimulating ovarian activity. b. Should be sprayed into one nostril every other day. c. Should be injected into subcutaneous tissue BID. d. Can cause her to experience some hot flashes and vaginal dryness.

d. Can cause her to experience some hot flashes and vaginal dryness.

40. The nurse providing care in a women's health care setting must be aware regarding which sexually transmitted infection that can be successfully treated and cured? a. Herpes b. Acquired immunodeficiency syndrome (AIDS) c. Venereal warts d. Chlamydia

d. Chlamydia

9. A client is concerned that her breasts are engorged and uncomfortable. What is the nurse's explanation for this physiologic change? a. Overproduction of colostrum b. Accumulation of milk in the lactiferous ducts and glands c. Hyperplasia of mammary tissue d. Congestion of veins and lymphatic vessels

d. Congestion of veins and lymphatic vessels

14. Which condition, not uncommon in pregnancy, is likely to require careful medical assessment during the puerperium? a. Varicosities of the legs b. Carpal tunnel syndrome c. Periodic numbness and tingling of the fingers d. Headaches

d. Headaches

With regard to the estimation and interpretation of the recurrence of risks for genetic disorders, nurses should be aware that: a. With a dominant disorder the likelihood of the second child also having the condition is 100%. b. An autosomal recessive disease carries a one in eight risk of the second child also having the disorder. c. Disorders involving maternal ingestion of drugs carry a one in four chance of being repeated in the second child. d. The risk factor remains the same no matter how many affected children are already in the family.

d. The risk factor remains the same no matter how many affected children are already in the family. Each pregnancy is an independent event. The risk factor (e.g., one in two, one in four) remains the same for each child, no matter how many children are born to the family. In a dominant disorder the likelihood of recurrence in subsequent children is 50% (one in two). An autosomal recessive disease carries a one in four chance of recurrence. Subsequent children would be at risk only if the mother continued to take drugs; the rate of risk would be difficult to calculate.

A pregnant woman's amniotic membranes rupture. Prolapsed umbilical cord is suspected. What intervention would be the top priority? a. Placing the woman in the knee-chest position b. Covering the cord in sterile gauze soaked in saline c. Preparing the woman for a cesarean birth d. Starting oxygen by face mask

A

A woman in preterm labor at 30 weeks of gestation receives two 12-mg doses of betamethasone intramuscularly. The purpose of this pharmacologic treatment is to: a. Stimulate fetal surfactant production. b. Reduce maternal and fetal tachycardia associated with ritodrine administration. c. Suppress uterine contractions. d. Maintain adequate maternal respiratory effort and ventilation during magnesium sulfate therapy.

A

A pregnant patient reports abdominal pain in the right lower quadrant, along with nausea and vomiting. The patient's urinalysis report shows an absence of any urinary tract infection in the patient. A chest x-ray also rules out lower-lobe pneumonia. Which condition does the nurse suspect in the patient? A. Appendicitis B. Cholelithiasis C. Placenta previa D. Uterine rupture

A (Abdominal pain in the right lower quadrant, accompanied by nausea and vomiting, indicates appendicitis in a pregnant patient. Cholelithiasis is characterized by right upper quadrant pain. Placenta previa is a condition wherein the placenta is implanted in the lower uterine segment covering the cervix, which causes bleeding when the cervix dilates. Uterine rupture is seen in a pregnant patient as a result of trauma, which may cause fetal death.)

In caring for the woman with disseminated intravascular coagulation (DIC), what order should the nurse anticipate? A. Administration of blood B. Preparation of the woman for invasive hemodynamic monitoring C. Restriction of intravascular fluids D. Administration of steroids

A (Administration of blood Primary medical management in all cases of DIC involves correction of the underlying cause, volume replacement, blood component therapy, optimization of oxygenation and perfusion status, and continued reassessment of laboratory parameters. Central monitoring would not be ordered initially in a woman with DIC because this can contribute to more areas of bleeding. Management of DIC includes volume replacement, not volume restriction. Steroids are not indicated for the management of DIC.)

A new mother asks whether she should feed her newborn colostrum, because it is not "real milk." The nurse's most appropriate answer is: a. Colostrum is high in antibodies, protein, vitamins, and minerals. b. Colostrum is lower in calories than milk and should be supplemented by formula. c. Giving colostrum is important in helping the mother learn how to breastfeed before she goes home. d. Colostrum is unnecessary for newborns.

A (Colostrum is important because it has high levels of the nutrients needed by the neonate and helps protect against infection. Supplementation is not necessary; it will decrease stimulation to the breast and decrease the production of milk. It is important for the mother to feel comfortable in this role before discharge; however, the importance of the colostrum to the infant is the top priority. Colostrum provides immunities and enzymes necessary to cleanse the gastrointestinal system, among other things.)

A woman with severe preeclampsia has been receiving magnesium sulfate by intravenous infusion for 8 hours. The nurse assesses the woman and documents the following findings: temperature 37.1° C, pulse rate 96 beats/min, respiratory rate 24 breaths/min, blood pressure 155/112 mm Hg, 3+ deep tendon reflexes, and no ankle clonus. The nurse calls the physician, anticipating an order for: A. hydralazine. B. magnesium sulfate bolus. C. diazepam. D. morphine

A (Hydralazine is an antihypertensive commonly used to treat hypertension in severe preeclampsia. An additional bolus of magnesium sulfate may be ordered for increasing signs of central nervous system irritability related to severe preeclampsia (e.g., clonus) or if eclampsia develops. Diazepam sometimes is used to stop or shorten eclamptic seizures. Calcium gluconate is used as the antidote for magnesium sulfate toxicity. The patient is not currently displaying any signs or symptoms of magnesium toxicity.)

The nurse tells the primary health care provider (PHP) that there is 15 mL of fetal blood in maternal circulation, as detected by Kleihauer-Betke test, in an Rh-negative patient. What does the nurse expect the PHP to prescribe to this patient? A. 300 mcg of intramuscular Rh immune globulin B. 400 mcg of intramuscular Rh immune globulin C. 100 mcg of intramuscular Rh immune globulin D. 200 mcg of intramuscular Rh immune globulin

A (If 15 mL of fetal blood is detected in the maternal circulation of an Rh-negative woman, as indicated by Kleihauer-Betke test, then 300 mcg (1 vial) of Rh immune globulin is usually sufficient to prevent maternal sensitization. A dose of 400 mcg of intramuscular Rh immune globulin may result in an overdosage. A dose of 100 mcg or 200 mcg of intramuscular Rh immune globulin is not sufficient to prevent maternal sensitization.)

The nurse is discussing storage of breast milk with a mother whose infant is preterm and in the special care unit. What statement would indicate that the mother needs additional teaching? a. "I can store my breast milk in the refrigerator for 3 months." b. "I can store my breast milk in the freezer for 3 months." c. "I can store my breast milk at room temperature for 8 hours." d. "I can store my breast milk in the refrigerator for 3 to 5 days."

A (If the mother states that she can store her breast milk in the refrigerator for 3 months, she needs additional teaching about safe storage. Breast milk can be stored at room temperature for 8 hours, in the refrigerator for 3 to 5 days, in the freezer for 3 months, or in a deep freezer for 6 to 12 months. It is accurate and does not require additional teaching if the mother states that she can store her breast milk in the freezer for 3 months, at room temperature for 8 hours, and in the refrigerator for 3 to 5 days.)

The nurse is caring for a pregnant patient who is receiving antibiotic therapy to treat a urinary tract infection (UTI). Which dietary changes does the nurse suggest for the pregnant patient who is receiving antibiotic therapy for UTI? A. "Include yogurt, cheese, and milk in your diet." B. "Avoid folic acid supplements until the end of therapy." C. "Include vitamins C and E supplementation in your diet." D. "Reduce your dietary fat intake by 40 to 50 g per day.

A (The antibiotic therapy kills normal flora in the genitourinary tract, as well as pathologic organisms. Therefore the nurse instructs the patient to include yogurt, cheese, and milk in daily diet because they contain active acidophilus cultures. Folic acid should not be avoided, because it may affect the fetal development. Vitamins C and E supplementation is usually included in the diet to treat preeclampsia in a patient. Dietary fat is reduced in patients with cholecystitis or cholelithiasis, because it may cause epigastric pain.)

A pregnant woman presents in labor at term, having had no prenatal care. After birth her infant is noted to be small for gestational age with small eyes and a thin upper lip. The infant also is microcephalic. On the basis of her infant's physical findings, this woman should be questioned about her use of which substance during pregnancy? a. Alcohol b. Cocaine c. Heroin d. Marijuana

A (The description of the infant suggests fetal alcohol syndrome, which is consistent with maternal alcohol consumption during pregnancy. Fetal brain, kidney, and urogenital system malformations have been associated with maternal cocaine ingestions. Heroin use in pregnancy frequently results in intrauterine growth restriction. The infant may have a shrill cry and sleep cycle disturbances and present with poor feeding, tachypnea, vomiting, diarrhea, hypothermia or hyperthermia, and sweating. Studies have found a higher incidence of meconium staining in infants born of mothers who used marijuana during pregnancy.)

Nursing follow-up care often includes home visits for the new mother and her infant. Which information related to home visits is correct? a. Ideally, the visit is scheduled within 72 hours after discharge. b. Home visits are available in all areas. c. Visits are completed within a 30-minute time frame. d. Blood draws are not a part of the home visit.

A (The home visit is ideally scheduled within 72 hours after discharge. This timing allows early assessment and intervention for problems with feedings, jaundice, newborn adaptation, and maternal-infant interaction. Because home visits are expensive, they are not available in all geographic areas. Visits are usually 60 to 90 minutes in length to allow enough time for assessment and teaching. When jaundice is found, the nurse can discuss the implications and check the transcutaneous bilirubin level or draw blood for testing.)

In the assessment of a preterm infant, the nurse notices continued respiratory distress even though oxygen and ventilation have been provided. The nurse should suspect: a. Hypovolemia and/or shock. b. A nonneutral thermal environment. c. Central nervous system injury. d. Pending renal failure.

A (The nurse should suspect hypovolemia and/or shock. Other symptoms could include hypotension, prolonged capillary refill, and tachycardia followed by bradycardia. Intervention is necessary.)

A postpartum patient who has an episiotomy is being discharged to home. Which instruction about medications is most important for the patient? A. Take stool softeners regularly. B. Continue prenatal vitamins. C. Include iron supplements. D. Take analgesics as prescribed.

A (The patient who has an episiotomy may have constipation due to discomfort during bowel movements. Therefore the nurse should instruct the patient to use stool softeners to help ease the passage of stools. Prenatal vitamins should be continued in all patients regardless of the episiotomy. All patients should take iron supplements to increase their hemoglobin levels. However, they do not ease the discomfort of episiotomy. Analgesics are usually prescribed for patients who underwent a cesarean.)

Nurses providing nutritional instruction should be cognizant of the uniqueness of human milk. Which statement is correct? a. Frequent feedings during predictable growth spurts stimulate increased milk production. b. The milk of preterm mothers is the same as the milk of mothers who gave birth at term. c. The milk at the beginning of the feeding is the same as the milk at the end of the feeding. d. Colostrum is an early, less concentrated, less rich version of mature milk.

A (These growth spurts (10 days, 3 weeks, 6 weeks, 3 months) usually last 24 to 48 hours, after which infants resume normal feeding. The milk of mothers of preterm infants is different from that of mothers of full-term infants to meet the needs of these newborns. Milk changes composition during feeding. The fat content of the milk increases as the infant feeds. Colostrum precedes mature milk and is more concentrated and richer in proteins and minerals (but not fat).)

During a prenatal examination, the woman reports having two cats at home. The nurse informs her that she should not be cleaning the litter box while she is pregnant. When the woman asks why, the nurse's best response would be: a. "Your cats could be carrying toxoplasmosis. This is a zoonotic parasite that can infect you and have severe effects on your unborn child." b. "You and your baby can be exposed to the human immunodeficiency virus (HIV) in your cats' feces." c. "It's just gross. You should make your husband clean the litter boxes." d. "Cat feces are known to carry Escherichia coli, which can cause a severe infection in both you and your baby."

A (Toxoplasmosis is a multisystem disease caused by the protozoal Toxoplasma gondii parasite, commonly found in cats, dogs, pigs, sheep, and cattle. About 30% of women who contract toxoplasmosis during gestation transmit the disease to their children. Clinical features ascribed to toxoplasmosis include hydrocephalus or microcephaly, chorioretinitis, seizures, or cerebral calcifications. HIV is not transmitted by cats. Although suggesting that the woman's husband clean the litter boxes may be a valid statement, it is not appropriate, does not answer the client's question, and is not the nurse's best response. E. coli is found in normal human fecal flora. It is not transmitted by cats.)

A woman diagnosed with marginal placenta previa gave birth vaginally 15 minutes ago. At the present time she is at the greatest risk for: A. hemorrhage. B. infection. C. urinary retention. D. thrombophlebitis.

A (hemorrhage. Hemorrhage is the most immediate risk because the lower uterine segment has limited ability to contract to reduce blood loss. Infection is a risk because of the location of the placental attachment site; however, it is not a priority concern at this time. Placenta previa poses no greater risk for urinary retention than with a normally implanted placenta. There is no greater risk for thrombophlebitis than with a normally implanted placenta.)

The nurse providing care for the laboring woman should understand that amnioinfusion is used to treat: A. Variable decelerations B. Late decelerations C. Fetal bradycardia D. Fetal tachycardia

A A. Correct: Amnioinfusion is used during labor either to dilute meconium-stained amniotic fluid or to supplement the amount of amniotic fluid to reduce the severity of variable decelerations caused by cord compression. B. Incorrect: Amnioinfusion has no bearing on this alteration in FHR tracings. C. Incorrect: Amnioinfusion has no bearing on this alteration in FHR tracings. D. Incorrect: Amnioinfusion has no bearing on this alteration in FHR tracings. p. 508

A normal uterine activity (UA) pattern in labor is characterized by: A. Contractions every 2 to 5 minutes B. Contractions lasting about 2 minutes C. Contractions about 1 minute apart D. A contraction intensity of about 1000 mm Hg with relaxation at 50 mm Hg

A A. Correct: Contractions normally occur every 2 to 5 minutes and last less than 90 seconds (intensity 800 mm Hg) with about 30 seconds in between (20 mm Hg or less). B. Incorrect: Contractions normally occur every 2 to 5 minutes and last less than 90 seconds (intensity 800 mm Hg) with about 30 seconds in between (20 mm Hg or less). C. Incorrect: Contractions normally occur every 2 to 5 minutes and last less than 90 seconds (intensity 800 mm Hg) with about 30 seconds in between (20 mm Hg or less). D. Incorrect: Contractions normally occur every 2 to 5 minutes and last less than 90 seconds (intensity 800 mm Hg) with about 30 seconds in between (20 mm Hg or less). p. 498

The nurse caring for a laboring woman is aware that maternal cardiac output can be increased by: A. Change in position B. Oxytocin administration C. Regional anesthesia D. Intravenous analgesic

A A. Correct: Maternal supine hypotension syndrome is caused by the weight and pressure of the gravid uterus on the ascending vena cava when the woman is in a supine position. This reduces venous return to the woman's heart, as well as cardiac output, and subsequently reduces her blood pressure. The nurse can encourage the woman to change positions and to avoid the supine position. B. Incorrect: This intervention may reduce maternal cardiac output. C. Incorrect: This intervention may reduce maternal cardiac output. D. Incorrect: This intervention may reduce maternal cardiac output. p. 503

Fetal tachycardia is most common during: A. Maternal fever B. Umbilical cord prolapse C. Regional anesthesia D. MgSO4 administration

A A. Maternal fever Correct: Fetal tachycardia can be considered an early sign of fetal hypoxemia and can also result from maternal or fetal infection. B. Umbilical cord prolapse Incorrect: This situation most likely would result in fetal bradycardia, not tachycardia. C. Regional anesthesia Incorrect: This situation most likely would result in fetal bradycardia, not tachycardia. D. MgSO4 administration Incorrect: This situation most likely would result in fetal bradycardia, not tachycardia. p. 505

D (Transvaginal ultrasound is useful for obese women whose thick abdominal layers cannot be penetrated adequately with the abdominal approach. A biophysical profile is a method of biophysical assessment of fetal well-being in the third trimester. An amniocentesis is performed after the fourteenth week of pregnancy. A MSAFP test is performed from week 15 to week 22 of the gestation (weeks 16 to 18 are ideal). An ultrasound is the method of biophysical assessment of the infant that is performed at this gestational age.)

A 40-year-old woman with a high body mass index is 10 weeks pregnant. Which diagnostic tool is appropriate to suggest to her at this time? A. Biophysical profile B. Amniocentesis C. Maternal serum alpha-fetoprotein (MSAFP) D. Transvaginal ultrasound

C (Rationale: Vibroacoustic stimulation is sometimes used with NST. CST is invasive if stimulation is by IV oxytocin but not if by nipple stimulation. No late decelerations indicate a positive CST. CST is contraindicated if the membranes have ruptured.)

A nurse providing care for the antepartum woman should understand that the contraction stress test (CST): A. sometimes uses vibroacoustic stimulation. B. is an invasive test; however, contractions are stimulated. C. is considered negative if no late decelerations are observed with the contractions. D. is more effective than nonstress test (NST) if the membranes have already been ruptured.

A (Three-dimensional (3D) or four-dimensional (4D) ultrasonography is advisable for women who want to see the fetus. MRI cannot be used in this case because it requires the fetus to be still for a long period of time for a clear image. CT uses ionizing radiation for imaging, which can be harmful to the fetus. Therefore CT is contraindicated for fetal imaging. NT is a specific ultrasonography screening procedure used to test for genetic abnormalities in the fetus.)

A patient in the sixth month of pregnancy expresses her wish to see the fetus. What investigation does the nurse suggest for the patient to help her see the fetus? A. Ultrasonography B. Magnetic resonance imaging (MRI) C. Computed tomography (CT) D. Nuchal translucency (NT)

C (Typically running should be replaced with walking around the seventh month of pregnancy. The nurse should inform the woman that she may need to reduce her exercise level as the pregnancy progresses. Physical activity promotes a feeling of well-being in pregnant women. It improves circulation, promotes relaxation and rest, and counteracts boredom. Simple measures should be initiated to prevent injuries, such as warm-up and stretching exercises to prepare the joints for more strenuous exercise.)

A pregnant woman at 10 weeks of gestation jogs three or four times per week. She is concerned about the effect of exercise on the fetus. The nurse should inform her: A. "You don't need to modify your exercising any time during your pregnancy." B. "Stop exercising because it will harm the fetus." C. "You may find that you need to modify your exercise to walking later in your pregnancy, around the seventh month." D. "Jogging is too hard on your joints; switch to walking now."

B (Women express a need to review their childbirth experience and evaluate their performance. After the mother's needs are met, she is more able to take an active role, not only in her own care but also in the care of her newborn. Short teaching sessions (using written materials to reinforce the content presented) are a more effective approach. The focus of the taking-in stage is nurturing the new mother by meeting her dependency needs for rest, comfort, hygiene, and nutrition. Test-Taking Tip: After choosing an answer, go back and reread the question stem along with your chosen answer. Does it fit correctly? The choice that grammatically fits the stem and contains the correct information is the best choice.)

A primiparous woman is in the taking-in stage of psychosocial recovery and adjustment following birth. Recognizing the needs of women during this stage, the nurse should: A. Foster an active role in the baby's care. B. Provide time for the mother to reflect on the events of and her behavior during childbirth. C. Recognize the woman's limited attention span by giving her written materials to read when she gets home rather than doing a teaching session now. D. Promote maternal independence by encouraging her to meet her own hygiene and comfort needs.

B (Undetected lacerations will bleed slowly and continuously. Bleeding from lacerations is uncontrolled by uterine contraction.)

A steady trickle of bright red blood from the vagina in the presence of a firm fundus suggests: a. uterine atony. b. lacerations of the genital tract. c. perineal hematoma. d. infection of the uterus.

Risk factors associated with necrotizing enterocolitis (NEC) include (Select all that apply): a. Polycythemia. b. Anemia. c. Congenital heart disease. d. Bronchopulmonary dysphasia. e. Retinopathy.

A, B, C (Risk factors for NEC include asphyxia, respiratory distress syndrome, umbilical artery catheterization, exchange transfusion, early enteral feedings, patent ductus arteriosus, congenital heart disease, polycythemia, anemia, shock, and gastrointestinal infection. Bronchopulmonary dysphasia and retinopathy are not associated with NEC.)

Vaginal examinations should be performed by the nurse under which of these circumstances. (Select all that apply.) A. An admission to the hospital at the start of labor B. When accelerations of the fetal heart rate (FHR) are noted C. On maternal perception of perineal pressure or the urge to bear down D. When membranes rupture E. When bright, red bleeding is observed

A. An admission to the hospital at the start of labor C. On maternal perception of perineal pressure or the urge to bear down D. When membranes rupture

A key finding from the Human Genome Project is: A. Approximately 20,000 to 25,000 genes make up the genome. B. All human beings are 80.99% identical at the DNA level. C. Human genes produce only one protein per gene; other mammals produce three proteins per gene. D. Single-gene testing will become a standardized test for all pregnant women in the future.

A. Approximately 20,000 to 25,000 genes make up the genome. Approximately 20,500 genes make up the human genome; this is only twice as many as make up the genomes of roundworms and flies. Human beings are 99.9% identical at the DNA level. Most human genes produce at least three proteins. Single-gene testing (e.g., alpha-fetoprotein) is already standardized for prenatal care

D (Biorhythmicity)

After birth, a crying infant may be soothed by being held in a position in which the newborn can hear the mother's heartbeat. This phenomenon is known as: A. Entrainment B. Reciprocity C. Synchrony D. Biorhythmicity

While caring for the patient who requires an induction of labor, the nurse should be cognizant that: a. Ripening the cervix usually results in a decreased success rate for induction. b. Labor sometimes can be induced with balloon catheters or laminaria tents. c. Oxytocin is less expensive than prostaglandins and more effective but creates greater health risks. d. Amniotomy can be used to make the cervix more favorable for labor.

B

The hormone necessary for milk production is: a. Estrogen. b. Prolactin. c. Progesterone. d. Lactogen.

B (Prolactin, secreted by the anterior pituitary, is a hormone that causes the breasts to produce milk. Estrogen decreases the effectiveness of prolactin and prevents mature breast milk from being produced. Progesterone decreases the effectiveness of prolactin and prevents mature breast milk from being produced. Human placental lactogen decreases the effectiveness of prolactin and prevents mature breast milk from being produced.)

The nurse is explaining the benefits associated with breastfeeding to a new mother. Which statement by the nurse would be inaccurate and provide conflicting information to the patient? a. Women who breastfeed have a decreased risk of breast cancer. b. Breastfeeding is an effective method of birth control. c. Breastfeeding increases bone density. d. Breastfeeding may enhance postpartum weight loss.

B (Women who breastfeed have a decreased risk of breast cancer, an increase in bone density, and a possibility of quicker postpartum weight loss. Breastfeeding delays the return of fertility; however, it is not an effective birth control method.)

What correctly matches the type of deceleration with its likely cause? A. Early deceleration—umbilical cord compression B. Late deceleration—uteroplacental inefficiency C. Variable deceleration—head compression D. Prolonged deceleration—cause unknown

B A. Incorrect: Early deceleration is caused by head compression. B. Correct: Late deceleration is caused by uteroplacental inefficiency. C. Incorrect: Variable deceleration is caused by umbilical cord compression. D. Incorrect: Prolonged deceleration has a variety of either benign or critical causes. p. 507

What three measures should the nurse implement to provide intrauterine resuscitation? Select the best response that indicates the priority of actions that should be taken, starting with the most important. A. Call the provider, reposition the mother, and perform a vaginal exam B. Reposition the mother, increase IV fluid, and provide oxygen via face mask C. Administer oxygen to the mother, increase IV fluid, and notify the care provider D. Perform a vaginal exam, reposition the mother, and provide oxygen via face mask

B A. Incorrect: The nurse should initiate intrauterine resuscitation in an ABC manner, similar to basic life support. The first priority is to open the maternal and fetal vascular systems by repositioning the mother for improved perfusion. The second priority is to increase blood volume by increasing the IV fluid. The third priority is to optimize oxygenation of the circulatory volume by providing oxygen via face mask. If these interventions do not resolve the fetal heart rate issue quickly, the primary provider should be notified immediately. B. Correct: These are the correct nursing actions for intrauterine resuscitation. C. Incorrect: The nurse should initiate intrauterine resuscitation in an ABC manner, similar to basic life support. The first priority is to open the maternal and fetal vascular systems by repositioning the mother for improved perfusion. The second priority is to increase blood volume by increasing the IV fluid. The third priority is to optimize oxygenation of the circulatory volume by providing oxygen via face mask. If these interventions do not resolve the fetal heart rate issue quickly, the primary provider should be notified immediately. D. Incorrect: The nurse should initiate intrauterine resuscitation in an ABC manner, similar to basic life support. The first priority is to open the maternal and fetal vascular systems by repositioning the mother for improved perfusion. The second priority is to increase blood volume by increasing the IV fluid. The third priority is to optimize oxygenation of the circulatory volume by providing oxygen via face mask. If these interventions do not resolve the fetal heart rate issue quickly, the primary provider should be notified immediately. p. 513

When caring for a newly delivered woman, what is the best measure to prevent abdominal distention after a cesarean birth? a. Rectal suppositories b. Early and frequent ambulation c. Tightening and relaxing abdominal muscles d. Carbonated beverages

B Activity will aid the movement of accumulated gas in the gastrointestinal tract. Rectal suppositories can be helpful after distention occurs; however, they do not prevent it. Ambulation is the best prevention. Carbonated beverages may increase distention.

A pregnant patient with chronic hypertension is at risk for placental abruption. Which symptoms of abruption does the nurse instruct the patient to be alert for? Select all that apply. A. Weight loss B. Abdominal pain C. Vaginal bleeding D. Shortness of breath E. Uterine tenderness

B, C, E (The nurse instructs the pregnant patient to be alert for abdominal pain, vaginal bleeding, and uterine tenderness as these indicates placental abruption. Weight loss indicates fluid and electrolyte loss and not placental abruption. Shortness of breath indicates inadequate oxygen, which is usually seen in a patient who is having cardiac arrest.)

The nurse recognizes that uterine hyperstimulation with oxytocin requires emergency interventions. What clinical cues would alert the nurse that the woman is experiencing uterine hyperstimulation (Select all that apply)? a. Uterine contractions lasting <90 seconds and occurring >2 minutes in frequency b. Uterine contractions lasting >90 seconds and occurring <2 minutes in frequency c. Uterine tone <20 mm Hg d. Uterine tone >20 mm Hg e. Increased uterine activity accompanied by a nonreassuring fetal heart rate (FHR) and pattern

B,D,E

A woman who is 39 weeks pregnant expresses fear about her impending labor and how she will manage. The nurse's best response is: A. "Don't worry about it. You'll do fine." B. "It's normal to be anxious about labor. Let's discuss what makes you afraid." C. "Labor is scary to think about, but the actual experience isn't." D. "You may have an epidural. You won't feel anything."

B. "It's normal to be anxious about labor. Let's discuss what makes you afraid." This statement negates the woman's fears and is not therapeutic. This statement allows the woman to share her concerns with the nurse and is a therapeutic communication tool. This statement negates the woman's fears and offers a false sense of security. This statement is not true. A number of criteria must be met for use of an epidural. Furthermore, many women still experience the feeling of pressure with an epidural.

Which characteristic is associated with false labor contractions? A. Painless B. Decrease in intensity with ambulation C. Regular pattern of frequency established D. Progressive in terms of intensity and duration

B. Decrease in intensity with ambulation True labor contractions are painful. Although false labor contractions decrease with activity, true labor contractions are enhanced or stimulated with activity such as ambulation. A regular pattern of frequency is a sign of true labor. A progression of intensity and duration indicates true labor.

What best describes the pattern of genetic transmission known as autosomal recessive inheritance? A. Disorders in which the abnormal gene for the trait is expressed even when the other member of the pair is normal B. Disorders in which both genes of a pair must be abnormal for the disorder to be expressed C. Disorders in which a single gene controls the particular trait D. Disorders in which the abnormal gene is carried on the X chromosome

B. Disorders in which both genes of a pair must be abnormal for the disorder to be expressed Autosomal dominant inheritance occurs when the abnormal gene for the trait is expressed, even when the other member of the pair is normal, such as Huntington disease or Marfan syndrome. An autosomal recessive inheritance disorder occurs when both genes of the pair are abnormal, such as phenylketonuria or sickle cell anemia. Disorders in which a single gene controls the particular trait describe the unifactorial inheritance. X-linked recessive inheritance occurs when the abnormal gene is carried on the X chromosome, such as hemophilia or Duchenne muscular dystrophy.

A (Biochemical findings such as an L/S ratio of 2:1, an S/A ratio of 60 mg/g, and the presence of PG in amniotic fluid indicate that the fetal lungs are well developed. The gestational age can be predicted only with the help of creatinine and lipid levels in the amniotic fluid. Creatinine levels greater than 2 mg/dL in amniotic fluid indicate that the gestational age is more than 36 weeks. The presence of alpha-fetoprotein (AFP) in the amniotic fluid indicates a neural tube defect in the fetus. The nurse needs to assess AFP levels in the amniotic fluid to determine whether the fetus has an open neural tube defect. A high AFP level in amniotic fluid after 15 weeks' gestation indicates that the fetus has an open neural tube defect.)

Biochemical examination of the amniotic fluid of a pregnant patient yields the following results: lecithin-to-sphingomyelin (L/S) ratio, 2:1; surfactant-to-albumin (S/A) ratio, 60 mg/g; and phosphatidylglycerol (PG) present. What conclusions will the nurse draw from this report? A. The fetal lungs are well developed. B. The gestational age is 36 weeks. C. The fetus has a neural tube defect. D. The fetus has an open neural tube defect.

In planning for home care of a woman with preterm labor, which concern must the nurse address? a. Nursing assessments will be different from those done in the hospital setting. b. Restricted activity and medications will be necessary to prevent recurrence of preterm labor. c. Prolonged bed rest may cause negative physiologic effects. d. Home health care providers will be necessary.

C

Prepidil (prostaglandin gel) has been ordered for a pregnant woman at 43 weeks of gestation. The nurse recognizes that this medication will be administered to: a. Enhance uteroplacental perfusion in an aging placenta. b. Increase amniotic fluid volume. c. Ripen the cervix in preparation for labor induction. d. Stimulate the amniotic membranes to rupture.

C

The nurse administers vitamin K to the newborn for which reason? a. Most mothers have a diet deficient in vitamin K, which results in the infant's being deficient. b. Vitamin K prevents the synthesis of prothrombin in the liver and must be given by injection. c. Bacteria that synthesize vitamin K are not present in the newborn's intestinal tract. d. The supply of vitamin K is inadequate for at least 3 to 4 months, and the newborn must be supplemented.

C (Bacteria that synthesize vitamin K are not present in the newborn's intestinal tract. Vitamin K is provided because the newborn does not have the intestinal flora to produce this vitamin for the first week. The maternal diet has no bearing on the amount of vitamin K found in the newborn. Vitamin K promotes the formation of clotting factors in the liver and is used for the prevention and treatment of hemorrhagic disease in the newborn. Vitamin K is not produced in the intestinal tract of the newborn until after microorganisms are introduced. By day 8, normal newborns are able to produce their own vitamin K.)

A mother expresses fear about changing her infant's diaper after he is circumcised. What does the woman need to be taught to take care of the infant when she gets home? a. Cleanse the penis with prepackaged diaper wipes every 3 to 4 hours. b. Apply constant, firm pressure by squeezing the penis with the fingers for at least 5 minutes if bleeding occurs. c. Cleanse the penis gently with water and put petroleum jelly around the glans after each diaper change. d. Wash off the yellow exudate that forms on the glans at least once every day to prevent infection.

C (Cleansing the penis gently with water and putting petroleum jelly around the glans after each diaper change are appropriate when caring for an infant who has had a circumcision. With each diaper change, the penis should be washed off with warm water to remove any urine or feces. If bleeding occurs, the nurse should apply gentle pressure to the site of the bleeding with a sterile gauze square. Yellow exudates cover the glans penis in 24 hours after the circumcision. This is part of normal healing and not an infective process. The exudates should not be removed.)

As related to the eventual discharge of the high risk newborn or transfer to a different facility, nurses and families should be aware that: a. Infants will stay in the neonatal intensive care unit (NICU) until they are ready to go home. b. Once discharged to home, the high risk infant should be treated like any healthy term newborn. c. Parents of high risk infants need special support and detailed contact information. d. If a high risk infant and mother need transfer to a specialized regional center, it is better to wait until after birth and the infant is stabilized.

C (High risk infants can cause profound parental stress and emotional turmoil. Parents need support, special teaching, and quick access to various resources available to help them care for their baby. Parents and their high risk infant should spend a night or two in a predischarge room, where care for the infant is provided away from the NICU. Just because high risk infants are discharged does not mean that they are normal, healthy babies. Follow-up by specialized practitioners is essential. Ideally, the mother and baby are transported with the fetus in utero; this reduces neonatal morbidity and mortality.)

An infant was born 2 hours ago at 37 weeks of gestation and weighing 4.1 kg. The infant appears chubby with a flushed complexion and is very tremulous. The tremors are most likely the result of: a. Birth injury. b. Hypocalcemia. c. Hypoglycemia d. Seizures.

C (Hypoglycemia is common in the macrosomic infant. Signs of hypoglycemia include jitteriness, apnea, tachypnea, and cyanosis.)

Infants of mothers with diabetes (IDMs) are at higher risk for developing: a. Anemia. b. Hyponatremia. c. Respiratory distress syndrome. d. Sepsis.

C (IDMs are at risk for macrosomia, birth injury, perinatal asphyxia, respiratory distress syndrome, hypoglycemia, hypocalcemia, hypomagnesemia, cardiomyopathy, hyperbilirubinemia, and polycythemia. They are not at risk for anemia, hyponatremia, or sepsis.)

The nurse is performing a gestational age and physical assessment on the newborn. The infant appears to have an excessive amount of saliva. The nurse recognizes that this finding: a. Is normal. b. Indicates that the infant is hungry. c. May indicate that the infant has a tracheoesophageal fistula or esophageal atresia. d. May indicate that the infant has a diaphragmatic hernia.

C (The presence of excessive saliva in a neonate should alert the nurse to the possibility of tracheoesophageal fistula or esophageal atresia.)

A new father of 1 day expresses concern to the nurse that his wife, who is normally very independent, is asking him to make all the decisions. The nurse can best explain this as a(n): a. Normal occurrence because the mother is in pain. b. Abnormal occurrence that needs to be assessed further. c. Normal occurrence because the mother is in the taking-in phase. d. Normal occurrence because the mother is frustrated with the care of the newborn.

C During the taking-in phase, the mother is focused primarily on her own need for fluid, food, and sleep. She may be passive and dependent. This is normal and lasts about 2 days.

A woman's cousin gave birth to an infant with a congenital heart anomaly. The woman asks the nurse when such anomalies occur during development. Which response by the nurse is most accurate? A. "We don't really know when such defects occur." B. "It depends on what caused the defect." C. "They occur between the third and fifth weeks of development." D. "They usually occur in the first 2 weeks of development."

C. "They occur between the third and fifth weeks of development." This is an inaccurate statement. Regardless of the cause, the heart is vulnerable during its period of development, the third to fifth weeks. The cardiovascular system is the first organ system to function in the developing human. Blood vessel and blood formation begins in the third week, and the heart is developmentally complete in the fifth week. This is an inaccurate statement.

Which description of the phases of the second stage of labor is accurate? A. Latent phase: feels sleepy, fetal station is 2+ to 4+, duration is 30 to 45 minutes B. Active phase: overwhelmingly strong contractions, Ferguson reflux activated, duration is 5 to 15 minutes C. Descent phase: significant increase in contractions, Ferguson reflux activated, average duration varies D. Transitional phase: woman "laboring down," fetal station is 0, duration is 15 minutes

C. Descent phase: significant increase in contractions, Ferguson reflux activated, average duration varies The latent phase is the lull, or "laboring down," period at the beginning of the second stage. It lasts 10 to 30 minutes on average. The second stage of labor has no active phase. The descent phase begins with a significant increase in contractions, the Ferguson reflex is activated, and the duration varies, depending on a number of factors. The transition phase is the final phase in the second stage of labor; contractions are strong and painful.

C, D (The average blood loss for a vaginal birth of a single fetus ranges from 300 to 500 mL (10% of blood volume). The typical blood loss for women who gave birth by cesarean is 500 to 1000 mL (15% to 30% of blood volume). During the first few days after birth the plasma volume decreases further as a result diuresis. Pregnancy-induced hypervolemia (an increase in blood volume of at least 35%) allows most women to tolerate considerable blood loss during childbirth.)

Changes in blood volume after childbirth depend on several factors such as blood loss during childbirth and the amount of extravascular water (physiologic edema) mobilized and excreted. A postpartum nurse anticipates blood loss of (Select all that apply): A. 100 mL B. 250 mL or less C. 300 to 500 mL D. 500 to 1000 mL E. 1500 mL or greater

A woman at 26 weeks of gestation is being assessed to determine whether she is experiencing preterm labor. What finding indicates that preterm labor is occurring? a. Estriol is not found in maternal saliva. b. Irregular, mild uterine contractions are occurring every 12 to 15 minutes. c. Fetal fibronectin is present in vaginal secretions. d. The cervix is effacing and dilated to 2 cm.

D

For a woman at 42 weeks of gestation, which finding would require further assessment by the nurse? a. Fetal heart rate of 116 beats/min b. Cervix dilated 2 cm and 50% effaced c. Score of 8 on the biophysical profile d. One fetal movement noted in 1 hour of assessment by the mother

D

In evaluating the effectiveness of magnesium sulfate for the treatment of preterm labor, what finding would alert the nurse to possible side effects? a. Urine output of 160 mL in 4 hours b. Deep tendon reflexes 2+ and no clonus c. Respiratory rate of 16 breaths/min d. Serum magnesium level of 10 mg/dL

D

Nurses should be aware that the induction of labor: a. Can be achieved by external and internal version techniques. b. Is also known as a trial of labor (TOL). c. Is almost always done for medical reasons. d. Is rated for viability by a Bishop score.

D

An Apgar score of 10 at 1 minute after birth would indicate a(n): a. Infant having no difficulty adjusting to extrauterine life and needing no further testing. b. Infant in severe distress who needs resuscitation. c. Prediction of a future free of neurologic problems. d. Infant having no difficulty adjusting to extrauterine life but who should be assessed again at 5 minutes after birth.

D (An initial Apgar score of 10 is a good sign of healthy adaptation; however, it must be repeated at the 5-minute mark.)

When providing an infant with a gavage feeding, which of the following should be documented each time? a. The infant's abdominal circumference after the feeding b. The infant's heart rate and respirations c. The infant's suck and swallow coordination d. The infant's response to the feeding

D (Documentation of a gavage feeding should include the size of the feeding tube, the amount and quality of the residual from the previous feeding, the type and quantity of the fluid instilled, and the infant's response to the procedure. Abdominal circumference is not measured after a gavage feeding. Vital signs may be obtained before feeding. However, the infant's response to the feeding is more important. Some older infants may be learning to suck, but the important factor to document would be the infant's response to the feeding (including attempts to suck).)

In a variation of rooming-in, called couplet care, the mother and infant share a room, and the mother shares the care of the infant with: a. The father of the infant. b. Her mother (the infant's grandmother). c. Her eldest daughter (the infant's sister). d. The nurse.

D (In couplet care the mother shares a room with the newborn and shares infant care with a nurse educated in maternity and infant care.)

Nurses should be able to teach breastfeeding mothers the signs that the infant has latched on correctly. Which statement indicates a poor latch? a. She feels a firm tugging sensation on her nipples but not pinching or pain. b. The baby sucks with cheeks rounded, not dimpled. c. The baby's jaw glides smoothly with sucking. d. She hears a clicking or smacking sound.

D (The clicking or smacking sound may indicate that the baby is having difficulty keeping the tongue out over the lower gum ridge. The mother should hope to hear the sound of swallowing. The tugging sensation without pinching is a good sign. Rounded cheeks are a positive indicator of a good latch. A smoothly gliding jaw is a good sign.)

A primiparous woman is delighted with her newborn son and wants to begin breastfeeding as soon as possible. The nurse can facilitate the infant's correct latch-on by helping the woman hold the infant: a. With his arms folded together over his chest. b. Curled up in a fetal position. c. With his head cupped in her hand. d. With his head and body in alignment.

D (The infant's head and body should be in correct alignment with the mother and the breast during latch-on and feeding. Holding the infant with his arms folded together over his chest, curled up in a fetal position, or with his head cupped in her hand are not ideal positions to facilitate latch-on.)

A macrosomic infant is born after a difficult forceps-assisted delivery. After stabilization the infant is weighed, and the birth weight is 4550 g (9 pounds, 6 ounces). The nurse's most appropriate action is to: a. Leave the infant in the room with the mother. b. Take the infant immediately to the nursery. c. Perform a gestational age assessment to determine whether the infant is large for gestational age. d. Monitor blood glucose levels frequently and observe closely for signs of hypoglycemia.

D (This infant is macrosomic (more than 4000 g) and is at high risk for hypoglycemia. Blood glucose levels should be monitored frequently, and the infant should be observed closely for signs of hypoglycemia. Observation may occur in the nursery or in the mother's room, depending on the condition of the fetus. Regardless of gestational age, this infant is macrosomic.)

A nurse might be called on to stimulate the fetal scalp: A. As part of fetal scalp blood sampling B. In response to tocolysis C. In preparation for fetal oxygen saturation monitoring D. To elicit an acceleration in the FHR

D A. Incorrect: Fetal scalp blood sampling involves swabbing the scalp with disinfectant before a sample is collected. The nurse would stimulate the fetal scalp to elicit an acceleration of the FHR. B. Incorrect: Tocolysis is relaxation of the uterus. The nurse would stimulate the fetal scalp to elicit an acceleration of the FHR. C. Incorrect: Fetal oxygen saturation monitoring involves the insertion of a sensor. The nurse would stimulate the fetal scalp to elicit an acceleration of the FHR. D. Correct: The scalp can be stimulated using digital pressure during a vaginal examination. p. 513

You are evaluating the fetal monitor tracing of your client, who is in active labor. Suddenly, you see the FHR drop from its baseline of 125 down to 80. You reposition the mother, provide oxygen, increase intravenous (IV) fluid, and perform a vaginal exam. The cervix has not changed. Five minutes have passed, and the fetal heart rates remain in the 80s. What additional nursing measures should you take? A. Scream for help B. Insert a Foley catheter C. Start pitocin D. Notify the care provider immediately

D A. Incorrect: This is an inappropriate nursing action. B. Incorrect: If the FHR were to continue in a nonreassuring pattern, a cesarean section may be warranted, which would require a Foley catheter. However, the physician must make that determination. C. Incorrect: Pitocin may put additional stress on the fetus. D. Correct: To relieve an FHR deceleration, the nurse can reposition the mother, increase IV fluid, and provide oxygen. Also, if oxytocin is infusing, it should be discontinued. If the FHR does not resolve, the primary care provider should be notified immediately. p. 511

Parents who have not already done so need to make time for newborn follow-up of the discharge. According to the American Academy of Pediatrics (AAP), when should a breastfeeding infant first need to be seen for a follow-up examination? a. 2 weeks of age b. 7 to 10 days after childbirth c. 4 to 5 days after hospital discharge d. 48 to 72 hours after hospital discharge

D Breastfeeding infants are routinely seen by the pediatric health care provider clinic within 3 to 5 days after birth or 48 to 72 hours after hospital discharge and again at 2 weeks of age. Formula-feeding infants may be seen for the first time at 2 weeks of age.

Because a full bladder prevents the uterus from contracting normally, nurses intervene to help the woman spontaneously empty her bladder as soon as possible. If all else fails, what tactic might the nurse use? a. Pouring water from a squeeze bottle over the womans perineum b. Placing oil of peppermint in a bedpan under the woman c. Asking the physician to prescribe analgesic agents d. Inserting a sterile catheter

D Invasive procedures are usually the last to be tried, especially with so many other simple and easy methods available (e.g., water, peppermint vapors, pain pills). Pouring water over the perineum may stimulate voiding. It is easy, noninvasive, and should be tried first. The oil of peppermint releases vapors that may relax the necessary muscles. It, too, is easy, noninvasive, and should be tried early on. If the woman is anticipating pain from voiding, then pain medications may be helpful. Other nonmedical means should be tried first, but medications still come before the insertion of a catheter.

The trend in the United States is for women to remain hospitalized no longer than 1 or 2 days after giving birth. Which scenario is not a contributor to this model of care? a. Wellness orientation model of care rather than a sick-care model b. Desire to reduce health care costs c. Consumer demand for fewer medical interventions and more family-focused experiences d. Less need for nursing time as a result of more medical and technologic advances and devices available at home that can provide information

D Nursing time and care are in demand as much as ever; the nurse simply has to do things more quickly. A wellness orientation model of care seems to focus on getting clients out the door sooner. In most cases, less hospitalization results in lower costs. People believe that the family gives more nurturing care than the institution.

The nurse should realize that the most common and potentially harmful maternal complication of epidural anesthesia is: A. Severe postpartum headache. B. Limited perception of bladder fullness. C. Increase in respiratory rate. D. Hypotension.

D. Hypotension. Headache is not a side effect since the spinal fluid is not disturbed by this anesthetic because it would be with a low spinal (saddle block) anesthetic. Limited perception of bladder fullness is an effect of epidural anesthesia but is not the most harmful. Respiratory depression is a potentially serious complication. Epidural anesthesia can lead to vasodilation and a drop in blood pressure that could interfere with adequate placental perfusion. The woman must be well hydrated before and during epidural anesthesia to prevent this problem and maintain an adequate blood pressure.

C (Desmopressin is the primary treatment of choice. This hormone can be administered orally, nasally, and intravenously. This medication promotes the release of factor VIII and vWf from storage.)

Despite popular belief, there is a rare type of hemophilia that affects women of childbearing age. Von Willebrand disease is the most common of the hereditary bleeding disorders and can affect males and females alike. It results from a factor VIII deficiency and platelet dysfunction. Although factor VIII levels increase naturally during pregnancy, there is an increased risk for postpartum hemorrhage from birth until 4 weeks postdelivery as levels of von Willebrand factor (vWf) and factor VIII decrease. The treatment that should be considered first for the client with von Willebrand disease who experiences a postpartum hemorrhage is: A. Cryoprecipitate B. Factor VIII and vWf C. Desmopressin D. Hemabate

D (Fetal kick count is a simple method to determine the presence of complications related to fetal oxygenation and activity level. The fetal kick count during the third trimester of pregnancy is approximately 30 kicks an hour; a count lower than that is an indication of poor health of the fetus. Fetal anomalies may not affect the oxygenation levels of the fetus. The nurse already knows the gestational age of the fetus; therefore the nurse need not refer the woman for ultrasonography to find the gestational age. Fetal position does not affect the activity level of the fetus.)

During a prenatal checkup, the patient who is 7 months pregnant reports that she is able to feel about two kicks in an hour. The nurse refers the patient for an ultrasound. What is the primary reason for this referral? To check: A. For fetal anomalies B. Gestational age C. Fetal position D. For fetal well-being

C (A parent that is grieving over a recent loss (in the process of detachment) will have the most difficulty bonding with the new baby. Knowledge of parent-infant attachment or being an only child are not related to successful bonding. A job loss does not have the impact that death of a family member does.)

During the early postpartum period, the nurse is evaluating a client's attachment to her neonate. Which type of parent has the most difficulty attaching to her newborn? A. One who has little knowledge of parent-infant attachment B. One who recently lost a job C. One whose father recently died D. One who is an only child

C (Fetal development concerns are more apparent in the second trimester when the woman is feeling fetal movement. Impact of a new baby on the family would be appropriate topics for the second trimester when the fetus becomes "real" as its movements are felt and its heartbeat heard. During this trimester a woman works on the task of, "I am going to have a baby." During the first trimester a woman is egocentric and concerned about how she feels. She is working on the task of accepting her pregnancy. Motivation to learn about childbirth techniques and breastfeeding is greatest for most women during the third trimester as the reality of impending birth and becoming a parent is accepted. A goal is to achieve a safe passage for herself and her baby.)

During the first trimester the pregnant woman would be most motivated to learn about: A. Fetal development. B. Impact of a new baby on family members. C. Measures to reduce nausea and fatigue so she can feel better. D. Location of childbirth preparation and breastfeeding classes

B (Maternal physiologic changes such as breast enlargement, nausea, fatigue, abdominal changes, perineal enlargement, leukorrhea, pelvic vasocongestion, and orgasmic responses may affect sexuality and sexual expression. Libido may be depressed in the first trimester but often increases during the second and third trimesters. During pregnancy, the breasts may become enlarged and tender; this tends to interfere with coitus, decreasing the desire to engage in sexual activity.)

During the first trimester, a woman can expect which of the following changes in her sexual desire? A. An increase, because of enlarging breasts B. A decrease, because of nausea and fatigue C. No change D. An increase, because of increased levels of female hormones

Chinese

Have an intrauterine device (IUD) inserted after the first child.

B (This is an expected view for an expectant father. Persistent refusal to talk about the fetus-newborn may be a sign of a problem and should be assessed further. This is an expected feeling for an expectant father. This is an expected finding with expectant fathers.)

If exhibited by an expectant father, what would be a warning sign of ineffective adaptation to his partner's first pregnancy? A. Views pregnancy with pride as a confirmation of his virility B. Consistently changes the subject when the topic of the fetus/newborn is raised C. Expresses concern that he might faint at the birth of his baby D. Experiences nausea and fatigue, along with his partner, during the first trimester

D (D&C allows examination of the uterine contents and removal of any retained placenta or membranes.)

If nonsurgical treatment for subinvolution is ineffective, which surgical procedure is appropriate to correct the cause of this condition? a. Hysterectomy b. Laparoscopy c. Laparotomy d. D&C

C (During the first 10 to 12 days after childbirth, values between 20,000 and 25,000/mm are common. Because this is a normal finding there is no reason to alert the physician. There is no need for reassessment or antibiotics because it is expected for the WBCs to be elevated.)

If the patient's white blood cell (WBC) count is 25,000/mm on her second postpartum day, the nurse should: A. Tell the physician immediately. B. Have the laboratory draw blood for reanalysis. C. Recognize that this is an acceptable range at this point postpartum. D. Begin antibiotic therapy immediately.

A (Washing both the infant's face and the mother's face.)

In the United States, the en face position is preferred immediately after birth. Nurses can facilitate this process by all of these actions except: A. Washing both the infant's face and the mother's face B. Placing the infant on the mother's abdomen or breast with their heads on the same plane C. Dimming the lights D. Delaying the instillation of prophylactic antibiotic ointment in the infant's eyes

A, B, C, E (Poor nutrition, maternal collagen disease, gestational hypertension, and smoking all are risk factors associated with IUGR. Premature rupture of membranes is associated with preterm labor, not IUGR.)

Intrauterine growth restriction (IUGR) is associated with numerous pregnancy-related risk factors (Select all that apply). A. Poor nutrition B. Maternal collagen disease C. Gestational hypertension D. Premature rupture of membranes E. Smoking

B (Infants can learn to distinguish their mother's voice from others soon after birth.)

Other early sensual contacts between infant and mother involve sound and smell. Nurses should be aware that despite what folk wisdom might say: A. High-pitched voices irritate newborns. B. Infants can learn to distinguish their mother's voice from others soon after birth. C. All babies in the hospital smell alike. D. A mother's breast milk has no distinctive odor.

A (The nails return to their prepregnancy consistency and strength. Some women have permanent darker pigmentation of the areolae and linea nigra. Striae gravidarum (stretch marks) usually do not completely disappear. For some women spider nevi persist indefinitely.)

Several changes in the integumentary system that appear during pregnancy disappear after birth, although not always completely. What change is almost certain to be completely reversed? A. Nail brittleness B. Darker pigmentation of the areolae and linea nigra C. Striae gravidarum on the breasts, abdomen, and thighs D. Spider nevi

Haitian

Take the placenta home to bury.

B (Lochia and infectious material are eliminated by gravity drainage.)

The client who is being treated for endometritis is placed in Fowler's position because it: a. promotes comfort and rest. b. facilitates drainage of lochia. c. prevents spread of infection to the urinary tract. d. decreases tension on the reproductive organs.

C (The initial management of excessive postpartum bleeding is firm massage of the uterine fundus.)

The first and most important nursing intervention when a nurse observes profuse postpartum bleeding is to: A. Call the woman's primary health care provider B. Administer the standing order for an oxytocic C. Palpate the uterus and massage it if it is boggy D. Assess maternal blood pressure and pulse for signs of hypovolemic shock

C (Tenderness, heat, and swelling are classic signs of thrombophlebitis that appear at the site of the inflammation.)

The mother-baby nurse must be able to recognize what sign of thrombophlebitis? a. Visible varicose veins b. Positive Homans' sign c. Local tenderness, heat, and swelling d. Pedal edema in the affected leg

D (A firm, muscular wall with less adipose tissue would ensure that the patient is able to regain the prepregnancy abdominal tone after delivery. Thus the nurse should advise the patient to do static abdominal exercises during pregnancy. The abdominal tone is not a factor based on which the nurse can determine whether the patient would have a normal vaginal delivery. Patients with weak abdominal muscles, especially those who have multifetal gestation or a large fetus, are at the risk of having diastasis recti abdominis. These abdominal striations usually do not fade away completely. Although the abdominal skin retains its tone, some striae always remain.)

The nurse advises a pregnant patient to do static abdominal exercises. How would these exercises benefit the patient? A. They will lead to a normal vaginal childbirth. B. The patient will have diastasis recti abdominis. C. The patient will not have any abdominal striations. D. They will help the patient to gain proper abdominal tone after delivery.

D (Most nurse-midwife births are managed in hospitals or birth centers; a few may be managed in the home. Nurse-midwives may practice with physicians or independently with an arrangement for physician backup. They must refer clients to physicians for complications. Care in a midwifery model is noninterventional, and the woman and family usually are encouraged to be active participants in the care. This does not imply that medications for pain control are prohibited. Midwives usually see low-risk obstetric clients. Care is often noninterventional with active involvement from the woman and her family. Nurse-midwives must refer clients to physicians for complications.)

The nurse advises the woman who wants to have a nurse-midwife provide obstetric care that: A. She will have to give birth at home. B. She must see an obstetrician as well as the midwife during pregnancy. C. She will not be able to have epidural analgesia for labor pain. D. She must be having a low-risk pregnancy.

A (Pointing out that the infant turned at the sound of his voice)

The nurse can help a father in his transition to parenthood by: A. Pointing out that the infant turned at the sound of his voice B. Encouraging him to go home to get some sleep C. Telling him to tape the infant's diaper a different way D. Suggesting that he let the infant sleep in the bassinet

A (The visual system continues to develop for the first 6 months. As soon as the amniotic fluid drains from the ear (minutes), the infant's hearing is similar to that of an adult. Newborns have a highly developed sense of smell. The newborn can distinguish and react to various tastes.)

The nurse caring for the newborn should be aware that the sensory system least mature at the time of birth is: a. Vision. b. Hearing. c. Smell. d. Taste.

D (Breast engorgement is caused by the temporary congestion of veins and lymphatics, not by overproduction of colostrum, overproduction of milk, or hyperplasia of mammary tissue.)

The nurse caring for the postpartum woman understands that breast engorgement is caused by: A. Overproduction of colostrum. B. Accumulation of milk in the lactiferous ducts. C. Hyperplasia of mammary tissue. D. Congestion of veins and lymphatics.

D, E (Internal hemorrhoids can evert while the woman is pushing the baby during childbirth. The patient has internal hemorrhoids due to pregnancy. During pregnancy, pressure from the fetus on the abdomen, as well as hormonal changes, cause hemorrhoids to enlarge. Delivery also leads to increased intraabdominal pressures, which result in internal hemorrhoids. An itching sensation and bleeding upon defecation are the symptoms of internal hemorrhoids. Hemorrhoids usually decrease in size within 6 weeks of childbirth. Abdominal tenderness is a sign of endometritis. White or yellow uterine discharge is a normal finding after delivery.)

The nurse is caring for a postpartum patient and finds that the patient has internal hemorrhoids. Which clinical manifestations would be consistent with the nurse's findings? Select all that apply. A. Abdominal tenderness B. White uterine discharge C. Yellow uterine discharge D. Bleeding upon defecation E. Itching sensation in the anus

C (No late decelerations is good news. Vibroacoustic stimulation is sometimes used with NST. CST is invasive if stimulation is by intravenous oxytocin but not if by nipple stimulation and is contraindicated if the membranes have ruptured.)

The nurse providing care for the antepartum woman should understand that contraction stress test (CST): A. Sometimes uses vibroacoustic stimulation. B. Is an invasive test; however, contractions are stimulated. C. Is considered negative if no late decelerations are observed with the contractions. D. Is more effective than nonstress test (NST) if the membranes have already been ruptured.

D (The NST is reactive (normal) when two or more FHR accelerations of at least 15 beats/min (each with a duration of at least 15 seconds) occur in a 20-minute period. A nonreactive result means that the heart rate did not accelerate during fetal movement. A positive result is not used with NST. Contraction stress test (CST) uses positive as a result term. A negative result is not used with NST. CST uses negative as a result term.)

The nurse recognizes that a nonstress test (NST) in which two or more fetal heart rate (FHR) accelerations of 15 beats/min or more occur with fetal movement in a 20-minute period is: A. Nonreactive B. Positive C. Negative D. Reactive

A (Doppler blood flow analysis allows the examiner to study the blood flow noninvasively in the fetus and the placenta. It is a helpful tool in the management of high risk pregnancies because of intrauterine growth restriction (IUGR), diabetes mellitus, multiple fetuses, or preterm labor. Because of the potential risk of inducing labor and causing fetal distress, CST is not performed on a woman whose fetus is preterm. Indications for amniocentesis include diagnosis of genetic disorders or congenital anomalies, assessment of pulmonary maturity, and diagnosis of fetal hemolytic disease, not IUGR. Fetal kick count monitoring is performed to monitor the fetus in pregnancies complicated by conditions that may affect fetal oxygenation. Although this may be a useful tool at some point later in this woman's pregnancy, it is not used to diagnose IUGR.)

The nurse sees a woman for the first time when she is 30 weeks pregnant. The woman has smoked throughout the pregnancy, and fundal height measurements now are suggestive of growth restriction in the fetus. In addition to ultrasound to measure fetal size, what other tool would be useful in confirming the diagnosis? A. Doppler blood flow analysis B. Contraction stress test (CST) C. Amniocentesis D. Daily fetal movement counts

C (The partner's main role in pregnancy is to nurture the pregnant woman and respond to her feelings of vulnerability. In older societies, the man enacted the ritual couvade. Changing cultural and professional attitudes have encouraged fathers' participation in the birth experience over the past 30 years.)

The nurse should be aware that the partner's main role in pregnancy is to: A. Provide financial support. B. Protect the pregnant woman from "old wives' tales." C. Support and nurture the pregnant woman. D. Make sure the pregnant woman keeps prenatal appointments.

A (Methergine provides long-sustained contraction of the uterus.)

The nurse should expect medical intervention for subinvolution to include: a. oral methylergonovine maleate (Methergine) for 48 hours. b. oxytocin intravenous infusion for 8 hours. c. oral fluids to 3000 mL/day. d. intravenous fluid and blood replacement.

B (The pinch test is used to determine whether the nipple is everted or inverted. Nipples must be everted to allow breastfeeding.)

The nurse should have knowledge of the purpose of the pinch test. It is used to: A. Check the sensitivity of the nipples. B. Determine whether the nipple is everted or inverted. C. Calculate the adipose buildup in the abdomen. D. See whether the fetus has become inactive.

D (An expectant father's experiencing pregnancy-like symptoms is called the couvade syndrome.)

The phenomenon of someone other than the mother-to-be experiencing pregnancy-like symptoms such as nausea and weight gain applies to the: A. Mother of the pregnant woman. B. Sister of the pregnant woman. C. Couple's teenage daughter. D. Expectant father.

B (Conjugation of bilirubin is the process of changing the bilirubin from a fat-soluble to a water-soluble product. This is the route by which part of the bile produced by the liver enters the intestine, is reabsorbed by the liver, and then is recycled into the intestine. Unconjugated bilirubin is fat soluble. Albumin binding is to attach something to a protein molecule.)

The process in which bilirubin is changed from a fat-soluble product to a water-soluble product is known as: a. Enterohepatic circuit. b. Conjugation of bilirubin. c. Unconjugation of bilirubin. d. Albumin binding.

C (Mastitis is infection in a breast, usually confined to a milk duct. Most women who suffer this are first-timers who are breastfeeding.)

What infection is contracted mostly by first-time mothers who are breastfeeding? A. Endometritis B. Wound infections C. Mastitis D. Urinary tract infections

B (A concern for modesty is a deterrent to many women seeking prenatal care. For some women, exposing body parts, especially to a man, is considered a major violation of their modesty. Many cultural variations are found in prenatal care. Even if the prenatal care described is familiar to a woman, some practices may conflict with the beliefs and practices of a subculture group to which she belongs.)

What type of cultural concern is the most likely deterrent to many women seeking prenatal care? A. Religion B. Modesty C. Ignorance D. Belief that physicians are evil

A (Mutuality)

When the infant's behavior and characteristics call forth a corresponding set of maternal behaviors and characteristics, this is called: a) Mutuality b) Bonding c) Claiming d) Acquaintance

A (Maternal serum can be analyzed for abnormal levels of alpha-fetoprotein, human chorionic gonadotropin, and estriol. The multiple-marker screening may predict chromosomal defects in the fetus. The L/S ratio is used to determine fetal lung maturity. A biophysical profile is used for evaluating fetal status during the antepartum period. Five variables are used, but none is concerned with chromosomal problems. The blood type and crossmatch would not predict chromosomal defects in the fetus.)

Which analysis of maternal serum may predict chromosomal abnormalities in the fetus? A. Multiple-marker screening B. Lecithin/sphingomyelin (L/S) ratio C. Biophysical profile D. Type and crossmatch of maternal and fetal serum

A (The goal of prenatal care is to foster a safe birth for the infant and mother. Although eating properly, driving carefully, and using proper body mechanics all are healthy measures that a mother can take, obtaining prenatal care is the optimal method for providing safety for both herself and her baby)

Which behavior indicates that a woman is "seeking safe passage" for herself and her infant? A. She keeps all prenatal appointments. B. She drives her car slowly. C. She "eats for two." D. She wears only low-heeled shoes.

D (Postpartum blues affects 50% to 70% of new mothers. It is believed to be related to hormonal fluctuations after childbirth.)

Which condition is a transient, self-limiting mood disorder that affects new mothers after childbirth? a. Postpartum depression b. Postpartum psychosis c. Postpartum bipolar disorder d. Postpartum blues

D (Headaches in the postpartum period can have a number of causes, some of which deserve medical attention. Total or nearly total regression of varicosities is expected after childbirth. Carpal tunnel syndrome is relieved in childbirth when the compression on the median nerve is lessened. Periodic numbness of the fingers usually disappears after birth unless carrying the baby aggravates the condition.)

Which condition, not uncommon in pregnancy, is likely to require careful medical assessment during the puerperium? A. Varicosities of the legs B. Carpal tunnel syndrome C. Periodic numbness and tingling of the fingers D. Headaches

B (Barrier methods such as condoms best protect against STIs and HIV. Periodic abstinence and hormonal methods ("the pill") offer no protection against STIs or HIV.)

Which contraceptive method best protects against sexually transmitted infections (STIs) and human immunodeficiency virus (HIV)? a. Periodic abstinence b. Barrier methods c. Hormonal methods d. They all offer about the same protection.

D (While no family follows just one pattern, in American culture the paternal grandparents are typically considered secondary to the maternal grandparents after the baby is born. This is because a woman may be closer to her own parents and feel more comfortable accepting their help than that of her partner's family. As a result, paternal grandparents may be naturally less involved with the newborn. There are no cultural rules in the American culture dictating that paternal grandparents care for the second-born child or bond with the baby only after 6 months. Grandparents are not typically responsible for the financial expenses of the baby in the United States.)

While caring for a patient, the nurse notices that the newborn's paternal grandparents visit the newborn and leave after an hour. The maternal grandparents, on the other hand, stay all day with the couple to look after the newborn. What could be the reason for this in terms of traditional paternal grandparents' cultural behaviors? A. They care for the second born child. B. Typically, they are called to care for baby after 6 months. C. They feel that they are responsible only for the financial expenses of the child. D. They do not feel responsible for the care of the newborn or new mother.

B (Children may feel insecure when parents pay more attention to a newborn sibling. The older child may develop feelings of sibling rivalry and exhibit negative behaviors, such as increased attention seeking and aggression toward the baby. Therefore trying to grab the attention of the parents indicates that the child has not adapted well to the newborn. Positive behaviors, such as increased interest and concern for the baby, indicate effective adaptation. Therefore expressing a desire to stay with the mother and newborn in hospital, watching the baby silently, and being willing to feed the baby does not indicate a lack of adaptation.)

While caring for a postpartum patient, the nurse observes that the patient's older child is not adapting well to the newborn and may have feelings of sibling rivalry. What behavior of the older child would make the nurse conclude this? A. Watches the baby silently in the hospital B. Always tries to grab the parents' attention C. Always wants to be with the mother and newborn D. Asks to feed the newborn with a milk bottle

A, D (A woman is unlikely to become pregnant after tubal ligation, although it is not 100% effective. It does not offer any protection against STIs. Sterilization offers no protection against STIs. Typically the menstrual cycle remains the same after a tubal ligation.)

You (the nurse) are reviewing the educational packet provided to a client about tubal ligation. What is an important fact you should point out? Choose all that apply. a. "It is highly unlikely that you will become pregnant after the procedure." b. "This is an effective form of 100% permanent sterilization. You won't be able to get pregnant." c. "Sterilization offers some form of protection against sexually transmitted infections (STIs)." d. "Sterilization offers no protection against STIs." e. "Your menstrual cycle will greatly increase after your sterilization."

A woman asks the nurse, "What protects my baby's umbilical cord from being squashed while the baby's inside of me?" The nurse's best response is: a. "Your baby's umbilical cord is surrounded by connective tissue called Wharton jelly, which prevents compression of the blood vessels and ensures continued nourishment of your baby." b. "Your baby's umbilical floats around in blood anyway." c. "You don't need to worry about things like that." d. "The umbilical cord is a group of blood vessels that are very well protected by the placenta."

a. "Your baby's umbilical cord is surrounded by connective tissue called Wharton jelly, which prevents compression of the blood vessels and ensures continued nourishment of your baby." "Your baby's umbilical cord is surrounded by connective tissue called Wharton jelly, which prevents compression of the blood vessels and ensures continued nourishment of your baby" is the most appropriate response. "Your baby's umbilical floats around in blood anyway" is inaccurate. "You don't need to worry about things like that" is not appropriate response. It negates the client's need for teaching and discounts her feelings. The placenta does not protect the umbilical cord. The cord is protected by the surrounding Wharton jelly.

The nurse is assessing the knowledge of new parents with a child born with maple syrup urine disease (MSUD). This is an autosomal recessive inherited disorder, which means that: a. Both genes of a pair must be abnormal for the disorder to be expressed. b. Only one copy of the abnormal gene is required for the disorder to be expressed. c. The disorder occurs in males and heterozygous females. d. The disorder is carried on the X chromosome.

a. Both genes of a pair must be abnormal for the disorder to be expressed. MSUD is a type of autosomal recessive inheritance disorder in which both genes of a pair must be abnormal for the disorder to be expressed. MSUD is not an X-linked dominant or recessive disorder or an autosomal dominant inheritance disorder.

13. Appendicitis may be difficult to diagnose in pregnancy because the appendix is: a. Displaced upward and laterally, high and to the right. b. Displaced upward and laterally, high and to the left. c. Deep at McBurney point. d. Displaced downward and laterally, low and to the right.

a. Displaced upward and laterally, high and to the right.

39. Fibrocystic changes in the breast most often appear in women in their 20s and 30s. The etiology is unknown, but it may be an imbalance of estrogen and progesterone. The nurse who cares for this client should be aware that treatment modalities are conservative. One proven modality that may provide relief is: a. Diuretic administration. b. Including caffeine daily in the diet. c. Increased vitamin C supplementation. d. Application of cold packs to the breast as necessary.

a. Diuretic administration.

20. A woman has a thick, white, lumpy, cottage cheese-like discharge, with patches on her labia and in her vagina. She complains of intense pruritus. The nurse practitioner would order which preparation for treatment? a. Fluconazole b. Tetracycline c. Clindamycin d. Acyclovir

a. Fluconazole

The nurse must be cognizant that an individual's genetic makeup is known as his or her: a. Genotype. b. Phenotype. c. Karyotype. d. Chromotype.

a. Genotype. The genotype comprises all the genes the individual can pass on to a future generation. The phenotype is the observable expression of an individual's genotype. The karyotype is a pictorial analysis of the number, form, and size of an individual's chromosomes. Genotype refers to an individual's genetic makeup.

16. Compared with contraction stress test (CST), nonstress test (NST) for antepartum fetal assessment: a. Has no known contraindications. b. Has fewer false-positive results. c. Is more sensitive in detecting fetal compromise. d. Is slightly more expensive.

a. Has no known contraindications.

5. During a client's physical examination the nurse notes that the lower uterine segment is soft on palpation. The nurse would document this finding as: a. Hegar's sign b. McDonald's sign c. Chadwick's sign d. Goodell's sign

a. Hegar's sign

25. Which viral sexually transmitted infection is characterized by a primary infection followed by recurrent episodes? a. Herpes simplex virus (HSV)-2 b. Human papillomavirus (HPV) c. Human immunodeficiency virus (HIV) d. Cytomegalovirus (CMV)

a. Herpes simplex virus (HSV)-2

15. Several delivery changes in the integumentary system that appear during pregnancy disappear after birth, although not always completely. What change is almost certain to be completely reversed? a. Nail brittleness b. Darker pigmentation of the areolae and linea nigra c. Striae gravidarum on the breasts, abdomen, and thighs d. Spider nevi

a. Nail brittleness

34. A patient has been prescribed adjuvant tamoxifen therapy. What common side effect might she experience? a. Nausea, hot flashes, and vaginal bleeding b. Vomiting, weight loss, and hair loss c. Nausea, vomiting, and diarrhea d. Hot flashes, weight gain, and headaches

a. Nausea, hot flashes, and vaginal bleeding

14. A woman who has completed one pregnancy with a fetus (or fetuses) reaching the stage of fetal viability is called a: a. Primipara. b. Primigravida. c. Multipara. d. Nulligravida.

a. Primipara.

29. An essential component of counseling women regarding safe sex practices includes discussion regarding avoiding the exchange of body fluids. The physical barrier promoted for the prevention of sexually transmitted infections and human immunodeficiency virus is the condom. Nurses can help motivate clients to use condoms by initiating a discussion related to a number of aspects of condom use. The most important of these is: a. Strategies to enhance condom use. b. Choice of colors and special features. c. Leaving the decision up to the male partner. d. Places to carry condoms safely.

a. Strategies to enhance condom use.

8. A woman gave birth to a 7-pound, 6-ounce infant girl 1 hour ago. The birth was vaginal and the estimated blood loss (EBL) was 1500 ml. When evaluating the woman's vital signs, which finding would be of greatest concern to the nurse? a. Temperature 37.9° C, heart rate 120 beats per minute (bpm), respirations 20 breaths per minute, and blood pressure 90/50 mm Hg b. Temperature 37.4° C, heart rate 88 bpm, respirations 36 breaths per minute, and blood pressure 126/68 mm Hg c. Temperature 38° C, heart rate 80 bpm, respirations 16 breaths per minute, and blood pressure 110/80 mm Hg d. Temperature 36.8° C, heart rate 60 bpm, respirations 18 breaths per minute, and blood pressure 140/90 mm Hg

a. Temperature 37.9° C, heart rate 120 beats per minute (bpm), respirations 20 breaths per minute, and blood pressure 90/50 mm Hg

37. What important, immediate postoperative care practice should the nurse remember when caring for a woman who has had a mastectomy? a. The blood pressure (BP) cuff should not be applied to the affected arm. b. Venipuncture for blood work should be performed on the affected arm. c. The affected arm should be used for intravenous (IV) therapy. d. The affected arm should be held down close to the woman's side.

a. The blood pressure (BP) cuff should not be applied to the affected arm.

10. A woman is in her seventh month of pregnancy. She has been complaining of nasal congestion and occasional epistaxis. The nurse suspects that: a. This is a normal respiratory change in pregnancy caused by elevated levels of estrogen. b. This is an abnormal cardiovascular change, and the nosebleeds are an ominous sign. c. The woman is a victim of domestic violence and is being hit in the face by her partner. d. The woman has been using cocaine intranasally.

a. This is a normal respiratory change in pregnancy caused by elevated levels of estrogen.

30. A patient in her first trimester complains of nausea and vomiting. She asks, "Why does this happen?" The nurse's best response is: a. "It is due to an increase in gastric motility." b. "It may be due to changes in hormones." c. "It is related to an increase in glucose levels." d. "It is caused by a decrease in gastric secretions."

b. "It may be due to changes in hormones."

1. A woman's obstetric history indicates that she is pregnant for the fourth time and all of her children from previous pregnancies are living. One was born at 39 weeks of gestation, twins were born at 34 weeks of gestation, and another child was born at 35 weeks of gestation. What is her gravidity and parity using the GTPAL system? a. 3-1-1-1-3 b. 4-1-2-0-4 c. 3-0-3-0-3 d. 4-2-1-0-3

b. 4-1-2-0-4

4. A 41-week pregnant multigravida presents in the labor and delivery unit after a nonstress test indicated that her fetus could be experiencing some difficulties in utero. Which diagnostic tool would yield more detailed information about the fetus? a. Ultrasound for fetal anomalies b. Biophysical profile (BPP) c. Maternal serum alpha-fetoprotein (MSAFP) screening d. Percutaneous umbilical blood sampling (PUBS)

b. Biophysical profile (BPP)

29. Physiologic anemia often occurs during pregnancy as a result of: a. Inadequate intake of iron. b. Dilution of hemoglobin concentration. c. The fetus establishing iron stores. d. Decreased production of erythrocytes.

b. Dilution of hemoglobin concentration.

3. The nurse teaches a pregnant woman about the presumptive, probable, and positive signs of pregnancy. The woman demonstrates understanding of the nurse's instructions if she states that a positive sign of pregnancy is: a. A positive pregnancy test. b. Fetal movement palpated by the nurse-midwife. c. Braxton Hicks contractions. d. Quickening.

b. Fetal movement palpated by the nurse-midwife.

21. To detect human immunodeficiency virus (HIV), most laboratory tests focus on the: a. virus. b. HIV antibodies. c. CD4 counts. d. CD8 counts.

b. HIV antibodies.

28. Which of the following statements about the various forms of hepatitis is accurate? a. A vaccine exists for hepatitis C but not for hepatitis B. b. Hepatitis A is acquired by eating contaminated food or drinking polluted water. c. Hepatitis B is less contagious than human immunodeficiency virus (HIV). d. The incidence of hepatitis C is decreasing.

b. Hepatitis A is acquired by eating contaminated food or drinking polluted water.

18. The viral sexually transmitted infection (STI) that affects most people in the United States today is: a. Herpes simplex virus type 2 (HSV-2). b. Human papillomavirus (HPV). c. Human immunodeficiency virus (HIV). d. Cytomegalovirus (CMV).

b. Human papillomavirus (HPV).

You are a maternal-newborn nurse caring for a mother who just delivered a baby born with Down syndrome. What nursing diagnosis would be the most essential in caring for the mother of this infant? a. Disturbed body image b. Interrupted family processes c. Anxiety d. Risk for injury

b. Interrupted family processes This mother likely will experience a disruption in the family process related to the birth of a baby with an inherited disorder. Women commonly experience body image disturbances in the postpartum period, but this is unrelated to giving birth to a child with Down syndrome. The mother likely will have a mix of emotions that may include anxiety, guilt, and denial, but this is not the most essential nursing diagnosis for this family. Risk for injury is not an applicable nursing diagnosis.

20. Risk factors tend to be interrelated and cumulative in their effect. While planning the care for a laboring client with diabetes mellitus, the nurse is aware that she is at a greater risk for: a. Oligohydramnios. b. Polyhydramnios. c. Postterm pregnancy. d. Chromosomal abnormalities.

b. Polyhydramnios.

28. The maternity nurse understands that vascular volume increases 40% to 60% during pregnancy to: a. Compensate for decreased renal plasma flow. b. Provide adequate perfusion of the placenta. c. Eliminate metabolic wastes of the mother. d. Prevent maternal and fetal dehydration.

b. Provide adequate perfusion of the placenta.

A man's wife is pregnant for the third time. One child was born with cystic fibrosis, and the other child is healthy. The man wonders what the chance is that this child will have cystic fibrosis. This type of testing is known as: a. Occurrence risk. b. Recurrence risk. c. Predictive testing. d. Predisposition testing.

b. Recurrence risk. The couple already has a child with a genetic disease; therefore they will be given a recurrence risk test. If a couple has not yet had children but are known to be at risk for having children with a genetic disease, they are given an occurrence risk test. This couple already has a child with a genetic disorder. Predictive testing is used to clarify the genetic status of an asymptomatic family member. Predisposition testing differs from presymptomatic testing in that a positive result does not indicate 100% risk of a condition developing.

6. While interviewing a 31-year-old woman before her routine gynecologic examination, the nurse collects data about the client's recent menstrual cycles. The nurse should collect additional information with which statement? a. The woman says her menstrual flow lasts 5 to 6 days. b. She describes her flow as very heavy. c. She reports that she has had a small amount of spotting midway between her periods for the past 2 months. d. She says the length of her menstrual cycle varies from 26 to 29 days.

b. She describes her flow as very heavy.

8. The musculoskeletal system adapts to the changes that occur during pregnancy. A woman can expect to experience what change? a. Her center of gravity will shift backward. b. She will have increased lordosis. c. She will have increased abdominal muscle tone. d. She will notice decreased mobility of her pelvic joints.

b. She will have increased lordosis.

4. A woman is at 14 weeks of gestation. The nurse would expect to palpate the fundus at which level? a. Not palpable above the symphysis at this time b. Slightly above the symphysis pubis c. At the level of the umbilicus d. Slightly above the umbilicus

b. Slightly above the symphysis pubis

17. To reassure and educate pregnant clients about changes in the uterus, nurses should be aware that: a. Lightening occurs near the end of the second trimester as the uterus rises into a different position. b. The woman's increased urinary frequency in the first trimester is the result of exaggerated uterine antireflexion caused by softening. c. Braxton Hicks contractions become more painful in the third trimester, particularly if the woman tries to exercise. d. The uterine souffle is the movement of the fetus.

b. The woman's increased urinary frequency in the first trimester is the result of exaggerated uterine antireflexion caused by softening.

19. The U.S. Centers for Disease Control and Prevention (CDC) recommends that HPV be treated with client-applied: a. Miconazole ointment. b. Topical podofilox 0.5% solution or gel. c. Penicillin given intramuscularly for two doses. d. Metronidazole by mouth.

b. Topical podofilox 0.5% solution or gel.

7. Maternal serum alpha-fetoprotein (MSAFP) screening indicates an elevated level. MSAFP screening is repeated and again is reported as higher than normal. What would be the next step in the assessment sequence to determine the well-being of the fetus? a. Percutaneous umbilical blood sampling (PUBS) b. Ultrasound for fetal anomalies c. Biophysical profile (BPP) for fetal well-being d. Amniocentesis for genetic anomalies

b. Ultrasound for fetal anomalies

19. Which description of postpartum restoration or healing times is accurate? a. The cervix shortens, becomes firm, and returns to form within a month postpartum. b. Vaginal rugae reappear by 3 weeks postpartum. c. Most episiotomies heal within a week. d. Hemorrhoids usually decrease in size within 2 weeks of childbirth.

b. Vaginal rugae reappear by 3 weeks postpartum.

12. In the first trimester, ultrasonography can be used to gain information on: a. Amniotic fluid volume. b. the presence of maternal abnormalities c. Placental location and maturity. d. Cervical length.

b. the presence of maternal abnormalities

The development of a strong emotional tie of a parent to a newborn is called _____________.

bonding

A couple has been counseled for genetic anomalies. They ask you, "What is karyotyping?" Your best response is: a. "Karyotyping will reveal if the baby's lungs are mature." b. "Karyotyping will reveal if your baby will develop normally." c. "Karyotyping will provide information about the gender of the baby and the number and structure of the chromosomes." d. "Karyotyping will detect any physical deformities the baby has."

c. "Karyotyping will provide information about the gender of the baby and the number and structure of the chromosomes." Karyotyping provides genetic information such as gender and chromosome structure. The L/S, not karyotyping, reveals lung maturity. Although karyotyping can detect genetic anomalies, the range of normal is nondescriptive. Although karyotyping can detect genetic anomalies, not all such anomalies display obvious physical deformities. The term deformities is a nondescriptive word. Furthermore, physical anomalies may be present that are not detected by genetic studies (e.g., cardiac malformations).

The measurement of lecithin in relation to sphingomyelin (L/S ratio) is used to determine fetal lung maturity. Which ratio reflects maturity of the lungs? a. 1.4:1 b. 1.8:1 c. 2:1 d. 1:1

c. 2:1 A ratio of 2:1 indicates a two-to-one ratio of L/S, an indicator of lung maturity. Ratios of 1.4:1, 1.8:1, and 1:1 indicate immaturity of the fetal lungs.

26. The nurse should know that once human immunodeficiency virus (HIV) enters the body, seroconversion to HIV positivity usually occurs within: a. 6 to 10 days. b. 2 to 4 weeks. c. 6 to 8 weeks. d. 6 months.

c. 6 to 8 weeks.

42. Which patient is most at risk for fibroadenoma of the breast? a. A 38-year-old woman b. A 50-year-old woman c. A 16-year-old girl d. A 27-year-old woman

c. A 16-year-old girl

43. The drug of choice for treatment of gonorrhea is: a. Penicillin G. b. Tetracycline. c. Ceftriaxone. d. Acyclovir.

c. Ceftriaxone.

17. When evaluating a patient for sexually transmitted infections (STIs), the nurse should be aware that the most common bacterial STI is: a. Gonorrhea. b. Syphilis. c. Chlamydia. d. Candidiasis.

c. Chlamydia.

23. Some pregnant clients may complain of changes in their voice and impaired hearing. The nurse can tell these clients that these are common reactions to: a. A decreased estrogen level. b. Displacement of the diaphragm, resulting in thoracic breathing. c. Congestion and swelling, which occur because the upper respiratory tract has become more vascular. d. Increased blood volume

c. Congestion and swelling, which occur because the upper respiratory tract has become more vascular.

31. Which diagnostic test is used to confirm a suspected diagnosis of breast cancer? a. Mammogram b. Ultrasound c. Fine-needle aspiration (FNA) d. CA 15.3

c. Fine-needle aspiration (FNA)

22. Which documentation on a woman's chart on postpartum day 14 indicates a normal involution process? a. Moderate bright red lochial flow b. Breasts firm and tender c. Fundus below the symphysis and nonpalpable d. Episiotomy slightly red and puffy

c. Fundus below the symphysis and nonpalpable

12. With regard to endometriosis, nurses should be aware that: a. It is characterized by the presence and growth of endometrial tissue inside the uterus. b. It is found more often in African-American women than in white or Asian women. c. It may worsen with repeated cycles or remain asymptomatic and disappear after menopause. d. It is unlikely to affect sexual intercourse or fertility.

c. It may worsen with repeated cycles or remain asymptomatic and disappear after menopause.

1. When assessing a patient for amenorrhea, the nurse should be aware that this is unlikely to be caused by: a. Anatomic abnormalities. b. Type 1 diabetes mellitus. c. Lack of exercise. d. Hysterectomy.

c. Lack of exercise.

20. To reassure and educate pregnant clients about changes in their breasts, nurses should be aware that: a. The visibility of blood vessels that form an intertwining blue network indicates full function of Montgomery's tubercles and possibly infection of the tubercles. b. The mammary glands do not develop until 2 weeks before labor. c. Lactation is inhibited until the estrogen level declines after birth. d. Colostrum is the yellowish oily substance used to lubricate the nipples for breastfeeding.

c. Lactation is inhibited until the estrogen level declines after birth.

44. The nurse providing education regarding breast care should explain to the woman that fibrocystic changes in breasts are: a. A disease of the milk ducts and glands in the breasts. b. A premalignant disorder characterized by lumps found in the breast tissue. c. Lumpiness with pain and tenderness found in varying degrees in the breast tissue of healthy women during menstrual cycles. d. Lumpiness accompanied by tenderness after menses.

c. Lumpiness with pain and tenderness found in varying degrees in the breast tissue of healthy women during menstrual cycles

8. A 36-year-old woman has been given a diagnosis of uterine fibroids. When planning care for this patient, the nurse should know that: a. Fibroids are malignant tumors of the uterus that require radiation or chemotherapy. b. Fibroids increase in size during the perimenopausal period. c. Menorrhagia is a common finding. d. The woman is unlikely to become pregnant as long as the fibroids are in her uterus.

c. Menorrhagia is a common finding.

7. When evaluating a patient whose primary complaint is amenorrhea, the nurse must be aware that lack of menstruation is most often the result of: a. Stress. b. Excessive exercise. c. Pregnancy. d. Eating disorders.

c. Pregnancy.

11. Which statement concerning cyclic perimenstrual pain and discomfort (CPPD) is accurate? a. Premenstrual dysphoric disorder (PMDD) is a milder form of premenstrual syndrome (PMS) and more common in younger women. b. Secondary dysmenorrhea is more intense and medically significant than primary dysmenorrhea. c. Premenstrual syndrome is a complex, poorly understood condition that may include any of a hundred symptoms. d. The causes of PMS have been well established.

c. Premenstrual syndrome is a complex, poorly understood condition that may include any of a hundred symptoms.

10. When nurses help their expectant mothers assess the daily fetal movement counts, they should be aware that: a. Alcohol or cigarette smoke can irritate the fetus into greater activity. b. "Kick counts" should be taken every half hour and averaged every 6 hours, with every other 6-hour stretch off. c. The fetal alarm signal should go off when fetal movements stop entirely for 12 hours. d. Obese mothers familiar with their bodies can assess fetal movement as well as average-size women.

c. The fetal alarm signal should go off when fetal movements stop entirely for 12 hours.

18. The nurse is providing instruction to the newly delivered client regarding postbirth uterine and vaginal discharge, called lochia. Which statement is the most appropriate? a. Lochia is similar to a light menstrual period for the first 6 to 12 hours. b. It is usually greater after cesarean births. c. Lochia will usually decrease with ambulation and breastfeeding. d. It should smell like normal menstrual flow unless an infection is present.

d. It should smell like normal menstrual flow unless an infection is present.

4. A woman gave birth to a healthy infant boy 5 days ago. What type of lochia does the nurse expect to find when evaluating this client? a. Lochia rubra b. Lochia sangra c. Lochia alba d. Lochia serosa

d. Lochia serosa

13. One of the alterations in cyclic bleeding that occurs between periods is called: a. Oligomenorrhea. b. Menorrhagia. c. Leiomyoma. d. Metrorrhagia.

d. Metrorrhagia.

2. When a nurse is counseling a woman for primary dysmenorrhea, which nonpharmacologic intervention might be recommended? a. Increasing the intake of red meat and simple carbohydrates b. Reducing the intake of diuretic foods such as peaches and asparagus c. Temporarily substituting physical activity for a sedentary lifestyle d. Using a heating pad on the abdomen to relieve cramping

d. Using a heating pad on the abdomen to relieve cramping

24. The recommended treatment for the prevention of human immunodeficiency virus (HIV) transmission to the fetus during pregnancy is: a. Acyclovir. b. Ofloxacin. c. Podophyllin. d. Zidovudine.

d. Zidovudine.

Complications and risks associated with cesarean births include (Select all that apply): a. Placental abruption. b. Wound dehiscence. c. Hemorrhage. d. Urinary tract infections. e. Fetal injuries.

B,C,D,E

41. A benign breast condition that includes dilation and inflammation of the collecting ducts is called: a. Ductal ectasia. b. Intraductal papilloma. c. Chronic cystic disease. d. Fibroadenoma.

a. Ductal ectasia.

3. Which client is most likely to experience strong and uncomfortable afterpains? a. A woman who experienced oligohydramnios b. A woman who is a gravida 4, para 4-0-0-4 c. A woman who is bottle-feeding her infant d. A woman whose infant weighed 5 pounds, 3 ounces

b. A woman who is a gravida 4, para 4-0-0-4

9. During her gynecologic checkup, a 17-year-old girl states that recently she has been experiencing cramping and pain during her menstrual periods. The nurse would document this complaint as: a. Amenorrhea. b. Dysmenorrhea. c. Dyspareunia. d. Premenstrual syndrome (PMS).

b. Dysmenorrhea.

5. Which hormone remains elevated in the immediate postpartum period of the breastfeeding woman? a. Estrogen b. Progesterone c. Prolactin d. Human placental lactogen

c. Prolactin

10. With regard to dysmenorrhea, nurses should be aware that: a. It is more common in older women. b. It is more common in leaner women who exercise strenuously. c. Symptoms can begin at any point in the ovulatory cycle. d. Pain usually occurs in the suprapubic area or lower abdomen.

d. Pain usually occurs in the suprapubic area or lower abdomen.

7. Numerous changes in the integumentary system occur during pregnancy. Which change persists after birth? a. Epulis b. Chloasma c. Telangiectasia d. Striae gravidarum

d. Striae gravidarum

When the father develops a bond with the new infant and has an intense interest in how the infant looks and responds, this is called ________________.

engrossment

External fetal monitoring cannot detect the ____________________ of uterine contractions.

intensity p. 500

The maternal adaptation phase in which the mother relinquishes her previous role as being childless and her old lifestyle is called the _______________ phase.

letting go

Fetal tachycardia is most common during: A. Maternal fever B. Umbilical cord prolapse C. Regional anesthesia D. MgSO4 administration

A A. Correct: Fetal tachycardia can be considered an early sign of fetal hypoxemia and can also result from maternal or fetal infection. B. Incorrect: This situation most likely would result in fetal bradycardia, not tachycardia. C. Incorrect: This situation most likely would result in fetal bradycardia, not tachycardia. D. Incorrect: This situation most likely would result in fetal bradycardia, not tachycardia. p. 505

At 1 minute after birth, the nurse assesses the infant and notes a heart rate of 80 beats/minute, some flexion of the extremities, a weak cry, grimacing, and a pink body with blue extremities. The nurse would calculate an Apgar score of: ________

5 (Each of the five signs the nurse noted would score an Apgar of 1 for a total of 5. Signs include heart rate, respiratory effort, muscle tone, reflex irritability, and color. The highest possible Apgar score is 10.)

The best reason for recommending formula over breastfeeding is that: a. The mother has a medical condition or is taking drugs that could be passed along to the infant via breast milk. b. The mother lacks confidence in her ability to breastfeed. c. Other family members or care providers also need to feed the baby. d. The mother sees bottle-feeding as more convenient.

A (Breastfeeding is contraindicated when mothers have certain viruses, are undergoing chemotherapy, or are using/abusing illicit drugs. A lack of confidence, the need for others to feed the baby, and the convenience of bottle-feeding are all honest reasons for not breastfeeding, although further education concerning the ease of breastfeeding and its convenience, benefits, and adaptability (expressing milk into bottles) could change some minds. In any case the nurse must provide information in a nonjudgmental manner and respect the mother's decision. Nonetheless, breastfeeding is definitely contraindicated when the mother has medical or drug issues of her own.)

In many hospitals, new mothers are routinely presented with gift bags containing samples of infant formula. This practice is inconsistent with what? a. Baby Friendly Hospital Initiative b. Promotion of longer periods of breastfeeding c. Perception of being supportive to both bottle feeding and breastfeeding mothers d. Association with earlier cessation of breastfeeding

A Infant formula should not be given to mothers who are breastfeeding. Such gifts are associated with early cessation of breastfeeding. Baby Friendly USA prohibits the distribution of any gift bags or formula to new mothers.

According to the recommendations of the American Academy of Pediatrics on infant nutrition: a. Infants should be given only human milk for the first 6 months of life. b. Infants fed on formula should be started on solid food sooner than breastfed infants. c. If infants are weaned from breast milk before 12 months, they should receive cow's milk, not formula. d. After 6 months mothers should shift from breast milk to cow's milk.

A (Breastfeeding/human milk should also be the sole source of milk for the second 6 months. Infants start on solids when they are ready, usually at 6 months, whether they start on formula or breast milk. If infants are weaned from breast milk before 12 months, they should receive iron-fortified formula, not cow's milk.)

The quantitative human chorionic gonadotropin (β-hCG) levels are high in a patient who is on methotrexate therapy for dissolving abdominal pregnancy. Which instruction does the nurse give to this patient? A. "Avoid sexual activity." B. "Avoid next pregnancy." C. "Avoid feeling sad and low." D. "Take folic acid without fail."

A (High β-hCG levels indicate that the abdominal pregnancy is not yet dissolved. Therefore the nurse advises the patient to avoid sexual activity until the β-hCG levels drop and the pregnancy is dissolved completely. If the patient engages in vaginal intercourse, the pelvic pressure may rupture the mass and cause pain. Abdominal pregnancy increases the chances of infertility or recurrent ectopic pregnancy in patients. However, the nurse need not instruct the patient to avoid further pregnancy, because it may increase the feelings of sadness and guilt in the patient. The nurse encourages the patient to share feelings of guilt or sadness related to pregnancy loss. Folic acid is contraindicated with methotrexate therapy, because it may exacerbate ectopic rupture.)

Parents have been asked by the neonatologist to provide breast milk for their newborn son, who was born prematurely at 32 weeks of gestation. The nurse who instructs them about pumping, storing, and transporting the milk needs to assess their knowledge of lactation. Which statement is valid? a. A premature infant more easily digests breast milk than formula. b. A glass of wine just before pumping will help reduce stress and anxiety. c. The mother should pump only as much as the infant can drink. d. The mother should pump every 2 to 3 hours, including during the night.

A (Human milk is the ideal food for preterm infants, with benefits that are unique in addition to those received by term, healthy infants. Greater physiologic stability occurs with breastfeeding compared with formula feeding. Consumption of alcohol during lactation is approached with caution. Excessive amounts can have serious effects on the infant and can adversely affect the mother's milk ejection reflex. To establish an optimal milk supply, the mother should be instructed to pump 8 to 10 times a day for 10 to 15 minutes on each breast.)

Which infant would be more likely to have Rh incompatibility? a. Infant of an Rh-negative mother and a father who is Rh positive and homozygous for the Rh factor b. Infant who is Rh negative and whose mother is Rh negative c. Infant of an Rh-negative mother and a father who is Rh positive and heterozygous for the Rh factor d. Infant who is Rh positive and whose mother is Rh positive

A (If the mother is Rh negative and the father is Rh positive and homozygous for the Rh factor, all the children will be Rh positive. Only Rh-positive children of an Rh-negative mother are at risk for Rh incompatibility. If the mother is Rh negative and the father is Rh positive and heterozygous for the factor, there is a 50% chance that each infant born of the union will be Rh positive and a 50% chance that each will be born Rh negative.)

With regard to injuries to the infant's plexus during labor and birth, nurses should be aware that: a. If the nerves are stretched with no avulsion, they should recover completely in 3 to 6 months. b. Erb palsy is damage to the lower plexus. c. Parents of children with brachial palsy are taught to pick up the child from under the axillae. d. Breastfeeding is not recommended for infants with facial nerve paralysis until the condition resolves.

A (If the nerves are stretched with no avulsion, they should recover completely in 3 to 6 months. However, if the ganglia are disconnected completely from the spinal cord, the damage is permanent. Erb palsy is damage to the upper plexus and is less serious than brachial palsy. Parents of children with brachial palsy are taught to avoid picking up the child under the axillae or by pulling on the arms. Breastfeeding is not contraindicated, but both the mother and infant will need help from the nurse at the start.)

An infant at 26 weeks of gestation arrives intubated from the delivery room. The nurse weighs the infant, places him under the radiant warmer, and attaches him to the ventilator at the prescribed settings. A pulse oximeter and cardiorespiratory monitor are placed. The pulse oximeter is recording oxygen saturations of 80%. The prescribed saturations are 92%. The nurse's most appropriate action would be to: a. Listen to breath sounds and ensure the patency of the endotracheal tube, increase oxygen, and notify a physician. b. Continue to observe and make no changes until the saturations are 75%. c. Continue with the admission process to ensure that a thorough assessment is completed. d. Notify the parents that their infant is not doing well.

A (Listening to breath sounds and ensuring the patency of the endotracheal tube, increasing oxygen, and notifying a physician are appropriate nursing interventions to assist in optimal oxygen saturation of the infant. Oxygenation of the infant is crucial. O2 saturation should be maintained above 92%. Oxygenation status of the infant is crucial. The nurse should delay other tasks to stabilize the infant. Notifying the parents that the infant is not doing well is not an appropriate action. Further assessment and intervention are warranted before determination of fetal status.)

Nurses can assist parents who are trying to decide whether their son should be circumcised by explaining: a. The pros and cons of the procedure during the prenatal period. b. That the American Academy of Pediatrics (AAP) recommends that all newborn boys be routinely circumcised. c. That circumcision is rarely painful and any discomfort can be managed without medication. d. That the infant will likely be alert and hungry shortly after the procedure.

A (Many parents find themselves making the decision during the pressure of labor. The AAP and other professional organizations note the benefits but stop short of recommendation for routine circumcision. Circumcision is painful and must be managed with environmental, nonpharmacologic, and pharmacologic measures. After the procedure the infant may be fussy for several hours, or he may be sleepy and difficult to awaken for feeding.)

A pregnant woman was admitted for induction of labor at 43 weeks of gestation with sure dates. A nonstress test (NST) in the obstetrician's office revealed a nonreactive tracing. On artificial rupture of membranes, thick, meconium-stained fluid was noted. The nurse caring for the infant after birth should anticipate: a. Meconium aspiration, hypoglycemia, and dry, cracked skin. b. Excessive vernix caseosa covering the skin, lethargy, and respiratory distress syndrome. c. Golden yellow- to green stained-skin and nails, absence of scalp hair, and an increased amount of subcutaneous fat. d. Hyperglycemia, hyperthermia, and an alert, wide-eyed appearance.

A (Meconium aspiration, hypoglycemia, and dry, cracked skin are consistent with a postmature infant. Excessive vernix caseosa covering the skin, lethargy, and respiratory distress syndrome would be consistent with a very premature infant. The skin may be meconium stained, but the infant would most likely have longer hair and decreased amounts of subcutaneous fat. Postmaturity with a nonreactive NST would indicate hypoxia. Signs and symptoms associated with fetal hypoxia are hypoglycemia, temperature instability, and lethargy.)

The nurse providing couplet care should understand that nipple confusion results when: a. Breastfeeding babies receive supplementary bottle feedings. b. The baby is weaned too abruptly. c. Pacifiers are used before breastfeeding is established. d. Twins are breastfed together.

A (Nipple confusion can result when babies go back and forth between bottles and breasts, especially before breastfeeding is established in 3 to 4 weeks, because the two require different skills. Abrupt weaning can be distressing to mother and/or baby but should not lead to nipple confusion. Pacifiers used before breastfeeding is established can be disruptive, but this does not lead to nipple confusion. Breastfeeding twins requires some logistical adaptations, but this should not lead to nipple confusion.)

A 3.8-kg infant was delivered vaginally at 39 weeks after a 30-minute second stage. There was a nuchal cord. After birth the infant is noted to have petechiae over the face and upper back. Information given to the infant's parents should be based on the knowledge that petechiae: a. Are benign if they disappear within 48 hours of birth. b. Result from increased blood volume. c. Should always be further investigated. d. Usually occur with forceps delivery.

A (Petechiae, or pinpoint hemorrhagic areas, acquired during birth may extend over the upper portion of the trunk and face. These lesions are benign if they disappear within 2 days of birth and no new lesions appear. Petechiae may result from decreased platelet formation. In this situation the presence of petechiae is most likely a soft-tissue injury resulting from the nuchal cord at birth. Unless they do not dissipate in 2 days, there is no reason to alarm the family. Petechiae usually occur with a breech presentation vaginal birth.)

The nurse providing care for the laboring woman should understand that accelerations with fetal movement: A. Are reassuring B. Are caused by umbilical cord compression C. Warrant close observation D. Are caused by uteroplacental insufficiency

A A. Correct: Episodic accelerations in the FHR occur during fetal movement and are indications of fetal well-being. B. Incorrect: Umbilical cord compression results in variable decelerations in the FHR. C. Incorrect: Accelerations in the FHR are an indication of fetal well-being and do not warrant close observation. D. Incorrect: Uteroplacental insufficiency would result in late decelerations in the FHR. p. 504

Which condition in a pregnant patient with severe preeclampsia is an indication for administering magnesium sulfate? A. Seizure activity B. Renal dysfunction C. Pulmonary edema D. Low blood pressure (BP)

A (Severe preeclampsia may cause seizure activity or eclampsia in the patient, which is treated with magnesium sulfate. Magnesium sulfate is not administered for renal dysfunction and can cause magnesium toxicity in the patient. Pulmonary enema can be prevented by restricting the patient's fluid intake to 125 mL/hr. Increasing magnesium toxicity can cause low BP in the patient.)

A careful review of the literature on the various recreational and illicit drugs reveals that: a. More longer-term studies are needed to assess the lasting effects on infants when mothers have taken or are taking illegal drugs. b. Heroin and methadone cross the placenta; marijuana, cocaine, and phencyclidine (PCP) do not. c. Mothers should discontinue heroin use (detox) any time they can during pregnancy. d. Methadone withdrawal for infants is less severe and shorter than heroin withdrawal.

A (Studies on the effects of marijuana and cocaine use by mothers are somewhat contradictory. More long-range studies are needed. Just about all these drugs cross the placenta, including marijuana, cocaine, and PCP. Drug withdrawal is accompanied by fetal withdrawal, which can lead to fetal death. Therefore, detoxification from heroin is not recommended, particularly later in pregnancy. Methadone withdrawal is more severe and more prolonged than heroin withdrawal.)

A premature infant with respiratory distress syndrome receives artificial surfactant. How would the nurse explain surfactant therapy to the parents? a. "Surfactant improves the ability of your baby's lungs to exchange oxygen and carbon dioxide." b. "The drug keeps your baby from requiring too much sedation." c. "Surfactant is used to reduce episodes of periodic apnea." d. "Your baby needs this medication to fight a possible respiratory tract infection."

A (Surfactant can be administered as an adjunct to oxygen and ventilation therapy. With administration of artificial surfactant, respiratory compliance is improved until the infant can generate enough surfactant on his or her own. Surfactant has no bearing on the sedation needs of the infant. Surfactant is used to improve respiratory compliance, including the exchange of oxygen and carbon dioxide. The goal of surfactant therapy in an infant with respiratory distress syndrome (RDS) is to stimulate production of surfactant in the type 2 cells of the alveoli. The clinical presentation of RDS and neonatal pneumonia may be similar. The infant may be started on broad-spectrum antibiotics to treat infection.)

With regard to umbilical cord care, nurses should be aware that: a. The stump can easily become infected. b. A nurse noting bleeding from the vessels of the cord should immediately call for assistance. c. The cord clamp is removed at cord separation. d. The average cord separation time is 5 to 7 days.

A (The cord stump is an excellent medium for bacterial growth. The nurse should first check the clamp (or tie) and apply a second one. If the bleeding does not stop, the nurse calls for assistance. The cord clamp is removed after 24 hours when it is dry. The average cord separation time is 10 to 14 days.)

Which deceleration of the FHR would NOT require the nurse to change the maternal position? A. Early decelerations B. Late decelerations C. Variable decelerations D. It is always a good idea to change the woman's position.

A A. Correct: Early decelerations (and accelerations) generally do not need any nursing intervention. B. Incorrect: Late decelerations suggest that the nurse should change the maternal position (lateral); variable decelerations also require a maternal position change (side to side). C. Incorrect: Late decelerations suggest that the nurse should change the maternal position (lateral); variable decelerations also require a maternal position change (side to side). D. Incorrect: Although changing positions throughout labor is recommended, it is not required in response to early decelerations. p. 505

The nurse caring for the laboring woman should understand that early decelerations are caused by: A. Altered fetal cerebral blood flow B. Umbilical cord compression C. Uteroplacental insufficiency D. Spontaneous rupture of membranes

A A. Correct: Early decelerations are the fetus's response to fetal head compression. B. Incorrect: Variable decelerations are associated with umbilical cord compression. C. Incorrect: Late decelerations are associated with uteroplacental insufficiency. D. Incorrect: Spontaneous rupture of membranes has no bearing on the FHR unless the umbilical cord prolapses, which would result in variable or prolonged bradycardia. p. 507

Fetal well-being during labor is assessed by: A. The response of the FHR to uterine contractions (UCs) B. Maternal pain control C. Accelerations in the FHR D. An FHR above 110 beats/min

A A. Correct: Fetal well-being during labor can be measured by the response of the FHR to UCs. In general, reassuring FHR patterns are characterized by an FHR baseline in the range of 110 to 160 beats/min with no periodic changes and a moderate baseline variability, and accelerations with fetal movement. B. Incorrect: Maternal pain control is not the measure used to determine fetal well-being in labor. C. Incorrect: Although FHR accelerations are a reassuring pattern, they are only one component of the criteria by which fetal well-being is assessed. D. Incorrect: Although an FHR above 110 beats/min may be reassuring, it is only one component of the criteria by which fetal well-being is assessed. More information would be needed to determine fetal well-being. p. 498

While evaluating an external monitor tracing of a woman in active labor, the nurse notes that the fetal heart rate (FHR) for five sequential contractions begins to decelerate late in the contraction, with the nadir of the decelerations occurring after the peak of the contraction. The nurse's first priority is to: A. Change the woman's position B. Notify the care provider C. Assist with amnioinfusion D. Insert a scalp electrode

A A. Correct: Late decelerations may be caused by maternal supine hypotension syndrome. They usually are corrected when the woman turns on her side to displace the weight of the gravid uterus from the vena cava. B. Incorrect: If the fetus does not respond to primary nursing interventions for late decelerations, the nurse would continue with subsequent intrauterine resuscitation measures, including notifying the care provider. C. Incorrect: An amnioinfusion may be used to relieve pressure on an umbilical cord that has not prolapsed. The fetal heart rate pattern associated with this situation most likely reveals variable deceleration. D. Incorrect: A fetal scalp electrode would provide accurate data for evaluating the well-being of the fetus; however, this is not a nursing intervention that would alleviate late decelerations, nor is it the nurse's first priority. p. 507

Perinatal nurses are legally responsible for: A. Correctly interpreting FHR patterns, initiating appropriate nursing interventions, and documenting the outcomes B. Greeting the client on arrival, assessing her, and starting an IV line C. Applying the external fetal monitor and notifying the care provider D. Making sure the woman is comfortable

A A. Correct: Nurses who care for women during childbirth are legally responsible for correctly interpreting FHR patterns, initiating appropriate nursing interventions based on those patterns, and documenting the outcomes of those interventions. B. Incorrect: This may be an activity that a nurse performs, but it is not an activity for which the nurse is legally responsible. C. Incorrect: This may be an activity that a nurse performs, but it is not an activity for which the nurse is legally responsible. D. Incorrect: This is one aspect of caring for a woman in labor, but it is not an activity for which the nurse is legally responsible. p. 511

The nurse caring for the woman in labor should understand that decreased variability of the fetal heart rate would be considered benign if caused by: A. A periodic fetal sleep state B. Uterine palpation C. Uterine contractions D. Maternal activity

A A. Correct: Periodic fetal sleep states usually last 20 to 30 minutes. B. Incorrect: Uterine palpations and contractions, as well as maternal activity, might be (probably) benign signs of increased variability. C. Incorrect: Uterine palpations and contractions, as well as maternal activity, might be (probably) benign signs of increased variability. D. Incorrect: Uterine palpations and contractions, as well as maternal activity, might be (probably) benign signs of increased variability. p. 502

When using IA to assess uterine activity, nurses should be aware that: A. The examiner's hand should be placed over the fundus before, during, and after contractions. B. The frequency and duration of contractions is measured in seconds for consistency. C. Contraction intensity is given a judgment number of 1 to 7 by the nurse and client together. D. The resting tone between contractions is described as either placid or turbulent.

A A. Correct: The assessment is done by palpation; duration, frequency, intensity, and resting tone must be assessed. B. Incorrect: The duration of contractions is measured in seconds; the frequency is measured in minutes. C. Incorrect: The intensity of contractions usually is described as mild, moderate, or strong. D. Incorrect: The resting tone usually is characterized as soft or relaxed. p. 500

In assisting with the two factors that have an effect on fetal status, namely pushing and positioning, nurses should: A. Encourage the woman's cooperation in avoiding the supine position B. Advise the woman to avoid the semi-Fowler position C. Encourage the woman to hold her breath and tighten her abdominal muscles to produce a vaginal response D. Instruct the woman to open her mouth and close her glottis, letting air escape after the push

A A. Correct: The woman should maintain a side-lying position. B. Incorrect: The semi-Fowler position is the recommended side-lying position with a lateral tilt to the uterus. C. Incorrect: This is the Valsalva maneuver, which should be avoided. D. Incorrect: Both the mouth and glottis should be open, letting air escape during the push. p. 516

Rho immune globulin will be ordered postpartum if which situation occurs? a. Mother Rh, baby Rh+ b. Mother Rh, baby Rh c. Mother Rh+, baby Rh+ d. Mother Rh+, baby Rh

A An Rh mother delivering an Rh+ baby may develop antibodies to fetal cells that entered her bloodstream when the placenta separated. The Rho immune globulin works to destroy the fetal cells in the maternal circulation before sensitization occurs. If mother and baby are both Rh+ or Rh the blood types are alike, so no antibody formation would be anticipated. If the Rh+ blood of the mother comes in contact with the Rh blood of the infant, no antibodies would develop because the antigens are in the mothers blood, not in the infants.

When should discharge instruction, or the teaching plan that tells the woman what she needs to know to care for herself and her newborn, officially begin? a. At the time of admission to the nurses unit b. When the infant is presented to the mother at birth c. During the first visit with the physician in the unit d. When the take-home information packet is given to the couple

A Discharge planning, the teaching of maternal and newborn care, begins on the womans admission to the unit, continues throughout her stay, and actually never ends as long as she has contact with medical personnel.

A hospital has a number of different perineal pads available for use. A nurse is observed soaking several of them and writing down what she sees. What goal is the nurse attempting to achieve by performing this practice? a. To improve the accuracy of blood loss estimation, which usually is a subjective assessment b. To determine which pad is best c. To demonstrate that other nurses usually underestimate blood loss d. To reveal to the nurse supervisor that one of them needs some time off

A Saturation of perineal pads is a critical indicator of excessive blood loss; anything done to help in the assessment is valuable. The nurse is noting the saturation volumes and soaking appearances. Instead of determining which pad is best, the nurse is more likely noting saturation volumes and soaking appearances to improve the accuracy of estimated blood loss. Nurses usually overestimate blood loss. Soaking perineal pads and writing down the results does not indicate the need for time off of work.

A primiparous woman is to be discharged from the hospital the following day with her infant girl. Which behavior indicates a need for further intervention by the nurse before the woman can be discharged? a. The woman is disinterested in learning about infant care. b. The woman continues to hold and cuddle her infant after she has fed her. c. The woman reads a magazine while her infant sleeps. d. The woman changes her infants diaper and then shows the nurse the contents of the diaper.

A The client should be excited, happy, and interested or involved in infant care. A woman who is sad, tearful, or disinterested in caring for her infant may be exhibiting signs of depression or postpartum blues and may require further intervention. Holding and cuddling her infant after feeding is an appropriate parent-infant interaction. Taking time for herself while the infant is sleeping is an appropriate maternal action. Showing the nurse the contents of the diaper is appropriate because the mother is seeking approval from the nurse and notifying the nurse of the infants elimination patterns.

The laboratory results for a postpartum woman are as follows: blood type, A; Rh status, positive; rubella titer, 1:8 (enzyme immunoassay [EIA] 0.8); hematocrit, 30%. How should the nurse best interpret these data? a. Rubella vaccine should be administered. b. Blood transfusion is necessary. c. Rh immune globulin is necessary within 72 hours of childbirth. d. Kleihauer-Betke test should be performed.

A This clients rubella titer indicates that she is not immune and needs to receive a vaccine. These data do not indicate that the client needs a blood transfusion. Rh immune globulin is indicated only if the client has an Rh-negative status and the infant has an Rh-positive status. A Kleihauer-Betke test should be performed if a large fetomaternal transfusion is suspected, especially if the mother is Rh negative. However, the data provided do not indicate a need for performing this test

The new mother comments that the newborn "has his father's eyes." The nurse recognizes this as: a. Part of the bonding process termed claiming. b. The mother trying to find signs of the baby's paternity. c. The mother trying to include the father in the bonding process. d. Part of the letting-go phase of maternal adaptation.

A Claiming or binding-in begins when the mother begins to identify specific features of the newborn. She then begins to relate features to family members.

D (Realize that the regression in habits and behaviors in the older child is a typical reaction and that he needs extra love and attention at this time)

A 30-year-old multiparous woman has a boy who is 2 1/2 years old and now an infant girl. She tells the nurse, "I don't know how I'll ever manage both children when I get home." Which suggestion would best help this woman alleviate sibling rivalry? A. Tell the older child that he is a big boy now and should love his new sister B. Let the older child stay with his grandparents for the first 6 weeks to allow him to adjust to the newborn C. Ask friends and relatives not to bring gifts to the older sibling because you do not want to spoil him D. Realize that the regression in habits and behaviors in the older child is a typical reaction and that he needs extra love and attention at this time

A (An ultrasound examination could be done to confirm the pregnancy and determine the gestational age of the fetus. It is too early in the pregnancy to perform MSAFP screening, amniocentesis, or NST. MSAFP screening is performed at 16 to 18 weeks of gestation, followed by amniocentesis if MSAFP levels are abnormal or if fetal/maternal anomalies are detected. NST is performed to assess fetal well-being in the third trimester.)

A 39-year-old primigravida thinks that she is about 8 weeks pregnant, although she has had irregular menstrual periods all her life. She has a history of smoking approximately one pack of cigarettes a day, but she tells you that she is trying to cut down. Her laboratory data are within normal limits. What diagnostic technique could be used with this pregnant woman at this time? A. Ultrasound examination B. Maternal serum alpha-fetoprotein (MSAFP) screening C. Amniocentesis D. Nonstress test (NST)

A (Fetal heart activity begins around 6 weeks, so 4 weeks is too early to detect fetal heart activity, and this is a normal finding. Absence of fetal heart activity at an advanced gestational age may indicate congenital anomalies, impaired growth, or cardiac disorders.)

A 4-week pregnant patient is undergoing an ultrasound. The report shows an absence of fetal heart activity. What does the nurse infer about the fetus from the report? A. Normal finding B. Congenital abnormality C. Impaired growth D. Cardiac disorder

D (Ultrasound would be performed at this gestational age for biophysical assessment of the infant. BPP would be a method of biophysical assessment of fetal well-being in the third trimester. Amniocentesis is performed after the fourteenth week of pregnancy. MSAFP screening is performed from week 15 to week 22 of gestation (weeks 16 to 18 are ideal).)

A 40-year-old woman is 10 weeks pregnant. Which diagnostic tool would be appropriate to suggest to her at this time? A. Biophysical profile (BPP) B. Amniocentesis C. Maternal serum alpha-fetoprotein (MSAFP) screening D. Transvaginal ultrasound

D (Rationale: A biophysical profile is a method of biophysical assessment of fetal well-being in the third trimester. An amniocentesis is performed after the fourteenth week of pregnancy. A MSAFP test is performed from week 15 to week 22 of the gestation (weeks 16 to 18 are ideal). An ultrasound is the method of biophysical assessment of the infant that is performed at this gestational age. Transvaginal ultrasound is especially useful for obese women whose thick abdominal layers cannot be penetrated adequately with the abdominal approach.)

A 40-year-old woman with a high body mass index (BMI) is 10 weeks pregnant. Which diagnostic tool is appropriate to suggest to her at this time? A. Biophysical profile B. Amniocentesis C. Maternal serum alpha-fetoprotein (MSAFP) D. Transvaginal ultrasound

B (Real-time ultrasound permits detailed assessment of the physical and physiologic characteristics of the developing fetus and cataloging of normal and abnormal biophysical responses to stimuli. BPP is a noninvasive, dynamic assessment of a fetus that is based on acute and chronic markers of fetal disease. An ultrasound for fetal anomalies would most likely have been performed earlier in the pregnancy. It is too late in the pregnancy to perform MSAFP screening. Also, MSAFP screening does not provide information related to fetal well-being. Indications for PUBS include prenatal diagnosis or inherited blood disorders, karyotyping of malformed fetuses, detection of fetal infection, determination of the acid-base status of a fetus with IUGR, and assessment and treatment of isoimmunization and thrombocytopenia in the fetus.)

A 41-week pregnant multigravida presents in the labor and delivery unit after a nonstress test indicated that her fetus could be experiencing some difficulties in utero. Which diagnostic tool would yield more detailed information about the fetus? A. Ultrasound for fetal anomalies B. Biophysical profile (BPP) C. Maternal serum alpha-fetoprotein (MSAFP) screening D. Percutaneous umbilical blood sampling (PUBS)

A (An accurate and appropriate response is, "Your placenta changes as your pregnancy progresses, and it is given a score that indicates the amount of calcium deposits it has. The more calcium deposits, the higher the grade, or number, that is assigned to the placenta. It also means that less blood and oxygen can be delivered to your baby." Although "Your placenta isn't working properly, and your baby is in danger" may be valid, it does not reflect therapeutic communication techniques and is likely to alarm the client. An ultrasound, not an amniocentesis, is the method of assessment used to determine placental maturation. The response "Don't worry about it. Everything is fine" is not appropriate and discredits the client's concerns.)

A client asks her nurse, "My doctor told me that he is concerned with the grade of my placenta because I am overdue. What does that mean?" The best response by the nurse is: A. "Your placenta changes as your pregnancy progresses, and it is given a score that indicates the amount of calcium deposits it has. The more calcium deposits, the higher the grade, or number, that is assigned to the placenta. It also means that less blood and oxygen can be delivered to your baby." B. "Your placenta isn't working properly, and your baby is in danger." C. "This means that we will need to perform an amniocentesis to detect if you have any placental damage." D. "Don't worry about it. Everything is fine."

B (Rationale: When caring for a client who has suffered perinatal loss, the nurse should provide opportunity for her to bond with the dead infant and for the infant to become part of the family unit. Parents not given that opportunity may have fantasies about the infant that are worse than reality. If the child has gross deformities, the nurse should prepare the client for these. If the client doesn't ask about her child, the nurse should encourage her to do so and provide any information she seems ready to hear. The client needs a full explanation of all factors related to the experience so she can grieve appropriately. Allowing the father to determine which information the client is given is inappropriate.)

A client gives birth to a stillborn infant at 36 weeks. When caring for this client, which strategy by the nurse would be most helpful? A. Be selective in providing the information that the client seeks B. Encourage the client to see, touch and hold the dead infant C. Provide information about the possible causes of the stillbirth only if the client requests it D. Let the child's father decide what information the mother receives.

A ("Your baby may lose heat by convection, which means that he will lose heat from his body to the cooler ambient air. You should keep him wrapped and prevent cool air from blowing on him" is an accurate statement. Conduction is the loss of heat from the body surface to cooler surfaces, not air, in direct contact with the newborn. Evaporation is loss of heat that occurs when a liquid is converted into a vapor. In the newborn heat loss by evaporation occurs as a result of vaporization of moisture from the skin. Cold stress may occur from excessive heat loss, but this does not imply that the infant will become stressed if not bundled at all times. Furthermore, excessive bundling may result in a rise in the infant's temperature.)

A client is warm and asks for a fan in her room for her comfort. The nurse enters the room to assess the mother and her infant and finds the infant unwrapped in his crib with the fan blowing over him on "high." The nurse instructs the mother that the fan should not be directed toward the newborn and the newborn should be wrapped in a blanket. The mother asks why. The nurse's best response is: a. "Your baby may lose heat by convection, which means that he will lose heat from his body to the cooler ambient air. You should keep him wrapped and prevent cool air from blowing on him." b. "Your baby may lose heat by conduction, which means that he will lose heat from his body to the cooler ambient air. You should keep him wrapped and prevent cool air from blowing on him." c. "Your baby may lose heat by evaporation, which means that he will lose heat from his body to the cooler ambient air. You should keep him wrapped and prevent cool air from blowing on him." d. "Your baby will get cold stressed easily and needs to be bundled up at all times."

B (Rationale: Cardiac output increases immediately after delivery as blood that had been diverted to the uterus reenters the central circulation. A client who cannot tolerate these changes may experience cardiac decompensation and cardiac failure. After delivery, renal function increases. There is usually not an increase in pain after delivery except for small increments attributable to uterine cramps, perineal discomfort and breast tenderness. Although hepatic blood flow decreases to normal levels after delivery, this does not affect cardiac function.)

A client with cardiac disease delivers a baby. Afterwards, the nurse assesses the client for signs of cardiac decompensation. During the postpartum period, which condition can cause cardiac decompensation? A. Increased pain B. Increased cardiac output C. Decreased renal function D. Decreased hepatic blood flow

A (Bleeding may occur during a spontaneous vaginal delivery as a result of the pressure against the maternal bony pelvis. The soft, irreducible fullness does not pulsate or bulge when the infant cries. Low forceps and other difficult extractions may result in bleeding. However, cephalhematomas can also occur spontaneously. The swelling may appear unilaterally or bilaterally and is usually minimal or absent at birth. It increases over the first 2 to 3 days of life. Cephalhematomas disappear gradually over 2 to 3 weeks. A less common condition results in calcification of the hematoma, which may persist for months.)

A collection of blood between the skull bone and its periosteum is known as a cephalhematoma. To reassure the new parents whose infant develops such a soft bulge, it is important that the nurse be aware that this condition: a. May occur with spontaneous vaginal birth. b. Happens only as the result of a forceps or vacuum delivery. c. Is present immediately after birth. d. Will gradually absorb over the first few months of life.

C (The woman has irregular menstrual cycles. The scenario does not indicate that she has had any testing related to this irregularity. Hormone analysis is performed to assess endocrine function of the hypothalamic-pituitary-ovarian axis when menstrual cycles are absent or irregular. Determination of blood levels of prolactin, FSH, luteinizing hormone (LH), estradiol, progesterone, and thyroid hormones may be necessary to diagnose the cause of irregular menstrual cycles. A testicular biopsy would be indicated only in cases of azoospermia (no sperm cells) or severe oligospermia (low number of sperm cells). Antisperm antibodies are produced by a man against his own sperm. This is unlikely to be the case here, because the husband has already produced children. Examination for testicular infection would be done before semen analysis. Furthermore, infection would affect spermatogenesis.)

A couple comes in for an infertility workup, having attempted to get pregnant for 2 years. The woman, 37, has always had irregular menstrual cycles but is otherwise healthy. The man has fathered two children from a previous marriage and had a vasectomy reversal 2 years ago. The man has had two normal semen analyses, but the sperm seem to be clumped together. What additional test is needed? a. Testicular biopsy b. Antisperm antibodies c. Follicle-stimulating hormone (FSH) level d. Examination for testicular infection

C (Venting negative feelings may unburden the couple. A support group may provide a safe haven for the couple to share their experiences and gain insight from others' experiences. Although talking about their feelings may unburden them of negative feelings, infertility can be a major stressor that affects the couple's relationships with family and friends. Limiting their interactions to other infertile couples may be a beginning point for addressing psychosocial needs, but depending on where the other couple is in their own recovery process, this may or may not help them. The statement about adoption proceedings is not supportive of the psychosocial needs of this couple and may be detrimental to their well-being)

A couple is trying to cope with an infertility problem. They want to know what they can do to preserve their emotional equilibrium. The nurse's most appropriate response is: a. "Tell your friends and family so they can help you." b. "Talk only to other friends who are infertile because only they can help." c. "Get involved with a support group. I'll give you some names." d. "Start adoption proceedings immediately because it is very difficult to obtain an infant."

C (Physiologic jaundice becomes visible when the serum bilirubin reaches a level of 5 mg/dL or greater, which occurs when the baby is approximately 3 days old. This finding is within normal limits for the newborn. Pathologic jaundice occurs during the first 24 hours of life. Pathologic jaundice is caused by blood incompatibilities, causing excessive destruction of erythrocytes, and must be investigated. Breast milk jaundice occurs in one third of breastfed infants at 2 weeks and is caused by an insufficient intake of fluids.)

A first-time dad is concerned that his 3-day-old daughter's skin looks "yellow." In the nurse's explanation of physiologic jaundice, what fact should be included? a. Physiologic jaundice occurs during the first 24 hours of life. b. Physiologic jaundice is caused by blood incompatibilities between the mother and infant blood types. c. The bilirubin levels of physiologic jaundice peak between the second and fourth days of life. d. This condition is also known as "breast milk jaundice."

A ("That's meconium, which is your baby's first stool. It's normal" is an accurate statement and the most appropriate response. Transitional stool is greenish brown to yellowish brown and usually appears by the third day after initiation of feeding. "That means your baby is bleeding internally" is not accurate. "Oh, don't worry about that. It's okay" is not an appropriate statement. It is belittling to the father and does not educate him about the normal stool patterns of his daughter.)

A first-time father is changing the diaper of his 1-day-old daughter. He asks the nurse, "What is this black, sticky stuff in her diaper?" The nurse's best response is: a. "That's meconium, which is your baby's first stool. It's normal." b. "That's transitional stool." c. "That means your baby is bleeding internally." d. "Oh, don't worry about that. It's okay."

C (Increased frequency of urination in a postpartum patient is termed postpartal diuresis. Postpartum patients have decreased estrogen and progesterone levels. In addition, removal of increased venous pressure in the lower extremities and loss of the remaining pregnancy-induced increase in blood volume may also cause diuresis. Diuresis helps get rid of excess fluids from the body. The levels of oxytocin are not related to postpartum diuresis. The aldosterone levels drop after childbirth and are not related to postpartum diuresis. hCG tends to disappear after the childbirth and has no role in diuresis.)

A lactating postpartum patient reports frequent urination. What could be the reason for increased frequency of urination in the patient? A decrease in the levels of: A. Estrogen and aldosterone B. Oxytocin and progesterone C. Progesterone and estrogen D. Human chorionic gonadotropin (hCG)

B ("Nonoxynol-9 does not provide protection against sexually transmitted infections, as originally thought; it has also been linked to an increase in the transmission of human immunodeficiency virus and can cause genital lesions" is a true statement. Nonoxynol-9 may cause vaginal irritation, has no effect on the quality of sexual activity, and has no effect on penile sensitivity.)

A male client asks the nurse why it is better to purchase condoms that are not lubricated with nonoxynol-9 (a common spermicide). The nurse's most appropriate response is: a. "The lubricant prevents vaginal irritation." b. "Nonoxynol-9 does not provide protection against sexually transmitted infections, as originally thought; it has also been linked to an increase in the transmission of human immunodeficiency virus and can cause genital lesions." c. "The additional lubrication improves sex." d. "Nonoxynol-9 improves penile sensitivity."

D (Use of tobacco, alcohol, and marijuana may affect sperm counts. "Your sperm count seems to be okay in the first semen analysis" is inaccurate. Sperm counts vary from day to day and depend on emotional and physical status and sexual activity. Therefore a single analysis may be inconclusive. A minimum of two analyses must be performed several weeks apart to assess male fertility. Use of tobacco, alcohol, and marijuana may affect sperm counts.)

A man smokes two packs of cigarettes a day. He wants to know if smoking is contributing to the difficulty he and his wife are having getting pregnant. The nurse's most appropriate response is: a. "Your sperm count seems to be okay in the first semen analysis." b. "Only marijuana cigarettes affect sperm count." c. "Smoking can give you lung cancer, even though it has no effect on sperm." d. "Smoking can reduce the quality of your sperm."

B (FAMs are effective with proper vigilance about ovulatory changes in the body and adherence to coitus intervals. They are effective if used correctly by a woman with a regular menstrual cycle. The typical failure rate for all FAMs is 25% during the first year of use. FAMs have no associated health risks. The use of birth control has associated health risks. In addition, taking a pill daily requires compliance on the client's part.)

A married couple is discussing alternatives for pregnancy prevention and has asked about fertility awareness methods (FAMs). The nurse's most appropriate reply is: a. "They're not very effective, and it's very likely you'll get pregnant." b. "They can be effective for many couples, but they require motivation." c. "These methods have a few advantages and several health risks." d. "You would be much safer going on the pill and not having to worry."

D (Low levels of MSAFP are associated with Down syndrome. Sickle cell anemia is not detected by the MSAFP. Cardiac defects would not be detected with the MSAFP. A triple marker test determines the levels of MSAFP along with serum levels of estriol and human chorionic gonadotropin; an elevated level is associated with open neural tube defects.)

A maternal serum alpha-fetoprotein (MSAFP) test is performed at 16 to 18 weeks of gestation. An elevated level has been associated with: A. Down syndrome. B. Sickle cell anemia. C. Cardiac defects. D. Open neural tube defects such as spina bifida.

C (Meconium stool is usually passed in the first 12 hours of life, and 99% of newborns have their first stool within 48 hours. If meconium is not passed by 48 hours, obstruction is suspected. Meconium stool is the first stool of the newborn and is made up of matter remaining in the intestines during intrauterine life. Meconium is dark and sticky.)

A meconium stool can be differentiated from a transitional stool in the newborn because the meconium stool is: a. Seen at age 3 days. b. The residue of a milk curd. c. Passed in the first 12 hours of life. d. Lighter in color and looser in consistency.

B (The depressed phase after pregnancy and delivery is known as postpartum blues, or "baby blues," where the mother may experience restlessness, insomnia, and headache. Episiotomy would cause other symptoms related to pain, such as restlessness and insomnia, but it would not cause the mother to experience headaches. The pink phase of the postpartum period is the period where the woman experiences a sense of heightened joy and feelings of well-being. Anesthesia is not given during vaginal delivery, so this is not a factor in her postpartum symptoms.)

A mother who had a vaginal delivery reports being restless and having insomnia and regular episodes of headache 4 days after childbirth. What would the nurse infer about the mother's clinical condition from these symptoms? The mother: A. Is experiencing pain from the episiotomy. B. Likely has postpartum blues. C. Is in the pink postpartum period. D. Was given spinal anesthesia during the delivery process.

C (The organisms are localized in the breast tissue and are not excreted in the breast milk.)

A mother with mastitis is concerned about breastfeeding while she has an active infection. The nurse should explain that: a. the infant is protected from infection by immunoglobulins in the breast milk. b. the infant is not susceptible to the organisms that cause mastitis. c. the organisms that cause mastitis are not passed to the milk. d. the organisms will be inactivated by gastric acid.

B (Treatment of excessive bleeding requires the collaboration of the physician and the nurses. Do not leave the client alone.)

A multiparous woman is admitted to the postpartum unit after a rapid labor and birth of a 4000 g infant. Her fundus is boggy, lochia is heavy, and vital signs are unchanged. The nurse has the woman void and massages her fundus, but her fundus remains difficult to find, and the rubra lochia remains heavy. The nurse should: a. continue to massage the fundus. b. notify the physician. c. recheck vital signs. d. insert a Foley catheter.

A (Acrocyanosis, or the appearance of slightly cyanotic hands and feet, is caused by vasomotor instability, capillary stasis, and a high hemoglobin level. Acrocyanosis is normal and appears intermittently over the first 7 to 10 days. Erythema toxicum (also called erythema neonatorum) is a transient newborn rash that resembles flea bites. The harlequin sign is a benign, transient color change in newborns. Half of the body is pale, and the other half is ruddy or bluish red with a line of demarcation. Vernix caseosa is a cheeselike, whitish substance that serves as a protective covering.)

A new mother states that her infant must be cold because the baby's hands and feet are blue. The nurse explains that this is a common and temporary condition called: a. Acrocyanosis. b. Erythema neonatorum. c. Harlequin color. d. Vernix caseosa.

B (Loss of heat must be controlled to protect the infant from the metabolic and physiologic effects of cold stress, and that is the primary reason for placing a newborn under a radiant heat warmer. Cold stress results in an increased respiratory rate and vasoconstriction.)

A newborn is placed under a radiant heat warmer, and the nurse evaluates the infant's body temperature every hour. Maintaining the newborn's body temperature is important for preventing: a. Respiratory depression. b. Cold stress. c. Tachycardia. d. Vasoconstriction.

B, C, E (Vibroacoustic stimulation is often used to stimulate fetal activity if the initial NST result is nonreactive and thus hopefully shortens the time required to complete the test (Greenberg, Druzin, and Gabbe, 2012). A nonreactive test requires further evaluation. The testing period is often extended, usually for an additional 20 minutes, with the expectation that the fetal sleep state will change and the test will become reactive. Care providers sometimes suggest that the woman drink orange juice or be given glucose to increase her blood sugar level and thereby stimulate fetal movements. Although this practice is common, there is no evidence that it increases fetal activity (Greenberg, Druzin, and Gabbe, 2012). A needle biopsy is not part of a NST. The FHR is recorded with a Doppler transducer, and a tocodynamometer is applied to detect uterine contractions or fetal movements. The tracing is observed for signs of fetal activity and a concurrent acceleration of FHR.)

A nonstress test (NST) is ordered on a pregnant woman at 37 weeks of gestation. What are the most appropriate teaching points to include when explaining the procedure to the woman? Select all that apply. A. After 20 minutes, a nonreactive reading indicates the test is complete. B. Vibroacoustic stimulation may be used during the test. C. Drinking orange juice before the test is appropriate. D. A needle biopsy may be needed to stimulate contractions. E. Two sensors are placed on the abdomen to measure contractions and fetal heart tones.

B, C, E (Rationale: A nonreactive test requires further evaluation. The testing period is often extended, usually for an additional 20 minutes, with the expectation that the fetal sleep state will change and the test will become reactive. During this time vibroacoustic stimulation (see later discussion) may be used to stimulate fetal activity. Vibroacoustic stimulation is often used to stimulate fetal activity if the initial NST result is nonreactive and thus hopefully shortens the time required to complete the test. Care providers sometimes suggest that the woman drink orange juice or be given glucose to increase her blood sugar level and thereby stimulate fetal movements. Although this practice is common, there is no evidence that it increases fetal activity. A needle biopsy is not part of a NST. The FHR is recorded with a Doppler transducer, and a tocodynamometer is applied to detect uterine contractions or fetal movements. The tracing is observed for signs of fetal activity and a concurrent acceleration of FHR.)

A nonstress test (NST) is ordered on a pregnant women at 37 weeks gestation. What are the most appropriate teaching points to include when explaining the procedure to the patient? (Select all that apply) A. After 20 minutes, a nonreactive reading indicates the test is complete. B. Vibroacoustic stimulation may be used during the test. C. Drinking orange juice before the test is appropriate. D. A needle biopsy may be needed to stimulate contractions. E. Two sensors are placed on the abdomen to measure contractions and fetal heart tones.

D (The condition will resolve itself within a few hours. For this common condition of newborns, surfactant acts to keep the expanded alveoli partially open between respirations. In vaginal births, absorption of remaining lung fluid is accelerated by the process of labor and delivery. Remaining lung fluid will move into interstitial spaces and be absorbed by the circulatory and lymphatic systems. This is a particularly common condition for infants delivered by cesarean section. Surfactant is produced by the lungs, so aspiration is not a concern.)

A nursing student is helping the nursery nurses with morning vital signs. A baby born 10 hours ago by cesarean section is found to have moist lung sounds. What is the best interpretation of these data? a. The nurse should notify the pediatrician stat for this emergency situation. b. The neonate must have aspirated surfactant. c. If this baby was born vaginally, it could indicate a pneumothorax. d. The lungs of a baby delivered by cesarean section may sound moist for 24 hours after birth.

A, B (Triple marker screening, which is performed in the first trimester of pregnancy, includes the measurement of two maternal biomarkers: PAPP-A and free β-hCG. High levels of free β-hCG and low levels of PAPP-A in the first trimester indicate that the fetus has Down syndrome, or trisomy 21.Inhibin-A is a placental hormone. Low levels of inhibin-A also indicate the possibility of Down syndrome, but inhibin-A levels are not measured in the triple marker screen; these levels are measured in quad screening. A low level of MSAFP and unconjugated estriol also indicate Down syndrome, but these can be measured only in the second and third trimesters.)

A patient in the first trimester of pregnancy undergoes a triple marker screening test. On reviewing the report, the nurse infers that the fetus may have Down syndrome. What clinical findings are noted by the nurse in the test reports? Select all that apply. A. High levels of beta-human chorionic gonadotropin (β-hCG) B. Low levels of pregnancy-associated placental protein (PAPP-A) C. Low levels of inhibin-A in the fetal blood D. Low levels of maternal serum alpha-fetoprotein (MSAFP) E. Low levels of unconjugated estriol in the fetal blood

B (Lochial bleeding is normal and decreases with time. Nonlochial bleeding occurs as a result of the tears in either the cervical or vaginal regions. In normal condition (lochial), the bleeding decreases with time. However, in this patient there is continuous bleeding even 4 weeks after childbirth. Normally, the color of the blood is bright red in the beginning (known as lochia rubra); after that, the color of the blood becomes slightly less pigmented. Therefore the patient has nonlochial discharge evident by the continuous bright red color bleeding. The odor of the lochia is the same as of the menstrual bleeding. The offensive odor of the lochia indicates presence of infection.)

A patient reports continuous bleeding 4 weeks after childbirth. Upon assessment, the nurse finds that the bleeding is bright red in color with an offensive odor. What does the nurse suspect as the cause of the bleeding? A. Lochial; the odor is caused by infection. B. Nonlochial; the odor is caused by infection. C. Lochial; the odor is normal in all postpartum patients. D. Nonlochial; the odor is normal in all postpartum patients

A, B, C (The patient reports having problems with lactation. Thus the nurse should assist in establishing and maintaining a successful breastfeeding process for the patient. To do this, the nurse should teach the patient about the infant's rooting and sucking reflexes. The patient should be asked to immediately breastfeed the child after birth. Then the nurse has to explain frequent milk expression and kangaroo care to the patient for optimized milk supply. Placing the newborn skin-to-skin with the parent immediately after birth is not useful for breastfeeding. The nurse has to talk to the patient about the infant's capabilities for interaction while teaching the patient how to nurture the infant in the first year of life, but these interventions are not in response to problems with lactation.)

A patient reports having difficulty ejecting milk while breastfeeding the infant. Which nursing interventions would help the patient breastfeed the newborn with greater ease? Select all that apply. A. Teach the patient about the infant's rooting and sucking reflexes. B. Ask the patient to breastfeed the newborn immediately after birth. C. Explain frequent milk expression and kangaroo care to the patient. D. Talk to the patient about the newborn's capabilities for interaction. E. Ensure the newborn has good contact with the patient immediately after birth.

D (Thromboembolism refers to the condition in which a blood vessel is blocked by a blood clot. As the postpartum period is characterized by a hypercoagulation state, the patient is at risk of thromboembolism. Thrombophlebitis is the inflammation of the vein and is not associated with hypercoagulation. Thrombocytopenia refers to the condition in which low levels of platelet are found in the blood. Thrombocytosis is a condition characterized by a significant increase in the number of platelets in the blood.)

A patient who had a cesarean birth is immobile in the immediate postoperative period. Which risk is increased in the patient as a result of the hypercoagulable state of the puerperal period? A. Thrombocytosis B. Thrombophlebitis C. Thrombocytopenia D. Thromboembolism

B (A rapid labor and delivery may cause exhaustion of the uterine muscle and prevent contraction.)

A postpartum client would be at increased risk for postpartum hemorrhage if she delivered a: a. 5-lb, 2-oz infant with outlet forceps. b. 6.5-lb infant after a 2-hour labor. c. 7-lb infant after an 8-hour labor. d. 8-lb infant after a 12-hour labor.

D (Stretch marks never disappear altogether; however, they gradually fade to silvery lines.)

A postpartum patient asks, "Will these stretch marks go away?" The nurse's best response is: A. "They will continue to fade and should be gone by your 6-week checkup." B. "No, never." C. "Yes, eventually." D. "They will fade to silvery lines but won`t disappear completely."

C (The pelvic floor is a broad sling of muscles, ligaments, and sheet-like tissues. It stretches from the pubic bone at the front of the body to the base of the spine at the back. It is responsible for movement, balance, stability, and flexibility. The supportive tissues of the pelvic floor are torn or stretched during childbirth. This results in urinary incontinence while coughing, sneezing, or performing exercises. Kegel exercises consist of repeated contractions and relaxations of the muscles that form the pelvic floor. These exercises help strengthen the perineal muscles and can prevent urinary incontinence. The intake of more fluids is recommended after childbirth if the woman has constipation. However, the administration of fluids for urinary incontinence causes irritation of the bladder. An epidural block is a medicine given in the epidural spaces for causing numbness in the lower half of the body. Therefore it causes urinary incontinence after childbirth. Personal hygiene is not related to urinary incontinence, but it helps prevent infections.)

A postpartum patient has urinary incontinence. What is the best nursing intervention to help this patient? A. Provide an epidural block. B. Encourage the patient to intake more fluids. C. Tell the patient to perform more Kegel exercises. D. Have the patient practice better personal hygiene.

D (Fluid retention and swelling are common during pregnancy and increase the pressure in the narrow and inflexible space in the hand, compressing the median nerve that runs through the hand. This results in numbness, tingling of fingers, and the inability to lift and carry objects and is a condition known as carpal tunnel syndrome, which this patient has. Itching and discomfort around the anus and bright red bleeding upon defecation are the symptoms of hemorrhoids. These symptoms are not observed in the patient; therefore the patient does not have hemorrhoids. Fever, pain, and abdominal tenderness during menstrual discharge are the symptoms of endometritis. The nurse does not find these symptoms in the patient, so the patient does not have endometritis. Vitamin A deficiency does not affect the muscles, nor does it cause periodic numbness or tingling of the fingers. Therefore the patient does not have a vitamin A deficiency.)

A pregnant patient complains to the nurse about periodic numbness, tingling of fingers, and the inability to lift and carry any object. What patient clinical condition does the nurse interpret from this assessment? A. Hemorrhoids B. Endometritis C. Vitamin A deficiency D. Carpal tunnel syndrome

D (MRI is a noninvasive technique that causes little pain. Therefore the patient undergoing MRI should be advised to not worry about pain. The patient undergoing MRI should be positioned in a supine position. The lithotomy position is not suitable for this procedure. The patient need not take fluids before the scan to ensure a full bladder because this procedure enables a full pelvic view without a full bladder. The patient should be instructed not to move during the scanning process because it may blur the images obtained.)

A pregnant patient is about to undergo magnetic resonance imaging (MRI). What information does the nurse give the patient before the procedure? The patient will: A. Be positioned in a lithotomy position. B. Need to take fluids to have a full bladder. C. Be able to move freely during the procedure. D. Not have pain during the process

D (Fingernails may regain strength to the prepregnancy state in a few weeks after delivery. Brittle and soft nails are caused by iron deficiency, not potassium deficiency. Carpal tunnel syndrome causes physiologic edema due to compression of the median nerve, but it does not cause brittle and soft nails. Moisture can cause soft and brittle fingers, so moisture should be reduced, not enhanced.)

A pregnant patient reports to the nurse, "My nails are soft and brittle, and I am worried about it." What is the nurse's best response to the patient? A. "You should make sure your nails are well moisturized." B. "You have low potassium, so take potassium supplements." C. "Your nails are soft and brittle due to carpal tunnel syndrome." D. "After delivery your nails should return to normal consistency and strength."

A (A normal fetal kick count is an indication of fetal activity. The patient has undergone nonstress testing, which may have high false-positive rates. Therefore the patient may be scheduled for a contraction stress test. Biophysical profile testing allows detailed assessment of the physical and physiologic characteristics of the fetus. Because the kick count is normal, biophysical profile testing is not needed. Maternal serum analysis is done to determine fetal abnormalities. It is not advised in this case, because the fetal kick count indicates adequate fetal activity. Doppler blood flow analysis is a noninvasive test for analyzing fetal circulation. It cannot be used to assess the fetal heart rate.)

A pregnant patient with a normal fetal kick count has come for a regular nonstress testing session. The nurse notices that there are no heart accelerations after 40 minutes of testing. What diagnostic testing will the nurse include in the plan of care? A. Contraction stress test B. Biophysical profile test C. Maternal serum analysis D. Doppler blood flow test

C (The nurse should inform the woman that she may need to reduce her exercise level as the pregnancy progresses. Physical activity promotes a feeling of well-being in pregnant women. It improves circulation, promotes relaxation and rest, and counteracts boredom. Typically, running should be replaced with walking around the seventh month of pregnancy. Simple measures should be initiated to prevent injuries, such as warm-up and stretching exercises to prepare the joints for more strenuous exercise.)

A pregnant woman at 10 weeks of gestation jogs three or four times per week. She is concerned about the effect of exercise on the fetus. The nurse should inform her: A. "You don't need to modify your exercising any time during your pregnancy." B. "Stop exercising, because it will harm the fetus." C. "You may find that you need to modify your exercise to walking later in your pregnancy, around the seventh month." D. "Jogging is too hard on your joints; switch to walking now."

B (Pelvic rock exercises may help stretch and strengthen the abdominal and lower back muscles and relieve low back pain. Kegel exercises increase the tone of the pelvic area, not the back. A softer mattress may not provide the support needed to maintain proper alignment of the spine and may contribute to back pain. Stretching and other exercises to relieve back pain should be performed several times a day.)

A pregnant woman at 18 weeks of gestation calls the clinic to report that she has been experiencing occasional backaches of mild-to-moderate intensity. The nurse would recommend that she: A. Do Kegel exercises. B. Do pelvic rock exercises. C. Use a softer mattress. D. Stay in bed for 24 hours.

D (Vital signs can be assessed next. Breathing into a paper bag is the solution for dizziness related to respiratory alkalosis associated with hyperventilation. Raising her legs will not solve the problem since pressure will still remain on the major abdominal blood vessels, thereby continuing to impede cardiac output. During a fundal height measurement the woman is placed in a supine position. This woman is experiencing supine hypotension as a result of uterine compression of the vena cava and abdominal aorta. Turning her on her side will remove the compression and restore cardiac output and blood pressure.)

A pregnant woman at 32 weeks of gestation complains of feeling dizzy and light-headed while her fundal height is being measured. Her skin is pale and moist. The nurse's initial response would be to: A. Assess the woman's blood pressure and pulse. B. Have the woman breathe into a paper bag. C. Raise the woman's legs. D. Turn the woman on her side.

C (Pointing toes can aggravate rather than relieve the cramp. Application of heat is recommended. Extending the leg and dorsiflexing the foot is the appropriate relief for a leg cramp. Bearing weight on the affected leg can help to relieve the leg cramp, so it should not be avoided.)

A pregnant woman demonstrates understanding of the nurse's instructions regarding relief of leg cramps if she: A. Wiggles and points her toes during the cramp. B. Applies cold compresses to the affected leg. C. Extends her leg and dorsiflexes her foot during the cramp. D. Avoids weight bearing on the affected leg during the cramp.

A (The normal biophysical score ranges from 8 to 10 points if the amniotic fluid volume is adequate. A normal score allows conservative treatment of high-risk patients. Delivery can be delayed if fetal well-being is indicated. Scores less than 4 should be investigated, and delivery could be initiated sooner than planned. This score is within normal range, and no further testing is required at this time. The results of the biophysical profile are usually available immediately after the procedure is performed.)

A pregnant woman's biophysical profile score is 8. She asks the nurse to explain the results. The nurse's best response is: A. "The test results are within normal limits." B. "Immediate delivery by cesarean birth is being considered." C. "Further testing will be performed to determine the meaning of this score." D. "An obstetric specialist will evaluate the results of this profile and, within the next week, will inform you of your options regarding delivery."

B (The initial management of excessive postpartum bleeding is firm massage of the uterine fundus.)

A primary nursing responsibility when caring for a woman experiencing an obstetric hemorrhage associated with uterine atony is to: A. Establish venous access B. Perform fundal massage C. Prepare the woman for surgical intervention D. Catheterize the bladder

B (Rationale: Based on the signs and symptoms presented by the client (especially the elevated temperature), the physician should be notified because the client probably has mastitis, an infection in the breast. An antibiotic that is tolerated by the infant as well as the mother may be prescribed. The mother should continue to nurse on both breasts but should start the infant on the unaffected breast while the affected breast lets down.)

A ten-day postpartum breastfeeding client telephones the postpartum unit complaining of a reddened, painful breast and elevated temperature. Based on assessment of the client's complaints, the nurse tells the client to: A. "Stop breastfeeding because you probably have an infection." B. "Notify your physician because you may need medication." C. "Continue breastfeeding because this is a normal response in breastfeeding mothers." D. "Breastfeed only with the unaffected breast."

A (Amniotic fluid embolism (AFE))

A thrombosis results from the formation of a blood clot or clots inside a blood vessel and is caused by inflammation or partial obstruction of the vessel. Three thromboembolic conditions are of concern during the postpartum period and include all except: a) Amniotic fluid embolism (AFE) b) Superficial venous thrombosis c) Deep vein thrombosis d) Pulmonary embolism

A (A WBC count in the upper ranges of normal (20,000 to 30,000) may indicate an infection.)

A white blood cell (WBC) count of 28,000 cells/mm3 on the morning of the first postpartum day indicates: a. possible infection. b. normal WBC limit. c. serious infection. d. suspicion of sexually transmitted disease.

D (Her family history of NTD, low BMI, and substance abuse all are high risk factors of pregnancy. The woman's BP is normal, and her age does not put her at risk. Her BMI is low and may indicate poor nutritional status, which would be a high risk. The woman's drug/alcohol use and family history put her in a high risk category, but her age does not. The woman's family history puts her in a high risk category. Her BMI is low and may indicate poor nutritional status, which would be high risk. Her BP is normal.)

A woman arrives at the clinic seeking confirmation that she is pregnant. The following information is obtained: She is 24 years old with a body mass index (BMI) of 17.5. She admits to having used cocaine "several times" during the past year and drinks alcohol occasionally. Her blood pressure (BP) is 108/70 mm Hg, her pulse rate is 72 beats/min, and her respiratory rate is 16 breaths/min. The family history is positive for diabetes mellitus and cancer. Her sister recently gave birth to an infant with a neural tube defect (NTD). Which characteristics place the woman in a high risk category? A. Blood pressure, age, BMI B. Drug/alcohol use, age, family history C. Family history, blood pressure, BMI D. Family history, BMI, drug/alcohol abuse

D (Increased activity can cause excessive bleeding. Therefore postpartum women should not lift heavy weights and should go up and down stairs slowly. If the bleeding starts to get heavier, the patient should take a rest from being on her feet. Hemorrhage can occur after delivery when the uterus does not shrink completely. It can be caused by infection in the uterus or a residual placenta. However, infection or residual placenta is not associated with excess loss of blood; instead there will be a steady loss of blood. Hemorrhage conditions typically present with continuous loss of blood, leading to shock-type symptoms. There is no evidence that the patient is experiencing a hemorrhage situation. Breastfeeding immediately after delivery and in the early postpartum days increases the release of oxytocin. Oxytocin helps to decrease blood loss and reduces the risk for postpartum hemorrhage. Oxytocin strengthens and coordinates the uterine contractions (UCs), which help compress the blood vessels and promote hemostasis.)

A woman complains of excess vaginal bleeding after childbirth. The patient reports that the presence of excess blood is not continuous and denies any headaches or dizziness. What does the nurse suspect to be the cause of this excess bleeding? A. Oxytocin B. Hemorrhage C. Breastfeeding D. Increased activity

D (The cervical cap can be inserted hours before sexual intercourse without the need for additional spermicide later. No additional spermicide is required for repeated acts of intercourse. Spermicide should be used inside the cap as an additional chemical barrier. The cervical cap should remain in place for 6 hours after the last act of intercourse. Repeated intercourse with the cervical cap is more convenient, because no additional spermicide is needed.)

A woman currently uses a diaphragm and spermicide for contraception. She asks the nurse what the major differences are between the cervical cap and diaphragm. The nurse's most appropriate response is: a. "No spermicide is used with the cervical cap, so it's less messy." b. "The diaphragm can be left in place longer after intercourse." c. "Repeated intercourse with the diaphragm is more convenient." d. "The cervical cap can safely be used for repeated acts of intercourse without adding more spermicide later."

B (Firmness of the uterus is necessary to control bleeding from the placental site. The nurse should first assess for firmness and massage the fundus as indicated.)

A woman delivered a 9-lb, 10-oz baby 1 hour ago. When you arrive to perform her 15-minute assessment, she tells you that she "feels all wet underneath." You discover that both pads are completely saturated and that she is lying in a 6-inch-diameter puddle of blood. What is your first action? a. Call for help. b. Assess the fundus for firmness. c. Take her blood pressure. d. Check the perineum for lacerations.

B (Excessive uterine bleeding)

A woman gave birth to a 7-lb, 3-oz boy 2 hours ago. The nurse determines that the woman's bladder is distended because her fundus is now 3 cm above the umbilicus and to the right of the midline. In the immediate postpartum period, the most serious consequence likely to occur from bladder distention is: a) Urinary tract infection b) Excessive uterine bleeding c) A ruptured bladder d) Bladder wall atony

B (Excessive bleeding can occur immediately after birth if the bladder becomes distended because it pushes the uterus up and to the side and prevents it from contracting firmly. A urinary tract infection may result from overdistention of the bladder, but it is not the most serious consequence. A ruptured bladder may result from a severely overdistended bladder. However, vaginal bleeding most likely would occur before the bladder reaches this level of overdistention. Bladder distention may result from bladder wall atony. The most serious concern associated with bladder distention is excessive uterine bleeding.)

A woman gave birth to a 7-pound, 3-ounce infant boy 2 hours ago. The nurse determines that the woman's bladder is distended because her fundus is now 3 cm above the umbilicus and to the right of the midline. In the immediate postpartum period, the most serious consequence likely to occur from bladder distention is: A. Urinary tract infection. B. Excessive uterine bleeding. C. A ruptured bladder. D. Bladder wall atony.

A (An EBL of 1500 mL with tachycardia and hypotension suggests hypovolemia caused by excessive blood loss. An increased respiratory rate of 36 may be secondary to pain from the birth. Temperature may increase to 38° C during the first 24 hours as a result of the dehydrating effects of labor. A BP of 140/90 is slightly elevated, which may be caused by the use of oxytocic medications.)

A woman gave birth to a 7-pound, 6-ounce infant girl 1 hour ago. The birth was vaginal, and the estimated blood loss (EBL) was approximately 1500 mL. When assessing the woman's vital signs, the nurse would be concerned to see: A. Temperature 37.9° C, heart rate 120, respirations 20, blood pressure (BP) 90/50. B. Temperature 37.4° C, heart rate 88, respirations 36, BP 126/68. C. Temperature 38° C, heart rate 80, respirations 16, BP 110/80. D. Temperature 36.8° C, heart rate 60, respirations 18, BP 140/90.

B (The first period of reactivity is the first phase of transition and lasts up to 30 minutes after birth. The infant is highly alert during this phase. The transition period is the phase between intrauterine and extrauterine existence. There is no such phase as the organizational stage. The second period of reactivity occurs roughly between 4 and 8 hours after birth, after a period of prolonged sleep.)

A woman gave birth to a healthy 7-pound, 13-ounce infant girl. The nurse suggests that the woman place the infant to her breast within 15 minutes after birth. The nurse knows that breastfeeding is effective during the first 30 minutes after birth because this is the: a. Transition period. b. First period of reactivity. c. Organizational stage. d. Second period of reactivity.

D (Lochia serosa, which consists of blood, serum, leukocytes, and tissue debris, generally occurs around day 3 or 4 after childbirth. Lochia rubra consists of blood and decidual and trophoblastic debris. The flow generally lasts 3 to 4 days and pales, becoming pink or brown. There is no such term as lochia sangra. Lochia alba occurs in most women after day 10 and can continue up to 6 weeks after childbirth.)

A woman gave birth to a healthy infant boy 5 days ago. What type of lochia would the nurse expect to find when assessing this woman? A. Lochia rubra B. Lochia sangra C. Lochia alba D. Lochia serosa

A (Within 12 hours after delivery the fundus may be approximately 1 cm above the umbilicus. The fundus descends about 1 to 2 cm every 24 hours. Within 12 hours after delivery the fundus may be approximately 1 cm above the umbilicus. By the sixth postpartum week the fundus normally is halfway between the symphysis pubis and the umbilicus. The fundus should be easily palpated using the maternal umbilicus as a reference point.)

A woman gave birth to an infant boy 10 hours ago. Where would the nurse expect to locate this woman's fundus? A. One centimeter above the umbilicus B. Two centimeters below the umbilicus C. Midway between the umbilicus and the symphysis pubis D. Nonpalpable abdominally

C (To minimize the side effect of nausea that occurs with high doses of estrogen and progestin, the woman can take an over-the-counter antiemetic 1 hour before each dose. Emergency contraception is used within 72 hours of unprotected intercourse to prevent pregnancy. Postcoital contraceptive use is 74% to 90% effective at preventing pregnancy. Oral emergency contraceptive regimens may include progestin-only and estrogen-progestin pills. Women with contraindications to estrogen use should use progestin-only pills.)

A woman had unprotected intercourse 36 hours ago and is concerned that she may become pregnant because it is her "fertile" time. She asks the nurse about emergency contraception. The nurse tells her that: a. It is too late; she needed to begin treatment within 24 hours after intercourse. b. Preven, an emergency contraceptive method, is 98% effective at preventing pregnancy. c. An over-the-counter antiemetic can be taken 1 hour before each contraceptive dose to prevent nausea and vomiting. d. The most effective approach is to use a progestin-only preparation.

B (The nurse can best help the woman and her husband regain a sense of control in their lives by providing support and encouragement (including active involvement in preparations and classes). The nurse can try to present opportunities for the couple to make as many choices as possible in prenatal care.)

A woman has been diagnosed with a high risk pregnancy. She and her husband come into the office in a very anxious state. She seems to be coping by withdrawing from the discussion, showing declining interest. The nurse can best help the couple by: A. Telling her that the physician will isolate the problem with more tests. B. Encouraging her and urging her to continue with childbirth classes. C. Becoming assertive and laying out the decisions the couple needs to make. D. Downplaying her risks by citing success rate studies.

A (The calendar method of conception control is based on the number of days in each cycle, counting from the first day of menses. The fertile period is determined after the lengths of menstrual cycles have been accurately recorded for 6 months. Weight gain or loss may be partly related to hormonal fluctuations, but it has no bearing on use of the calendar method. Integumentary changes may be related to hormonal changes, but they are not indicators for use of the calendar method. Exploring previous experiences with conception control may demonstrate client understanding and compliancy, but it is not the most important aspect to assess for discussion of the calendar method.)

A woman has chosen the calendar method of conception control. During the assessment process it is most important that the nurse: a. Obtain a history of menstrual cycle lengths for the past 6 to 12 months. b. Determine the client's weight gain and loss pattern for the previous year. c. Examine skin pigmentation and hair texture for hormonal changes. d. Explore the client's previous experiences with conception control.

A, B, E (Immunization with live or attenuated live viruses is contraindicated during pregnancy because of potential teratogenicity. Vaccines consisting of killed viruses may be used. Immunizations that may be administered during pregnancy include tetanus, diphtheria, recombinant hepatitis B, and rabies vaccines. Live-virus vaccines include those for measles (rubeola and rubella), chickenpox, and mumps.)

A woman has just moved to the United States from Mexico. She is 3 months pregnant and has arrived for her first prenatal visit. During her assessment interview, you discover that she has not had any immunizations. Which immunizations should she receive at this point in her pregnancy (Select all that apply)? A. Tetanus B. Diphtheria C. Chickenpox D. Rubella E. Hepatitis B

B (Herbs that a woman should avoid while trying to conceive include licorice root, yarrow, wormwood, ephedra, fennel, golden seal, lavender, juniper, flaxseed, pennyroyal, passionflower, wild cherry, cascara, sage, thyme, and periwinkle. Nettle leaf, dong quai, and vitamin E are all remedies that promote fertility. Vitamin E, calcium, and magnesium may promote fertility and conception. Although most herbal remedies have not been proven clinically to promote fertility, women should avoid the following herbs while trying to conceive: licorice root, yarrow, wormwood, ephedra, fennel, golden seal, lavender, juniper, flaxseed, pennyroyal, passionflower, wild cherry, cascara, sage, thyme, and periwinkle.)

A woman inquires about herbal alternative methods for improving fertility. Which statement by the nurse is the most appropriate when instructing the client in which herbal preparations to avoid while trying to conceive? a. "You should avoid nettle leaf, dong quai, and vitamin E while you are trying to get pregnant." b. "You may want to avoid licorice root, lavender, fennel, sage, and thyme while you are trying to conceive." c. "You should not take anything with vitamin E, calcium, or magnesium. They will make you infertile." d. "Herbs have no bearing on fertility."

A (The most common technique for medical termination of a pregnancy in the second trimester is D&E. It is usually performed between 13 and 16 weeks. Hypertonic solutions injected directly into the uterus account for less than 1% of all abortions because other methods are safer and easier to use. Intravenous administration of Pitocin is used to induce labor in a woman with a third-trimester fetal demise. Vacuum aspiration is used for abortions in the first trimester.)

A woman is 16 weeks pregnant and has elected to terminate her pregnancy. The nurse knows that the most common technique used for medical termination of a pregnancy in the second trimester is: a. Dilation and evacuation (D&E). b. Instillation of hypertonic saline into the uterine cavity. c. Intravenous administration of Pitocin. d. Vacuum aspiration.

With regard to dysfunctional uterine bleeding (DUB), the nurse should be aware that: A) it is most commonly caused by anovulation. B) it most often occurs in middle age. C) the diagnosis of DUB should be the first considered for abnormal menstrual bleeding. D) the most effective medical treatment involves steroids.

A) it is most commonly caused by anovulation. Anovulation may occur because of hypothalamic dysfunction or polycystic ovary syndrome. DUB most often occurs when the menstrual cycle is being established or when it draws to a close at menopause. A diagnosis of DUB is made only after all other causes of abnormal menstrual bleeding have been ruled out. The most effective medical treatment is oral or intravenous estrogen.

D (The statement "Hormonal changes during pregnancy commonly result in mood swings" is accurate and the most appropriate response by the nurse. The statement "Don't worry about it; you'll feel better in a month or so" dismisses the client's concerns and is not the most appropriate response. Although women should be encouraged to share their feelings, "Have you talked to your husband about how you feel" is not the most appropriate response and does not provide the client with a rationale for the psychosocial dynamics of her pregnancy. "Perhaps you really don't want to be pregnant" is completely inappropriate and deleterious to the psychologic well-being of the woman. Hormonal and metabolic adaptations often cause mood swings in pregnancy. The woman's responses are normal. She should be reassured about her feelings.)

A woman is 3 months pregnant. At her prenatal visit, she tells the nurse that she doesn't know what is happening; one minute she's happy that she is pregnant, and the next minute she cries for no reason. Which response by the nurse is most appropriate? A. "Don't worry about it; you'll feel better in a month or so." B. "Have you talked to your husband about how you feel?" C. "Perhaps you really don't want to be pregnant." D. "Hormonal changes during pregnancy commonly result in mood swings."

C (Rationale: Since the client is already hemorrhaging, it is inappropriate to initiate a pad count. Fundal massage and administration of oxytocics would be indicated if the hemorrhage is due to uterine atony. If a full bladder is displacing the uterus and preventing it from contracting, insertion of an indwelling catheter would be an appropriate response.)

A woman is experiencing an early postpartum hemorrhage. Which of the following actions would be inappropriate? A. Insertion of an indwelling urinary catheter B. Fundal massage C. Pad count D. Administration of oxytocics

A (Adequate uterine activity necessary for a CST consists of the presence of three contractions in a 10-minute time frame. If no decelerations are observed in the FHR pattern with the contractions, the findings are considered to be negative. A positive CST indicates the presence of repetitive later FHR decelerations. Satisfactory and unsatisfactory are not applicable terms.)

A woman is undergoing a nipple-stimulated contraction stress test (CST). She is having contractions that occur every 3 minutes. The fetal heart rate (FHR) has a baseline of approximately 120 beats/min without any decelerations. The interpretation of this test is said to be: A. Negative. B. Positive. C. Satisfactory. D. Unsatisfactory.

D (The absence of a temperature decrease most likely is the result of lack of ovulation. Pregnancy cannot occur without ovulation (which is being measured using the BBT method). A comment such as "Don't worry; it's probably nothing" discredits the client's concerns. Illness would most likely cause an increase in basal body temperature.)

A woman is using the basal body temperature (BBT) method of contraception. She calls the clinic and tells the nurse, "My period is due in a few days, and my temperature has not gone up." The nurse's most appropriate response is: a. "This probably means that you're pregnant." b. "Don't worry; it's probably nothing." c. "Have you been sick this month?" d. "You probably didn't ovulate during this cycle."

A (Women with a history of TSS should not use a cervical cap. Condoms, vaginal films, and vaginal sheaths are not contraindicated for a woman with a history of TSS.)

A woman was treated recently for toxic shock syndrome (TSS). She has intercourse occasionally and uses over-the-counter protection. On the basis of her history, what contraceptive method should she and her partner avoid? a. Cervical cap b. Condom c. Vaginal film d. Vaginal sheath

B (Because the liver metabolizes oral contraceptives, their effectiveness is reduced when they are taken simultaneously with anticonvulsants. "Your current medications will reduce the effectiveness of the pill" is a true statement, but it is not the most appropriate response. The anticonvulsant will reduce the effectiveness of the pill, not the other way around. "This is a good choice for a woman of your age and personal history" does not teach the client that the effectiveness of the pill may be reduced because of her anticonvulsant therapy.)

A woman who has a seizure disorder and takes barbiturates and phenytoin sodium daily asks the nurse about the pill as a contraceptive choice. The nurse's most appropriate response would be: a. "This is a highly effective method, but it has some side effects." b. "Your current medications will reduce the effectiveness of the pill." c. "The pill will reduce the effectiveness of your seizure medication." d. "This is a good choice for a woman of your age and personal history."

C (Pain and tenderness in the extremities, which show warmth, redness, and hardness, is likely thrombophlebitis. A Doppler ultrasound is a common noninvasive way to confirm diagnosis.)

A woman who has recently given birth complains of pain and tenderness in her leg. Upon physical examination, the nurse notices warmth and redness over an enlarged, hardened area. The nurse should suspect _____ and should confirm the diagnosis by _____. A. Disseminated intravascular coagulation; asking for laboratory tests B. von Willebrand disease; noting whether bleeding times have been extended C. Thrombophlebitis; using real time and color Doppler ultrasound D. Coagulopathies; drawing blood for laboratory analysis

D (Regardless of which trimester the woman has reached, no amount of alcohol during pregnancy has been deemed safe for the fetus. Neither one drink per night nor three drinks per week is a safe recommendation. Although the first trimester is a crucial period of fetal development, pregnant women of all gestations are counseled to eliminate all alcohol from their diet. A safe level of alcohol consumption during pregnancy has not yet been established. Although the consumption of occasional alcoholic beverages may not be harmful to the mother or her developing fetus, complete abstinence is strongly advised.)

A woman who is 14 weeks pregnant tells the nurse that she always had a glass of wine with dinner before she became pregnant. She has abstained during her first trimester and would like to know if it is safe for her to have a drink with dinner now. The nurse tells her: A. "Because you're in your second trimester, there's no problem with having one drink with dinner." B. "One drink every night is too much. One drink three times a week should be fine." C. "Because you're in your second trimester, you can drink as much as you like." D. "Because no one knows how much or how little alcohol it takes to cause fetal problems, the best course is to abstain throughout your pregnancy."

46. Examples of sexual risk behaviors associated with exposure to a sexually transmitted infection (STI) include (Select all that apply): a. Fellatio. b. Unprotected anal intercourse. c. Multiple sex partners. d. Dry kissing. e. Abstinence.

A, B, C

D (The statement "Because no one knows how much or how little alcohol it takes to cause fetal problems, the best course is to abstain throughout your pregnancy" is accurate. A safe level of alcohol consumption during pregnancy has not yet been established. Although the consumption of occasional alcoholic beverages may not be harmful to the mother or her developing fetus, complete abstinence is strongly advised.)

A woman who is 14 weeks pregnant tells the nurse that she always had a glass of wine with dinner before she became pregnant. She has abstained during her first trimester and would like to know if it is safe for her to have a drink with dinner now. The nurse would tell her: A. "Since you're in your second trimester, there's no problem with having one drink with dinner." B. "One drink every night is too much. One drink three times a week should be fine." C. "Since you're in your second trimester, you can drink as much as you like." D. "Because no one knows how much or how little alcohol it takes to cause fetal problems, the best course is to abstain throughout your pregnancy."

B (Constipation is a normal discomfort of pregnancy that occurs in the second and third trimesters. An alteration in the pattern or amount of fetal movement may indicate fetal jeopardy. Heart palpitations are a normal change related to pregnancy. This is most likely to occur during the second and third trimesters. As the pregnancy progresses, edema in the ankles and feet at the end of the day is not uncommon.)

A woman who is 32 weeks pregnant is informed by the nurse that a danger sign of pregnancy could be: A. Constipation. B. Alteration in the pattern of fetal movement. C. Heart palpitations. D. Edema in the ankles and feet at the end of the day.

B (An alteration in the pattern or amount of fetal movement may indicate fetal jeopardy. Constipation, heart palpitations, and ankle and foot edema are normal discomforts of pregnancy that occur in the second and third trimesters.)

A woman who is 32 weeks' pregnant is informed by the nurse that a danger sign of pregnancy could be: A. Constipation. B. Alteration in the pattern of fetal movement. C. Heart palpitations. D. Edema in the ankles and feet at the end of the day.

D (The nonstress test is one of the most widely used techniques to determine fetal well-being and is accomplished by monitoring fetal heart rate in conjunction with fetal activity and movements. An ultrasound is the test that requires a full bladder. An amniocentesis is the test that a pregnant woman should be driven home afterward. A maternal alpha-fetoprotein test is used in conjunction with unconjugated estriol levels, and human chorionic gonadotropin helps to determine Down syndrome.)

A woman who is at 36 weeks of gestation is having a nonstress test. Which statement by the woman indicates a correct understanding of the test? A. "I will need to have a full bladder for the test to be done accurately." B. "I should have my husband drive me home after the test because I may be nauseous." C. "This test will help to determine if the baby has Down syndrome or a neural tube defect." D. "This test will observe for fetal activity and an acceleration of the fetal heart rate to determine the well-being of the baby."

D (Rationale: An ultrasound is the test that requires a full bladder. An amniocentesis would be the test that a pregnant woman should be driven home afterward. A maternal alpha-fetoprotein test is used in conjunction with unconjugated estriol levels, and human chorionic gonadotropin helps to determine Down syndrome. The nonstress test is one of the most widely used techniques to determine fetal well-being and is accomplished by monitoring fetal heart rate in conjunction with fetal activity and movements.)

A woman who is at 36 weeks of gestation is having a nonstress test. Which statement by the woman would indicate a correct understanding of the test? A. "I will need to have a full bladder for the test to be done accurately." B. "I should have my husband drive me home after the test because I may be nauseous." C. "This test will help to determine if the baby has Down syndrome or a neural tube defect." D. None of the above

C (To maintain adequate hormone levels for contraception and to enhance compliance, clients should take oral contraceptives at the same time each day. If contraceptives are to be started at any time other than during normal menses or within 3 weeks after birth or abortion, another method of contraception should be used through the first week to prevent the risk of pregnancy. Taken exactly as directed, oral contraceptives prevent ovulation, and pregnancy cannot occur. No strong pharmacokinetic evidence indicates a link between the use of broad-spectrum antibiotics and altered hormone levels in oral contraceptive users. If the client misses two pills during week 1, she should take two pills a day for 2 days, finish the package, and use a backup method the next 7 consecutive days.)

A woman will be taking oral contraceptives using a 28-day pack. The nurse should advise this woman to protect against pregnancy by: a. Limiting sexual contact for one cycle after starting the pill. b. Using condoms and foam instead of the pill for as long as she takes an antibiotic. c. Taking one pill at the same time every day. d. Throwing away the pack and using a backup method if she misses two pills during week 1 of her cycle.

A (Rationale: Sudden dyspnea, diaphoresis and confusion are the classic signs of the dislodgment of a thrombus (stationary blood clot) from a varicose vein and its travel to and its becoming enlodged in the pulmonary circulation. Chills and fever would indicate infection. A person with a pulmonary embolism would be hypotensive and not hypertensive.)

A woman with a past history of varicose veins has just delivered and the nurse suspects she has developed a pulmonary embolism. Which of the data below would lead to this nursing judgment? A. Sudden dyspnea and confusion B. Hypertension C. Chills and fever D. Leg pain

What symptom described by a woman is characteristic of premenstrual syndrome (PMS)? A) "I feel irritable and moody a week before my period is supposed to start." B) "I have lower abdominal pain beginning the third day of my menstrual period." C) "I have nausea and headaches after my period starts, and they last 2 to 3 days." D) "I have abdominal bloating and breast pain after a couple days of my period."

A) "I feel irritable and moody a week before my period is supposed to start." PMS is a cluster of physical, psychologic, and behavioral symptoms that begin in the luteal phase of the menstrual cycle and resolve within a couple of days of the onset of menses. PMS begins in the luteal phase and resolves as menses occurs. It does not start after menses has begun. This complaint is associated with PMS. However, the timing reflected in this statement is inaccurate. PMS begins in the luteal phase and resolves as menses occurs. It does not start after menses has begun. Abdominal bloating and breast pain are likely to occur a few days prior to menses, not after it has begun.

Many common drugs of abuse cause significant physiologic and behavioral problems in infants who are breastfed by mothers currently using (Select all that apply): a. Amphetamine. b. Heroin. c. Nicotine. d. PCP. e. Morphine.

A, B, C, D (Amphetamine, heroin, nicotine, and PCP are contraindicated during breastfeeding because of the reported effects on the infant. Morphine is a medication that often is used to treat neonatal abstinence syndrome.)

Which conditions during pregnancy can result in preeclampsia in the patient? Select all that apply. A. Genetic abnormalities B. Dietary deficiencies C. Abnormal trophoblast invasion D. Cardiovascular changes E. Maternal hypotension

A, B, C, D (Current theories consider that genetic abnormalities and dietary deficiencies can result in preeclampsia. Abnormal trophoblast invasion causes fetal hypoxia and results in maternal hypertension. Cardiovascular changes stimulate the inflammatory system and result in preeclampsia in the pregnant patient. Maternal hypertension, and not hypotension, after 20 weeks' gestation is known as preeclampsia.)

Pain should be assessed regularly in all newborn infants. If the infant is displaying physiologic or behavioral cues indicating pain, measures should be taken to manage the pain. Examples of nonpharmacologic pain management techniques include (Select all that apply): a. Swaddling. b. Nonnutritive sucking. c. Skin-to-skin contact with the mother. d. Sucrose. e. Acetaminophen.

A, B, C, D (Swaddling, nonnutritive sucking, skin-to-skin contact with the mother, and sucrose are all appropriate nonpharmacologic techniques used to manage pain in neonates. Acetaminophen is a pharmacologic method of treating pain.)

26. Intrauterine growth restriction (IUGR) is associated with numerous pregnancy-related risk factors (Select all that apply). a. Poor nutrition b. Maternal collagen disease c. Gestational hypertension d. Premature rupture of membranes e. Smoking

A, B, C, E

27. Transvaginal ultrasonography is often performed during the first trimester. While preparing your 6-week gestation patient for this procedure, she expresses concerns over the necessity for this test. The nurse should explain that this diagnostic test may be indicated for a number of situations (Select all that apply). a. Multifetal gestation b. Obesity c. Fetal abnormalities d. Amniotic fluid volume e. Ectopic pregnancy

A, B, C, E

A nurse is discussing the signs and symptoms of mastitis with a mother who is breastfeeding. What signs and symptoms should the nurse include in her discussion (Select all that apply)? a. Breast tenderness b. Warmth in the breast c. An area of redness on the breast often resembling the shape of a pie wedge d. A small white blister on the tip of the nipple e. Fever and flulike symptoms

A, B, C, E (Breast tenderness, breast warmth, breast redness, and fever and flulike symptoms are commonly associated with mastitis and should be included in the nurse's discussion of mastitis. A small white blister on the tip of the nipple generally is not associated with mastitis. It is commonly seen in women who have a plugged milk duct.)

Which hypertensive disorders can occur during pregnancy? Select all that apply. A. Chronic hypertension B. Preeclampsia-eclampsia C. Hyperemesis gravidarum D. Gestational hypertension E. Gestational trophoblastic disease

A, B, D (Chronic hypertension refers to hypertension that developed in the pregnant patient before 20 weeks' gestation. Preeclampsia refers to hypertension and proteinuria that develops after 20 weeks' gestation. Eclampsia is the onset of seizure activity in a pregnant patient with preeclampsia. Gestational hypertension is the onset of hypertension after 20 weeks' gestation. Gestational trophoblastic disease and hyperemesis gravidarum are not hypertensive disorders. Gestational trophoblastic disease refers to a disorder without a viable fetus that is caused by abnormal fertilization. Hyperemesis gravidarum is excessive vomiting during pregnancy that may result in weight loss and electrolyte imbalance.)

The nurse is caring for a lactating patient with a body temperature of 102° F (38.9° C). The nurse finds that the patient's breasts are engorged, swollen, hard, and red. Which interventions related to patient care would be helpful in managing breast engorgement? Select all that apply. A. Taking warm showers before breastfeeding B. Nursing the baby frequently C. Using a tight supportive bra or a breast binder D. Applying cold cabbage leaves to the breasts E. Avoiding use of lanolin or hydrogel pads

A, B, D (Engorgement in a breastfeeding woman requires careful management to preserve the milk supply while managing the increased blood flow to the breasts. Taking warm showers can increase milk flow. Frequent feedings will permit the breasts to empty fully and establish the supply-demand cycle that is appropriate for the infant. Cold cabbage leaves work well to reduce pain and swelling and should be applied every 4 hours. Binding the breasts is not appropriate because it decreases the milk supply. To ease the discomfort associated with sore nipples, the mother may apply topical preparations such as purified lanolin or hydrogel pads.)

If a woman is at risk for thrombus and is not ready to ambulate, which nursing intervention would the nurse use? (Select all that apply.) a. Putting her in antiembolic stockings (thromboembolic deterrent [TED] hose) and/or sequential compression device (SCD) boots b. Having her flex, extend, and rotate her feet, ankles, and legs c. Having her sit in a chair d. Immediately notifying the physician if a positive Homans sign occurs e. Promoting bed rest

A, B, D Sitting immobile in a chair does not help; bed exercise and prophylactic footwear might. TED hose and SCD boots are recommended. The client should be encouraged to ambulate with assistance, not remain in bed. Bed exercises are useful. A positive Homans sign (calf muscle pain or warmth, redness, tenderness) requires the physicians immediate attention

A 24-year-old primipara, who is 18 weeks pregnant, has been having increasing vomiting since she was 8 weeks pregnant. Upon arrival at the emergency department, her skin turgor is diminished, temperature is 99.2F (o), pulse is 102, respiration is 18, blood pressure is 102/68, and she has deep furrows on her tongue. What would the nurse expect to do to care for her? Select all that apply. A. Start an intravenous infusion. B. Check her urine for ketones C. Cross match blood for a transfusion. D. Obtain a complete history. E. Obtain blood for a complete blood count

A, B, D, E (Whenever a pregnant woman has nausea and vomiting, the first priority is a thorough assessment to determine the severity of the problem. In most cases the woman should be told to come immediately to the health care provider's office or the emergency department because the severity of the illness often is difficult to determine by telephone conversation. The assessment should include frequency, severity, and duration of episodes of nausea and vomiting. If the woman reports vomiting, the assessment also should include the approximate amount and color of the vomitus. The woman is asked to report any precipitating factors relating to the onset of her symptoms. Any pharmacologic or nonpharmacologic treatment measures used should be recorded. Prepregnancy weight and documented weight gain or loss during pregnancy are important to note. The woman's weight and vital signs are measured, and a complete physical examination is performed, with attention to signs of fluid and electrolyte imbalance and nutritional status. The most important initial laboratory test to be obtained is a determination of ketonuria. Other laboratory tests that may be prescribed are a urinalysis, a complete blood cell count, electrolytes, liver enzymes, and bilirubin levels. At this time, there is no supportive evidence that a blood transfusion is required. Based on provided objective data that the patient has deep furrows on her tongue, this may suggest a vitamin B deficiency which should be investigated further.)

The nurse is caring for a woman who is at 24 weeks of gestation with suspected severe preeclampsia. Which signs and symptoms should the nurse expect to observe? Select all that apply. A. Decreased urinary output and irritability B. Transient headache and +1 proteinuria C. Ankle clonus and epigastric pain D. Platelet count of less than 100,000/mm3 and visual problems E. Seizure activity and hypotension

A, C, D (Decreased urinary output and irritability are signs of severe eclampsia. Ankle clonus and epigastric pain are signs of severe eclampsia. Platelet count of less than 100,000/mm3 and visual problems are signs of severe preeclampsia. A transient headache and +1 proteinuria are signs of preeclampsia and should be monitored.Seizure activity and hyperreflexia are signs of eclampsia. Test-Taking Tip: Do not worry if you select the same numbered answer repeatedly, because there usually is no pattern to the answers.)

A nurse is evaluating a woman's breastfeeding technique. Which of the following actions would indicate that the woman needs further instruction regarding breastfeeding to ensure success? (Circle all that apply.) a. Washes her breasts and nipples thoroughly with soap and water twice a day b. Massages a small amount of breast milk into her nipple and areola before and after each feeding c. Lines her bra with a thick plastic-lined pad to absorb leakage d. Positions baby supporting back and shoulders securely and then brings her breast toward the baby, putting the nipple in the baby's mouth e. Feeds her baby every 2 to 3 hours f. Inserts her finger into the corner of her baby's mouth between the gums before removing him from the breast

A, C, D (Nipples should not be washed using soap; plastic liners can keep nipples and areola moist and increase the risk for tissue breakdown; bring baby to breast, not breast to baby.)

Infants born between 34 0/7 and 36 6/7 weeks of gestation are called late-preterm infants because they have many needs similar to those of preterm infants. Because they are more stable than early-preterm infants, they may receive care that is much like that of a full-term baby. The mother-baby or nursery nurse knows that these babies are at increased risk for (Select all that apply): a. Problems with thermoregulation b. Cardiac distress c. Hyperbilirubinemia d. Sepsis e. Hyperglycemia

A, C, D (Thermoregulation problems, hyperbilirubinemia, and sepsis are all conditions related to immaturity and warrant close observation. After discharge the infant is at risk for rehospitalization related to these problems. AWHONN launched the Near-Term Infant Initiative to study the problem and ways to ensure that these infants receive adequate care. The nurse should ensure that this infant is feeding adequately before discharge and that parents are taught the signs and symptoms of these complications. Late-preterm infants are also at increased risk for respiratory distress and hypoglycemia.)

Which practices contribute to the prevention of postpartum infection? (Select all that apply.) a. Not allowing the mother to walk barefoot at the hospital b. Educating the client to wipe from back to front after voiding c. Having staff members with conditions such as strep throat, conjunctivitis, and diarrhea stay home d. Instructing the mother to change her perineal pad from front to back each time she voids or defecates e. Not permitting visitors with cough or colds to enter the postpartum unit

A, C, D Proper perineal care helps prevent infection and aids in the healing process. Educating the woman to wipe from front to back (urethra to anus) after voiding or defecating is a simple first step. Walking barefoot and getting back into bed can contaminate the linens. Clients should wear shoes or slippers. Staff members with infections need to stay home until they are no longer contagious. The client should also wash her hands before and after these functions. Visitors with any signs of illness should not be allowed entry to the postpartum unit.

Hearing loss is one of the genetic disorders included in the universal screening program. Auditory screening of all newborns within the first month of life is recommended by the American Academy of Pediatrics. Reasons for having this testing performed include (Select all that apply): a. Prevention or reduction of developmental delay. b. Reassurance for concerned new parents. c. Early identification and treatment. d. Helping the child communicate better. e. Recommendation by the Joint Committee on Infant Hearing.

A, C, D, E (New parents are often anxious regarding this test and the impending results; however, it is not the reason for the screening to be performed. Auditory screening is usually done before hospital discharge. It is important for the nurse to ensure that the infant receives the appropriate testing and that the test is fully explained to the parents. For infants who are referred for further testing and follow-up, it is important for the nurse to provide further explanation and emotional support. All other responses are appropriate reasons for auditory screening of the newborn. Infants who do not pass the screening test should have it repeated. If they still do not pass the test, they should have a full audiologic and medical evaluation by 3 months of age. If necessary, the infant should be enrolled in early intervention by 6 months of age.)

The nurse should teach a pregnant woman that which substances are teratogens? (Select all that apply) A. Cigarette smoke B. Isotretinoin (Retin A) C. Vitamin C D. Salicylic acid E. Rubella

A. Cigarette smoke B. Isotretinoin (Retin A) E. Rubella Vitamin C and salicylic acid are not known teratogens.

Many new mothers experience some type of nipple pain during the first weeks of initiating breastfeeding. Should this pain be severe or persistent, it may discourage or inhibit breastfeeding altogether. Which factors might contribute to this pain? (Select all that apply.) a. Improper feeding position b. Large-for-gestational age infant c. Fair skin d. Progesterone deficiency e. Flat or retracted nipples

A, C, E Nipple lesions may manifest as chapped, cracked, bleeding, sore, erythematous, edematous, or blistered nipples. Factors that contribute to nipple pain include improper positioning or a failure to break suction before removing the baby from the breast. Flat or retracted nipples along with the use of nipple shields, breast shells, or plastic breast pads also contribute to nipple pain. Women with fair skin are more likely to develop sore and cracked nipples. Preventing nipple soreness is preferable to treating soreness after it appears. Vigorous feeding may be a contributing factor, which may be the case with any size infant, not just infants who are large for gestational age. Estrogen or dietary deficiencies can contribute to nipple soreness.

47. The exact cause of breast cancer remains undetermined. Researchers have found that there are many common risk factors that increase a woman's chance of developing a malignancy. It is essential for the nurse who provides care to women of any age to be aware of which of the following risk factors (Select all that apply)? a. Family history b. Late menarche c. Early menopause d. Race e. Nulliparity or first pregnancy after age 30

A, D, E

Nurses play a critical role in educating parents regarding measures to prevent infant abduction. Which instructions contribute to infant safety and security? (Select all that apply.) a. The mother should check the photo identification (ID) of any person who comes to her room. b. The baby should be carried in the parents arms from the room to the nursery. c. Because of infant security systems, the baby can be left unattended in the clients room. d. Parents should use caution when posting photographs of their infant on the Internet. e. The mom should request that a second staff member verify the identity of any questionable person.

A, D, E Nurses must discuss infant security precautions with the mother and her family because infant abduction continues to be a concern. The mother should be taught to check the identity of any person who comes to remove the baby from her room. Hospital personnel usually wear picture identification patches. On some units, staff members also wear matching scrubs or special badges that are unique to the perinatal unit. As a rule, the baby is never carried in arms between the mothers room and the nursery, but rather the infant is always wheeled in a bassinet. The infant should never be left unattended, even if the facility has an infant security system. Parents should be instructed to use caution when posting photographs of their new baby on the Internet and on other public forums.

Induction of labor is considered an acceptable obstetric procedure if it is in the best interest to deliver the fetus. The charge nurse on the labor and delivery unit is often asked to schedule patients for this procedure and therefore must be cognizant of the specific conditions appropriate for labor induction. These include (Select all that apply): a. Rupture of membranes at or near term. b. Convenience of the woman or her physician. c. Chorioamnionitis (inflammation of the amniotic sac). d. Post-term pregnancy. e. Fetal death.

A,C,D,E

When managing the care of a woman in the second stage of labor, the nurse uses various measures to enhance the progress of fetal descent. These measures include: A. Encouraging the woman to try various upright positions, including squatting and standing. B. Telling the woman to start pushing as soon as her cervix is fully dilated. C. Continuing an epidural anesthetic so that pain is reduced and the woman can relax. D. Coaching the woman to use sustained, 10- to 15-second, closed-glottis bearing-down efforts with each contraction.

A. Encouraging the woman to try various upright positions, including squatting and standing. Upright positions and squatting may enhance the progress of fetal descent. Many factors dictate when a woman will begin pushing. Complete cervical dilation is necessary, but it is only one factor. If the fetal head is still in a higher pelvic station, the physician or midwife may allow the woman to "labor down" (allowing more time for fetal descent, thereby reducing the amount of pushing needed) if she is able. The epidural may mask the sensations and muscle control needed for the woman to push effectively. Closed-glottic breathing may trigger the Valsalva maneuver, which increases intrathoracic and cardiovascular pressure, reducing cardiac output and inhibiting perfusion of the uterus and placenta. In addition, holding the breath for longer than 5 to 7 seconds diminishes the perfusion of oxygen across the placenta, resulting in fetal hypoxia.

Most of the genetic tests now offered in clinical practice are tests for: A. Single-gene disorders. B. Carrier screening. C. Predictive values. D. Predispositional testing.

A. Single-gene disorders. Most tests now offered are tests for single-gene disorders in clients with clinical symptoms or who have a family history of a genetic disease. Carrier screening is used to identify individuals who have a gene mutation for a genetic condition but do not display symptoms. Predictive testing is used only to clarify the genetic status of asymptomatic family members. Predispositional testing differs from the other types of genetic screening in that a positive result does not indicate a 100% chance of developing the condition.

For the labor nurse, care of the expectant mother begins with which situations? (Select all that apply.) A. The onset of progressive, regular contractions B. The bloody, or pink, show C. The spontaneous rupture of membranes D. Formulation of the woman's plan of care for labor E. Moderately painful contractions

A. The onset of progressive, regular contractions B. The bloody, or pink, show C. The spontaneous rupture of membranes Labor care begins with the onset of progressive, regular contractions. The woman and the nurse can formulate their plan of care before labor or during treatment. Labor care begins when the blood-tinged mucoid vaginal discharge appears. The woman and the nurse can formulate their plan of care before labor or during treatment. Labor care begins when amniotic fluid is discharged from the vagina. The woman and the nurse can formulate their plan of care before labor or during treatment. Labor care begins when progressive, regular contractions begin, the blood-tinged mucoid vaginal discharge appears, or fluid is discharged from the vagina. The woman and the nurse can formulate their plan of care before labor or during treatment. Pain is subjective. The onset of progressive, regular contractions signals the beginning of labor; not the intensity of the pain.

Achieving and maintaining euglycemia comprise the primary goals of medical therapy for the pregnant woman with diabetes. These goals are achieved through a combination of diet, insulin, exercise, and blood glucose monitoring. The target blood glucose levels 1 hour after a meal should be: _________________

ANS: 130 to 140 mg/dL Target levels of blood glucose during pregnancy are lower than nonpregnant values. Accepted fasting levels are between 65 and 95 mg/dL, and 1-hour postmeal levels should be less than 130 to 140 mg/dL. Two-hour postmeal levels should be 120 mg/dL or less. PTS: 1 DIF: Cognitive Level: Application REF: 273 OBJ: Nursing Process: Implementation MSC: Client Needs: Health Promotion and Maintenance

Which presentation is accurately described in terms of both the resenting part and the frequency of occurrence? a.Cephalic: occiput, at least 96% b.Breech: sacrum, 10% to 15% c.Shoulder: scapula, 10% to 15% d.Cephalic: cranial, 80% to 85%

ANS: A In cephalic presentations (head first), the presenting part is the occiput; this presentation occurs in 96% of births. In a breech birth, the sacrum emerges first; this presentation occurs in approximately 3% of births. In shoulder presentations, the scapula emerges first; this presentation occurs in only 1% of births. In a cephalic presentation, the part of the head or cranium that emerges first is the occiput; cephalic presentations occur in 96% of births.

At least five factors affect the process of labor and birth. These are easily remembered as the five Ps. Which factors are included in this process? (Select all that apply.) a.Passenger b.Passageway c.Powers d.Pressure e.Psychologic response

ANS: A, B, C, E The five Ps are passenger (fetus and placenta), passageway (birth canal), powers (contractions), position of the mother, and psychologic response. Pressure is not one of the five Ps.

Which factors influence cervical dilation? (Select all that apply.) a.Strong uterine contractions b.Force of the presenting fetal part against the cervix c.Size of the woman d.Pressure applied by the amniotic sac e.Scarring of the cervix

ANS: A, B, D, E Dilation of the cervix occurs by the drawing upward of the musculofibrous components of the cervix, which is caused by strong uterine contractions. Pressure exerted by the amniotic fluid while the membranes are intact or by the force applied by the presenting part can also promote cervical dilation. Scarring of the cervix as a result of a previous infection or surgery may slow cervical dilation. Pelvic size or the size of the woman does not affect cervical dilation.

Diabetes refers to a group of metabolic diseases characterized by hyperglycemia resulting from defects in insulin action, insulin secretion, or both. Over time, diabetes causes significant changes in the microvascular and macrovascular circulations. These complications include: a. Atherosclerosis. b. Retinopathy. c. IUFD. d. Nephropathy. e. Autonomcs neuropathy.

ANS: A, B, D, E These structural changes are most likely to affect a variety of systems, including the heart, eyes, kidneys, and nerves. Intrauterine fetal death (stillbirth) remains a major complication of diabetes in pregnancy; however, this is a fetal complication. PTS: 1 DIF: Cognitive Level: Comprehension REF: 268 OBJ: Nursing Process: Diagnosis MSC: Client Needs: Physiologic Integrity

Which are nursing measures that can promote parent-infant bonding and attachment? (Select all that apply.) a. Provide comfort and ample time for rest. b. Keep the baby wrapped to avoid cold stress. c. Position the infant face to face with the mother. d. Point out the characteristics of the infant in a positive way. e. Limit the amount of modeling so the mother doesn't feel insecure.

ANS: A, C, D Provide comfort and ample time for rest, because the mother must replenish her energy and be relatively free of discomfort before she can progress to initiating care of the infant. Position the infant in an en face position and discuss the infant's ability to see the parent's face. Face to face and eye to eye contact is a first step in establishing mutual interaction between the infant and parent. Point out the characteristics of the infant in a positive way: "She has such pretty little hands and beautiful eyes." The baby should be kept warm, but parents should be assisted to unwrap the baby (keeping or rewrapping the body part not being inspected) to inspect the toes, fingers, and body. The nurse should model behaviors by holding the infant close, making eye contact with the infant, and speaking in high-pitched, soothing tones.

Signs that precede labor include (Select all that apply): a. Lightening. b. Exhaustion. c. Bloody show. d. Rupture of membranes. e. Decreased fetal movement.

ANS: A, C, D Signs that precede labor may include lightening, urinary frequency, backache, weight loss, surge of energy, bloody show, and rupture of membranes. Many women experience a burst of energy before labor. A decrease in fetal movement is an ominous sign that does not always correlate with labor.

In the United States today: a. More than 20% of pregnancies meet the definition of high risk to either the mother or the infant b. Other than biophysical criteria, sociodemographic factors place both the mother and baby at risk c. High risk pregnancy status extends from first confirmation of pregnancy to birth d. High risk pregnancy is less critical a medical concern because of the reduction in family size and the decrease in unwanted pregnancies

ANS: B Sociodemographic risks include lack of prenatal care, low income, marital status, and ethnicity. Approximately 500,000 of the 4 million births (12.5%) in the United States are categorized as high risk. The high risk status for the mother extends through 30 days after childbirth. The reduction in family size and the decrease in unwanted pregnancies, along with technologic advances that facilitate pregnancies in previously infertile couples and advance the potential of care, have enhanced emphasis on delivering babies safely.

Nurses can help their clients by keeping them informed about the distinctive stages of labor. Which description of the phases of the first stage of labor is accurate? a.Latent: Mild, regular contractions; no dilation; bloody show; duration of 2 to 4 hours b.Active: Moderate, regular contractions; 4- to 7-cm dilation; duration of 3 to 6 hours c.Lull: No contractions; dilation stable; duration of 20 to 60 minutes d.Transition: Very strong but irregular contractions; 8- to 10-cm dilation; duration of 1 to 2 hours

ANS: B The active phase is characterized by moderate, regular contractions; 4- to 7-cm dilation; and a duration of 3 to 6 hours. The latent phase is characterized by mild-to-moderate and irregular contractions; dilation up to 3 cm; brownish-to-pale pink mucus, and a duration of 6 to 8 hours. No official "lull" phase exists in the first stage. The transition phase is characterized by strong- to-very strong and regular contractions; 8- to 10-cm dilation; and a duration of 20 to 40 minutes.

The nurse has received a report regarding a client in labor. The woman's last vaginal examination was recorded as 3 cm, 30%, and -2. What is the nurse's interpretation of this assessment? a.Cervix is effaced 3 cm and dilated 30%; the presenting part is 2 cm above the ischial spines. b.Cervix is dilated 3 cm and effaced 30%; the presenting part is 2 cm above the ischial spines. c.Cervix is effaced 3 cm and dilated 30%; the presenting part is 2 cm below the ischial spines. d.Cervix is dilated 3 cm and effaced 30%; the presenting part is 2 cm below the ischial spines.

ANS: B The sterile vaginal examination is recorded as centimeters of cervical dilation, percentage of cervical dilation, and the relationship of the presenting part to the ischial spines (either above or below). For this woman, the cervix is dilated 3 cm and effaced 30%, and the presenting part is 2 cm above the ischial spines. The first interpretation of this vaginal examination is incorrect; the cervix is dilated 3 cm and is 30% effaced. However, the presenting part is correct at 2 cm above the ischial spines. The remaining two interpretations of this vaginal examination are incorrect. Although the dilation and effacement are correct at 3 cm and 30%, the presenting part is actually 2 cm above the ischial spines.

Concerning the use and abuse of legal drugs or substances, nurses should be aware that: a. Although cigarette smoking causes a number of health problems, it has little direct effect on maternity-related health. b. Caucasian women are more likely to experience alcohol-related problems. c. Coffee is a stimulant that can interrupt body functions and has been related to birth defects. d. Prescription psychotherapeutic drugs taken by the mother do not affect the fetus; otherwise, they would not have been prescribed.

ANS: B African-American and poor women are more likely to use illicit substances, particularly cocaine, whereas Caucasian and educated women are more likely to use alcohol. Cigarette smoking impairs fertility and is a cause of low birth weight. Caffeine consumption has not been related to birth defects. Psychotherapeutic drugs have some effect on the fetus, and that risk must be weighed against their benefit to the mother. PTS: 1 DIF: Cognitive Level: Knowledge REF: 297 OBJ: Nursing Process: Assessment MSC: Client Needs: Psychosocial Integrity

Diabetes in pregnancy puts the fetus at risk in several ways. Nurses should be aware that: a. With good control of maternal glucose levels, sudden and unexplained stillbirth is no longer a major concern. b. The most important cause of perinatal loss in diabetic pregnancy is congenital malformations. c. Infants of mothers with diabetes have the same risks for respiratory distress syndrome because of the careful monitoring. d. At birth the neonate of a diabetic mother is no longer in any risk.

ANS: B Congenital malformations account for 30% to 50% of perinatal deaths. Even with good control, sudden and unexplained stillbirth remains a major concern. Infants of diabetic mothers are at increased risk for respiratory distress syndrome. The transition to extrauterine life often is marked by hypoglycemia and other metabolic abnormalities. PTS: 1 DIF: Cognitive Level: Comprehension REF: 271 OBJ: Nursing Process: Diagnosis MSC: Client Needs: Physiologic Integrity

Preconception counseling is critical to the outcome of diabetic pregnancies because poor glycemic control before and during early pregnancy is associated with: a. Frequent episodes of maternal hypoglycemia. b. Congenital anomalies in the fetus. c. Polyhydramnios. d. Hyperemesis gravidarum.

ANS: B Preconception counseling is particularly important because strict metabolic control before conception and in the early weeks of gestation is instrumental in decreasing the risks of congenital anomalies. Frequent episodes of maternal hypoglycemia may occur during the first trimester (not before conception) as a result of hormone changes and the effects on insulin production and usage. Hydramnios occurs about 10 times more often in diabetic pregnancies than in nondiabetic pregnancies. Typically it is seen in the third trimester of pregnancy. Hyperemesis gravidarum may exacerbate hypoglycemic events because the decreased food intake by the mother and glucose transfer to the fetus contribute to hypoglycemia. PTS: 1 DIF: Cognitive Level: Comprehension REF: 270 OBJ: Nursing Process: Planning MSC: Client Needs: Physiologic Integrity

The nurse has received report regarding her patient in labor. The woman's last vaginal examination was recorded as 3 cm, 30%, and ?2-2. The nurse's interpretation of this assessment is that: a. The cervix is effaced 3 cm, it is dilated 30%, and the presenting part is 2 cm above the ischial spines. b. The cervix is 3 cm dilated, it is effaced 30%, and the presenting part is 2 cm above the ischial spines. c. The cervix is effaced 3 cm, it is dilated 30%, and the presenting part is 2 cm below the ischial spines. d. The cervix is dilated 3 cm, it is effaced 30%, and the presenting part is 2 cm below the ischial spines.

ANS: B The correct description of the vaginal examination for this woman in labor is the cervix is 3 cm dilated, it is effaced 30%, and the presenting part is 2 cm above the ischial spines. The sterile vaginal examination is recorded as centimeters of cervical dilation, percentage of cervical dilation, and the relationship of the presenting part to the ischial spines (either above or below).

When assessing the fetus using Leopold's maneuvers, the nurse feels a round, firm, and movable fetal part in the fundal portion of the uterus and a long, smooth surface in the mother's right side close to midline. What is the position of the fetus? a.ROA b.LSP c.RSA d.LOA

ANS: C Fetal position is denoted with a three-letter abbreviation. The first letter indicates the presenting part in either the right or the left side of the maternal pelvis. The second letter indicates the anatomic presenting part of the fetus. The third letter stands for the location of the presenting part in relationship to the anterior, posterior, or transverse portion of the maternal pelvis. Palpation of a round, firm fetal part in the fundal portion of the uterus would be the fetal head, indicating that the fetus is in a breech position with the sacrum as the presenting part in the maternal pelvis. Palpation of the fetal spine along the mother's right side denotes the location of the presenting part in the mother's pelvis. The ability to palpate the fetal spine indicates that the fetus is anteriorly positioned in the maternal pelvis. This fetus is anteriorly positioned in the right side of the maternal pelvis with the sacrum as the presenting part. RSA is the correct three-letter abbreviation to indicate this fetal position. ROA denotes a fetus that is anteriorly positioned in the right side of the maternal pelvis with the occiput as the presenting part. LSP describes a fetus that is posteriorly positioned in the left side of the pelvis with the sacrum as the presenting part. A fetus that is LOA would be anteriorly positioned in the left side of the pelvis with the occiput as the presenting part.

6.A 30-year-old gravida 3, para 2-0-0-2 is at 18 weeks of gestation. What screening test should be suggested to her? a. Biophysical profile b. Chorionic villi sampling c. Maternal serum alpha-fetoprotein (MSAFP) screening d. Screening for diabetes mellitus

ANS: C The biochemical assessment MSAFP test is performed from week 15 to week 20 of gestation (weeks 16 to 18 are ideal). A biophysical profile is a method of biophysical assessment of fetal well-being in the third trimester. Chorionic villi sampling is a biochemical assessment of the fetus that should be performed from the tenth to twelfth weeks of gestation. Screening for diabetes mellitus begins with the first prenatal visit.

A (500 mL in the first 24 hours after vaginal delivery.)

Early postpartum hemorrhage is defined as a blood loss greater than: a. 500 mL in the first 24 hours after vaginal delivery. b. 750 mL in the first 24 hours after vaginal delivery. c. 1000 mL in the first 48 hours after cesarean delivery. d. 1500 mL in the first 48 hours after cesarean delivery.

A labor and delivery nurse should be cognizant of which information regarding how the fetus moves through the birth canal? a.Fetal attitude describes the angle at which the fetus exits the uterus. b.Of the two primary fetal lies, the horizontal lie is that in which the long axis of the fetus is parallel to the long axis of the mother. c.Normal attitude of the fetus is called general flexion. d.Transverse lie is preferred for vaginal birth.

ANS: C The normal attitude of the fetus is called general flexion. The fetal attitude is the relationship of the fetal body parts to each one another. The horizontal lie is perpendicular to the mother; in the longitudinal (or vertical) lie, the long axes of the fetus and the mother are parallel. Vaginal birth cannot occur if the fetus stays in a transverse lie.

When assessing a woman in labor, the nurse is aware that the relationship of the fetal body parts to one another is called fetal: a. Lie. b. Presentation. c. Attitude. d. Position.

ANS: C Attitude is the relation of the fetal body parts to one another. Lie is the relation of the long axis (spine) of the fetus to the long axis (spine) of the mother. Presentation refers to the part of the fetus that enters the pelvic inlet first and leads through the birth canal during labor at term. Position is the relation of the presenting part to the four quadrants of the mother's pelvis.

_____ use/abuse during pregnancy causes vasoconstriction and decreased placental perfusion, resulting in maternal and neonatal complications. a. Alcohol b. Caffeine c. Tobacco d. Chocolate

ANS: C Smoking in pregnancy is known to cause a decrease in placental perfusion and has serious health risks, including bleeding complications, low birth weight, prematurity, miscarriage, stillbirth, and sudden infant death syndrome. Prenatal alcohol exposure is the single greatest preventable cause of mental retardation. Alcohol use during pregnancy can cause high blood pressure, miscarriage, premature birth, stillbirth, and anemia. Caffeine and chocolate may safely be consumed in small quantities during pregnancy. PTS: 1 DIF: Cognitive Level: Knowledge REF: 298 OBJ: Nursing Process: Assessment MSC: Client Needs: Health Promotion and Maintenance

When assessing the fetus using Leopold maneuvers, the nurse feels a round, firm, movable fetal part in the fundal portion of the uterus and a long, smooth surface in the mother's right side close to midline. What is the likely position of the fetus? a. ROA b. LSP c. RSA d. LOA

ANS: C The fetus is positioned anteriorly in the right side of the maternal pelvis with the sacrum as the presenting part. RSA is the correct three-letter abbreviation to indicate this fetal position. The first letter indicates the presenting part in either the right or left side of the maternal pelvis. The second letter indicates the anatomic presenting part of the fetus. The third letter stands for the location of the presenting part in relation to the anterior, posterior, or transverse portion of the maternal pelvis. Palpation of a round, firm fetal part in the fundal portion of the uterus would be the fetal head, indicating that the fetus is in a breech position with the sacrum as the presenting part in the maternal pelvis. Palpation of the fetal spine along the mother's right side denotes the location of the presenting part in the mother's pelvis. The ability to palpate the fetal spine indicates that the fetus is anteriorly positioned in the maternal pelvis.

With regard to the turns and other adjustments of the fetus during the birth process, known as the mechanism of labor, nurses should be aware that: a. The seven critical movements must progress in a more or less orderly sequence. b. Asynclitism sometimes is achieved by means of the Leopold maneuver. c. The effects of the forces determining descent are modified by the shape of the woman's pelvis and the size of the fetal head. d. At birth the baby is said to achieve "restitution" (i.e., a return to the C-shape of the womb).

ANS: C The size of the maternal pelvis and the ability of the fetal head to mold also affect the process. The seven identifiable movements of the mechanism of labor occur in combinations simultaneously, not in precise sequences. Asynclitism is the deflection of the baby's head; the Leopold maneuver is a means of judging descent by palpating the mother's abdomen. Restitution is the rotation of the baby's head after the infant is born.

In caring for a pregnant woman with sickle cell anemia, the nurse is aware that signs and symptoms of sickle cell crisis include: a. Anemia. b. Endometritis. c. Fever and pain. d. Urinary tract infection.

ANS: C Women with sickle cell anemia have recurrent attacks (crisis) of fever and pain, most often in the abdomen, joints, and extremities. These attacks are attributed to vascular occlusion when RBCs assume the characteristic sickled shape. Crises are usually triggered by dehydration, hypoxia, or acidosis. Women with sickle cell anemia are not iron deficient. Therefore, routine iron supplementation, even that found in prenatal vitamins, should be avoided in order to prevent iron overload. Women with sickle cell trait usually are at greater risk for postpartum endometritis (uterine wall infection); however, this is not likely to occur in pregnancy and is not a sign of crisis. These women are at an increased risk for UTIs; however, this is not an indication of sickle cell crisis. PTS: 1 DIF: Cognitive Level: Comprehension REF: 291 OBJ: Nursing Process: Assessment MSC: Client Needs: Physiologic Integrity

When caring for a pregnant woman with cardiac problems, the nurse must be alert for signs and symptoms of cardiac decompensation, which include: a. A regular heart rate and hypertension. b. An increased urinary output, tachycardia, and dry cough. c. Shortness of breath, bradycardia, and hypertension. d. Dyspnea; crackles; and an irregular, weak pulse.

ANS: D Signs of cardiac decompensation include dyspnea; crackles; an irregular, weak, rapid pulse; rapid respirations; a moist, frequent cough; generalized edema; increasing fatigue; and cyanosis of the lips and nail beds. A regular heart rate and hypertension are not generally associated with cardiac decompensation. Tachycardia would indicate cardiac decompensation, but increased urinary output and a dry cough would not. Shortness of breath would indicate cardiac decompensation, but bradycardia and hypertension would not. PTS: 1 DIF: Cognitive Level: Comprehension REF: 288 OBJ: Nursing Process: Assessment MSC: Client Needs: Physiologic Integrity

A new mother asks the nurse when the "soft spot" on her son's head will go away. The nurse's answer is based on the knowledge that the anterior fontanel closes after birth by _____ months. a. 2 b. 8 c. 12 d. 18

ANS: D The larger of the two fontanels, the anterior fontanel, closes by 18 months after birth.

Which assessment is not included in the fetal biophysical profile (BPP)? a. Fetal movement b. Fetal tone c. Fetal heart rate d. Amniotic fluid index e. Placental grade

ANS: E Fetal movement, tone, heart rate, and amniotic fluid index are all assessed in a BPP. The placental grade is determined by ultrasound (as is a BPP), but it is not included in the criteria of assessment factors for a BPP.

B (Most postpartum women often experience a "blue" period 2 days after childbirth, during which women may be emotional and cry for no explainable reason. This is called postpartum blues and is the natural hormonal reaction after giving birth. The nurse may instruct the patient to soak in a tub for 20 minutes on a regular basis to promote relaxation and help the patient cope with the postpartum blues. Postpartum women do not typically have trouble staying warm, so this is not the reason for the nurse's suggestion. Soaking in the tub does not facilitate eye contact, so the nurse would more likely encourage the mother to hold the baby en face position if this were the problem.)

After assessing a postpartum patient 2 days after childbirth, the nurse instructs the patient to soak in a warm water tub for 20 minutes every day. What is the most likely reason for this instruction? The patient: A. Is unable to stay warm. B. Cries easily for no apparent reason. C. Is joyful and has a feeling of well-being. D. Has trouble maintaining direct eye contact with the infant.

D (Provide time for the woman to bathe her infant after she views an infant bath demonstration)

After giving birth to a healthy infant boy, a primiparous woman, 16, is admitted to the postpartum unit. An appropriate nursing diagnosis for her at this time is "risk for impaired parenting related to deficient knowledge of newborn care." In planning for the woman's discharge, what should the nurse be certain to include in the plan of care? A. Tell the woman how to feed and bathe her infant B. Give the woman written information on bathing her infant C. Advise the woman that all mothers instinctively know how to care for their infants D. Provide time for the woman to bathe her infant after she views an infant bath demonstration

B (Exposure to nicotine from maternal smoking has been reported to increase the fetal S/D ratio. An elevated S/D ratio indicates a poorly perfused placenta. To improve the blood supply to the placenta, the patient should quit smoking as soon as possible. The AFV cannot be assessed through Doppler umbilical blood flow study. Moreover, smoking does not affect amniotic fluid volume. Smoking increases the S/D ratio; it does not decrease it.)

After reviewing the Doppler umbilical flow reports of a pregnant patient, the nurse advises the patient to quit smoking immediately. Which finding in the report could be the reason for this instruction? A. High amniotic fluid volume (AFV) B. High systolic-to-diastolic (S/D) ratio C. Low amniotic fluid volume (AFV) D. Low systolic-to-diastolic (S/D) ratio

D (If the BPP score is 8 to 10, then the test should be repeated weekly or twice weekly. If the BPP score is 0 to 2, then chronic asphyxia may be suspected. In this case the testing time should be extended to 120 minutes. If the BPP score is 4 after 36 weeks' gestation, then clinical conditions exist that may lead to an eminent delivery. If the BPP score is 4 before 32 weeks' gestation, the test should be repeated. If the BPP score is 6 at 36 to 37 weeks' gestation with positive fetal pulmonary testing, then delivery can be performed. If the BPP score is 6 before 36 weeks' gestation with negative pulmonary testing, then BPP can be repeated in 4 to 6 hours, and if oligohydramnios is present, then delivery can be done. The BPP provides an insight into fetal maturity and well-being and as such should be used as a diagnostic tool to plan and evaluate management of care. Findings are related to several factors involving both maternal and fetal characteristics.)

After reviewing the biophysical profile (BPP) reports of a pregnant patient close to term, the nurse advises the patient to repeat the test on a weekly basis. What BPP score did the nurse find in the report? A. 1 B. 4 C. 6 D. 9

A (An AFI less than 5 cm indicates oligohydramnios. Oligohydramnios is associated with intrauterine growth restriction and congenital anomalies. An AFI of 10 cm or greater indicates that the fetus is normal. AFI values between 5 and 10 cm are considered low normal, indicating a comparatively low risk for congenital anomalies. An AFI greater than 25 cm indicates polyhydramnios. This is associated with neural tube defects and obstruction of the fetal gastrointestinal tract.)

After reviewing the reports of a pregnant patient, the nurse infers that there might be a high risk for intrauterine growth restriction (IUGR). What could be the reason for this? The amniotic fluid index (AFI) is: A. Less than 5 cm. B. Equal to or more than 10 cm. C. Between 5 and 10 cm. D. More than 25 cm

C (Specialized or targeted ultrasound scans are performed only if a patient is suspected of carrying an anatomically or physiologically abnormal fetus. Limited ultrasound examination is used to estimate the amniotic fluid volume. Standard ultrasound scan is used to see the detailed anatomy of the fetus. Ultrasound scan is not used to find genetic abnormalities in the fetus.)

After reviewing the standard ultrasound scan reports of a pregnant patient, the nurse advises the patient to undergo a specialized ultrasound scan. What is the nurse's rationale for this suggestion? A. To estimate the amniotic fluid volume B. To identify the detailed fetal anatomy C. To assess for physiologic abnormalities D. To assess for fetal genetic abnormalities

B, D, E (The triple marker screen measures the levels of three maternal serum markers: unconjugated estriol, hCG, and MSAFP. Low values of unconjugated estriol, hCG, and MSAFP indicate that the fetus has trisomy 18. The quad screen has an additional serum marker: inhibin-A. A low inhibin-A level indicates the possibility of Down syndrome. NT is not a serum marker protein. Moreover, elevated NT indicates that the fetus has a chromosomal abnormality but does not specifically indicate that the fetus has trisomy 18.)

After reviewing the triple marker screen reports of a patient who is in the second trimester of pregnancy, the nurse concludes that the fetus has trisomy 18. What factors in the report led to the nurse's conclusion? Select all that apply. A. Low level of inhibin-A in the maternal serum B. Low level of unconjugated estriol in serum C. Elevated nuchal translucency (NT) in the fetus D. Low level of maternal human chorionic gonadotrophin (hCG) E. Low level of maternal serum alpha-fetoprotein (MSAFP)

D (Breastfeeding immediately after childbirth increases the release of the oxytocin hormone, which decreases blood loss and reduces the risk of postpartum hemorrhage. Therefore the nurse instructs the patient to immediately start breastfeeding. Oxytocin does not have any effect on hemorrhoids; therefore it does not help reduce the risk of hemorrhoids. Breastfeeding may not affect endometritis. Breastfeeding prevents infection in the child, but it does not prevent infection in the mother.)

After the delivery of a baby, the nurse instructs the patient to immediately start breastfeeding. Which complication is the nurse trying to prevent by giving this instruction? A. Hemorrhoids B. Endometritis C. Maternal infection D. Postpartum hemorrhage

D (Although the method of payment is important, obtaining this information is not the responsibility of the nurse. It is also of note that 14 states have mandated some form of insurance to assist couples with coverage for infertility. Risks of multiple gestation is indeed a risk of treatment of which the couple needs to be aware. To minimize the chance of multiple gestation, generally only three or fewer embryos are transferred. The couple should be informed that there may be a need for multifetal reduction. Nurses can provide anticipatory guidance on this matter. Depending on the therapy chosen, there may be a need for donor oocytes, sperm, embryos, or a surrogate mother. Couples who have excess embryos frozen for later transfer must be fully informed before consenting to the procedure. A decision must be made regarding the disposal of embryos in the event of death or divorce or if the couple no longer wants the embryos at a future time.)

Although remarkable developments have occurred in reproductive medicine, assisted reproductive therapies are associated with a number of legal and ethical issues. Nurses can provide accurate information about the risks and benefits of treatment alternatives so couples can make informed decisions about their choice of treatment. Which issue would not need to be addressed by an infertile couple before treatment? a. Risks of multiple gestation b. Whether or how to disclose the facts of conception to offspring c. Freezing embryos for later use d. Financial ability to cover the cost of treatment

D (A Mongolian spot is a bluish black area of pigmentation that may appear over any part of the exterior surface of the body. It is more commonly noted on the back and buttocks and most frequently is seen on infants whose ethnic origins are Mediterranean, Latin American, Asian, or African. Lanugo is the fine, downy hair seen on a term newborn. A vascular nevus, commonly called a strawberry mark, is a type of capillary hemangioma. A nevus flammeus, commonly called a port-wine stain, is most frequently found on the face.)

An African-American woman noticed some bruises on her newborn girl's buttocks. She asks the nurse who spanked her daughter. The nurse explains that these marks are called: a. Lanugo. b. Vascular nevi. c. Nevus flammeus. d. Mongolian spots.

B (The Ortolani maneuver is a technique for checking hip integrity. Unequal movement suggests that the hip is dislocated. The physician should be notified.)

An examiner who discovers unequal movement or uneven gluteal skin folds during the Ortolani maneuver would then: a. Tell the parents that one leg may be longer than the other, but they will equal out by the time the infant is walking. b. Alert the physician that the infant has a dislocated hip. c. Inform the parents and physician that molding has not taken place. d. Suggest that, if the condition does not change, surgery to correct vision problems may be needed.

D (Some spotting can normally occur as a result of the increased fragility and vascularity of the cervix and vagina during pregnancy. Intercourse can continue as long as the pregnancy is progressing normally. Safer-sex practices are always recommended; rupture of the membranes may require abstaining from intercourse. Uterine contractions that accompany orgasm can stimulate labor and would be problematic if the woman were at risk for or had a history of preterm labor.)

An expectant couple asks the nurse about intercourse during pregnancy and if it is safe for the baby. The nurse should tell the couple that: A. Intercourse should be avoided if any spotting from the vagina occurs afterward. B. Intercourse is safe until the third trimester. C. Safer-sex practices should be used once the membranes rupture. D. Intercourse and orgasm are often contraindicated if a history or signs of preterm labor are present.

C (Although this statement is appropriate, it does not answer the father's question. Mood swings are a normal finding in the first trimester; the woman does not need counseling. This is the most appropriate response since it gives an explanation and a time frame for when the mood swings may stop. This statement is judgmental and not appropriate.)

An expectant father confides in the nurse that his pregnant wife, 10 weeks of gestation, is driving him crazy. "One minute she seems happy, and the next minute she is crying over nothing at all. Is there something wrong with her?" The nurse's BEST response would be: A. "This is normal behavior and should begin to subside by the second trimester." B. "She may be having difficulty adjusting to pregnancy; I will refer her to a counselor that I know." C. "This is called emotional liability and is related to hormone changes and anxiety during pregnancy. The mood swings will eventually subside as she adjusts to being pregnant." D. "You seem impatient with her. Perhaps this is precipitating her behavior."

B (Disadvantages of IUDs include an increased risk of PID in the first 20 days after insertion and the risks of bacterial vaginosis and uterine perforation. The IUD offers no protection against STIs or the human immunodeficiency virus. Because this woman has multiple sex partners, she is at higher risk of developing an STI. The IUD does not protect against infection, as does a barrier method. Although "The IUD does not interfere with sex" may be correct, it is not the most appropriate response. The IUD offers no protection from STIs. The typical failure rate of the IUD ranges from 0.8% to 2%.)

An unmarried young woman describes her sex life as "active" and involving "many" partners. She wants a contraceptive method that is reliable and does not interfere with sex. She requests an intrauterine device (IUD). The nurse's most appropriate response is: a. "The IUD does not interfere with sex." b. "The risk of pelvic inflammatory disease (PID) will be higher for you." c. "The IUD will protect you from sexually transmitted infections (STIs)." d. "Pregnancy rates are high with the IUDs."

A (A newborn who has not voided in 24 hours may have any of a number of problems, some of which deserve the attention of the pediatrician. Formula-fed infants tend to void more frequently in the first 3 days; breastfed infants void less during this time because the mother's breast milk has not come in yet. Brick dust may be uric acid crystals; blood spotting could be caused by withdrawal of maternal hormones (pseudomenstruation) or a circumcision. The physician must be notified only if there is no apparent cause of bleeding. Weight loss from fluid loss may take 14 days to regain.)

As related to the normal functioning of the renal system in newborns, nurses should be aware that: a. The pediatrician should be notified if the newborn has not voided in 24 hours. b. Breastfed infants likely will void more often during the first days after birth. c. "Brick dust" or blood on a diaper is always cause to notify the physician. d. Weight loss from fluid loss and other normal factors should be made up in 4 to 7 days.

C (Excess fluid loss through other means occurs as well. Kidney function usually returns to normal in about a month. Diastasis recti abdominis is the separation of muscles in the abdominal wall; it has no effect on the voiding reflex. Bladder tone usually is restored 5 to 7 days after childbirth.)

As relates to the condition and reconditioning of the urinary system after childbirth, nurses should be aware that: A. Kidney function returns to normal a few days after birth. B. Diastasis recti abdominis is a common condition that alters the voiding reflex. C. Fluid loss through perspiration and increased urinary output accounts for a weight loss of more than 2 kg during the puerperium. D. With adequate emptying of the bladder, bladder tone usually is restored 2 to 3 weeks after childbirth.

A (A father typically goes through three phases of development to reach acceptance of fatherhood: the announcement phase, the moratorium phase, and the focusing phase. The father-child attachment can be as strong as the mother-child relationship and can also begin during pregnancy. In the last 2 months of pregnancy, many expectant fathers work hard to improve the environment of the home for the child. Typically the expectant father's ambivalence ends by the first trimester, and he progresses to adjusting to the reality of the situation and then to focusing on his role.)

As relates to the father's acceptance of the pregnancy and preparation for childbirth, the maternity nurse should know that: A. The father goes through three phases of acceptance of his own. B. The father's attachment to the fetus cannot be as strong as that of the mother because it does not start until after birth. C. In the last 2 months of pregnancy, most expectant fathers suddenly get very protective of their established lifestyle and resist making changes to the home. D. Typically men remain ambivalent about fatherhood right up to the birth of their child.

C (Amniocentesis would be performed to assess fetal lung maturity in the event of a preterm birth. Indications for PUBS include prenatal diagnosis or inherited blood disorders, karyotyping of malformed fetuses, detection of fetal infection, determination of the acid-base status of a fetus with intrauterine growth restriction, and assessment and treatment of isoimmunization and thrombocytopenia in the fetus. Typically, fetal size is determined by ultrasound during the second trimester and is not indicated in this scenario. NST measures the fetal response to fetal movement in a noncontracting mother.)

At 35 weeks of pregnancy a woman experiences preterm labor. Tocolytics are administered and she is placed on bed rest, but she continues to experience regular uterine contractions, and her cervix is beginning to dilate and efface. What would be an important test for fetal well-being at this time? A. Percutaneous umbilical blood sampling (PUBS) B. Ultrasound for fetal size C. Amniocentesis for fetal lung maturity D. Nonstress test (NST)

A maternal indication for the use of vacuum extraction is: a. A wide pelvic outlet. c. A history of rapid deliveries. b. Maternal exhaustion. d. Failure to progress past 0 station.

B

A pregnant woman at 29 weeks of gestation has been diagnosed with preterm labor. Her labor is being controlled with tocolytic medications. She asks when she would be able to go home. Which response by the nurse is most accurate? a. "After the baby is born." b. "When we can stabilize your preterm labor and arrange home health visits." c. "Whenever the doctor says that it is okay." d. "It depends on what kind of insurance coverage you have."

B

In planning for an expected cesarean birth for a woman who has given birth by cesarean previously and who has a fetus in the transverse presentation, which information would the nurse include? a. "Because this is a repeat procedure, you are at the lowest risk for complications." b. "Even though this is your second cesarean birth, you may wish to review the preoperative and postoperative procedures." c. "Because this is your second cesarean birth, you will recover faster." d. "You will not need preoperative teaching because this is your second cesarean birth."

B

Nurses should know some basic definitions concerning preterm birth, preterm labor, and low birth weight. For instance: a. The terms preterm birth and low birth weight can be used interchangeably. b. Preterm labor is defined as cervical changes and uterine contractions occurring between 20 and 37 weeks of pregnancy. c. Low birth weight is anything below 3.7 pounds. d. In the United States early in this century, preterm birth accounted for 18% to 20% of all births.

B

The nurse is caring for a client whose labor is being augmented with oxytocin. He or she recognizes that the oxytocin should be discontinued immediately if there is evidence of: a. Uterine contractions occurring every 8 to 10 minutes. b. A fetal heart rate (FHR) of 180 with absence of variability. c. The client's needing to void. d. Rupture of the client's amniotic membranes.

B

The nurse providing care for a woman with preterm labor who is receiving terbutaline would include which intervention to identify side effects of the drug? a. Assessing deep tendon reflexes (DTRs) b. Assessing for chest discomfort and palpitations c. Assessing for bradycardia d. Assessing for hypoglycemia

B

With regard to dysfunctional labor, nurses should be aware that: a. Women who are underweight are more at risk. b. Women experiencing precipitous labor are about the only "dysfunctionals" not to be exhausted. c. Hypertonic uterine dysfunction is more common than hypotonic dysfunction. d. Abnormal labor patterns are most common in older women.

B

A breastfeeding woman asks the nurse about what birth control she should use during the postpartum period. Which is the best recommendation for a safe, yet effective method during the first 6 weeks after birth? a. Combination oral contraceptive that she used before she was pregnant b. Barrier method using a combination of a condom and spermicide foam c. Resume using the diaphragm she used prior to getting pregnant d. Complete breastfeeding—baby only receives breast milk for nourishment

B (A combination hormonal contraceptive could decrease the milk supply if given before lactation is well established during the first 6 weeks after birth; after 6 weeks, a progestin-only contraceptive could be used because it is the least likely hormonal contraceptive to affect lactation; even complete breastfeeding is not considered to be a reliable method because ovulation can occur unexpectedly even before the first menstrual period; diaphragm used before pregnancy would have to be checked to see whether it fits properly before the woman uses it again.)

Necrotizing enterocolitis (NEC) is an acute inflammatory disease of the gastrointestinal mucosa that can progress to perforation of the bowel. Approximately 2% to 5% of premature infants succumb to this fatal disease. Care is supportive; however, known interventions may decrease the risk of NEC. To develop an optimal plan of care for this infant, the nurse must understand which intervention has the greatest effect on lowering the risk of NEC: a. Early enteral feedings b. Breastfeeding c. Exchange transfusion d. Prophylactic probiotics

B (A decrease in the incidence of NEC is directly correlated with exclusive breastfeeding. Breast milk enhances maturation of the gastrointestinal tract and contains immune factors that contribute to a lower incidence or severity of NEC, Crohn's disease, and celiac illness. The neonatal intensive care unit nurse can be very supportive of the mother in terms of providing her with equipment to pump breast milk, ensuring privacy, and encouraging skin-to-skin contact with the infant. Early enteral feedings of formula or hyperosmolar feedings are a risk factor known to contribute to the development of NEC. The mother should be encouraged to pump or feed breast milk exclusively. Exchange transfusion may be necessary; however, it is a known risk factor for the development of NEC. Although still early, a study in 2005 found that the introduction of prophylactic probiotics appeared to enhance the normal flora of the bowel and therefore decrease the severity of NEC when it did occur. This treatment modality is not as widespread as encouraging breastfeeding; however, it is another strategy that the care providers of these extremely fragile infants may have at their disposal.)

To prevent the abduction of newborns from the hospital, the nurse should: a. Instruct the mother not to give her infant to anyone except the one nurse assigned to her that day. b. Apply an electronic and identification bracelet to mother and infant. c. Carry the infant when transporting him or her in the halls. d. Restrict the amount of time infants are out of the nursery.

B (A measure taken by many facilities is to band both the mother and the baby with matching identification bracelets and band the infant with an electronic device that will alarm if the infant is removed from the maternity unit. It is impossible for one nurse to be on call for one mother and baby for the entire shift, so parents need to be able to identify the nurses who are working on the unit. Infants should always be transported in their bassinette, for both safety and security reasons. All maternity unit nursing staff should have unique identification bracelets in comparison with the rest of the hospital. Infants should remain with their parents and spend as little time in the nursery as possible.)

When caring for a newly delivered woman, the nurse is aware that the best measure to prevent abdominal distention after a cesarean birth is: a. Rectal suppositories. b. Early and frequent ambulation. c. Tightening and relaxing abdominal muscles. d. Carbonated beverages.

B (Activity will aid the movement of accumulated gas in the gastrointestinal tract. Rectal suppositories can be helpful after distention occurs; however, they do not prevent it. Ambulation is the best prevention. Carbonated beverages may increase distention.)

During a home visit, the mother of a 1-week-old infant son tells the nurse that she is very concerned about whether her baby is getting enough breast milk. The nurse would tell this mother that at 1 week of age a well-nourished newborn should exhibit which of the following? a. Weight gain sufficient to reach his birthweight b. A minimum of three bowel movements each day c. Approximately 10 to 12 wet diapers each day d. Breastfeeding at a frequency of every 4 hours or about 6 times each day

B (Birth weight is regained in 10 to 14 days; 6 to 8 wet diapers are expected at this time; should be fed every 2 to 3 hours for a total of 8 to 10 times per day.)

The most common cause of pathologic hyperbilirubinemia is: a. Hepatic disease. b. Hemolytic disorders in the newborn. c. Postmaturity. d. Congenital heart defect.

B (Hemolytic disorders in the newborn are the most common cause of pathologic jaundice. Hepatic damage may be a cause of pathologic hyperbilirubinemia, but it is not the most common cause. Prematurity would be a potential cause of pathologic hyperbilirubinemia in neonates, but it is not the most common cause. Congenital heart defect is not a common cause of pathologic hyperbilirubinemia in neonates.)

With regard to small for gestational age (SGA) infants and intrauterine growth restrictions (IUGR), nurses should be aware that: a. In the first trimester diseases or abnormalities result in asymmetric IUGR. b. Infants with asymmetric IUGR have the potential for normal growth and development. c. In asymmetric IUGR weight is slightly more than SGA, whereas length and head circumference are somewhat less than SGA. d. Symmetric IUGR occurs in the later stages of pregnancy.

B (IUGR is either symmetric or asymmetric. The symmetric form occurs in the first trimester; SGA infants have reduced brain capacity. The asymmetric form occurs in the later stages of pregnancy. Weight is less than the 10th percentile; head circumference is greater than the 10th percentile. Infants with asymmetric IUGR have the potential for normal growth and development.)

The nurse is assessing blood loss in a postpartum patient by observing the perineal pad. The nurse finds that 1.5 cm of the pad is saturated. What patient clinical observation should the nurse infer from this finding? A. Light bleeding B. Scanty bleeding C. Heavy bleeding D. Moderate bleeding

B (If the area of saturated pad is less than 2.5 cm, it indicates that the patient had scanty bleeding. If it is less than 10 cm, then the patient had light bleeding. If the pad is saturated within 2 hours, the patient had heavy bleeding. If it is 10 cm or more, the patient had moderate bleeding.)

To prevent nipple trauma, the nurse should instruct the new mother to: a. Limit the feeding time to less than 5 minutes. b. Position the infant so the nipple is far back in the mouth. c. Assess the nipples before each feeding. d. Wash the nipples daily with mild soap and water.

B (If the infant's mouth does not cover as much of the areola as possible, the pressure during sucking will be applied to the nipple, thus causing trauma to the area. Stimulating the breast for less than 5 minutes will not produce the extra milk the infant may need. This will also limit access to the higher-fat "hindmilk." Assessing the nipples for trauma is important; however, this action alone will not prevent sore nipples. Soap can be drying to the nipples and should be avoided during breastfeeding.)

The breastfeeding mother should be taught a safe method to remove the breast from the baby's mouth. Which suggestion by the nurse is most appropriate? a. Slowly remove the breast from the baby's mouth when the infant has fallen asleep and the jaws are relaxed. b. Break the suction by inserting your finger into the corner of the infant's mouth. c. A popping sound occurs when the breast is correctly removed from the infant's mouth. d. Elicit the Moro reflex to wake the baby and remove the breast when the baby cries.

B (Inserting a finger into the corner of the baby's mouth between the gums to break the suction avoids trauma to the breast. The infant who is sleeping may lose grasp on the nipple and areola, resulting in "chewing" on the nipple that makes it sore. A popping sound indicates improper removal of the breast from the baby's mouth and may cause cracks or fissures in the breast. Most mothers prefer the infant to continue to sleep after the feeding. Gentle wake-up techniques are recommended.)

A new father is ready to take his wife and newborn son home. He proudly tells the nurse who is discharging them that within the next week he plans to start feeding the infant cereal between breastfeeding sessions. The nurse can explain to him that beginning solid foods before 4 to 6 months may: a. Decrease the infant's intake of sufficient calories. b. Lead to early cessation of breastfeeding. c. Help the infant sleep through the night. d. Limit the infant's growth.

B (Introduction of solid foods before the infant is 4 to 6 months of age may result in overfeeding and decreased intake of breast milk. It is not true that feeding of solids helps infants sleep through the night. The proper balance of carbohydrate, protein, and fat for an infant to grow properly is in the breast milk or formula.)

The nurse's initial action when caring for an infant with a slightly decreased temperature is to: a. Notify the physician immediately. b. Place a cap on the infant's head and have the mother perform kangaroo care. c. Tell the mother that the infant must be kept in the nursery and observed for the next 4 hours. d. Change the formula because this is a sign of formula intolerance.

B (Keeping the head well covered with a cap will prevent further heat loss from the head, and having the mother place the infant skin to skin should increase the infant's temperature. Nursing actions are needed first to correct the problem. If the problem persists after interventions, notification may then be necessary. A slightly decreased temperature can be treated in the mother's room. This would be an excellent time for parent teaching on prevention of cold stress. Mild temperature instability is an expected deviation from normal during the first days as the infant adapts to external life.)

In caring for the preterm infant, what complication is thought to be a result of high arterial blood oxygen level? a. Necrotizing enterocolitis (NEC) b. Retinopathy of prematurity (ROP) c. Bronchopulmonary dysplasia (BPD) d. Intraventricular hemorrhage (IVH)

B (ROP is thought to occur as a result of high levels of oxygen in the blood. NEC is caused by the interference of blood supply to the intestinal mucosa. Necrotic lesions occur at that site. BPD is caused by the use of positive pressure ventilation against the immature lung tissue. IVH results from rupture of the fragile blood vessels in the ventricles of the brain. It is most often associated with hypoxic injury, increased blood pressure, and fluctuating cerebral blood flow.)

During labor a fetus with an average heart rate of 135 beats/min over a 10-minute period would be considered to have: A. Bradycardia B. A normal baseline heart rate C. Tachycardia D. Hypoxia

B A. Incorrect: Bradycardia is an FHR below 110 beats/min for 10 minutes or longer. B. Correct: The baseline heart rate is measured over 10 minutes; a normal range is 110 to 160 beats/min. C. Incorrect: Tachycardia is an FHR over 160 beats/min for 10 minutes or longer. D. Incorrect: Hypoxia is an inadequate supply of oxygen; no indication of this condition exists with a baseline heart rate in the normal range. p. 502

A pregnant patient has a systolic blood pressure that exceeds 160 mm Hg. Which action should the nurse take for this patient? A. Administer magnesium sulfate intravenously. B. Obtain a prescription for antihypertensive medications. C. Restrict intravenous and oral fluids to 125 mL/hr. D. Monitor fetal heart rate (FHR) and uterine contractions (UCs).

B (Systolic blood pressure exceeding 160 mm Hg indicates severe hypertension in the patient. The nurse should alert the health care provider and obtain a prescription for antihypertensive medications, such as nifedipine (Adalat) and labetalol hydrochloride (Normodyne). Magnesium sulfate would be administered if the patient was experiencing eclamptic seizures. Oral and intravenous fluids are restricted when the patient is at risk for pulmonary edema. Monitoring FHR and UCs is a priority when the patient experiences a trauma so that any complications can be addressed immediately.)

A new father wants to know what medication was put into his infant's eyes and why it is needed. The nurse explains to the father that the purpose of the Ilotycin ophthalmic ointment is to: a. Destroy an infectious exudate caused by Staphylococcus that could make the infant blind. b. Prevent gonorrheal and chlamydial infection of the infant's eyes potentially acquired from the birth canal. c. Prevent potentially harmful exudate from invading the tear ducts of the infant's eyes, leading to dry eyes. d. Prevent the infant's eyelids from sticking together and help the infant see.

B (The purpose of the Ilotycin ophthalmic ointment is to prevent gonorrheal and chlamydial infection of the infant's eyes potentially acquired from the birth canal. Prophylactic ophthalmic ointment is instilled in the eyes of all neonates to prevent gonorrheal or chlamydial infection. Prophylactic ophthalmic ointment is not instilled to prevent dry eyes. Prophylactic ophthalmic ointment has no bearing on vision other than to protect against infection that may lead to vision problems.)

A 24-year-old primipara, 10 weeks pregnant, who has been experiencing vomiting every morning for the past few weeks, asks the nurse at her check-up how long this "morning sickness" will continue. Which statement by the nurse is most accurate? A. "It will end by the 15th week of pregnancy." B. "It usually subsides by the 20th week of pregnancy." C. "It's a very common but not serious problem." D. "In some women, it can last throughout the pregnancy and become serious."

B (This discomfort of pregnancy usually subsides by the 20th week of pregnancy. An absolute definite end of vomiting during pregnancy can never be stated. Test-Taking Tip: Identifying content and what is being asked about that content is critical to your choosing the correct response. Be alert for words in the stem of the item that are the same or similar in nature to those in one or two of the options. Example: If the item relates to and identifies stroke rehabilitation as its focus and only one of the options contains the word stroke in relation to rehabilitation, you are safe in identifying this choice as the correct response.)

A woman with severe preeclampsia is receiving a magnesium sulfate infusion. The nurse becomes concerned after assessment when the woman exhibits: A. a sleepy, sedated affect. B. a respiratory rate of 10 breaths/min. C. deep tendon reflexes of 2+. D. absent ankle clonus.

B (a respiratory rate of 10 breaths/min. Because magnesium sulfate is a central nervous system (CNS) depressant, the client will most likely become sedated when the infusion is initiated. A respiratory rate of 10 breaths/min indicates that the client is experiencing respiratory depression (bradypnea) from magnesium toxicity. Deep tendon reflexes of 2+ are a normal finding. Absent ankle clonus is a normal finding.)

The most prevalent clinical manifestation of abruptio placentae (as opposed to placenta previa) is: A. bleeding. B. intense abdominal pain. C. uterine activity. D. cramping.

B (intense abdominal pain. Bleeding may be present in varying degrees for both placental conditions. Pain is absent with placenta previa and may be agonizing with abruptio placentae. Uterine activity may be present with both placental conditions. Cramping is a form of uterine activity that may be present in both placental conditions.)

A woman with severe preeclampsia is being treated with an IV infusion of magnesium sulfate. This treatment is considered successful if: A. blood pressure is reduced to prepregnant baseline. B. seizures do not occur. C. deep tendon reflexes become hypotonic. D. diuresis reduces fluid retention.

B (seizures do not occur. A temporary decrease in blood pressure can occur; however, this is not the purpose of administering this medication. Magnesium sulfate is a central nervous system (CNS) depressant given primarily to prevent seizures. Hypotonia is a sign of an excessive serum level of magnesium. It is critical that calcium gluconate be on hand to counteract the depressant effects of magnesium toxicity. Diuresis is not an expected outcome of magnesium sulfate administration.)

The nurse providing care for the laboring woman should understand that variable FHR decelerations are caused by: A. Altered fetal cerebral blood flow B. Umbilical cord compression C. Uteroplacental insufficiency D. Fetal hypoxemia

B A. Incorrect: Altered fetal cerebral blood flow would result in early decelerations in the FHR. B. Correct: Variable decelerations can occur any time during the uterine contracting phase and are caused by compression of the umbilical cord. C. Incorrect: Uteroplacental insufficiency would result in late decelerations in the FHR. D. Incorrect: Fetal hypoxemia would result in tachycardia initially, then bradycardia if hypoxia continues. p. 507

A new client and her partner arrive on the labor, delivery, recovery, and postpartum (LDRP) unit for the birth of their first child. You apply the EFM to the woman. Her partner asks you to explain what is printing on the graph, referring to the EFM strip. He wants to know what the baby's heart rate should be. Your best response is: A. "Don't worry about that machine; that's my job." B. "The top line graphs the baby's heart rate. Generally, the heart rate is between 110 and 160. The heart rate will fluctuate in response to what is happening during labor." C. "The top line graphs the baby's heart rate, and the bottom line lets me know how strong the contractions are." D. "Your doctor will explain all of that later."

B A. Incorrect: This discredits the partner's feelings and does not provide the teaching he is requesting. B. Correct: This statement educates the partner about fetal monitoring and provides support and information to alleviate his fears. C. Incorrect: This statement provides inaccurate information and does not address the partner's concerns about the fetal heart rate. The EFM graphs the frequency and duration of the contractions, not the intensity. D. Incorrect: Nurses should take every opportunity to provide client and family teaching, especially when information is requested. pp. 501-502

A woman gave birth 48 hours ago to a healthy infant girl. She has decided to bottle feed. During the assessment, the nurse notices that both breasts are swollen, warm, and tender on palpation. Which guidance should the nurse provide to the client at this time? a. Run warm water on her breasts during a shower. b. Apply ice to the breasts for comfort. c. Express small amounts of milk from the breasts to relieve the pressure. d. Wearing a loose-fitting bra to prevent nipple irritation.

B Applying ice packs and cabbage leaves to the breasts for comfort is an appropriate intervention for treating engorgement in a mother who is bottle feeding. The ice packs should be applied for 15 minutes on and 45 minutes off to avoid rebound engorgement. A bottle-feeding mother should avoid any breast stimulation, including pumping or expressing milk. A bottle-feeding mother should continuously wear a well-fitted support bra or breast binder for at least the first 72 hours after giving birth. A loose-fitting bra will not aid lactation suppression. Furthermore, the shifting of the bra against the breasts may stimulate the nipples and thereby stimulate lactation.

What information should the nurse understand fully regarding rubella and Rh status? a. Breastfeeding mothers cannot be vaccinated with the live attenuated rubella virus. b. Women should be warned that the rubella vaccination is teratogenic and that they must avoid pregnancy for at least 1 month after vaccination. c. Rh immunoglobulin is safely administered intravenously because it cannot harm a nursing infant. d. Rh immunoglobulin boosts the immune system and thereby enhances the effectiveness of vaccinations.

B Women should understand that they must practice contraception for at least 1 month after being vaccinated. Because the live attenuated rubella virus is not communicable in breast milk, breastfeeding mothers can be vaccinated. Rh immunoglobulin is administered intramuscular (IM); it should never be administered to an infant. Rh immunoglobulin suppresses the immune system and therefore might thwart the rubella vaccination

With regard to the diagnosis and management of amenorrhea, nurses should be aware that: A) it probably is the result of a hormone deficiency that can be treated with medication. B) it may be caused by stress or excessive exercise or both. C) it likely will require the client to eat less and exercise more. D) it often goes away on its own.

B) it may be caused by stress or excessive exercise or both. Amenorrhea may be the result of a decrease in follicle-stimulating hormone (FSH) and luteinizing hormone (LH). This is usually caused by stress, body fat to lean ratio, and in rare occurrences a pituitary tumor. It cannot be treated by medication. Amenorrhea usually is the result of stress and/or an inappropriate ratio of body fat to lean tissue, possibly as a result of excessive exercise. Management includes counseling and education about the causes and possible lifestyle changes. In most cases a client will need to decrease her amount of exercise and increase her body weight in order to resume menstruation. Management of stress and eating disorders is usually necessary to manage this condition.

When providing care to a young single woman just diagnosed with acute pelvic inflammatory disease, the nurse should: A) point out that inappropriate sexual behavior caused the infection. B) position the woman in a semi-Fowler position. C) explain to the woman that infertility is a likely outcome of this type of infection. D) tell her that antibiotics need to be taken until pelvic pain is relieved.

B) position the woman in a semi-Fowler position. Although sexual behavior may have contributed to the infection, the nurse must discuss these practices in a nonjudgmental manner and provide information about prevention measures. The position of comfort is the semi-Fowler position. In addition, the foot of the bed could be elevated to keep the uterus in a dependent position and reduce discomfort. Until treatment is complete and healing has occurred, the outcome is unknown and should not be suggested. The nurse should emphasize that medication must be continued until follow-up assessment indicates that the infection has been treated successfully.

The emergency department nurse is assessing a pregnant trauma victim who just arrived at the hospital. What are the nurse's MOST appropriate actions? (Select all that apply.) A. Place the patient in a supine position. B. Assess for point of maximal impulse at fourth intercostal space. C. Collect urine for urinalysis and culture. D. Frequent vital sign monitoring. E. Assist with ambulation to decrease risk of thrombosis.

B, C, D (Passive regurgitation may occur if patient is supine, leading to high risk for aspiration. Placental perfusion is decreased when the patient is in a supine position as well. The heart is displaced upward and to the left in pregnant patients. During pregnancy, there is dilation of the ureters and urethra, and the bladder is displaced forward placing the pregnant trauma patient at higher risk for urinary stasis, infection, and bladder trauma. The trauma patient can suffer blood loss and other complications, necessitating frequent monitoring of vital signs. While the pregnant patient is at risk for thrombus formation, the patient must be cleared by the health care provider before ambulating. The pregnant trauma patient is at higher risk for pelvic fracture, and therefore this condition must be ruled out first as well.)

A pregnant patient with severe preeclampsia who is being transported to a tertiary care center needs to be administered magnesium sulfate injection for seizure activity. What actions does the nurse take when administering the drug? Select all that apply. A. A 10-g dose is administered in the buttock. B. A local anesthetic is added to the solution. C. The Z-track technique is used to inject the drug. D. The injection site is massaged after the injection. E. The subcutaneous route is used to inject the drug.

B, C, D (The nurse adds a local anesthetic to the solution to reduce pain that is caused by the injection. The Z-track technique is used to inject the drug so that the drug is injected in the intramuscular (IM) tissue safely. The nurse gently massages the site after administering the injection to reduce pain. The nurse administers two separate injections of 5 g in each buttock. Magnesium sulfate injections are administered in the IM layer and not the subcutaneous layer.)

A nurse counseling a client with endometriosis understands which statements regarding the management of endometriosis is accurate? (Select all that apply) A) Bone loss from hypoestrogenism is not reversible. B) Side effects from the steroid danazol include masculinizing traits. C) Surgical intervention often is needed for severe or acute symptoms. D) Women without pain and who do not want to become pregnant need no treatment. E) Women with mild pain who may want a future pregnancy may take nonsteroidal antiinflammatory drugs (NSAIDs).

B, C, D B) Side effects from the steroid danazol include masculinizing traits, C) Surgical intervention often is needed for severe or acute symptoms and D) Women without pain and who do not want to become pregnant need no treatment. Bone loss is mostly reversible within 12 to 18 months after the medication is stopped. Such masculinizing traits as hirsutism, a deepening voice, and weight gain occur with danazol but are reversible. Surgical intervention often is needed when symptoms are incapacitating. The type of surgery is influenced by the woman's age and desire to have children. Treatment is not needed for women without pain or the desire to have children. In women with mild pain who may desire a future pregnancy, treatment may be limited to use of NSAIDs during menstruation.

A pregnant woman presents to the emergency department complaining of persistent nausea and vomiting. She is diagnosed with hyperemesis gravidarum. The nurse should include which information when teaching about diet for hyperemesis? (Select all that apply.) A. Eat three larger meals a day. B. Eat a high-protein snack at bedtime. C. Ice cream may stay down better than other foods. D. Avoid ginger tea or sweet drinks. E. Eat what sounds good to you even if your meals are not well-balanced.

B, C, E (The diet for hyperemesis includes: • Avoid an empty stomach. Eat frequently, at least every 2 to 3 hours. Separate liquids from solids and alternate every 2 to 3 hours. • Eat a high-protein snack at bedtime. • Eat dry, bland, low-fat, and high-protein foods. Cold foods may be better tolerated than those served at a warm temperature. • In general eat what sounds good to you rather than trying to balance your meals. • Follow the salty and sweet approach; even so-called junk foods are okay. • Eat protein after sweets. • Dairy products may stay down more easily than other foods. • If you vomit even when your stomach is empty, try sucking on a Popsicle. • Try ginger tea. Peel and finely dice a knuckle-sized piece of ginger and place it in a mug of boiling water. Steep for 5 to 8 minutes and add brown sugar to taste. • Try warm ginger ale (with sugar, not artificial sweetener) or water with a slice of lemon. • Drink liquids from a cup with a lid.)

A pregnant woman presents to the emergency department complaining of persistent nausea and vomiting. She is diagnosed with hyperemesis gravidarum. The nurse should include which information when teaching about diet for hyperemesis? Select all that apply. A. Eat three larger meals a day. B. Eat a high-protein snack at bedtime. C. Ice cream may stay down better than other foods. D. Avoid ginger tea or sweet drinks. E. Eat what sounds good to you even if your meals are not well-balanced.

B, C, E (The diet for hyperemesis includes: (1) Avoid an empty stomach. Eat frequently, at least every 2 to 3 hours. Separate liquids from solids and alternate every 2 to 3 hours. (2) Eat a high-protein snack at bedtime. (3) Eat dry, bland, low-fat, and high-protein foods. Cold foods may be better tolerated than those served at a warm temperature. (4) In general eat what sounds good to you rather than trying to balance your meals. (5) Follow the salty and sweet approach; even so-called junk foods are okay. (6) Eat protein after sweets. (7) Dairy products may stay down more easily than other foods. (8) If you vomit even when your stomach is empty, try sucking on a Popsicle. (9) Try ginger tea. Peel and finely dice a knuckle-sized piece of ginger and place it in a mug of boiling water. Steep for 5 to 8 minutes and add brown sugar to taste. (10) Try warm ginger ale (with sugar, not artificial sweetener) or water with a slice of lemon. (11) Drink liquids from a cup with a lid.)

Postpartum fatigue (PPF) is more than just feeling tired. It is a complex phenomenon affected by physiologic, psychologic, and situational variables. Which factors contribute to this phenomenon? (Select all that apply.) a. Precipitous labor b. Hospital routines c. Bottle feeding d. Anemia e. Excitement

B, D, E Physical fatigue and exhaustion are often associated with a long labor or cesarean birth, hospital routines, breastfeeding, and infant care. PPF is also attributed to anemia, infection, or thyroid dysfunction. The excitement and exhilaration of delivering a new infant along with well-intentioned visitors may make rest difficult.

The nurse should include which information when teaching a 15-year-old about genital tract infection prevention? (Select all that apply.) A) Wear nylon undergarments. B) Avoid tight-fitting jeans. C) Use floral scented bath salts. D) Decrease sugar intake. E) Do not douche. F) Limit time spent wearing a wet bathing suit

B, D, E, F B) Avoid tight-fitting jeans, D) Decrease sugar intake, E) Do not douche & F) Limit time spent wearing a wet bathing suit Patient teaching for the prevention of genital tract infections in women includes the following guidelines: • Practice genital hygiene. • Choose underwear or hosiery with a cotton crotch. • Avoid tight-fitting clothing (especially tight jeans). • Select cloth car seat covers instead of vinyl. • Limit the time spent in damp exercise clothes (especially swimsuits, leotards, and tights). • Limit exposure to bath salts or bubble bath. • Avoid colored or scented toilet tissue. • If sensitive, discontinue use of feminine hygiene deodorant sprays. • Use condoms. • Void before and after intercourse. • Decrease dietary sugar. • Drink yeast-active milk and eat yogurt (with lactobacilli). • Do not douche.

A patient who has had a cesarean birth has been on bed rest for 8 hours after surgery and has warmth and redness in the left lower limb. Which interventions taken by the nurse would be most beneficial to the patient? Select all that apply. A. Advise the patient to apply a hot compress at the reddened site. B. Inform the primary health care provider (PHP) about the patient's condition immediately. C. Advise the patient to apply an antinflammatory ointment at the reddened site. D. Have the patient sit upright and lower the reddened leg. E. Have the patient remain in bed with reddened limb elevated on pillows.

B, E (A patient who has had a cesarean birth and has remained in the bed for more than 8 hours is at risk of venous thromboembolism. If a thrombus is suspected, as evidenced by warmth, redness, or tenderness in the leg, the nurse should notify the PHP immediately. Meanwhile, the patient should remain in bed with the affected limb elevated on pillows. Applying heat increases discomfort because the affected limb is already warm. Applying antiinflammatory ointment to the leg at the reddened site would not be useful because the redness is caused by embolism, not inflammation.)

A woman is 8 months pregnant. She tells the nurse that she knows her baby listens to her, but her husband thinks she is imagining things. Which response by the nurse is most appropriate? A. "Many women imagine what their baby is like." B. "A baby in utero does respond to the mother's voice." C. "You'll need to ask the doctor if the baby can hear yet." D. "Thinking that your baby hears will help you bond with the baby."

B. "A baby in utero does respond to the mother's voice." Although this statement is accurate, it is not the most appropriate response. Fetuses respond to sound by 24 weeks. The fetus can be soothed by the sound of the mother's voice. This statement is not appropriate. The mother should be instructed that her fetus can hear at 24 weeks and can respond to the sound of her voice. The statement is not appropriate. It gives the impression that her baby cannot hear her. It also belittles the mother's interpretation of her fetus's behaviors.

With regard to abnormalities of chromosomes, nurses should be aware that: A. They occur in approximately 10% of newborns. B. Abnormalities of number are the leading cause of pregnancy loss. C. Down syndrome is a result of an abnormal chromosomal structure. D. Unbalanced translocation results in a mild abnormality that the child will outgrow.

B. Abnormalities of number are the leading cause of pregnancy loss. Chromosomal abnormalities occur in less than 1% of newborns. Aneuploidy is an abnormality of number that also is the leading genetic cause of mental retardation. Down syndrome is the most common form of trisomal abnormality, an abnormality of chromosome number (47 chromosomes). Unbalanced translocation is an abnormality of chromosome structure that often has serious clinical effects.

Which test is performed to determine if membranes are ruptured? A. Urine analysis B. Fern test C. Leopold maneuvers D. Artificial Rupture of Membranes (AROM)

B. Fern test A urine analysis should be performed on admission to labor and delivery. This test is used to identify the presence of glucose and protein. In many instances a sterile speculum examination and a Nitrazine (pH) and fern test are performed to confirm that fluid seepage is indeed amniotic fluid. The nurse performs Leopold maneuvers to identify fetal lie, presenting part, and attitude. AROM is the procedure of artificially rupturing membranes, usually with a device known as an amnihook.

E (NOT: Each pregnancy problem can be attributed to a number of related risk factors. Polyhydramnios may also be the result of poorly controlled diabetes mellitus. Other maternal causes of IUGR include hypertensive disorders, diabetes, chronic renal disease, vascular disease, thrombophilia, poor weight gain, and cyanotic heart disease. Fetoplacental causes of IUGR may be related to chromosomal abnormalities, congenital malformations, intrauterine infection, or genetic syndromes. Other contributors to oligohydramnios are renal agenesis, prolonged pregnancy, uteroplacental insufficiency, and paternal hypertensive disorders. Although advanced maternal age is a well-known cause of chromosomal abnormalities, other causes include parental chromosome rearrangements and pregnancy with autosomal trisomy.)

Biophysical risks include factors that originate with either the mother or the fetus and affect the functioning of either one or both. The nurse who provides prenatal care should have an understanding of these risk factors. Match the specific pregnancy problem with the related risk factor. Abnormal placenta development A. Polyhydramnios B. Intrauterine growth restriction (maternal cause) C. Oligohydramnios D. Chromosomal abnormalities E. Intrauterine growth restriction (fetoplacental cause)

D (NOT: Each pregnancy problem can be attributed to a number of related risk factors. Polyhydramnios may also be the result of poorly controlled diabetes mellitus. Other maternal causes of IUGR include hypertensive disorders, diabetes, chronic renal disease, vascular disease, thrombophilia, poor weight gain, and cyanotic heart disease. Fetoplacental causes of IUGR may be related to chromosomal abnormalities, congenital malformations, intrauterine infection, or genetic syndromes. Other contributors to oligohydramnios are renal agenesis, prolonged pregnancy, uteroplacental insufficiency, and paternal hypertensive disorders. Although advanced maternal age is a well-known cause of chromosomal abnormalities, other causes include parental chromosome rearrangements and pregnancy with autosomal trisomy.)

Biophysical risks include factors that originate with either the mother or the fetus and affect the functioning of either one or both. The nurse who provides prenatal care should have an understanding of these risk factors. Match the specific pregnancy problem with the related risk factor. Advanced maternal age A. Polyhydramnios B. Intrauterine growth restriction (maternal cause) C. Oligohydramnios D. Chromosomal abnormalities E. Intrauterine growth restriction (fetoplacental cause)

A (NOT: Each pregnancy problem can be attributed to a number of related risk factors. Polyhydramnios may also be the result of poorly controlled diabetes mellitus. Other maternal causes of IUGR include hypertensive disorders, diabetes, chronic renal disease, vascular disease, thrombophilia, poor weight gain, and cyanotic heart disease. Fetoplacental causes of IUGR may be related to chromosomal abnormalities, congenital malformations, intrauterine infection, or genetic syndromes. Other contributors to oligohydramnios are renal agenesis, prolonged pregnancy, uteroplacental insufficiency, and paternal hypertensive disorders. Although advanced maternal age is a well-known cause of chromosomal abnormalities, other causes include parental chromosome rearrangements and pregnancy with autosomal trisomy.)

Biophysical risks include factors that originate with either the mother or the fetus and affect the functioning of either one or both. The nurse who provides prenatal care should have an understanding of these risk factors. Match the specific pregnancy problem with the related risk factor. Fetal congenital anomalies A. Polyhydramnios B. Intrauterine growth restriction (maternal cause) C. Oligohydramnios D. Chromosomal abnormalities E. Intrauterine growth restriction (fetoplacental cause)

C (NOT: Each pregnancy problem can be attributed to a number of related risk factors. Polyhydramnios may also be the result of poorly controlled diabetes mellitus. Other maternal causes of IUGR include hypertensive disorders, diabetes, chronic renal disease, vascular disease, thrombophilia, poor weight gain, and cyanotic heart disease. Fetoplacental causes of IUGR may be related to chromosomal abnormalities, congenital malformations, intrauterine infection, or genetic syndromes. Other contributors to oligohydramnios are renal agenesis, prolonged pregnancy, uteroplacental insufficiency, and paternal hypertensive disorders. Although advanced maternal age is a well-known cause of chromosomal abnormalities, other causes include parental chromosome rearrangements and pregnancy with autosomal trisomy.)

Biophysical risks include factors that originate with either the mother or the fetus and affect the functioning of either one or both. The nurse who provides prenatal care should have an understanding of these risk factors. Match the specific pregnancy problem with the related risk factor. Premature rupture of membranes A. Polyhydramnios B. Intrauterine growth restriction (maternal cause) C. Oligohydramnios D. Chromosomal abnormalities E. Intrauterine growth restriction (fetoplacental cause)

B (NOT: Each pregnancy problem can be attributed to a number of related risk factors. Polyhydramnios may also be the result of poorly controlled diabetes mellitus. Other maternal causes of IUGR include hypertensive disorders, diabetes, chronic renal disease, vascular disease, thrombophilia, poor weight gain, and cyanotic heart disease. Fetoplacental causes of IUGR may be related to chromosomal abnormalities, congenital malformations, intrauterine infection, or genetic syndromes. Other contributors to oligohydramnios are renal agenesis, prolonged pregnancy, uteroplacental insufficiency, and paternal hypertensive disorders. Although advanced maternal age is a well-known cause of chromosomal abnormalities, other causes include parental chromosome rearrangements and pregnancy with autosomal trisomy.)

Biophysical risks include factors that originate with either the mother or the fetus and affect the functioning of either one or both. The nurse who provides prenatal care should have an understanding of these risk factors. Match the specific pregnancy problem with the related risk factor. Smoking, alcohol, and illicit drug use A. Polyhydramnios B. Intrauterine growth restriction (maternal cause) C. Oligohydramnios D. Chromosomal abnormalities E. Intrauterine growth restriction (fetoplacental cause)

B (The WBC count is high the first day of birth and then declines rapidly. Delayed clamping of the cord results in an increase in hemoglobin and the red blood cell count. The platelet count essentially is the same for newborns and adults. Clotting is sufficient to prevent hemorrhage unless the vitamin K deficiency is significant.)

By knowing about variations in infants' blood count, nurses can explain to their clients that: a. A somewhat lower than expected red blood cell count could be the result of delay in clamping the umbilical cord. b. The early high white blood cell (WBC) count is normal at birth and should decrease rapidly. c. Platelet counts are higher than in adults for a few months. d. Even a modest vitamin K deficiency means a problem with the ability of the blood to clot properly.

A primigravida at 40 weeks of gestation is having uterine contractions every 1.5 to 2 minutes and says that they are very painful. Her cervix is dilated 2 cm and has not changed in 3 hours. The woman is crying and wants an epidural. What is the likely status of this woman's labor? a. She is exhibiting hypotonic uterine dysfunction. b. She is experiencing a normal latent stage. c. She is exhibiting hypertonic uterine dysfunction. d. She is experiencing pelvic dystocia.

C

A woman is having her first child. She has been in labor for 15 hours. Two hours ago her vaginal examination revealed the cervix to be dilated to 5 cm and 100% effaced, and the presenting part was at station 0. Five minutes ago her vaginal examination indicated that there had been no change. What abnormal labor pattern is associated with this description? a. Prolonged latent phase c. Arrest of active phase b. Protracted active phase d. Protracted descent

C

As relates to the use of tocolytic therapy to suppress uterine activity, nurses should be aware that: a. The drugs can be given efficaciously up to the designated beginning of term at 37 weeks. b. There are no important maternal (as opposed to fetal) contraindications. c. Its most important function is to afford the opportunity to administer antenatal glucocorticoids. d. If the client develops pulmonary edema while receiving tocolytics, intravenous (IV) fluids should be given.

C

Surgical, medical, or mechanical methods may be used for labor induction. Which technique is considered a mechanical method of induction? a. Amniotomy c. Transcervical catheter b. Intravenous Pitocin d. Vaginal insertion of prostaglandins

C

The least common cause of long, difficult, or abnormal labor (dystocia) is: a. Midplane contracture of the pelvis. b. Compromised bearing-down efforts as a result of pain medication. c. Disproportion of the pelvis. d. Low-lying placenta.

C

The nurse providing care to a woman in labor should understand that cesarean birth: a. Is declining in frequency in the twenty-first century in the United States. b. Is more likely to be performed for poor women in public hospitals who do not receive the nurse counseling as do wealthier clients. c. Is performed primarily for the benefit of the fetus. d. Can be either elected or refused by women as their absolute legal right.

C

The priority nursing care associated with an oxytocin (Pitocin) infusion is: a. Measuring urinary output. b. Increasing infusion rate every 30 minutes. c. Monitoring uterine response. d. Evaluating cervical dilation.

C

To provide safe care for the woman, the nurse understands that which condition is a contraindication for an amniotomy? a. Dilation less than 3 cm c. -2 station b. Cephalic presentation d. Right occiput posterior position

C

The nurse is preparing to discharge a 30-year-old woman who has experienced a miscarriage at 10 weeks of gestation. Which statement by the woman would indicate a correct understanding of the discharge instructions? A. "I will not experience mood swings since I was only at 10 weeks of gestation." B. "I will avoid sexual intercourse for 6 weeks and pregnancy for 6 months." C. "I should eat foods that are high in iron and protein to help my body heal." D. "I should expect the bleeding to be heavy and bright red for at least 1 week."

C ("I should eat foods that are high in iron and protein to help my body heal." After a miscarriage a woman may experience mood swings and depression from the reduction of hormones and the grieving process. Sexual intercourse should be avoided for 2 weeks or until the bleeding has stopped and should avoid pregnancy for 2 months. A woman who has experienced a miscarriage should be advised to eat foods that are high in iron and protein to help replenish her body after the loss. The woman should not experience bright red, heavy, profuse bleeding; this should be reported to the health care provider.)

The nurse is caring for a 24-hour-postpartum patient who had a cesarean birth with general anesthesia. The patient complains of abdominal discomfort and gas pains. What would be the most suitable nursing intervention in this situation? A. Encourage the patient to drink coffee. B. Administer analgesic medications to patient. C. Encourage the patient to use a rocking chair. D. Offer soups and beverages to the patient

C (A patient who complains of abdominal discomfort and gas pains should be encouraged to use a rocking chair because it stimulates the passage of flatus and relieves discomfort. The patient should not be encouraged to drink coffee because the caffeine present in it intensifies the pain by increasing bowel movements. Analgesic medication does not relieve gas, but the administration of antigas or antiflatulent medications may help relieve gas. Offering soups and beverages may cause more discomfort and gas in the patient.)

The nurse is preparing to discharge a 30-year-old woman who has experienced a miscarriage at 10 weeks of gestation. Which statement by the woman indicates a correct understanding of the discharge instructions? A. "I will not experience mood swings since I was only at 10 weeks of gestation." B. "I will avoid sexual intercourse for 6 weeks and pregnancy for 6 months." C. "I should eat foods that are high in iron and protein to help my body heal." D. "I should expect the bleeding to be heavy and bright red for at least 1 week."

C (A woman who has experienced a miscarriage should be advised to eat foods that are high in iron and protein to help replenish her body after the loss. After a miscarriage, a woman may experience mood swings and depression from the reduction of hormones and the grieving process. Sexual intercourse should be avoided for 2 weeks or until the bleeding has stopped and should avoid pregnancy for 2 months. The woman should not experience bright red, heavy, profuse bleeding; this should be reported to the health care provider.)

As related to central nervous system injuries that could occur to the infant during labor and birth, nurses should be aware that: a. Intracranial hemorrhage (ICH) as a result of birth trauma is more likely to occur in the preterm, low-birth-weight infant. b. Subarachnoid hemorrhage (the most common form of ICH) occurs in term infants as a result of hypoxia. c. In many infants signs of hemorrhage in a full-term infant are absent and are diagnosed only through laboratory tests. d. Spinal cord injuries almost always result from forceps-assisted deliveries.

C (Abnormalities in lumbar punctures or red blood cell counts, for instance, or in visuals on computed tomography scan may reveal a hemorrhage. ICH as a result of birth trauma is more likely to occur in the full-term, large infant. Subarachnoid hemorrhage in term infants is a result of trauma; in preterm infants it is a result of hypoxia. Spinal cord injuries are almost always from breech births; they are rare today because cesarean birth often is used for breech presentation.)

With regard to the nutrient needs of breastfed and formula-fed infants, nurses should be understand that: a. Breastfed infants need extra water in hot climates. b. During the first 3 months breastfed infants consume more energy than do formula-fed infants. c. Breastfeeding infants should receive oral vitamin D drops daily at least during the first 2 months. d. Vitamin K injections at birth are not needed for infants fed on specially enriched formula.

C (Human milk contains only small amounts of vitamin D. Neither breastfed nor formula-fed infants need to be given water, even in very hot climates. During the first 3 months formula-fed infants consume more energy than do breastfed infants and therefore tend to grow more rapidly. Vitamin K shots are required for all infants because the bacteria that produce it are absent from the baby's stomach at birth.)

As related to laboratory tests and diagnostic tests in the hospital after birth, nurses should be aware that: a. All states test for phenylketonuria (PKU), hypothyroidism, cystic fibrosis, and sickle cell diseases. b. Federal law prohibits newborn genetic testing without parental consent. c. If genetic screening is done before the infant is 24 hours old, it should be repeated at age 1 to 2 weeks. d. Hearing screening is now mandated by federal law.

C (If done very early, genetic screening should be repeated. States all test for PKU and hypothyroidism, but other genetic defects are not universally covered. Federal law mandates newborn genetic screening, but not screening for hearing problems (although more than half the states do mandate hearing screening).)

Postpartal overdistention of the bladder and urinary retention can lead to which complications? a. Postpartum hemorrhage and eclampsia b. Fever and increased blood pressure c. Postpartum hemorrhage and urinary tract infection d. Urinary tract infection and uterine rupture

C (Incomplete emptying and overdistention of the bladder can lead to urinary tract infection. Overdistention of the bladder displaces the uterus and prevents contraction of the uterine muscle, thus leading to postpartum hemorrhage. There is no correlation between bladder distention and high blood pressure or eclampsia. The risk of uterine rupture decreases after the birth of the infant.)

The nurse practicing in the perinatal setting should promote kangaroo care regardless of an infant's gestational age. This intervention: a. Is adopted from classical British nursing traditions. b. Helps infants with motor and central nervous system impairment. c. Helps infants to interact directly with their parents and enhances their temperature regulation. d. Gets infants ready for breastfeeding.

C (Kangaroo care is skin-to-skin holding in which the infant, dressed only in a diaper, is placed directly on the parent's bare chest and then covered. The procedure helps infants interact with their parents and regulates their temperature, among other developmental benefits.)

With regard to basic care of the breastfeeding mother, nurses should be able to advise her that she: a. Will need an extra 1000 calories a day to maintain energy and produce milk. b. Can go back to prepregnancy consumption patterns of any drinks, as long as she ingests enough calcium. c. Should avoid trying to lose large amounts of weight. d. Must avoid exercising because it is too fatiguing.

C (Large weight loss would release fat-stored contaminants into her breast milk. It would also likely involve eating too little and/or exercising too much. A breastfeeding mother need add only 200 to 500 extra calories to her diet to provide extra nutrients for the infant. The mother can go back to her consumption patterns of any drinks as long as she ingests enough calcium, only if she does not drink alcohol, limits coffee to no more than two cups (caffeine in chocolate, tea, and some sodas), and reads the herbal tea ingredients carefully. The mother needs her rest, but moderate exercise is healthy.)

Signs of a threatened abortion (miscarriage) are noted in a woman at 8 weeks of gestation. What is an appropriate management approach for this type of abortion? A. Prepare the woman for a dilation and curettage (D&C). B. Place the woman on bed rest for at least 1 week and reevaluate. C. Prepare the woman for an ultrasound and blood work. D. Comfort the woman by telling her that if she loses this baby, she may attempt to get pregnant again in 1 month.

C (Prepare the woman for an ultrasound and blood work. D&C is not considered until signs of the progress to an inevitable abortion are noted or the contents are expelled and incomplete. Bed rest is recommended for 48 hours initially. Repetitive transvaginal ultrasounds and measurement of human chorionic gonadotropin (hCG) and progesterone levels may be performed to determine if the fetus is alive and within the uterus. If the pregnancy is lost, the woman should be guided through the grieving process. Telling the client that she can get pregnant again soon is not a therapeutic response because it discounts the importance of this pregnancy.)

Providing care for the neonate born to a mother who abuses substances can present a challenge for the health care team. Nursing care for this infant requires a multisystem approach. The first step in the provision of this care is: a. Pharmacologic treatment. b. Reduction of environmental stimuli. c. Neonatal abstinence syndrome scoring. d. Adequate nutrition and maintenance of fluid and electrolyte balance.

C (Neonatal abstinence syndrome (NAS) is the term used to describe the cohort of symptoms associated with drug withdrawal in the neonate. The Neonatal Abstinence Scoring System evaluates central nervous system (CNS), metabolic, vasomotor, respiratory, and gastrointestinal disturbances. This evaluation tool enables the care team to develop an appropriate plan of care. The infant is scored throughout the length of stay, and the treatment plan is adjusted accordingly. Pharmacologic treatment is based on the severity of withdrawal symptoms. Symptoms are determined by using a standard assessment tool. Medications of choice are morphine, phenobarbital, diazepam, or diluted tincture of opium. Swaddling, holding, and reducing environmental stimuli are essential in providing care to the infant who is experiencing withdrawal. These nursing interventions are appropriate for the infant who displays CNS disturbances. Poor feeding is one of the gastrointestinal symptoms common to this client population. Fluid and electrolyte balance must be maintained and adequate nutrition provided. These infants often have a poor suck reflex and may need to be fed via gavage.)

What does the nurse advise a pregnant patient who is prescribed phenazopyridine (Pyridium) for cystitis? A. "Avoid sweet foods in diet." B. "Limit exposure to sunlight." C. "Do not wear contact lenses." D. "Restrict oral fluids to 125 mL per hour."

C (Phenazopyridine (Pyridium) colors the tears orange. Therefore the nurse instructs the patient to avoid wearing contact lenses. Sweet foods are avoided in patients with diabetes mellitus, because they can cause fluctuating glucose levels, which may harm the fetus. Exposure to sunlight is avoided when the patient is receiving methotrexate therapy, because it causes photosensitivity. Oral fluids are restricted in patients who are at risk for pulmonary edema.)

What instruction does the nurse provide to a pregnant patient with mild preeclampsia? A. "You need to be hospitalized for fetal evaluation." B. "Nonstress testing can be done once every month." C. "Fetal movement counts need to be evaluated daily." D. "Take complete bed rest during the entire pregnancy."

C (Preeclampsia can affect the fetus and may cause fetal growth restrictions, decreased amniotic fluid volume, abnormal fetal oxygenation, low birth weight, and preterm birth. Therefore the fetal movements need to be evaluated daily. Patients with mild preeclampsia can be managed at home effectively and need not be hospitalized. Nonstress testing is performed once or twice per week to determine fetal well-being. Patients need to restrict activity, but complete bed rest is not advised because it may cause cardiovascular deconditioning, muscle atrophy, and psychological stress.)

For clinical purposes, preterm and post-term infants are defined as: a. Preterm before 34 weeks if appropriate for gestational age (AGA) and before 37 weeks if small for gestational age (SGA). b. Post-term after 40 weeks if large for gestational age (LGA) and beyond 42 weeks if AGA. c. Preterm before 37 weeks, and post-term beyond 42 weeks, no matter the size for gestational age at birth. d. Preterm, SGA before 38 to 40 weeks, and post-term, LGA beyond 40 to 42 weeks.

C (Preterm and post-term are strictly measures of time—before 37 weeks and beyond 42 weeks, respectively—regardless of size for gestational age.)

Which intervention will help prevent the risk of pulmonary edema in a pregnant patient with severe preeclampsia? A. Assess fetal heart rate (FHR) abnormalities regularly. B. Place the patient on bed rest in a darkened environment. C. Restrict total intravenous (I.V.) and oral fluids to 125 mL/hr. D. Ensure that magnesium sulfate is administered as prescribed.

C (Pulmonary edema may be seen in patients with severe preeclampsia. Therefore the nurse needs to restrict total intravenous (I.V.) and oral fluids to 125 mL/hr. FHR monitoring helps assess any fetal complications. The patient is placed on bed rest in a darkened environment to prevent stress. Magnesium sulfate is administered to prevent eclamptic seizures.)

Which type of formula is not diluted before being administered to an infant? a. Powdered b. Concentrated c. Ready-to-use d. Modified cow's milk

C (Ready-to-use formula can be poured directly from the can into baby's bottle and is good (but expensive) when a proper water supply is not available. Formula should be well mixed to dissolve the powder and make it uniform in consistency. Improper dilution of concentrated formula may cause malnutrition or sodium imbalances. Cow's milk is more difficult for the infant to digest and is not recommended, even if it is diluted.)

Signs of a threatened abortion (miscarriage) are noted in a woman at 8 weeks of gestation. What is an appropriate management approach for this type of abortion? A. Prepare the woman for a dilation and curettage (D&C). B. Place the woman on bed rest for at least 1 week and reevaluate. C. Prepare the woman for an ultrasound and bloodwork. D. Comfort the woman by telling her that if she loses this baby, she may attempt to get pregnant again in 1 month.

C (Repetitive transvaginal ultrasounds and measurement of human chorionic gonadotropin (hCG) and progesterone levels may be performed to determine if the fetus is alive and within the uterus. If the pregnancy is lost , the woman should be guided through the grieving process. D&C is not considered until signs of the progress to an inevitable abortion are noted or the contents are expelled and incomplete. Bed rest is recommended for 48 hours initially. Telling the woman that she can get pregnant again soon is not a therapeutic response because it discounts the importance of this pregnancy.)

The nurse is preparing to administer rubella vaccine to a patient during the postpartum period. At the follow-up visit, the patient reports to the nurse that she has rashes on her skin. What does the nurse expect the primary health care provider (PHP) to prescribe in this situation? A. Oxytocin (Pitocin) B. Rh immune globulin C. Adrenaline (Epinephrine) D. Magnesium sulfate

C (Rubella vaccine is made from duck eggs; therefore women who are allergic to duck eggs can develop a hypersensitivity reaction to the vaccine. As a result, the patient might develop rashes on her skin. The PHP would prescribe adrenaline to combat hypersensitivity reactions. Oxytocin is injected to increase the tone of the uterine muscles but not to combat hypersensitivity. Rh immune globulin suppresses the immune system, which would worsen the condition; therefore this medication is unlikely to be prescribed. Magnesium sulfate is used for preeclampsia and is not used to minimize hypersensitivity reactions caused by rubella vaccine.)

If a woman is at risk for thrombus and is not ready to ambulate, nurses may intervene by performing a number of interventions. Which intervention should the nurse avoid? a. Putting the patient in antiembolic stockings (TED hose) and/or sequential compression device (SCD) boots. b. Having the patient flex, extend, and rotate her feet, ankles, and legs. c. Having the patient sit in a chair. d. Notifying the physician immediately if a positive Homans' sign occurs.

C (Sitting immobile in a chair will not help. Bed exercise and prophylactic footwear may. TED hose and SCD boots are recommended. Bed exercises, such as flexing, extending, and rotating her feet, ankles, and legs, are useful. A positive Homans' sign (calf muscle pain or warmth, redness, or tenderness) requires the physician's immediate attention.)

In caring for the mother who has abused (or is abusing) alcohol and for her infant, nurses should be aware that: a. The pattern of growth restriction of the fetus begun in prenatal life is halted after birth, and normal growth takes over. b. Two thirds of newborns with fetal alcohol syndrome (FAS) are boys. c. Alcohol-related neurodevelopmental disorders not sufficient to meet FAS criteria (learning disabilities, speech and language problems) are often not detected until the child goes to school. d. Both the distinctive facial features of the FAS infant and the diminished mental capacities tend toward normal over time.

C (Some learning problems do not become evident until the child is at school. The pattern of growth restriction persists after birth. Two thirds of newborns with FAS are girls. Although the distinctive facial features of the FAS infant tend to become less evident, the mental capacities never become normal.)

In the classification of newborns by gestational age and birth weight, the appropriate for gestational age (AGA) weight would: a. Fall between the 25th and 75th percentiles for the infant's age. b. Depend on the infant's length and the size of the head. c. Fall between the 10th and 90th percentiles for the infant's age. d. Be modified to consider intrauterine growth restriction (IUGR).

C (The AGA range is large: between the 10th and the 90th percentiles for the infant's age. The infant's length and size of the head are measured, but they do not affect the normal weight designation. IUGR applies to the fetus, not the newborn's weight.)

An infant is being discharged from the neonatal intensive care unit after 70 days of hospitalization. The infant was born at 30 weeks of gestation with several conditions associated with prematurity, including respiratory distress syndrome, mild bronchopulmonary dysplasia, and retinopathy of prematurity requiring surgical treatment. During discharge teaching the infant's mother asks the nurse whether her baby will meet developmental milestones on time, as did her son who was born at term. The nurse's most appropriate response is: a. "Your baby will develop exactly like your first child did." b. "Your baby does not appear to have any problems at the present time." c. "Your baby will need to be corrected for prematurity. Your baby is currently 40 weeks of postconceptional age and can be expected to be doing what a 40-week-old infant would be doing." d. "Your baby will need to be followed very closely."

C (The age of a preterm newborn is corrected by adding the gestational age and the postnatal age. The infant's responses are evaluated accordingly against the norm expected for the corrected age of the infant. Although it is impossible to predict with complete accuracy the growth and development potential of each preterm infant, certain measurable factors predict normal growth and development. The preterm infant experiences catch-up body growth during the first 2 to 3 years of life. The growth and developmental milestones are corrected for gestational age until the child is approximately 2.5 years old. Stating that the baby does not appear to have any problems at the present time is inaccurate. Development will need to be evaluated over time.)

As a result of large body surface in relation to weight, the preterm infant is at high risk for heat loss and cold stress. By understanding the four mechanisms of heat transfer (convection, conduction, radiation, and evaporation), the nurse can create an environment for the infant that prevents temperature instability. While evaluating the plan that has been implemented, the nurse knows that the infant is experiencing cold stress when he or she exhibits: a. Decreased respiratory rate. b. Bradycardia followed by an increased heart rate. c. Mottled skin with acrocyanosis. d. Increased physical activity.

C (The infant has minimal to no fat stores. During times of cold stress the skin will become mottled, and acrocyanosis will develop, progressing to cyanosis. Even if the infant is being cared for on a radiant warmer or in an isolette, the nurse's role is to observe the infant frequently to prevent heat loss and respond quickly if signs and symptoms occur. The respiratory rate increases followed by periods of apnea. The infant initially tries to conserve heat and burns more calories, after which the metabolic system goes into overdrive. In the preterm infant experiencing heat loss, the heart rate initially increases, followed by periods of bradycardia. In the term infant, the natural response to heat loss is increased physical activity. However, in a term infant experiencing respiratory distress or in a preterm infant, physical activity is decreased.)

In assisting the breastfeeding mother position the baby, nurses should keep in mind that: a. The cradle position usually is preferred by mothers who had a cesarean birth. b. Women with perineal pain and swelling prefer the modified cradle position. c. Whatever the position used, the infant is "belly to belly" with the mother. d. While supporting the head, the mother should push gently on the occiput.

C (The infant inevitably faces the mother, belly to belly. The football position usually is preferred after cesarean birth. Women with perineal pain and swelling prefer the side-lying position because they can rest while breastfeeding. The mother should never push on the back of the head. It may cause the baby to bite, hyperextend the neck, or develop an aversion to being brought near the breast.)

A plan of care for an infant experiencing symptoms of drug withdrawal should include: a. Administering chloral hydrate for sedation. b. Feeding every 4 to 6 hours to allow extra rest. c. Swaddling the infant snugly and holding the baby tightly. d. Playing soft music during feeding.

C (The infant should be wrapped snugly to reduce self-stimulation behaviors and protect the skin from abrasions. Phenobarbital or diazepam may be administered to decrease central nervous system (CNS) irritability. The infant should be fed in small, frequent amounts and burped well to diminish aspiration and maintain hydration. The infant should not be stimulated (such as with music) because this will increase activity and potentially increase CNS irritability.)

A newborn is jaundiced and receiving phototherapy via ultraviolet bank lights. An appropriate nursing intervention when caring for an infant with hyperbilirubinemia and receiving phototherapy by this method would be to: a. Apply an oil-based lotion to the newborn's skin to prevent dying and cracking. b. Limit the newborn's intake of milk to prevent nausea, vomiting, and diarrhea. c. Place eye shields over the newborn's closed eyes. d. Change the newborn's position every 4 hours.

C (The infant's eyes must be protected by an opaque mask to prevent overexposure to the light. Eye shields should cover the eyes completely but not occlude the nares. Lotions and ointments should not be applied to the infant because they absorb heat, and this can cause burns. The lights increase insensible water loss, placing the infant at risk for fluid loss and dehydration. Therefore, it is important that the infant be adequately hydrated. The infant should be turned every 2 hours to expose all body surfaces to the light.)

The normal term infant has little difficulty clearing the airway after birth. Most secretions are brought up to the oropharynx by the cough reflex. However, if the infant has excess secretions, the mouth and nasal passages can be cleared easily with a bulb syringe. When instructing parents on the correct use of this piece of equipment, it is important that the nurse teach them to: a. Avoid suctioning the nares. b. Insert the compressed bulb into the center of the mouth. c. Suction the mouth first. d. Remove the bulb syringe from the crib when finished.

C (The mouth should be suctioned first to prevent the infant from inhaling pharyngeal secretions by gasping as the nares are suctioned. The nasal passages should be suctioned one nostril at a time. After compression of the bulb it should be inserted into one side of the mouth. If the bulb is inserted into the center of the mouth, the gag reflex is likely to be initiated. When the infant's cry no longer sounds as though it is through mucus or a bubble, suctioning can be stopped. The bulb syringe should remain in the crib so that it is easily accessible if needed again.)

A blunt abdominal trauma causes fetal hemorrhage in a pregnant patient. The nurse finds that the patient is Rh negative. What action does the nurse take? A. Initiate magnesium sulfate per protocol. B. Administer oxytocin (pitocin). C. Administer prescribed Rho (D) immunoglobulin. D. Prepare the patient for magnetic resonance imaging (MRI).

C (The nurse administers the prescribed Rho(D) immunoglobulin to the patient to protect the patient from isoimmunization. The nurse needs to obtain a prescription for magnesium sulfate if there are eclamptic seizures in a patient with preeclampsia. Oxytocin (Pitocin) is administered to prevent bleeding after birth or the evacuation of the uterus. Magnetic resonance imaging (MRI) is used to assess injuries in a patient after trauma.)

Near the end of the first week of life, an infant who has not been treated for any infection develops a copper-colored, maculopapular rash on the palms and around the mouth and anus. The newborn is showing signs of: a. Gonorrhea. b. Herpes simplex virus infection. c. Congenital syphilis. d. Human immunodeficiency virus.

C (The rash is indicative of congenital syphilis. The lesions may extend over the trunk and extremities.)

A lactating patient who gave birth to twins 1 month earlier approaches the primary health care provider (PHP) for a general checkup. What suggestion does the nurse give to the patient about the recommended calorie intake? A. Less than 1800 kcal/day B. Less than 2200 kcal/day C. More than 2700 kcal/day D. Should be 1800 to 2200 kcal/day

C (The recommended caloric intake for a lactating mother who breastfeeds more than one infant is more than 2700 kcal/day. If a lactating mother of twins takes less than 2200 kcal/day, she may not produce enough milk. An intake of 1800 to 2200 kcal/day is recommended for nonlactating mothers.)

A 25-year-old gravida 2, para 2-0-0-2 gave birth 4 hours ago to a 9-pound, 7-ounce boy after augmentation of labor with Pitocin. She puts on her call light and asks for her nurse right away, stating, "I'm bleeding a lot." The most likely cause of postpartum hemorrhage in this woman is: a. Retained placental fragments. c. Uterine atony. b. Unrepaired vaginal lacerations. d. Puerperal infection

C (This woman gave birth to a macrosomic boy after Pitocin augmentation. The most likely cause of bleeding 4 hours after delivery, combined with these risk factors, is uterine atony. Although retained placental fragments may cause postpartum hemorrhage, this typically would be detected in the first hour after delivery of the placenta and is not the most likely cause of hemorrhage in this woman. Although unrepaired vaginal lacerations may cause bleeding, they typically would occur in the period immediately after birth. Puerperal infection can cause subinvolution and subsequent bleeding; however, this typically would be detected 24 hours after delivery.)

At a 2-month well-baby examination, it was discovered that a breastfed infant had only gained 10 ounces in the past 4 weeks. The mother and the nurse agree that, to gain weight faster, the infant needs to: a. Begin solid foods. b. Have a bottle of formula after every feeding. c. Add at least one extra breastfeeding session every 24 hours. d. Start iron supplements.

C (Usually the solution to slow weight gain is to improve the feeding technique. Position and latch-on are evaluated, and adjustments are made. It may help to add a feeding or two in a 24-hour period. Solid foods should not be introduced to an infant for at least 4 to 6 months. Bottle-feeding may cause nipple confusion and limit the supply of milk. Iron supplements have no bearing on weight gain.)

A primigravida has just delivered a healthy infant girl. The nurse is about to administer erythromycin ointment in the infant's eyes when the mother asks, "What is that medicine for?" The nurse responds: a. "It is an eye ointment to help your baby see you better." b. "It is to protect your baby from contracting herpes from your vaginal tract." c. "Erythromycin is given prophylactically to prevent a gonorrheal infection." d. "This medicine will protect your baby's eyes from drying out over the next few days."

C (With the prophylactic use of erythromycin, the incidence of gonococcal conjunctivitis has declined to less than 0.5%. Eye prophylaxis is administered at or shortly after birth to prevent ophthalmia neonatorum. Erythromycin has no bearing on enhancing vision, is used to prevent an infection caused by gonorrhea, not herpes, and is not used for eye lubrication.)

According to demographic research, the woman least likely to breastfeed and therefore most likely to need education regarding the benefits and proper techniques of breastfeeding would be: a. A woman who is 30 to 35 years of age, Caucasian, and employed part time outside the home. b. A woman who is younger than 25 years of age, Hispanic, and unemployed. c. A woman who is younger than 25 years of age, African-American, and employed full time outside the home. d. A woman who is 35 years of age or older, Caucasian, and employed full time at home.

C (Women least likely to breastfeed typically are younger than 25 years of age, have a lower income, are less educated, are employed full time outside the home, and are African-American.)

The nurse providing care for the laboring woman should understand that late FHR decelerations are caused by: A. Altered cerebral blood flow B. Umbilical cord compression C. Uteroplacental insufficiency D. Meconium fluid

C A. Incorrect: Altered fetal cerebral blood flow would result in early decelerations in the FHR. B. Incorrect: Umbilical cord compression would result in variable decelerations in the FHR. C. Correct: Uteroplacental insufficiency would result in late decelerations in the FHR. D. Incorrect: Meconium-stained fluid may or may not produce changes in the fetal heart rate, depending on the gestational age of the fetus and whether other causative factors associated with fetal distress are present. p. 507

According to standard professional thinking, nurses should auscultate the FHR: A. Every 15 minutes in the active phase of the first stage of labor in the absence of risk factors B. Every 20 minutes in the second stage regardless of whether risk factors are present C. Before and after ambulation and rupture of membranes D. More often in a woman's first pregnancy

C A. Incorrect: In the active phase of the first stage of labor, the FHR should be auscultated every 30 minutes if no risk factors are involved; with risk factors, it should be auscultated every 15 minutes. B. Incorrect: In the second stage of labor, the FHR should be auscultated every 15 minutes if no risk factors are involved; with risk factors, it should be auscultated every 5 minutes. C. Correct: The FHR should be auscultated before and after administration of medications and induction of anesthesia. D. Incorrect: The fetus of a first-time mother is automatically at greater risk. p. 499

The nurse caring for the woman in labor should understand that maternal hypotension can result in: A. Early decelerations B. Fetal dysrhythmias C. Uteroplacental insufficiency D. Spontaneous rupture of membranes

C A. Incorrect: Maternal hypotension is not associated with this condition. B. Incorrect: Maternal hypotension is not associated with this condition. C. Correct: Low maternal blood pressure reduces placental blood flow during uterine contractions, resulting in fetal hypoxemia. D. Incorrect: Maternal hypotension is not associated with this condition. p. 503

The nurse caring for the woman in labor should understand that increased variability of the fetal heart rate might be caused by: A. Narcotics B. Barbiturates C. Methamphetamines D. Tranquilizers

C A. Incorrect: Narcotics, barbiturates, and tranquilizers might be causes of decreased variability; methamphetamines might cause increased variability. B. Incorrect: Narcotics, barbiturates, and tranquilizers might be causes of decreased variability; methamphetamines might cause increased variability. C. Correct: Narcotics, barbiturates, and tranquilizers might be causes of decreased variability; methamphetamines might cause increased variability. D. Incorrect: Narcotics, barbiturates, and tranquilizers might be causes of decreased variability; methamphetamines might cause increased variability. p. 503-504

A (The diagnosis of DIC is made according to clinical findings and laboratory markers. Physical examination reveals unusual bleeding. Petechiae may appear around a blood pressure cuff on the woman's arm. Excessive bleeding may occur from the site of a slight trauma, such as venipuncture sites.)

In caring for an immediate postpartum client, you note petechiae and oozing from her IV site. You would monitor her closely for the clotting disorder: A. Disseminated intravascular coagulation B. Amniotic fluid embolism C. Hemorrhage D. HELLP syndrome

The nurse caring for a woman in labor understands that prolonged decelerations: A. Are a continuing pattern of benign decelerations that do not require intervention B. Constitute a baseline change when they last longer than 5 minutes C. Usually are isolated events that end spontaneously D. Require the usual fetal monitoring by the nurse

C A. Incorrect: Prolonged decelerations usually are isolated events that end spontaneously. However, in certain combinations with late and/or variable decelerations, they are a danger sign that requires the nurse to notify the physician or midwife immediately. B. Incorrect: A deceleration that lasts longer than 10 minutes constitutes a baseline change. Prolonged decelerations usually are isolated events that end spontaneously. However, in certain combinations with late and/or variable decelerations, they are a danger sign that requires the nurse to notify the physician or midwife immediately. C. Correct: Prolonged decelerations usually are isolated events that end spontaneously. However, in certain combinations with late and/or variable decelerations, they are a danger sign that requires the nurse to notify the physician or midwife immediately. D. Incorrect: Prolonged decelerations usually are isolated events that end spontaneously. However, in certain combinations with late and/or variable decelerations, they are a danger sign that requires the nurse to notify the physician or midwife immediately. p. 509

A number of methods to assist in the assessment of fetal well-being have been developed for use in conjunction with electronic fetal monitoring (EFM). These various technologies assist in supporting interventions for a nonreassuring fetal heart rate pattern when necessary. The labor and delivery nurse should be aware that one of these modalities, fetal oxygen saturation monitoring, includes the use of: A. A fetal acoustic stimulator B. Fetal blood sampling C. Fetal pulse oximetry D. Umbilical cord acid-base determination

C A. Incorrect: Stimulation of the fetus is done in an effort to elicit a fetal heart rate response. The two acceptable methods of achieving this result are the use of fetal scalp stimulation or vibroacoustic stimulation. Vibroacoustic stimulation is performed by using an artificial larynx or fetal acoustic stimulation device over the fetal head for 1 or 2 seconds. B. Incorrect: Sampling of the fetal scalp blood was designed to assess fetal pH, O2, and CO2. The sample is obtained from the fetal scalp through a dilated cervix. This test is usually done in tertiary care centers, where results can be immediately available. It has recently fallen out of favor because test results vary widely. C. Correct: Continuous monitoring of the fetal O2 saturation by fetal pulse oximetry is a method that was approved for clinical use in 2000 by the FDA. This process works in a similar method to obtaining a pulse oximetry in a child or adult. A specially designed sensor is inserted into the uterus and lies against the fetus's temple or cheek. A normal result is 30% to 70%, with 30% being the cutoff for further intervention. D. Incorrect: This test is not completed until after birth. Cord blood is drawn from the umbilical artery and tested for pH, O2, and CO2. pp. 513-516

Fetal bradycardia is most common during: A. Intraamniotic infection B. Fetal anemia C. Prolonged umbilical cord compression D. Tocolytic treatment using ritodrine

C A. Incorrect: This circumstance most likely would result in fetal tachycardia. B. Incorrect: This circumstance most likely would result in fetal tachycardia. C. Correct: Fetal bradycardia can be considered a later sign of fetal hypoxia and is known to occur before fetal death. Bradycardia can result from placental transfer of drugs, prolonged compression of the umbilical cord, maternal hypothermia, and maternal hypotension. D. Incorrect: This circumstance most likely would result in fetal tachycardia. p. 503

A recently delivered mother and her baby are at the clinic for a 6-week postpartum checkup. Which response by the client alerts the nurse that psychosocial outcomes have not been met? a. The woman excessively discusses her labor and birth experience. b. The woman feels that her baby is more attractive and clever than any others. c. The woman has not given the baby a name. d. The woman has a partner or family members who react very positively about the baby.

C If the mother is having difficulty naming her new infant, it may be a signal that she is not adapting well to parenthood. Other red flags include a refusal to hold or feed the baby, a lack of interaction with the infant, and becoming upset when the baby vomits or needs a diaper change. A new mother who is having difficulty is unwilling to discuss her labor and birth experience. An appropriate nursing diagnosis might be Impaired parenting, related to a long, difficult labor or unmet expectations of birth. A mother who is willing to discuss her birth experience is making a healthy personal adjustment. The mother who is not coping well finds her baby unattractive and messy. She may also be overly disappointed in the babys sex. The client might voice concern that the baby reminds her of a family member whom she does not like. Having a partner and/or other family members react positively is an indication that this new mother has a good support system in place. This support system helps reduce anxiety related to her new role as a mother.

Postpartum overdistention of the bladder and urinary retention can lead to which complications? a. Postpartum hemorrhage and eclampsia b. Fever and increased blood pressure c. Postpartum hemorrhage and urinary tract infection d. Urinary tract infection and uterine rupture

C Incomplete emptying and overdistention of the bladder can lead to urinary tract infection. Overdistention of the bladder displaces the uterus and prevents contraction of the uterine muscle, thus leading to postpartum hemorrhage. No correlation exists between bladder distention and high blood pressure or eclampsia. The risk of uterine rupture decreases after the birth of the infant.

Under the Newborns and Mothers Health Protection Act, all health plans are required to allow new mothers and newborns to remain in the hospital for a minimum of _____ hours after a normal vaginal birth and for _____ hours after a cesarean birth. What is the correct interpretation of this legislation? a. 24; 72 b. 24; 96 c. 48; 96 d. 48; 120

C The specified stays are 48 hours (2 days) for a vaginal birth and 96 hours (4 days) for a cesarean birth. The attending provider and the mother together can decide on an earlier discharge. A client may be discharged either 24 hours after a vaginal birth or 72 hours after a cesarean birth if she is stable and her provider is in agreement. A client is unlikely to remain in the hospital for 120 hours after a cesarean birth unless complications have developed.

When making a visit to the home of a postpartum woman 1 week after birth, the nurse should recognize that the woman would characteristically: a. Express a strong need to review events and her behavior during the process of labor and birth. b. Exhibit a reduced attention span, limiting readiness to learn. c. Attempt to meet the needs of the infant and is eager to learn about infant care. d. Have reestablished her role as a spouse and partner.

C One week after birth the woman should exhibit behaviors characteristic of the taking-hold phase. This stage lasts for as long as 4 to 5 weeks after birth. Responses A and B are characteristic of the taking-in stage, which lasts for the first few days after birth. Response D reflects the letting-go stage, which indicates that psychosocial recovery is complete.

The day after giving birth, the woman complains that she did not lose all the weight she had gained during the pregnancy. The nurse can best respond to the mother with the knowledge that: a. She has lost the most of the weight and the rest will be gone within 1 week. b. She has lost some of the weight and the rest will slowly disappear within 6 weeks. c. It will take about 6 to 12 months for all the weight gained with the pregnancy to disappear. d. Most women do not lose all the weight gained with each pregnancy.

C Women are very concerned about regaining their normal figure. Nurses must emphasize that weight loss should be gradual and that about 6 to 12 months is usually required to lose most weight gained during pregnancy.

The nurse is caring for a 2-day postpartum patient who is breastfeeding. The patient reports breast irritation. Which intervention would be beneficial to the patient? A. Apply ice packs to the breasts between feedings. B. Place hydrogel pads to the breasts between feedings. C. Tell the patient to wear breast shells. D. Apply cold cabbage leaves to the breasts between the feedings.

C (To reduce breast irritation, the nurse advises the patient to wear breast shells. This will increase comfort during breastfeeding. Application of ice packs between feedings reduces breast engorgement. Hydrogel pads can be applied if the patient has sore nipples between feedings. Cold cabbage leaves applied to the breasts for 15 to 20 minutes between feedings can reduce breast engorgement by reducing tissue swelling and facilitating the flow of milk.)

Several noted health risks are associated with menopause. These risks include all except: A) osteoporosis. B) coronary heart disease. C) breast cancer. D) obesity.

C) Breast Cancer Osteoporosis is a major health problem in the United States. It is associated with an increase in hip and vertebral fractures in postmenopausal women. A woman's risk of developing and dying of cardiovascular disease increases significantly after menopause. Breast cancer may be associated with the use of hormone replacement therapy for women who have a family history of breast cancer. Women tend to become more sedentary in midlife. The metabolic rate decreases after menopause, which may require an adjustment in lifestyle and eating patterns.

An effective relief measure for primary dysmenorrhea would be to: A) reduce physical activity level until menstruation ceases. B) begin taking prostaglandin synthesis inhibitors on the first day of the menstrual flow. C) decrease intake of salt and refined sugar about 1 week before menstruation is about to occur. D) use barrier methods rather than the oral contraceptive pill (OCP) for birth control.

C) decrease intake of salt and refined sugar about 1 week before menstruation is about to occur. Staying active is helpful since it facilitates menstrual flow and increases vasodilation to reduce ischemia. Prostaglandin inhibitors should be started a few days before the onset of menstruation. Decreasing intake of salt and refined sugar can reduce fluid retention. OCPs are beneficial in relieving primary dysmenorrhea as a result of inhibition of ovulation and prostaglandin synthesis.

Self-care instructions for a woman following a modified radical mastectomy would include that she: A) wears clothing with snug sleeves to support her affected arm. B) use depilatory creams instead of shaving the axilla of her affected arm. C) expect a decrease in sensation or tingling in her affected arm as her body heals. D) empty surgical drains once a day or every other day.

C) expect a decrease in sensation or tingling in her affected arm as her body heals. Loose clothing should be worn since tight clothing could impede circulation in the affected arm. The axilla of the affected arm should not be shaved nor should depilatory creams or strong deodorants be used. A decrease in sensation and tingling in the affected arm and in the incision are expected for weeks to months after the surgery. Drains should be emptied at least twice a day and more often if necessary.

A postpartum patient has chosen not to breastfeed. What instructions should the nurse provide to the patient to prevent discomfort caused by breast engorgement? Select all that apply. A. Express the milk from both breasts. B. Perform regular breast stimulation. C. Wear a well-fitted support bra. D. Use a breast binder. E. Apply ice packs on the breasts

C, D, E (Patients who choose not to breastfeed may experience breast engorgement and related discomfort. The nurse should instruct the patient to wear a well-fitted support bra or use a breast binder to support the breasts, which can relieve discomfort. Applying ice packs with a 15-minutes-on, 45-minutes-off schedule also helps relieve breast engorgement and reduce discomfort. Expressing milk from the breast or performing nipple stimulation may increase milk production and may worsen breast engorgement.)

A couple has been counseled for genetic anomalies. They ask you, "What is karyotyping?" Your best response is: A. "Karyotyping will reveal if the baby's lungs are mature." B. "Karyotyping will reveal if your baby will develop normally." C. "Karyotyping will provide information about the gender of the baby, and the number and structure of the chromosomes." D. "Karyotyping will detect any physical deformities the baby has."

C. "Karyotyping will provide information about the gender of the baby, and the number and structure of the chromosomes." The lecithin/sphingomyelin ratio, not karyotyping, reveals lung maturity. Although karyotyping can detect genetic anomalies, the range of normal is nondescriptive. Karyotyping provides genetic information, such as gender and chromosomal structure. Although karyotyping can detect genetic anomalies, not all such anomalies display obvious physical deformities. The term deformities is a nondescriptive word. Furthermore, physical anomalies may be present that are not detected by genetic studies (e.g., cardiac malformations).

The student nurse is giving a presentation about milestones in embryonic development. Which information should he or she include? A. At 8 weeks of gestation, primary lung and urethral buds appear. B. At 12 weeks of gestation, the vagina is open or the testes are in position for descent into the scrotum. C. At 20 weeks of age, the vernix caseosa and lanugo appear. D. At 24 weeks of age, the skin is smooth, and subcutaneous fat is beginning to collect.

C. At 20 weeks of age, the vernix caseosa and lanugo appear. The primary lung and urethral buds appear at 6 weeks of gestation. The vagina is open or the testes are in position for descent into the scrotum at 16 weeks. Two milestones that occur at 20 weeks are the appearance of the vernix caseosa and lanugo. The appearance of smooth skin occurs at 28 weeks, and subcutaneous fat begins to collect at 30 to 31 weeks.

A nurse is providing genetic counseling for an expectant couple who already have a child with trisomy 18. The nurse should: A. Tell the couple they need to have an abortion within 2 to 3 weeks. B. Explain that the fetus has a 50% chance of having the disorder. C. Discuss options with the couple, including amniocentesis to determine whether the fetus is affected. D. Refer the couple to a psychologist for emotional support.

C. Discuss options with the couple, including amniocentesis to determine whether the fetus is affected. The couple should be given information about the likelihood of having another baby with this disorder so that they can make an informed decision. A genetic counselor is the best source for determining genetic probability ratios. Genetic testing, including amniocentesis, would need to be performed to determine whether the fetus is affected. The couple eventually may need emotional support, but the status of the pregnancy must be determined first.

Evidence-based care practices designed to support normal labor and birth recommend which practice during the immediate newborn period? A. The healthy newborn should be taken to the nursery for a complete assessment. B. After drying, the infant should be given to the mother wrapped in a receiving blanket. C. Encourage skin-to-skin contact of mother and baby. D. The father or support person should be encouraged to hold the infant while awaiting delivery of the placenta.

C. Encourage skin-to-skin contact of mother and baby. Although this is the practice in many facilities, it is neither evidence-based nor supportive of family-centered care. This is a common practice and more family friendly than separating mother and baby; however, ideally the baby should be placed skin to skin. The unwrapped infant should be placed on the woman's bare chest or abdomen, then covered with a warm blanket. Skin-to-skin contact keeps the newborn warm, prevents neonatal infection, enhances physiologic adjustment to extrauterine life, and fosters early breastfeeding. The father or support person is likely anxious to hold and admire the newborn. This can happen after the infant has been placed skin to skin and breastfeeding has been initiated.

A woman at 35 weeks of gestation has had an amniocentesis. The results reveal that surface-active phospholipids are present in the amniotic fluid. The nurse is aware that this finding indicates: A The fetus is at risk for Down syndrome. B. The woman is at high risk for developing preterm labor. C. Lung maturity. D. Meconium is present in the amniotic fluid.

C. Lung maturity. The presence of surface-active phospholipids is not an indication of Down syndrome. This result reveals the fetal lungs are mature and in no way indicates risk for preterm labor. The detection of the presence of pulmonary surfactants, surface-active phospholipids, in amniotic fluid has been used to determine fetal lung maturity, or the ability of the lungs to function after birth. This occurs at approximately 35 weeks of gestation. Meconium should not be present in the amniotic fluid.

B (With the increase in the blood flow to the left atrium from the lungs, the pressure is increased, and the foramen ovale is functionally closed. The pressure in the right atrium decreases at birth. It is higher during fetal life. Blood flow increases to the left ventricle after birth. The hepatic blood flow changes, but that is not the reason for the closure of the foramen ovale.)

Cardiovascular changes that cause the foramen ovale to close at birth are a direct result of: a. Increased pressure in the right atrium. b. Increased pressure in the left atrium. c. Decreased blood flow to the left ventricle. d. Changes in the hepatic blood flow.

D (The woman can pretend that she is attempting to stop the passing of gas or the flow of urine midstream. This will replicate the sensation of the muscles drawing upward and inward. Each contraction should be as intense as possible without contracting the abdomen, buttocks, or thighs. Guidelines suggest that these exercises should be done 24 to 100 times per day. Positive results are shown with a minimum of 24 to 45 repetitions per day. The best position to learn Kegel exercises is to lie supine with knees bent. A secondary position is on the hands and knees.)

Childbirth may result in injuries to the vagina and uterus. Pelvic floor exercises also known as Kegel exercises will help to strengthen the perineal muscles and encourage healing. The nurse knows that the client understands the correct process for completing these conditioning exercises when she reports: A. "I contract my thighs, buttocks, and abdomen." B. "I do 10 of these exercises every day." C. "I stand while practicing this new exercise routine." D. "I pretend that I am trying to stop the flow of urine midstream."

As a perinatal nurse, you realize that an FHR that is tachycardic, bradycardic, has late decelerations, or loss of variability is nonreassuring and is associated with: A. Hypotension B. Cord compression C. Maternal drug use D. Hypoxemia

D A. Incorrect: Fetal bradycardia may be associated with maternal hypotension. B. Incorrect: Fetal variable decelerations are associated with cord compression. C. Incorrect: Maternal drug use is associated with fetal tachycardia. D. Correct: Nonreassuring heart rate patterns are associated with fetal hypoxemia. pp. 502-503

The nurse practicing in a labor setting knows that the woman most at risk for uterine rupture is: a. A gravida 3 who has had two low-segment transverse cesarean births. b. A gravida 2 who had a low-segment vertical incision for delivery of a 10-pound infant. c. A gravida 5 who had two vaginal births and two cesarean births. d. A gravida 4 who has had all cesarean births.

D

The priority nursing intervention after an amniotomy should be to: a. Assess the color of the amniotic fluid. b. Change the patient's gown. c. Estimate the amount of amniotic fluid. d. Assess the fetal heart rate.

D

The standard of care for obstetrics dictates that an internal version may be used to manipulate the: a. Fetus from a breech to a cephalic presentation before labor begins. b. Fetus from a transverse lie to a longitudinal lie before cesarean birth. c. Second twin from an oblique lie to a transverse lie before labor begins. d. Second twin from a transverse lie to a breech presentation during vaginal birth.

D

Which assessment is least likely to be associated with a breech presentation? a. Meconium-stained amniotic fluid b. Fetal heart tones heard at or above the maternal umbilicus c. Preterm labor and birth d. Post-term gestation

D

With regard to the care management of preterm labor, nurses should be aware that: a. Because all women must be considered at risk for preterm labor and prediction is so hit-and-miss, teaching pregnant women the symptoms probably causes more harm through false alarms. b. Braxton Hicks contractions often signal the onset of preterm labor. c. Because preterm labor is likely to be the start of an extended labor, a woman with symptoms can wait several hours before contacting the primary caregiver. d. The diagnosis of preterm labor is based on gestational age, uterine activity, and progressive cervical change.

D

A postpartum woman telephones about her 4-day-old infant. She is not scheduled for a weight check until the infant is 10 days old, and she is worried about whether breastfeeding is going well. Effective breastfeeding is indicated by the newborn who: a. Sleeps for 6 hours at a time between feedings. b. Has at least one breast milk stool every 24 hours. c. Gains 1 to 2 ounces per week. d. Has at least six to eight wet diapers per day.

D (After day 4, when the mother's milk comes in, the infant should have six to eight wet diapers every 24 hours. Sleeping for 6 hours between feedings is not an indication of whether the infant is breastfeeding well. Typically infants sleep 2 to 4 hours between feedings, depending on whether they are being fed on a 2- to 3-hour schedule or cluster fed. The infant should have a minimum of three bowel movements in a 24-hour period. Breastfed infants typically gain 15 to 30 g/day.)

With regard to hemolytic diseases of the newborn, nurses should be aware that: a. Rh incompatibility matters only when an Rh-negative child is born to an Rh-positive mother. b. ABO incompatibility is more likely than Rh incompatibility to precipitate significant anemia. c. Exchange transfusions frequently are required in the treatment of hemolytic disorders. d. The indirect Coombs' test is performed on the mother before birth; the direct Coombs' test is performed on the cord blood after birth.

D (An indirect Coombs' test may be performed on the mother a few times during pregnancy. Only the Rh-positive child of an Rh-negative mother is at risk. ABO incompatibility is more common than Rh incompatibility but causes less severe problems; significant anemia, for instance, is rare with ABO. Exchange transfers are needed infrequently because of the decrease in the incidence of severe hemolytic disease in newborns from Rh incompatibility.)

What does the nurse administer to a patient if there is excessive bleeding after suction curettage? A. Nifedipine (Procardia) B. Methyldopa (Aldomet) C. Hydralazine (Apresoline) D. Ergonovine (Methergine)

D (Ergonovine (Methergine) is an ergot product, which is administered to contract the uterus when there is excessive bleeding after suction curettage. Nifedipine (Procardia) is prescribed for gestational hypertension or severe preeclampsia. Methyldopa (Aldomet) is an antihypertensive medication indicated for pregnant patients with hypertension. Hydralazine (Apresoline) is also an antihypertensive medication used for treating hypertension intrapartum.)

Two hours after giving birth a primiparous woman becomes anxious and complains of intense perineal pain with a strong urge to have a bowel movement. Her fundus is firm, at the umbilicus, and midline. Her lochia is moderate rubra with no clots. The nurse suspects: a) Bladder distention b) Uterine atony c) Constipation d) Hematoma formation

D (Hematoma formation)

On reviewing the medical reports of a postpartum patient, the nurse finds that the patient has Homans' sign. What does the nurse interpret from this finding? A. Risk of uterine atony B. Hypotensive shock C. Risk of developing mastitis D. Venous thromboembolism (VTE)

D (Homans' sign is an assessment test used to determine whether the patient has VTE. Presence of Homans' sign indicates that the patient may have VTE. Uterine atony can be assessed by palpating the uterine fundus. Hypotensive shock can be assessed by checking the patient's vitals. Mastitis can be assessed by the examining the patient's breasts.)

Which fetal risk is associated with an ectopic pregnancy? A. Miscarriage B. Fetal anemia C. Preterm birth D. Fetal deformity

D (In an ectopic pregnancy, the risk for fetal deformity is high because of the pressure deformities caused by oligohydramnios. There may be facial or cranial asymmetry, various joint deformities, limb deficiency, and central nervous system (CNS) anomalies. Miscarriage is not likely to happen in an ectopic pregnancy. Instead, the patient is at risk for pregnancy-related death resulting from ectopic rupture. Fetal anemia is a risk associated with placenta previa. Preterm birth is not possible because the pregnancy is dissolved when it is diagnosed or a surgery is performed to remove the fetus.)

A woman is trying to calm her fussy baby daughter in preparation for feeding. She exhibits a need for further instruction if she does which of the following? a. Swaddles the baby b. Dims lights in the room and turns off the television c. Gently rocks the baby and talks to her in a low voice d. Attempts to get the baby to latch on immediately

D (Mother should be encouraged to let her newborn begin to suck on her clean finger until the baby begins to calm down then switch to the breast; a, b, and c are all appropriate actions to calm a fussy baby.)

Which FHR finding would concern the nurse during labor? A. Accelerations with fetal movement B. Early decelerations C. An average FHR of 126 beats/min D. Late decelerations

D A. Incorrect: Accelerations in the FHR are an indication of fetal well-being. B. Incorrect: Early decelerations in the FHR are associated with head compression as the fetus descends into the maternal pelvic outlet; they generally are not a concern during normal labor. C. Incorrect: This FHR finding is normal and not a concern. D. Correct: Late decelerations are caused by uteroplacental insufficiency and are associated with fetal hypoxemia. They are considered ominous if persistent and uncorrected. p. 507

All parents are entitled to a birthing environment in which breastfeeding is promoted and supported. The Baby Friendly Hospital Initiative endorsed by WHO and UNICEF was founded to encourage institutions to offer optimal levels of care for lactating mothers. Which instruction is not included in the "Ten Steps to Successful Breastfeeding for Hospitals"? a. Give newborns no food or drink other than breast milk. b. Have a written breastfeeding policy that is communicated to all staff. c. Help mothers initiate breastfeeding within one half hour of birth. d. Give artificial teats or pacifiers as necessary.

D (No artificial teats or pacifiers (also called dummies or soothers) should be given to breastfeeding infants. No other food or drink should be given to the newborn unless medically indicated. The breastfeeding policy should be routinely communicated to all health care staff. All staff should be trained in the skills necessary to maintain this policy. Breastfeeding should be initiated within one half hour of birth, and all mothers need to be shown how to maintain lactation even if they are separated from their babies.)

The nurse should teach breastfeeding mothers about breast care measures to preserve the integrity of the nipples and areola. Which of the following should the nurse include in these instructions? a. Cleanse nipples and areola twice a day with mild soap and water. b. Apply vitamin E cream to nipples and areola at least four times each day before a feeding. c. Insert plastic-lined pads into the bra to absorb leakage and protect clothing. d. Place a nipple shell into the bra if nipples are sore.

D (No soap should be used because it could dry the areola and increase the risk for irritation; vitamin E should not be used because it is a fat-soluble vitamin that the infant could ingest when breastfeeding; lanolin or colostrum/milk are the preferred substances to be applied to the area; plastic liners can trap moisture and lead to sore nipples.)

What bacterial infection is definitely decreasing because of effective drug treatment? a. Escherichia coli infection b. Tuberculosis c. Candidiasis d. Group B streptococcal infection

D (Penicillin has significantly decreased the incidence of group B streptococcal infection. E. coli may be increasing, perhaps because of the increasing use of ampicillin (resulting in a more virulent E. coli resistant to the drug). Tuberculosis is increasing in the United States and Canada. Candidiasis is a fairly benign fungal infection.)

When responding to the question "Will I produce enough milk for my baby as she grows and needs more milk at each feeding?" the nurse should explain that: a. The breast milk will gradually become richer to supply additional calories. b. As the infant requires more milk, feedings can be supplemented with cow's milk. c. Early addition of baby food will meet the infant's needs. d. The mother's milk supply will increase as the infant demands more at each feeding.

D (The amount of milk produced depends on the amount of stimulation of the breast. Increased demand with more frequent and longer breastfeeding sessions results in more milk available for the infant. Mature breast milk will stay the same. The amounts will increase as the infant feeds for longer times. Supplementation will decrease the amount of stimulation of the breast and decrease the milk production. Solids should not be added until about 4 to 6 months, when the infant's immune system is more mature. This will decrease the chance of allergy formations.)

The most common cause of decreased variability in the FHR that lasts 30 minutes or less is: A. Altered cerebral blood flow B. Fetal hypoxemia C. Umbilical cord compression D. Fetal sleep cycles

D A. Incorrect: Altered fetal cerebral blood flow would result in early decelerations in the FHR. B. Incorrect: Fetal hypoxemia would be evidenced by tachycardia initially and then bradycardia. A persistent decrease or loss of FHR variability may be seen. C. Incorrect: Umbilical cord compression would result in variable decelerations in the FHR. D. Correct: A temporary decrease in variability can occur when the fetus is in a sleep state. These sleep states do not usually last longer than 30 minutes. p. 502

Early this morning, an infant boy was circumcised using the PlastiBell method. The nurse tells the mother that she and the infant can be discharged after: a. The bleeding stops completely. b. Yellow exudate forms over the glans. c. The PlastiBell rim falls off. d. The infant voids.

D (The infant should be observed for urination after the circumcision. Bleeding is a common complication after circumcision. The nurse will check the penis for 12 hours after a circumcision to assess and provide appropriate interventions for prevention and treatment of bleeding. Yellow exudates cover the glans penis in 24 hours after the circumcision. This is part of normal healing and not an infective process. The PlastiBell remains in place for about a week and falls off when healing has taken place.)

While completing a newborn assessment, the nurse should be aware that the most common birth injury is: a. To the soft tissues. b. Caused by forceps gripping the head on delivery. c. Fracture of the humerus and femur. d. Fracture of the clavicle.

D (The most common birth injury is fracture of the clavicle (collarbone). It usually heals without treatment, although the arm and shoulder may be immobilized for comfort.)

A newly delivered mother who intends to breastfeed tells her nurse, "I am so relieved that this pregnancy is over so I can start smoking again." The nurse encourages the client to refrain from smoking. However, this new mother insists that she will resume smoking. The nurse will need to adapt her health teaching to ensure that the client is aware that: a. Smoking has little or no effect on milk production. b. There is no relation between smoking and the time of feedings. c.The effects of secondhand smoke on infants are less significant than for adults. d. The mother should always smoke in another room.

D (The new mother should be encouraged not to smoke. If she continues to smoke, she should be encouraged to always smoke in another room removed from the baby. Smoking may impair milk production. When the products of tobacco are broken down, they cross over into the breast milk. Tobacco also results in a reduction of the immunologic properties of breast milk. Research supports that mothers should not smoke within 2 hours before a feeding. The effects of secondhand smoke on infants include sudden infant death syndrome.)

A pregnant woman at 37 weeks of gestation has had ruptured membranes for 26 hours. A cesarean section is performed for failure to progress. The fetal heart rate (FHR) before birth is 180 beats/min with limited variability. At birth the newborn has Apgar scores of 6 and 7 at 1 and 5 minutes and is noted to be pale and tachypneic. On the basis of the maternal history, the cause of this newborn's distress is most likely to be: a. Hypoglycemia. b. Phrenic nerve injury. c. Respiratory distress syndrome. d. Sepsis.

D (The prolonged rupture of membranes and the tachypnea (before and after birth) both suggest sepsis. An FHR of 180 beats/min is also indicative. This infant is at high risk for sepsis.)

Premature infants who exhibit 5 to 10 seconds of respiratory pauses followed by 10 to 15 seconds of compensatory rapid respiration are: a. Suffering from sleep or wakeful apnea. b. Experiencing severe swings in blood pressure. c. Trying to maintain a neutral thermal environment. d. Breathing in a respiratory pattern common to premature infants.

D (This pattern is called periodic breathing and is common to premature infants. It may still require nursing intervention of oxygen and/or ventilation. Apnea is a cessation of respirations for 20 seconds or longer. It should not be confused with periodic breathing.)

When assessing the relative advantages and disadvantages of internal and external electronic fetal monitoring, nurses should be aware that both: A. Can be used when membranes are intact B. Measure the frequency, duration, and intensity of UCs C. May need to rely on the woman to indicate when UA is occurring D. Can be used during the antepartum and intrapartum periods

D A. Incorrect: For internal monitoring, the membranes must have ruptured and the cervix must be sufficiently dilated. B. Incorrect: Internal monitoring measures the intensity of contractions; external monitoring cannot do this. C. Incorrect: With external monitoring, the woman may need to alert the nurse that UA is occurring; internal monitoring does not require this. D. Correct: External monitoring can be used in both periods; internal monitoring can be used only in the intrapartum period. p. 500

Nurses should be aware that accelerations in the fetal heart rate: A. Are indications of fetal well-being when they are periodic B. Are greater and longer in preterm gestations C. Are usually seen with breech presentations when they are episodic D. May visibly resemble the shape of the uterine contraction

D A. Incorrect: Periodic accelerations occur with UC and usually are seen with breech presentations. Episodic accelerations occur during fetal movement and are indications of fetal well-being. B. Incorrect: Preterm accelerations peak at 10 beats/min above the baseline and last for at least 10 seconds, not 15 seconds. C. Incorrect: Periodic accelerations occur with UC and usually are seen with breech presentations. Episodic accelerations occur during fetal movement and are indications of fetal well-being. D. Correct: They may resemble the shape of the uterine contraction or may be spikelike. p. 507

While evaluating an external monitor tracing of a woman in active labor whose labor is being induced, the nurse notes that the FHR begins to decelerate at the onset of several contractions and returns to baseline before each contraction ends. The nurse should: A. Change the woman's position B. Discontinue the oxytocin infusion C. Insert an internal monitor D. Document the finding in the client's record

D A. Incorrect: The FHR indicates early decelerations, which are not an ominous sign and do not require any intervention. The nurse should simply document these findings. B. Incorrect: The presence of early decelerations is not an ominous sign and does not require any intervention. C. Incorrect: The presence of early decelerations is not an ominous sign and does not require any intervention. D. Correct: The FHR indicates early decelerations, which are not an ominous sign and do not require any intervention. The nurse should simply document these findings. p. 509

When using IA for FHR, nurses should be aware that: A. They can be expected to cover only two or three clients when IA is the primary method of fetal assessment. B. The best course is to use the descriptive terms associated with EFM when documenting results. C. If the heartbeat cannot be found immediately, a shift must be made to electronic monitoring. D. Ultrasound can be used to find the fetal heartbeat and reassure the mother if initial difficulty was a factor.

D A. Incorrect: When used as the primary method of fetal assessment, auscultation requires a nurse-to-client ratio of one to one. B. Incorrect: Documentation should use only terms that can be numerically defined; the usual visual descriptions of EFM are inappropriate. C. Incorrect: Locating fetal heartbeats often takes time. Mothers can be reassured verbally and by the ultrasound pictures if that device is used to help locate the heartbeat. D. Correct: Locating fetal heartbeats often takes time. Mothers can be reassured verbally and by the ultrasound pictures if that device is used to help locate the heartbeat. p. 500

A 25-year-old multiparous woman gave birth to an infant boy 1 day ago. Today her husband brings a large container of brown seaweed soup to the hospital. When the nurse enters the room, the husband asks for help with warming the soup so that his wife can eat it. What is the nurses most appropriate response? a. Didnt you like your lunch? b. Does your physician know that you are planning to eat that? c. What is that anyway? d. Ill warm the soup in the microwave for you.

D Offering to warm the food shows cultural sensitivity to the dietary preferences of the woman and is the most appropriate response. Cultural dietary preferences must be respected. Women may request that family members bring favorite or culturally appropriate foods to the hospital. Asking the woman to identify her food does not show cultural sensitivity.

On examining a woman who gave birth 5 hours ago, the nurse finds that the woman has completely saturated a perineal pad within 15 minutes. What is the nurses highest priority at this time? a. Beginning an intravenous (IV) infusion of Ringers lactate solution b. Assessing the womans vital signs c. Calling the womans primary health care provider d. Massaging the womans fundus

D The nurse should first assess the uterus for atony by massaging the womans fundus. Uterine tone must be established to prevent excessive blood loss. The nurse may begin an IV infusion to restore circulatory volume, but this would not be the first action. Blood pressure is not a reliable indicator of impending shock from impending hemorrhage; assessing vital signs should not be the nurses first action. The physician would be notified after the nurse completes the assessment of the woman.

A woman gave birth vaginally to a 9-pound, 12-ounce girl yesterday. Her primary health care provider has written orders for perineal ice packs, use of a sitz bath three times daily, and a stool softener. Which information regarding the clients condition is most closely correlated with these orders? a. Woman is a gravida 2, para 2. b. Woman had a vacuum-assisted birth. c. Woman received epidural anesthesia. d.Woman has an episiotomy.

D These orders are typical interventions for a woman who has had an episiotomy, lacerations, and hemorrhoids. A multiparous classification is not an indication for these orders. A vacuum-assisted birth may be used in conjunction with an episiotomy, which would indicate these interventions. The use of an epidural anesthesia has no correlation with these orders.

With regard to the estimation and interpretation of the recurrence of risks for genetic disorders, nurses should be aware that: A. With a dominant disorder, the likelihood of the second child also having the condition is 100%. B. An autosomal recessive disease carries a one in eight risk of the second child also having the disorder. C. Disorders involving maternal ingestion of drugs carry a one in four chance of being repeated in the second child. D. The risk factor remains the same no matter how many affected children are already in the family.

D. The risk factor remains the same no matter how many affected children are already in the family. In a dominant disorder, the likelihood of recurrence in subsequent children is 50% (one in two). An autosomal recessive disease carries a one in four chance of recurrence. Subsequent children would be at risk only if the mother continued to use drugs; the rate of risk would be difficult to calculate. Each pregnancy is an independent event. The risk factor (e.g., one in two, one in four) remains the same for each child, no matter how many children are born to the family.

A (The appealing facial expression of the infant is a normal behavioral pattern and is indicative of the infant's desire to seek the parents' attention. If the infant seeks attention from other adults, it indicates that the infant is not getting required attention from the parents. Jerky movements upon touching indicate that the infant is not reacting in a normal way to the parents and may suggest that the parents are not comfortable while handling the infant. If the infant does not respond properly to the care giving tasks of the parents, it indicates that the infant lacks adequate attachment with the parents.)

During a home visit, the nurse observes that a newborn baby is well attached to the parents. Which infant behavior did the nurse most likely observe in the baby? A. The baby used appealing facial expressions. B. The baby sought attention from other adults. C. The baby's body movements were jerky when touched. D. The baby was unresponsive to the parents' caregiving.

C (Postpartum blues)

During a phone follow-up conversation with a woman who is 4 days postpartum, the woman tells the nurse, "I don't know what's wrong. I love my son, but I feel so let down. I seem to cry for no reason!" The nurse would recognize that the woman is experiencing: A. Taking-in B. Postpartum depression (PPD) C. Postpartum blues D. Attachment difficulty

D (A psychologic factor is not one of the essential factors in the initiation of breathing; the fourth factor is sensory. The sensory factors include handling by the provider, drying by the nurse, lights, smells, and sounds. Chemical factors are essential for the initiation of breathing. During labor, decreased levels of oxygen and increased levels of carbon dioxide seem to have a cumulative effect that is involved in the initiation of breathing. Clamping of the cord may also contribute to the start of respirations. Prostaglandins are known to inhibit breathing, and clamping of the cord results in a drop in the level of prostaglandins. Mechanical factors also are necessary to initiate respirations. As the infant passes through the birth canal, the chest is compressed. With birth the chest is relaxed, which allows for negative intrathoracic pressure that encourages air to flow into the lungs. The profound change in temperature between intrauterine and extrauterine life stimulates receptors in the skin to communicate with the receptors in the medulla. This also contributes to the initiation of breathing.)

During life in utero, oxygenation of the fetus occurs through transplacental gas exchange. When birth occurs, four factors combine to stimulate the respiratory center in the medulla. The initiation of respiration then follows. Which is not one of these essential factors? a. Chemical b. Mechanical c. Thermal d. Psychologic

B, D (After the baby's birth, the patient feels irritated and stressed due to lack of sleep. The nurse should suggest interventions that help the patient rest. For example, if the patient's family can be called upon to help with the household chores while the patient looks after the baby and rests, this can help the patient feel more relaxed in the early days of parenting. The patient should try to adjust her schedule to nap when the baby sleeps if possible. Installing bright lights is not an effective technique, as it does not promote sleep. Organizing a dinner party may increase the number of visitors and guests in the home, which, in turn, can increase the household work, causing the patient to become even more fatigued and tired. Coffee contains caffeine and may cause insomnia by stimulating the central nervous system (CNS). Therefore, drinking coffee 3 times a day does not support relaxation.)

During the postnatal visit, the patient tells the nurse, "I'm feeling irritated and tired. I haven't slept properly for days." The nurse suggests a few interventions to help the patient with relaxation and rest. Which techniques adopted by the patient indicate effective learning? Select all that apply. A. Installed bright lights in the room B. Asked a family member to stay overnight C. Organized a small dinner party at her home D. Takes a nap when the baby is sleeping E. Started drinking coffee 3 times a day

D (Women who are HIV positive are discouraged from breastfeeding. Although hepatitis B antigen has not been shown to be transmitted through breast milk, as an added precaution infants born to HBsAg-positive women should receive the hepatitis B vaccine and immune globulin immediately after birth. Everted nipples are functional for breastfeeding. Newly diagnosed breast cancer would be a contraindication to breastfeeding.)

For what reason would breastfeeding be contraindicated? A. Hepatitis B B. Everted nipples C. History of breast cancer 3 years ago D. Human immunodeficiency virus (HIV) positive

A (Primary medical management in all cases of DIC involves correction of the underlying cause, volume replacement, blood component therapy, optimization of oxygenation and perfusion status, and continued reassessment of laboratory parameters.)

In caring for the woman with disseminated intravascular coagulation (DIC), what order should the nurse anticipate? A. Administration of blood B. Preparation of the client for invasive hemodynamic monitoring C. Restriction of intravascular fluids D. Administration of steroids

D (Mothers need to reestablish their own well-being to effectively care for their baby. The woman should not be told what to do and needs to care for her own well-being. Because the woman needs to rest does not indicate ineffective maternal-newborn attachment. The behavior described is typical of this stage and is not a reflection of ineffective attachment unless it persists. Test-Taking Tip: Be alert for grammatical inconsistencies. If the response is intended to complete the stem (an incomplete sentence) but makes no grammatical sense to you, it might be a distractor rather than the correct response. Question writers typically try to eliminate these inconsistencies.)

Four hours after a difficult labor and birth, a primiparous woman refuses to feed her baby, stating that she is too tired and just wants to sleep. The nurse should: A. Tell the woman she can rest after she feeds her baby. B. Recognize this as a behavior of the taking-hold stage. C. Record the behavior as ineffective maternal-newborn attachment. D. Take the baby back to the nursery, reassuring the woman that her rest is a priority at this time.

C (Blue cohosh)

Herbal remedies have been used with some success to control PPH after initial management. Some herbs have homeostatic actions, whereas others work as oxytocic agents to contract the uterus. ________________ is a commonly used oxytocic herbal remedy. a) Witch hazel b) Lady's mantel c) Blue cohosh d) Yarrow

B (An abnormal odor of the lochia indicates infection in the uterus.)

If the nurse suspects a uterine infection in the postpartum client, she should assess the: a. pulse and blood pressure. b. odor of the lochia. c. episiotomy site. d. abdomen for distention.

C (The bowel is initially sterile in the newborn, and vitamin K cannot be synthesized until food is introduced into the bowel. Vitamin K is necessary to activate blood clotting factors. The platelet count in term newborns is near adult levels. Vitamin K is necessary to activate prothrombin and other clotting factors.)

In administering vitamin K to the infant shortly after birth, the nurse understands that vitamin K is: a. Important in the production of red blood cells. b. Necessary in the production of platelets. c. Not initially synthesized because of a sterile bowel at birth. d. Responsible for the breakdown of bilirubin and prevention of jaundice.

D (The transvaginal examination allows pelvic anatomy to be evaluated in greater detail and allows intrauterine pregnancies to be diagnosed earlier. The abdominal examination requires a full bladder; the transvaginal examination requires an empty bladder. The transvaginal examination is more useful in the first trimester; the abdominal examination works better after the first trimester. Neither method should be painful, although with the transvaginal examination the woman feels pressure as the probe is moved.)

In comparing the abdominal and transvaginal methods of ultrasound examination, nurses should explain to their clients that: A. Both require the woman to have a full bladder. B. The abdominal examination is more useful in the first trimester. C. Initially the transvaginal examination can be painful. D. The transvaginal examination allows pelvic anatomy to be evaluated in greater detail.

D (The infant seeks attention from any adult in the room.)

In follow-up appointments or visits with parents and their new baby, it is useful if the nurse can identify infant behaviors that can either facilitate or inhibit attachment. What is an inhibiting behavior? A. The infant cries only when hungry or wet. B. The infant's activity is somewhat predictable. C. The infant clings to the parents. D. The infant seeks attention from any adult in the room.

B (The parents hover around the infant, directing attention to and pointing at the infant)

In follow-up appointments or visits with parents and their new baby, it may be useful if the nurse can identify parental behaviours that can either facilitate or inhibit attachment. What is a facilitating behavior? a) The parents have difficulty naming the infant b) The parents hover around the infant, directing attention to and pointment at the infant c) The parents make no effort to interpret the actions or needs of the infant d) The parents do not move from fingertip touch to palmar contact and holding

D (The purpose of all cultural practices is to protect the mother and fetus during pregnancy. Although many cultures consider pregnancy normal, certain practices are expected of women of all cultures to ensure a good outcome. Cultural prescriptions tell women what to do, and cultural proscriptions establish taboos. The purposes of these practices are to prevent maternal illness resulting from a pregnancy-induced imbalanced state and to protect the vulnerable fetus.)

In her work with pregnant women of various cultures, a nurse practitioner has observed various practices that seemed strange or unusual. She has learned that cultural rituals and practices during pregnancy seem to have one purpose in common. Which statement best describes that purpose? A. To promote family unity B. To ward off the "evil eye" C. To appease the gods of fertility D. To protect the mother and fetus during pregnancy

C (Gestational age cohorts comprise the groups, with approximately 8 to 12 women in each group. This group remains intact throughout the pregnancy. Individual follow-up visits are scheduled as needed. Group sessions begin at 12 to 16 weeks of gestation and end with an early postpartum visit. Before group sessions the client has an individual assessment, physical examination, and history. At the beginning of each group meeting, clients measure their own blood pressure, weight, and urine dips and enter these in their record. Fetal heart rate assessment and fundal height are obtained by the nurse. Results evaluating this approach have been very promising. In a study of adolescent clients, there was a decrease in low-birth-weight infants and an increase in breastfeeding rates.)

In response to requests by the U.S. Public Health Service for new models of prenatal care, an innovative new approach to prenatal care known as centering pregnancy was developed. Which statement would accurately apply to the centering model of care? A. Group sessions begin with the first prenatal visit. B. At each visit, blood pressure, weight, and urine dipsticks are obtained by the nurse. C. Eight to 12 women are placed in gestational-age cohort groups. D. Outcomes are similar to those of traditional prenatal care.

D (Nurses should be open to and ready to help with a variety of medical and nonmedical approaches.)

In their role of implementing a plan of care for infertile couples, nurses should: a. Be comfortable with their sexuality and nonjudgmental about others to effectively counsel their clients. b. Know about such nonmedical remedies as diet, exercise, and stress management. c. Be able to direct clients to sources of information about what herbs to take that might help and which ones to avoid. d. Do all of the above plus be knowledgeable about potential drug and surgical remedies.

B (During the first trimester, ultrasound examination is performed to obtain information regarding the number, size, and location of gestatials sacs; the presence or absence of fetal cardiac and body movements; the presences or absence of uterine abnormalities (e.g., bicornuate uterus or fibroids) or adnexal masses (e.g., ovarian cysts or an ectopic pregnancy); and pregnancy dating.)

In the first trimester, ultrasonography can be used to gain information on: A. Amniotic fluid volume. B. Location of Gestational sacs C. Placental location and maturity. D. Cervical length.

A (A full bladder displaces the uterus and prevents contraction of the uterus and uterine atony is the primary cause of postpartum hemorrhage. Shock, infection and DIC are not related to bladder distention)

In the fourth stage of labor, a full bladder increases the risk for A. Hemorrhage B. Dissesminated intravascular coagulation C. Infection D. Shock

C (This category is correctly referred to as sociodemographic risk. These factors stem from the mother and her family. Ethnicity may be one of the risks to pregnancy; however, it is not the only factor in this category. Low income, lack of prenatal care, age, parity, and marital status also are included. Biophysical is one of the broad categories used for determining risk. These include genetic considerations, nutritional status, and medical and obstetric disorders. Psychosocial risks include smoking, caffeine, drugs, alcohol, and psychologic status. All of these adverse lifestyles can have a negative effect on the health of the mother or fetus. Environmental risks are risks that can affect both fertility and fetal development. These include infections, chemicals, radiation, pesticides, illicit drugs, and industrial pollutants.)

In the past, factors to determine whether a woman was likely to develop a high risk pregnancy were evaluated primarily from a medical point of view. A broader, more comprehensive approach to high-risk pregnancy has been adopted today. There are now four categories based on threats to the health of the woman and the outcome of pregnancy. Which of the following is not one of these categories? A. Biophysical B. Psychosocial C. Geographic D. Environmental

B (Mood swings are natural and are likely to affect every woman to some degree. A woman may dislike being pregnant, refuse to accept it, and still love and accept the child. Ambivalent feelings about pregnancy are normal for mature or immature women, younger or older women. Conflicts such as not wanting to be pregnant or childrearing and career-related decisions need to be resolved. The baby ends the pregnancy but not all the issues.)

In understanding and guiding a woman through her acceptance of pregnancy, a maternity nurse should be aware that: A. Nonacceptance of the pregnancy very often equates to rejection of the child. B. Mood swings most likely are the result of worries about finances and a changed lifestyle as well as profound hormonal changes. C. Ambivalent feelings during pregnancy usually are seen only in emotionally immature or very young mothers. D. Conflicts such as not wanting to be pregnant or childrearing and career-related decisions need not be addressed during pregnancy because they will resolve themselves naturally after birth.

A (A woman's eggs are collected from her ovaries, fertilized in the laboratory with sperm, and transferred to her uterus after normal embryonic development has occurred. The statement, "A donor embryo will be transferred into your wife's uterus" describes therapeutic donor insemination. "Donor sperm will be used to inseminate your wife" describes the procedure for a donor embryo. "Don't worry about the technical stuff; that's what we are here for" discredits the client's need for teaching and is not the most appropriate response.)

In vitro fertilization-embryo transfer (IVF-ET) is a common approach for women with blocked fallopian tubes or unexplained infertility and for men with very low sperm counts. A husband and wife have arrived for their preprocedural interview. The husband asks the nurse to explain what the procedure entails. The nurse's most appropriate response is: a. "IVF is a type of assisted reproductive therapy that involves collecting eggs from your wife's ovaries, fertilizing them in the lab with your sperm, and transferring the embryo to her uterus." b. "A donor embryo will be transferred into your wife's uterus." c. "Donor sperm will be used to inseminate your wife." d. "Don't worry about the technical stuff; that's what we are here for."

B (Cephalhematomas are characterized by bleeding between the bone and its covering, the periosteum. Because of the breakdown of the red blood cells within a hematoma, the infants are at greater risk for jaundice. Cephalhematomas do not increase the risk for infections. Caput is an edematous area on the head from pressure against the cervix. Erythema toxicum is a benign rash of unknown cause that consists of blotchy red areas.)

Infants in whom cephalhematomas develop are at increased risk for: a. Infection. b. Jaundice. c. Caput succedaneum. d. Erythema toxicum.

9. A woman is undergoing a nipple-stimulated contraction stress test (CST). She is having contractions that occur every 3 minutes. The fetal heart rate (FHR) has a baseline of approximately 120 beats/min without any decelerations. The interpretation of this test is said to be: a. Negative. b. Positive. c. Satisfactory. d. Unsatisfactory.

a. Negative.

C (Advantages of DMPA include a contraceptive effectiveness comparable to that of combined oral contraceptives with the requirement of only four injections a year. Disadvantages of injectable progestins are prolonged amenorrhea and uterine bleeding. Use of injectable progestin carries an increased risk of venous thrombosis and thromboembolism. To be effective, DMPA injections must be administered every 11 to 13 weeks. Access to health care is necessary to prevent pregnancy or potential complications.)

Injectable progestins (DMPA, Depo-Provera) are a good contraceptive choice for women who: a. Want menstrual regularity and predictability. b. Have a history of thrombotic problems or breast cancer. c. Have difficulty remembering to take oral contraceptives daily. d. Are homeless or mobile and rarely receive health care.

D (Placenta abruptio is premature separation of the placenta as opposed to partial or complete adherence. This occurs between the 20th week of gestation and delivery in the area of the decidua basilis. Symptoms include localized pain and bleeding.)

It is important for the perinatal nurse to be knowledgeable regarding conditions of abnormal adherence of the placenta. This occurs when the zygote implants in an area of defective endometrium and results in little to no zone separation between the placenta and decidua. Which classification of separation is not recognized as an abnormal adherence pattern? A. Placenta accreta B. Placenta increta C. Placenta percreta D. Placenta abruptio

A (Breast tenderness should persist for 24 to 48 hours after lactation begins. That movable, noncancerous mass is a filled milk sac. Colostrum is present for a few days whether the mother breastfeeds or not. A mother who does not want to breastfeed should also avoid stimulating her nipples.)

Knowing that the condition of the new mother's breasts will be affected by whether she is breastfeeding, nurses should be able to tell their clients all the following statements except: A. Breast tenderness is likely to persist for about a week after the start of lactation. B. As lactation is established, a mass may form that can be distinguished from cancer by its position shift from day to day. C. In nonlactating mothers colostrum is present for the first few days after childbirth. D. If suckling is never begun (or is discontinued), lactation ceases within a few days to a week.

B ("Grandparents can help you with parenting skills and also help preserve family traditions.")

Many first-time parents do not plan on their parents' help immediately after the newborn arrives. What statement by the nurse is the most appropriate when counseling new parents about the involvement of grandparents? A. "You should tell your parents to leave you alone." B. "Grandparents can help you with parenting skills and also help preserve family traditions." C. "Grandparent involvement can be very disruptive to the family." D. "They are getting old. You should let them be involved while they can."

B (If MSAFP findings are abnormal, follow-up procedures include genetic counseling for families with a history of neural tube defect, repeated MSAFP screening, ultrasound examination, and possibly amniocentesis. Indications for use of PUBS include prenatal diagnosis of inherited blood disorders, karyotyping of malformed fetuses, detection of fetal infection, determination of the acid-base status of fetuses with intrauterine growth restriction, and assessment and treatment of isoimmunization and thrombocytopenia in the fetus. BPP is a method of assessing fetal well-being in the third trimester. Before amniocentesis is considered, the client first would have an ultrasound for direct visualization of the fetus.)

Maternal serum alpha-fetoprotein (MSAFP) screening indicates an elevated level. MSAFP screening is repeated and again is reported as higher than normal. What would be the next step in the assessment sequence to determine the well-being of the fetus? A. Percutaneous umbilical blood sampling (PUBS) B. Ultrasound for fetal anomalies C. Biophysical profile (BPP) for fetal well-being D. Amniocentesis for genetic anomalies

A, B, D

Medications used to manage postpartum hemorrhage include (choose all that apply): A. Pitocin B. Methergine C. Terbutaline D. Hemabate E. Magnesium sulfate

A (Attachment, or bonding, is a process that occurs over time and does not require early contact.)

New parents express concern that because of the mother's emergency cesarean birth under general anesthesia, they did not have the opportunity to hold and bond with their daughter immediately after her birth. The nurse's response should convey to the parents that: A. Attachment, or bonding, is a process that occurs over time and does not require early contact. B. The time immediately after birth is a critical period for humans. C. Early contact is essential for optimum parent-infant relationships. D. They should just be happy that the infant is healthy.

A (Because the infant is wet with amniotic fluid and blood, heat loss by evaporation occurs quickly. Heat loss by convection occurs when drafts come from open doors and air currents created by people moving around. If the heat loss is caused by placing the baby near cold surfaces or equipment, it is referred to as a radiation heat loss. Conduction heat loss occurs when the baby comes in contact with cold objects or surfaces.)

Nurses can prevent evaporative heat loss in the newborn by: a. Drying the baby after birth and wrapping the baby in a dry blanket. b. Keeping the baby out of drafts and away from air conditioners. c. Placing the baby away from the outside wall and the windows. d. Warming the stethoscope and the nurse's hands before touching the baby.

D (Early PPH is also known as primary, or acute, PPH; late PPH is known as secondary PPH.)

Nurses need to know the basic definitions and incidence data about postpartum hemorrhage. For instance: A. PPH is easy to recognize early; after all, the woman is bleeding. B. Traditionally, it takes more than 1000 ml of blood after vaginal birth and 2500 ml after cesarean birth to define the condition as PPH. C. If anything, nurses and doctors tend to overestimate the amount of blood loss. D. Traditionally, PPH has been classified as early or late with respect to birth.

D (MSAFP is a screening tool, not a diagnostic tool. CVS provides a rapid result, but it is declining in popularity because of advances in noninvasive screening techniques. MSAFP screening is recommended for all pregnant women. MSAFP screening, not PUBS, is part of the triple-marker tests for Down syndrome.)

Nurses should be aware of the strengths and limitations of various biochemical assessments during pregnancy, including that: A. Chorionic villus sampling (CVS) is becoming more popular because it provides early diagnosis. B. Maternal serum alpha-fetoprotein (MSAFP) screening is recommended only for women at risk for neural tube defects. C. Percutaneous umbilical blood sampling (PUBS) is one of the triple-marker tests for Down syndrome. D. MSAFP is a screening tool only; it identifies candidates for more definitive procedures.

D (MSAFP is a screening tool, not a diagnostic tool. Further diagnostic testing is indicated after an abnormal MSAFP. CVS does provide a rapid result, but it is declining in popularity because of advances in noninvasive screening techniques. MSAFP screening is recommended for all pregnant women. MSAFP, not PUBS, is part of the quad-screen tests for Down syndrome.)

Nurses should be aware of the strengths and limitations of various biochemical assessments during pregnancy, including that: A. Chorionic villus sampling (CVS) is becoming more popular because it provides early diagnosis. B. Screening for maternal serum alpha-fetoprotein (MSAFP) levels is recommended only for women at risk for neural tube defects. C. Percutaneous umbilical blood sampling (PUBS) is one of the quad-screen tests for Down syndrome. D. MSAFP is a screening tool only; it identifies candidates for more definitive procedures.

D (Rationale: CVS does provide a rapid result, but it is declining in popularity because of advances in noninvasive screening techniques. MSAFP screening is recommended for all pregnant women. MSAFP, not PUBS, is part of the quad-screen tests for Down syndrome. This is correct. MSAFP is a screening tool, not a diagnostic tool. Further diagnostic testing is indicated after an abnormal MSAFP.)

Nurses should be aware of the strengths and limitations of various biochemical assessments during pregnancy, including that: A. chorionic villus sampling (CVS) is becoming more popular because it provides early diagnosis. B. screening for maternal serum alpha-fetoprotein (MSAFP) levels is recommended only for women at risk for neural tube defects. C. percutaneous umbilical blood sampling (PUBS) is one of the quad-screen tests for Down syndrome. D. MSAFP is a screening tool only; it identifies candidates for more definitive procedures.

A (Women tend to be more stressed about infertility tests and to place more importance on having children. The prevalence of infertility is stable among the overall population, but it increases with a woman's age, especially over age 40. Of cases with an identifiable cause, about 40% are related to female factors, 40% to male factors, and 20% to both partners. Sterility is the inability to conceive. Infertility is a state of requiring a prolonged time to conceive, or subfertility.)

Nurses should be aware that infertility: a. Is perceived differently by women and men. b. Has a relatively stable prevalence among the overall population and throughout a women's potential reproductive years. c. Is more likely the result of a physical flaw in the woman than in her male partner. d. Is the same thing as sterility.

A (An abnormal BPP score is an indication that labor should be induced. The BPP evaluates the health of the fetus, requires many different measures, and is a noninvasive procedure.)

Nurses should be aware that the biophysical profile (BPP): A. Is an accurate indicator of impending fetal well-being. B. Is a compilation of health risk factors of the mother during the later stages of pregnancy. C. Consists of a Doppler blood flow analysis and an amniotic fluid index. D. Involves an invasive form of ultrasound examination.

B (The leading cause of PPH is uterine atony, which complicates one in 20 births. The uterus is overstretched and contracts poorly after the birth.)

Nurses should first look for the most common cause of PPH, _____, by _____. A. Lacerations of the genital tract; checking for the source of blood B. Uterine atony; evaluating the contractility of the uterus C. Inversion of the uterus; feeling for a smooth mass through the dilated cervix D. Retained placenta; noting the type of bleeding

B (This is the primary step of this nursing assessment and necessary before completing the process and moving on to a nursing diagnosis. Once the client's level of knowledge is determined, the nurse can interact with the woman to compare options, reliability, cost, comfort level, protection from sexually transmitted infections, and a partner's willingness to participate. Although important, obtaining data about the frequency of coitus is not the first action that the nurse should undertake when completing an assessment. Data should include not only the frequency of coitus but also the number of sexual partners, level of contraceptive involvement, and partner's objections. Assessing the woman's willingness to touch herself is a key factor for the nurse to discuss should the client express interest in using one of the fertility awareness methods of contraception. The nurse must be aware of the client's plan regarding whether she is attempting to prevent conception, delay conception, or conceive.)

Nurses, certified nurse-midwives, and other advanced practice nurses have the knowledge and expertise to assist women in making informed choices regarding contraception. A multidisciplinary approach should ensure that the woman's social, cultural, and interpersonal needs are met. Which action should the nurse take first when meeting with a new client to discuss contraception? a. Obtain data about the frequency of coitus. b. Determine the woman's level of knowledge about contraception and commitment to any particular method. c. Assess the woman's willingness to touch her genitals and cervical mucus. d. Evaluate the woman's contraceptive life plan.

D (An environment that fosters as much privacy as possible should be created)

Nursing activities that promote parent-infant attachment are many and varied. One activity that should not be overlooked is the management of the environment. While providing routine mother-baby care, the nurse should ensure that: A. The baby is able to return to the nursery at night so that the new mother can sleep B. Routine times for care are established to reassure the parents C. The father should be encouraged to go home at night to prepare for mother-baby discharge D. An environment that fosters as much privacy as possible should be created

C (Adequate fluid intake prevents urinary stasis, dilutes urine, and flushes out waste products.)

Nursing measures that help prevent postpartum urinary tract infection include: a. promoting bed rest for 12 hours after delivery. b. discouraging voiding until the sensation of a full bladder is present. c. forcing fluids to at least 3000 mL/day. d. encouraging the intake of orange, grapefruit, or apple juice.

C (Adolescent mothers have a higher documented incidence of child abuse.)

Of the many factors that influence parental responses, nurses should be aware that all of these statements regarding age are true except: A. An adolescent mother's egocentricity and unmet developmental needs interfere with her ability to parent effectively. B. An adolescent mother is likely to use less verbal instruction, be less responsive, and interact less positively than other mothers. C. Adolescent mothers have a higher documented incidence of child abuse. D. Mothers older than 35 often deal with more stress related to work and career issues, as well as decreasing libido.

D (Postpartum or puerperal infection is any clinical infection of the genital canal that occurs within 28 days after miscarriage, induced abortion, or childbirth. The definition used in the United States continues to be the presence of a fever of 38° C (100.4° F) or higher on 2 successive days of the first 10 postpartum days, starting 24 hours after birth.)

One of the first symptoms of puerperal infection to assess for in the postpartum woman is: A. Fatigue continuing for longer than 1 week B. Pain with voiding C. Profuse vaginal bleeding with ambulation D. Temperature of 38° C (100.4° F) or higher on 2 successive days starting 24 hours after birth

D (The vulnerability of the brain likely is to the result of the cerebellum growth spurt. The neuromuscular system is almost completely developed at birth. The reflex system is not relevant. The various sleep-wake states are not relevant.)

One reason the brain is vulnerable to nutritional deficiencies and trauma in early infancy is the: a. Incompletely developed neuromuscular system. b. Primitive reflex system. c. Presence of various sleep-wake states. d. Cerebellum growth spurt.

A (Having the sibling make or choose a gift for the new baby helps to make the child feel a part of the process. Special time should be set aside just for the other children without interruption from the newborn. Someone other than the mother should carry the baby into the home so she can give full attention to greeting her other children. Children should be actively involved in the care of the baby according to their ability without overwhelming them.)

Parents can facilitate the adjustment of their other children to a new baby by: A. Having the children choose or make a gift to give to the new baby on its arrival home. B. Emphasizing activities that keep the new baby and other children together. C. Having the mother carry the new baby into the home so she can show him or her to the other children. D. Reducing stress on other children by limiting their involvement in the care of the new baby.

A (In normal infant respiration the chest and abdomen rise synchronously, and breaths are shallow and irregular. Breathing with nasal flaring is a sign of respiratory distress. Diaphragmatic breathing with chest retraction is a sign of respiratory distress. Infant breaths are not deep with a regular rhythm.)

Part of the health assessment of a newborn is observing the infant's breathing pattern. A full-term newborn's breathing pattern is predominantly: a. Abdominal with synchronous chest movements. b. Chest breathing with nasal flaring. c. Diaphragmatic with chest retraction. d. Deep with a regular rhythm.

B (As the infant's skin begins to dry, the creases will appear more prominent, and the infant's gestation could be misinterpreted. Footprinting will not interfere with the creases. Heel sticks will not interfere with the creases. The creases will appear more prominent after 24 hours.)

Plantar creases should be evaluated within a few hours of birth because: a. The newborn has to be footprinted. b. As the skin dries, the creases will become more prominent. c. Heel sticks may be required. d. Creases will be less prominent after 24 hours.

D (An offensive odor usually indicates an infection. Lochia flow should approximate a heavy menstrual period for the first 2 hours and then steadily decrease. Less lochia usually is seen after cesarean births and usually increases with ambulation and breastfeeding.)

Postbirth uterine/vaginal discharge, called lochia: A. Is similar to a light menstrual period for the first 6 to 12 hours. B. Is usually greater after cesarean births. C. Will usually decrease with ambulation and breastfeeding. D. Should smell like normal menstrual flow unless an infection is present.

d (Should smell like normal menstrual flow unless an infection is present)

Postbirth uterine/vaginal discharge, called lochia: a) Is similar to a light menstrual period for the first 6 to 12 hours b) Is usually greater after cesarean births c) Will usually decrease with ambulation and breastfeeding d) Should smell like normal menstrual flow unless an infection is present

A (Testing for the antibody to HIV is strongly recommended for all pregnant women. A HIV test is recommended for all women, regardless of risk factors. Women who test positive for HIV can be treated, reducing the risk of transmission to the fetus.)

Prenatal testing for human immunodeficiency virus (HIV) is recommended for: A. All women, regardless of risk factors. B. A woman who has had more than one sexual partner. C. A woman who has had a sexually transmitted infection. D. A woman who is monogamous with her partner.

A (Rationale: Because of the possibility of fetomaternal hemorrhage, administering RhoD immunoglobulin to the woman who is Rh negative is standard practice after an amniocentesis. Anticoagulants are not administered before amniocentesis as this would increase the risk of bleeding when the needle is inserted transabdominally. A CT is not required before amniocentesis, because the procedure is ultrasound guided. The mother is not exposed to radiation during amniocentesis.)

Prior to the patient undergoing amniocentesis, the most appropriate nursing intervention is to: A. administer RhoD immunoglobulin. B. administer anticoagulant. C. send the patient for a computed tomography (CT) scan before the procedure. D. assure the mother that short-term radiation exposure is not harmful to the fetus.

B (Polyhydramnios (amniotic fluid >2000 mL) is 10 times more likely to occur in diabetic compared with nondiabetic pregnancies. Polyhydramnios puts the mother at risk for premature rupture of membranes, premature labor, and postpartum hemorrhage. Prolonged rupture of membranes, intrauterine growth restriction, intrauterine fetal death, and renal agenesis (Potter syndrome) all put the client at risk for developing oligohydramnios. Anencephaly, placental insufficiency, and perinatal hypoxia all contribute to the risk for postterm pregnancy. Maternal age older than 35 and balanced translocation (maternal and paternal) are risk factors for chromosome abnormalities.)

Risk factors tend to be interrelated and cumulative in their effect. While planning the care for a laboring client with diabetes mellitus, the nurse is aware that she is at a greater risk for: A. Oligohydramnios. B. Polyhydramnios. C. Postterm pregnancy. D. Chromosomal abnormalities.

A, B, C (Vaginal bleeding, rupture of membranes, and severe headaches all are signs of potential complications in pregnancy. Clients should be advised to report these signs to the health care provider. Decreased libido and urinary frequency are common discomforts of pregnancy that do not require immediate health care interventions.)

Signs and symptoms that a woman should report immediately to her health care provider include (Select all that apply): A. Vaginal bleeding. B. Rupture of membranes. C. Heartburn accompanied by severe headache. D. Decreased libido. E. Urinary frequency.

C (The normal value of AFI is 10 cm or greater, with the upper limit of normal around 25 cm. An AFI less than 5 cm indicates oligohydramnios. This condition is associated with renal agenesis in the fetus. A high AFI indicates neural tube defects and fetal hydrops. The AFI is not directly related to fetal movement. Fetal activity can be assessed using ultrasonography.)

The amniotic fluid index (AFI) of a pregnant patient is 3 cm. What clinical information related to the fetus does the nurse infer from this? A. Neural tube defect B. Fetal hydrops C. Renal defects D. Low activity level

D (If the BPP score is less than 2, regardless of gestational age, delivery can be performed. If the BPP score is 0 to 2 and chronic asphyxia is suspected, then testing time should be extended to 120 minutes. If the BPP score is 8 to 10 and a low risk for chronic asphyxia is suspected, then the test should be repeated at twice-weekly intervals. If the fetal pulmonary test result is negative and the BPP score is 6, then the BPP profile should be repeated in 4 to 6 hours.)

The biophysical profile (BPP) testing report of a pregnant patient gives the following information: one episode of fetal breathing movement lasting for 30 seconds in a 30-minute observation; three limb movements of the fetus in 30 minutes; an amniotic fluid index greater than 5; a reactive nonstress test; and a BPP score of 1. The test is performed for 120 minutes. What does the nurse expect the primary health care provider to do? A. Extend the test time to 120 minutes. B. Repeat the test twice a week. C. Repeat the test in 4 to 6 hours. D. Consider delivery of the fetus.

A (Wear a snug, supportive bra)

The breasts of a bottle-feeding woman are engorged. The nurse should instruct her to A) Wear a snug, supportive bra B) Allow warm water to soothe the breasts during a shower C) Express milk from breasts occasionally to relieve discomfort D) Place absorbent pads with plastic liners into her bra to absorb leakage

C (PPD can easily go undetected)

The early postpartum period is a time of emotional and physical vulnerability. Many mothers can easily become psychologically overwhelmed by the reality of their new parental responsibilities. Fatigue compounds these issues. Although the baby blues are a common occurrence in the postpartum period, about one-half million women in America experience a more severe syndrome known as postpartum depression (PPD). Which statement regarding PPD is essential for the nurse to be aware of when attempting to formulate a nursing diagnosis? a) PPD symptoms are consistently severe b) This syndrome affects only new members c) PPD can easily go undetected d) Only mental health professionals should teach new parents about this condition

A (This protection, vernix caseosa, is needed because the infant's skin is so thin. Surfactant is a protein that lines the alveoli of the infant's lungs. Caput succedaneum is the swelling of the tissue over the presenting part of the fetal head. Acrocyanosis is cyanosis of the hands and feet that results in a blue coloring.)

The cheeselike, whitish substance that fuses with the epidermis and serves as a protective coating is called: a. Vernix caseosa. b. Surfactant. c. Caput succedaneum. d. Acrocyanosis.

B (Rho(D) immune globulin (RhoGam) is given to the Rho(D)-negative mother, within 72 hours after delivery of an Rho(D)-positive baby (if the Coombs is negative). RhoGam is never given to the baby.)

The client has just given birth to a healthy, full-term infant. The client is Rho(D) negative and her baby is Rho(D) positive. Which intervention will take place to reduce the possibility of isoimmunization? A. Administering Rho(D) immune globulin to the baby, IM, within 72 hours B. Administering Rho(D) immune globulin to the mother, IM, within 72 hours C. Administering Rho(D) immune globulin to the mother, IM, at her 6-week visit D. Administering Rho(D) immune globulin to the mother, IM, within 3 months

D (Rationale: A woman that develops postpartum psychosis usually does so within four weeks of delivery. Only 1% of women develop this disorder. Suicide and infanticide are common and the disorder is considered a medical emergency. Delusions and hallucinations accompany the disorder and the woman usually has a past history of a psychiatric disorder and treatment.)

The clinic nurse is caring for a woman who is suspected of developing postpartum psychosis. Which of the following statements characterizes this disorder: A. Symptoms start within several days of delivery B. The disorder is common in postpartum women C. Suicide and infanticide are uncommon in this disorder D. Delusions and hallucinations accompany this disorder

B (NOT: Other clusters of neonatal behavior include motor performance, quality of movement and tone and reflexes, and assessment of neonatal reflexes.)

The healthy infant must accomplish both behavioral and biologic tasks to develop normally. Behavioral characteristics form the basis of the social capabilities of the infant. Newborns pass through a hierarchy of developmental challenges as they adapt to their environment and caregivers. This progression in behavior is the basis for the Brazelton Neonatal Behavioral Assessment (NBAS). Please match the cluster of neonatal behavior with the correct level on the NBAS scale. Ability to attend to visual and auditory stimuli while alert. a. Habituation b. Orientation c. Range of state d. Autonomic stability e. Regulation of state

A (NOT: Other clusters of neonatal behavior include motor performance, quality of movement and tone and reflexes, and assessment of neonatal reflexes.)

The healthy infant must accomplish both behavioral and biologic tasks to develop normally. Behavioral characteristics form the basis of the social capabilities of the infant. Newborns pass through a hierarchy of developmental challenges as they adapt to their environment and caregivers. This progression in behavior is the basis for the Brazelton Neonatal Behavioral Assessment (NBAS). Please match the cluster of neonatal behavior with the correct level on the NBAS scale. Ability to respond to discrete stimuli while asleep a. Habituation b. Orientation c. Range of state d. Autonomic stability e. Regulation of state

E (NOT: Other clusters of neonatal behavior include motor performance, quality of movement and tone and reflexes, and assessment of neonatal reflexes.)

The healthy infant must accomplish both behavioral and biologic tasks to develop normally. Behavioral characteristics form the basis of the social capabilities of the infant. Newborns pass through a hierarchy of developmental challenges as they adapt to their environment and caregivers. This progression in behavior is the basis for the Brazelton Neonatal Behavioral Assessment (NBAS). Please match the cluster of neonatal behavior with the correct level on the NBAS scale. How the infant responds when aroused a. Habituation b. Orientation c. Range of state d. Autonomic stability e. Regulation of state

C (NOT: Other clusters of neonatal behavior include motor performance, quality of movement and tone and reflexes, and assessment of neonatal reflexes.)

The healthy infant must accomplish both behavioral and biologic tasks to develop normally. Behavioral characteristics form the basis of the social capabilities of the infant. Newborns pass through a hierarchy of developmental challenges as they adapt to their environment and caregivers. This progression in behavior is the basis for the Brazelton Neonatal Behavioral Assessment (NBAS). Please match the cluster of neonatal behavior with the correct level on the NBAS scale. Measure of general arousability. a. Habituation b. Orientation c. Range of state d. Autonomic stability e. Regulation of state

D (NOT: Other clusters of neonatal behavior include motor performance, quality of movement and tone and reflexes, and assessment of neonatal reflexes.)

The healthy infant must accomplish both behavioral and biologic tasks to develop normally. Behavioral characteristics form the basis of the social capabilities of the infant. Newborns pass through a hierarchy of developmental challenges as they adapt to their environment and caregivers. This progression in behavior is the basis for the Brazelton Neonatal Behavioral Assessment (NBAS). Please match the cluster of neonatal behavior with the correct level on the NBAS scale. Signs of stress related to homeostatic adjustment a. Habituation b. Orientation c. Range of state d. Autonomic stability e. Regulation of state

D (The puerperium, also called the fourth trimester or the postpartum period of pregnancy, lasts about 3 to 6 weeks. Involution marks the end of the puerperium, or the fourth trimester of pregnancy. Lochia refers to the various vaginal discharges during the puerperium, or fourth trimester of pregnancy.)

The interval between the birth of the newborn and the return of the reproductive organs to their normal nonpregnant state is called the: A. Involutionary period because of what happens to the uterus. B. Lochia period because of the nature of the vaginal discharge. C. Mini-tri period because it lasts only 3 to 6 weeks. D. Puerperium, or fourth trimester of pregnancy.

C (Strict adherence by all health care personnel to aseptic techniques during childbirth and the postpartum period is very important and the least expensive measure to prevent infection.)

The most effective and least expensive treatment of puerperal infection is prevention. What is important in this strategy? A. Large doses of vitamin C during pregnancy B. Prophylactic antibiotics C. Strict aseptic technique, including handwashing, by all health care personnel D. Limited protein and fat intake

A (The most likely cause of the infant's vomiting is insufficient attachment between infant and parent. Therefore the nurse should advise the parent to maintain skin contact with the child while feeding. To accomplish this, the nurse should teach the mother kangaroo care, which promotes attachment between the infant and the mother. Breast milk provides optimal nutrition to the baby. If the patient is able to breastfeed the baby, the nurse should encourage the patient to continue breastfeeding rather than switching to formula or cow's milk. The nurse's primary goal should be to improve the attachment between the infant and mother so that the nurse does not need to involve other family members for feeding unless absolutely necessary.)

The mother of a newborn reports that the child vomits milk immediately after breastfeeding. The infant's laboratory report does not show any abnormalities. What is the most appropriate nursing intervention in this situation? A. Teach the mother kangaroo care. B. Ask the mother to give the child formula. C. Ask the mother to feed the child cow's milk. D. Teach other family members to feed the baby

A (The maternal serum level of alpha-fetoprotein is used to screen for Down syndrome, neural tube defects, and other chromosome anomalies. The multiple marker test would not detect diaphragmatic hernia, congenital cardiac abnormality, or anencephaly. Additional testing, such as ultrasonography and amniocentesis, would be required to diagnose these conditions.)

The multiple marker test is used to assess the fetus for which condition? A. Down syndrome B. Congenital cardiac abnormality C. Diaphragmatic hernia D. Anencephaly

A (During the early postpartum period, lochia rubra should be moderate to significant. Scant lochia may indicate that large clots are blocking the flow. Thirst, fatigue and a temperature up to 100.4oF (38oC) are normal within the first 24 hours. Immediately after delivery, vasomotor changes may cause a shaking chill.)

The nurse assesses a client who delivered 24 hours ago. Which of the following suggests the need for further assessment? A. Scant lochia rubra B. Chills C. Thirst and fatigue D. A temperature of 100.2oF (37.9oC)

D (Retained placental fragments or infection cause subinvolution of the uterus. Therefore the nurse should assess the patient for any placental fragments in the uterus. Estrogen and progesterone stimulate massive growth of the uterus during pregnancy. In the postpartum stage, the hormone levels are reduced and, therefore, do not affect involution of the uterus. Platelet aggregation causes uterine muscle contraction, but it does not result in involution of the uterus.)

The nurse assesses a postpartum patient several hours after delivery and suspects that the uterus is subinvoluted. What could be a potential etiology for this finding? A. Estrogen levels B. Progesterone levels C. Impaired platelet aggregation D. Retained placental fragments

D (The most critical adjustment of a newborn at birth is the establishment of respirations. The cardiovascular system changes markedly after birth as a result of fetal respiration, which reduces pulmonary vascular resistance to the pulmonary blood flow and initiates a chain of cardiac changes that support the cardiovascular system. The infant relies on passive immunity received from the mother for the first 3 months of life. After the establishment of respirations, heat regulation is critical to newborn survival.)

The nurse assessing a newborn knows that the most critical physiologic change required of the newborn is: a. Closure of fetal shunts in the circulatory system. b. Full function of the immune defense system at birth. c. Maintenance of a stable temperature. d. Initiation and maintenance of respirations.

A (A nulliparous woman has prominent rugae in introitus along with erythema and edema. Nulliparous women may have mild uterine cramping resulting in fewer or less severe afterpains compared to multiparous woman. Single gestation may cause mild afterpains, but it does not cause prominence of rugae or erythema or edema in the introitus. A multiparous woman usually has more afterpains compared with a nulliparous woman. Rugae are also seen in a multiparous woman, but the rugae are less prominent and flattened. Multiple gestation usually causes severe afterpains.)

The nurse assessing a patient finds prominent rugae erythema and edema in the vaginal introitus. The patient reports having mild afterpains. What does the nurse interpret about the patient's clinical A. Nulliparous B. Multiparous C. Single gestation D. Multiple gestation

B (Prenatal care ideally should begin soon after the first missed menstrual period. Regular prenatal visits offer opportunities to ensure the health of the expectant mother and her infant.)

The nurse caring for a newly pregnant woman would advise her that ideally prenatal care should begin: A. Before the first missed menstrual period. B. After the first missed menstrual period. C. After the second missed menstrual period. D. After the third missed menstrual period.

C (The nonstress test is the most widely used technique for prenatal evaluation of the fetus. The results are either nonreactive or reactive. In a nonreactive test, there are less than two qualifying accelerations of the fetal heart rate in a 20-minute period. Absence of fetal heart rate accelerations during the nonstress test indicates that the fetus is sleeping. In a reactive test, there are at least two qualifying accelerations in a 20-minute time period. More than two fetal heart rate accelerations within a 20-minute time period also would be considered a reactive test.)

The nurse finds that the nonstress test of a pregnant patient is nonreactive. Which factor in the report might have led the nurse to this finding? A. No qualifying accelerations in a 20-minute period B. Two qualifying accelerations in a 20-minute period C. Less than two qualifying accelerations in a 20-minute period D. More than two qualifying accelerations in a 20-minute period

C (Claiming)

The nurse hears a primiparous woman talking to her son and telling him that his chin is just like his dad's chin. This woman's statement reflects: A. Mutuality B. Synchrony C. Claiming D. Reciprocity

C (Couples that become new parents may develop new challenges in their personal relationship because they will spend most of their time taking care of the baby. Therefore the couple needs to make time to spend together rather than spending all of their time focused on the baby. This can be accomplished by scheduling time for one another, apart from the time dedicated to caring for the infant. The couple has to share their expectations with each other and should appreciate each other for their assistance in the child care activities.)

The nurse instructs a group of nursing students about relationship issues in couples who are first-time parents. The nurse asks a student for suggestions to give new parents to prevent relationship problems. Which student response indicates the need for further teaching? A. "Appreciate each other for the assistance." B. "Assess your relationship on a regular basis." C. "You should both spend more time with the baby." D. "Share your personal expectations with each other.

D (Fibrinogen is normally increased during pregnancy and remains elevated in the immediate puerperium. Fibrinogen may lead to thromboembolism due to immobility. Therefore the nurse instructs the patient to exercise the lower limbs frequently to prevent the risk of thromboembolism. Hemorrhoids occur as a result of the pressure from the fetus on the abdomen, hormonal changes, and increased intraabdominal pressures during pregnancy. Exercise of the lower limbs is not helpful in reducing the risk of hemorrhoids. Lower-limb exercise does not reduce the risk of endometritis either, because it is associated with uterine discharge. Lower-limb exercises do not produce any effect on uterine discharge. Estrogen deficiency is responsible for a decreased amount of vaginal lubrication, and it leads to dyspareunia. Exercise of the lower limbs is not increase estrogen.)

The nurse instructs the pregnant patient to exercise her lower limbs frequently. What clinical condition is the nurse trying to prevent? A. Hemorrhoids B. Endometritis C. Dyspareunia D. Thromboembolism

C (Maternal hypertension can cause serious adverse effects on the fetus. A blood pressure reading of 150/90 mm Hg indicates that the mother is hypertensive. To assess the effect of maternal hypertension on the fetus, the nurse should refer the patient for a Doppler blood flow analysis. It is a noninvasive ultrasonic technique used to study fetal blood flow. NT is a technique used to assess genetic abnormalities in the fetus. CVS is a prenatal test used to diagnose structural defects in the fetus. PUBS is used to assess the fetal circulation.)

The nurse is assessing a pregnant patient and finds that her blood pressure is 150/90 mm Hg. What procedure does the nurse recommend for this patient? A. Nuchal translucency (NT) test B. Chorionic villus sampling (CVS) C. Doppler blood flow analysis D. Percutaneous umbilical blood sampling (PUBS)

A (The couple does not attempt to talk to the infant. This indicates that they lack interest in developing a bond with the infant. Calling the infant by name indicates the couple is giving importance to the individuality of the infant. In Chinese culture, the newborn is taken care of by the grandparents. This is done to promote rest and recovery in the mother after childbirth. Maintaining eye contact with the infant is a sign of attention by the parents toward the infant.)

The nurse is caring for a Chinese couple who have a newborn. What patient behavior indicates that the couple lacks interest in developing an attachment with the child? The couple: A. Never talks to the infant. B. Always calls the infant by name. C. Allows grandparents to take care of the child. D. Maintains good eye contact with the infant.

C (Postpartum hemostasis is facilitated by uterine muscle contractions, which results in the compression of intramyometrial blood vessels. Hemostasis is influenced by the hormone oxytocin. The uterine contractions (UCs) decrease during the first 1 to 2 hours after delivery due to low levels of oxytocin. Therefore the nurse needs to administer exogenous oxytocin to the pregnant woman. Postpartum hemostasis does not take place by platelet aggregation or clot formation. Moreover, administering platelet aggregators may increase the risk of clot formation within the blood vessels. Estrogen and progesterone play a vital role in the development and maintenance of pregnancy, but they do not induce any uterine muscle contractions.)

The nurse is caring for a pregnant patient who just delivered a baby. The woman has continuous, heavy vaginal bleeding after the delivery. What should be the immediate medication intervention? A. Platelet aggregators B. Exogenous estrogen C. Exogenous oxytocin D. Exogenous progesterone

C (The infant wants to interact with the mother by looking at the mother's eyes and facial expressions. Visually impaired mothers may show impassive facial expressions, which makes the infant uninterested. The infant may abandon the mother and try to interact with other family members. The mother can interact efficiently by nodding and smiling frequently while talking. This conceals the impassive facial expressions of the mother. Because the father cannot improve the facial expressions of the mother by standing nearby, the father cannot improve the interaction between the mother and infant by standing nearby. As the infant looks at the face of the mother to interact, holding the newborn's hand while talking will not improve the interaction either. The mother should give spontaneous responses to the child's actions. Waiting for others to tell her about the infant's actions may not be helpful in this situation.)

The nurse is caring for a North American, postpartum patient who is visually impaired. The nurse finds that the mother is having a difficult time establishing attachment with the infant. What suggestion would the nurse give the mother in order to create an effective interaction with the infant? A. "Ask the infant's father to stand nearby." B. "Hold the newborn's hand while talking." C. "Nod and smile while talking to the infant." D. "Get informed of the actions of the infant."

A (Native American patients do not initiate breastfeeding until their breast milk comes in. They avoid feeding the first milk (colostrum) to the child. In this situation, the nurse should inform the patient about the health benefits of feeding colostrum to the child. Mother's milk is the best source of nutrition for the baby; the formula milk is not as nutritious. The mother is also unlikely to have problems ejecting milk. Native American mothers tend to avoid feeding the baby unless the colostrum stops ejecting and the mother starts ejecting milk. Because the patient has not yet started breastfeeding, it is unlikely that the patient has sore nipples, so the nurse would not need to apply anti-inflammatory ointment.)

The nurse is caring for a postpartum Native American patient who is unwilling to breastfeed the baby until after the breast milk has come in. What will the nurse do in this situation? A. Explain the importance of first milk to the patient. B. Ask the patient to give formula milk to the infant. C. Use additional measures for milk ejection in the patient. D. Apply anti-inflammatory ointment around the patient's nipples.

B (Lochia serosa is a pink or brown fluid containing old blood, serum, leukocytes, and tissue debris. The lochia serosa starts 3 to 4 days after childbirth. Lochia alba is a yellow to white fluid containing leukocytes, serum, epithelial cells, bacteria, and decidua. It starts 10 days after childbirth in most women. In the case of a vaginal tear, the patient would have bright red bleeding for more than 2 hours after delivery. Lochia rubra, a bright red fluid containing small clots, starts from the end of the childbirth and disappears within 2 hours.)

The nurse is caring for a postpartum patient and finds that the patient has brown vaginal discharge. What is the cause of the discharge? A. Lochia alba B. Lochia serosa C. Lochia rubra D. Vaginal or cervical tear

C (Vaginal deliveries cause the pelvic muscles and ligaments to stretch and weaken. Kegel exercises help strengthen the pelvic floor muscles and thereby can prevent uterine complications, such as prolapse. The physical activity of climbing stairs may delay the process of healing from an episiotomy, so it is usually avoided. However, avoiding stairs does not prevent uterine prolapse. A diet high in protein is necessary to build muscle strength, but it cannot prevent uterine prolapse. Because the patient has already undergone delivery, sleeping in prone position does not cause any harm.)

The nurse is caring for a postpartum patient who had a normal vaginal delivery. The nurse tells the patient, "This will help you prevent uterine prolapse in later stages of life." Which instruction from the primary health care provider (PHP) is the nurse most likely explaining to the patient? A. "Avoid climbing of the stairs." B. "Maintain a high-protein diet." C. "Do Kegel exercises every day." D. "Avoid sleeping in the prone position."

D (Orthostatic hypotension develops as a result of splanchnic engorgement after birth, which causes dizziness immediately upon standing upright. Decreased blood pressure results from hypovolemia due to hemorrhage. Manifestations of endometritis include pain, fever, and abdominal tenderness, along with continued flow of lochia serosa or alba up to 3 to 4 weeks. Manifestations of hemorrhoids include itching, discomfort, and bright red bleeding upon defecation. Puerperal sepsis manifests by an increase in the maternal temperature up to 38° C (100.4° F) 24 hours after childbirth. This increased temperature persists or recurs for about 2 days.)

The nurse is caring for a postpartum patient who reports dizziness upon standing. What does the nurse believe to be the most likely cause for this occurrence? A. Endometritis B. Hemorrhoids C. Puerperal sepsis D. Orthostatic hypotension

D (While interacting with the parents of an infant who are hearing-impaired, the nurse should suggest that the parents show video recordings to the infant to improve vocalization, as this helps the newborn become familiar with human voices. Interacting with the child using sign language can enhance communication, but may not help to improve vocalization. Playing rhymes and music at a very high volume may damage the eardrum of the infant and can lead to hearing impairment. Asking a family member or friend to talk to the baby is a good solution, but it may not be a practical solution if the parents care for the child independently.)

The nurse is caring for a postpartum patient with a hearing impairment. The nurse finds that the patient's partner is also hearing-impaired. What does the nurse suggest to both parents to improve vocalization in the newborn? A. Interact with the infant using sign language. B. Play rhymes and music at a very high volume. C. Ask a family member or friend to talk to the baby. D. Show the infant recordings of television programs.

C (Estrogen and progesterone play a vital role in the development of the uterus during pregnancy. They are responsible for the growth of the uterus and may cause hypertrophy and hyperplasia of the uterine muscle cells. A decrease in estrogen and progesterone results in decreased growth of the uterus, which might even lead to miscarriage. Low estrogen and progesterone may not cause increased UCs. Moreover, the pituitary hormone oxytocin is primarily responsible for UCs during labor. Low levels of estrogen and progesterone lead to abnormally low growth of the uterus. Estrogen and progesterone also increase the blood circulation in the mother. Low levels of these hormones would decrease blood circulation.)

The nurse is caring for a pregnant woman who has low levels of estrogen and progesterone. What does the nurse expect may occur as a result of the low hormone levels? A. Massive growth of the uterus during pregnancy B. Increased uterine contractions (UCs) during labor C. Decreased growth of the uterus during pregnancy D. Increased blood circulation to the uterus during pregnancy

A (Rationale: The indirect Coombs test is a screening tool for Rh incompatibility. If the maternal titer for Rh antibodies is greater than 1:8, amniocentesis for determination of bilirubin in amniotic fluid is indicated to establish the severity of fetal hemolytic anemia. Hemoglobin reveals the oxygen carrying capacity of the blood. hCG is the hormone of pregnancy. Maternal serum alpha-fetoprotein (MSAFP) levels are used as a screening tool for NTDs in pregnancy)

The nurse is reviewing lab values to determine Rh incompatibility between mother and fetus. The nurse should assess which specific lab result? A. Indirect Coombs test B. Hemoglobin level C. hCG level D. Maternal serum alpha-fetoprotein (MSAFP)

A (The indirect Coombs test is a screening tool for Rh incompatibility. If the maternal titer for Rh antibodies is greater than 1:8, amniocentesis for determination of bilirubin in amniotic fluid is indicated to establish the severity of fetal hemolytic anemia. Hemoglobin reveals the oxygen carrying capacity of the blood. hCG is the hormone of pregnancy. Maternal serum alpha-fetoprotein (MSAFP) levels are used as a screening tool for NTDs in pregnancy.)

The nurse is reviewing lab values to determine Rh incompatibility between mother and fetus. Which specific lab result should the nurse assess? A. Indirect Coombs test B. Hemoglobin level C. hCG level D. Maternal serum alpha-fetoprotein (MSAFP)

B (Meconium is normally stored in the infant's intestines until after birth, but sometimes (in cases of fetal distress and hypoxia) it is expelled into the amniotic fluid before birth. The amniotic fluid is then said to be meconium stained. Fewer than three contractions in 10 minutes or late decelerations occurring with 50% or more of contractions constitute positive CST results. Positive CST results are associated with meconium-stained amniotic fluid. Negative CST results indicate that the fetus is normal. Suspicious or unsatisfactory CST results are not associated with any other fetal conditions.)

The nurse is reviewing the contraction stress test (CST) reports of a pregnant patient. The nurse expects the fetus to have meconium-stained amniotic fluid. What would be the reason for that conclusion? A. Negative CST results B. Positive CST results C. Suspicious CST results D. Unsatisfactory CST results

A (What appears to be a lack of interest in the newborn is in fact the Vietnamese way of demonstrating intense love by attempting to ward off evil spirits.)

The nurse notes that a Vietnamese woman does not cuddle or interact with her newborn other than to feed him, change his diapers or soiled clothes, and put him to bed. In evaluating the woman's behavior with her infant, the nurse realizes that: A. What appears to be a lack of interest in the newborn is in fact the Vietnamese way of demonstrating intense love by attempting to ward off evil spirits. B. The woman is inexperienced in caring for newborns. C. The woman needs a referral to a social worker for further evaluation of her parenting behaviors once she goes home with the newborn. D. Extra time needs to be planned for assisting the woman in bonding with her newborn.

B (The woman should be encouraged to hold her infant in the en face position and make eye contact with him. Talking and cooing to her son is a normal infant-parent interaction. Cuddling is a normal infant-parent interaction. Sharing her son's success at feeding is a normal infant-parent interaction.)

The nurse observes several interactions between a postpartum woman and her new son. What behavior (if exhibited by this woman) does the nurse identify as a possible maladaptive behavior regarding parent-infant attachment? A. Talks and coos to her son B. Seldom makes eye contact with her son C. Cuddles her son close to her D. Tells visitors how well her son is feeding

B (Show the mother how the infant initiates interaction and attends to her.)

The nurse observes that a 15-year-old mother seems to ignore her newborn. A strategy that the nurse can use to facilitate mother-infant attachment in this mother is: A. Tell the mother she must pay attention to her infant B. Show the mother how the infant initiates interaction and attends to her C. Demonstrate for the mother different positions for holding her infant while feeding D. Arrange for the mother to watch a video on parent-infant interaction

A (Mexican mothers may believe that excessive admiration may result in evil eye (mal de ojo). They feel that children are more susceptible to evil eye. This behavior does not indicate that the mother does not have a good attachment with the baby. Similarly, this evasive behavior does not indicate the mother is worried about infection or the baby becoming fussy after exposure to new faces.)

The nurse observes that a Spanish-speaking patient of Mexican descent does not like the nurses to lean in and admire her newborn. What is the most likely reason for this behavior? The mother: A. Wants to protect the infant from evil eye. B. Does not have good attachment with the baby. C. Feels that the baby may acquire infection. D. Feels that the baby may become fussy seeing new faces.

A (Patients of Southeast Asian descent believe that minimal touching and cuddling of the newborn after birth protects the child from evil spirits. If the patient had postpartum blues, the patient may show different symptoms such as crying episodically. If the patient does not hold or feed the child, it would indicate that the patient is afraid of handling the child. Avoiding cuddling and touching the newborn does not indicate that the patient is improving the baby's strength and immunity. Southeast Asians believe that massaging the newborn would be helpful in improving the baby's strength and immunity.)

The nurse observes that a patient of Southeast Asian descent avoids cuddling and touching her newborn. What can the nurse infer about the patient from these observations? The patient: A. May be trying to protect the child from evil spirits. B. Is depressed because of postpartum blues. C. Is afraid of handling the child all by herself. D. Is improving the baby's strength and immunity

D (While learning to breastfeed, women may refrain from talking or smiling excessively so that the infant will not be distracted and will continue to feed. The patient does not avoid talking to soothe and quiet the infant; instead, she may gently stroke the infant to provide comfort after feeding. Avoiding speaking with the infant is not helpful for coping with depression and is not a typical behavior of postpartum blues. The mother's quiet behavior during breastfeeding does not indicate that she has not properly bonded with the baby.)

The nurse observes that a postpartum patient does not talk or smile during breastfeeding but instead quietly maintains her gaze on the infant. What does the nurse infer from this observation? The patient: A. Aims to soothe and quiet the infant. B. Is experiencing postpartum blues. C. Is not properly bonding with the infant. D. Wants the infant to suck the milk without interruption.

A (When parents speak, infants tend to get excited and dance in tune with the parent's voice. This indicates that the infant is responding well to the patient. The newborn should ideally look at the parent when the parent tries to communicate. If the newborn looks toward the wall, it indicates that the newborn is not responding well to the parent. If the newborn keeps the upper and lower limbs still when the parent interacts, it indicates that the newborn is not responding well to the patient.)

The nurse observes that the newborn is responding well to the parent. Which behavior from the newborn did the nurse observe to come to this conclusion? The newborn: A. Was dancing in tune with the parent's voice. B. Looked at the wall upon hearing the parent's voice. C. Was not kicking its legs in tune to the parent's voice. D. Was not waving its arms in tune to the parent's voice.

A (Doppler blood flow analysis allows the examiner to study the blood flow noninvasively in the fetus and placenta. It is a helpful tool in the management of high risk pregnancies because of intrauterine growth restriction (IUGR), diabetes mellitus, multiple fetuses, or preterm labor. Because of the potential risk of inducing labor and causing fetal distress, a CST is not performed on a woman whose fetus is preterm. Indications for an amniocentesis include diagnosis of genetic disorders or congenital anomalies, assessment of pulmonary maturity, and the diagnosis of fetal hemolytic disease, not IUGR. Fetal kick count monitoring is performed to monitor the fetus in pregnancies complicated by conditions that may affect fetal oxygenation. Although this may be a useful tool at some point later in this woman's pregnancy, it is not used to diagnose IUGR.)

The nurse sees a woman for the first time when she is 30 weeks pregnant. The woman has smoked throughout the pregnancy, and fundal height measurements now are suggestive of growth restriction in the fetus. In addition to ultrasound to measure fetal size, what tool is useful in confirming the diagnosis? A. Doppler blood flow analysis B. Contraction stress test (CST) C. Amniocentesis D. Daily fetal movement counts

A (Rationale: Doppler blood flow analysis allows the examiner to study the blood flow noninvasively in the fetus and the placenta. It is a helpful tool in the management of high-risk pregnancies because of intrauterine growth restriction (IUGR), diabetes mellitus, multiple fetuses, or preterm labor. Because of the potential risk of inducing labor and causing fetal distress, a CST is not performed on a woman whose fetus is preterm. Indications for an amniocentesis include diagnosis of genetic disorders or congenital anomalies, assessment of pulmonary maturity, and the diagnosis of fetal hemolytic disease, not IUGR. Fetal kick count monitoring is performed to monitor the fetus in pregnancies complicated by conditions that may affect fetal oxygenation. Although this may be a useful tool at some point later in this woman's pregnancy, it is not used to diagnose IUGR.)

The nurse sees a woman for the first time when she is 30 weeks pregnant. The woman has smoked throughout the pregnancy, and fundal height measurements now are suggestive of growth restriction in the fetus. In addition to ultrasound to measure fetal size, what would be another tool useful in confirming the diagnosis? A. Doppler blood flow analysis B. Contraction stress test (CST) C. Amniocentesis D. Daily fetal movement counts

A (An infant who is dusky and becomes cyanotic when crying is showing poor adaptation to extrauterine life. Acrocyanosis is an expected finding during the early neonatal life. This is within normal range for a newborn. Infants enter the period of deep sleep when they are about 1 hour old.)

The nurse should immediately alert the physician when: a. The infant is dusky and turns cyanotic when crying. b. Acrocyanosis is present at age 1 hour. c. The infant's blood glucose level is 45 mg/dL. d. The infant goes into a deep sleep at age 1 hour.

B ("Infants can track their parent's eyes and distinguish patterns; they prefer complex patterns" is an accurate statement. Development of the visual system continues for the first 6 months of life. Visual acuity is difficult to determine, but the clearest visual distance for the newborn appears to be 19 cm. Infants prefer to look at complex patterns, regardless of the color. Infants prefer low illumination and withdraw from bright light.)

The parents of a newborn ask the nurse how much the newborn can see. The parents specifically want to know what type of visual stimuli they should provide for their newborn. The nurse responds to the parents by telling them: a. "Infants can see very little until about 3 months of age." b. "Infants can track their parent's eyes and distinguish patterns; they prefer complex patterns." c. "The infant's eyes must be protected. Infants enjoy looking at brightly colored stripes." d. "It's important to shield the newborn's eyes. Overhead lights help them see better."

B (Almost all instances of acute mastitis can be avoided by proper breastfeeding technique to prevent cracked nipples.)

The perinatal nurse assisting with establishing lactation is aware that acute mastitis can be minimized by: A. Washing the nipples and breasts with mild soap and water once a day B. Using proper breastfeeding techniques C. Wearing a nipple shield for the first few days of breastfeeding D. Wearing a supportive bra 24 hours a day

A (Late PPH may be the result of subinvolution of the uterus, pelvic infection, or retained placental fragments.)

The perinatal nurse caring for the postpartum woman understands that late postpartum hemorrhage is most likely caused by: A. Subinvolution of the placental site B. Defective vascularity of the decidua C. Cervical lacerations D. Coagulation disorders

A (Uterine atony is marked hypotonia of the uterus. It is the leading cause of postpartum hemorrhage.)

The perinatal nurse is caring for a woman in the immediate postbirth period. Assessment reveals that the woman is experiencing profuse bleeding. The most likely etiology for the bleeding is: A. Uterine atony B. Uterine inversion C. Vaginal hematoma D. Vaginal laceration

D (NOT: During the first 24 hours postpartum, temperature may increase to 38¦ C as a result of the dehydrating effects of labor. After 24 hours the woman should be afebrile. Other causes of fever include mastitis, endometritis, urinary tract infection, and other systemic infections. Pulse, along with stroke volume and cardiac output, remains elevated for the first hour or so after childbirth. A rapid pulse, or one that is increasing, may indicate hypovolemia as a result of hemorrhage. Hypoventilation may occur after an unusually high subarachnoid block or epidural narcotic after a cesarean birth. An increased reading in blood pressure may result from the excessive use of the vasopressor or oxytocic medication. Because gestational hypertension can persist into or occur first in the postpartum period, routine evaluation of blood pressure is necessary.)

The physiologic changes that occur during the reversal of the processes of pregnancy are distinctive; however, they are normal. To provide care during this recovery period the nurse must synthesize knowledge regarding anticipated maternal changes and deviations from normal. Excessive use of oxytocin is related to: a. Elevated temperature within the first 24 hours b. Rapid pulse c. Elevated temperature at 36 hours postpartum d. Hypertension e. Hypoventilation

B (NOT: During the first 24 hours postpartum, temperature may increase to 38¦ C as a result of the dehydrating effects of labor. After 24 hours the woman should be afebrile. Other causes of fever include mastitis, endometritis, urinary tract infection, and other systemic infections. Pulse, along with stroke volume and cardiac output, remains elevated for the first hour or so after childbirth. A rapid pulse, or one that is increasing, may indicate hypovolemia as a result of hemorrhage. Hypoventilation may occur after an unusually high subarachnoid block or epidural narcotic after a cesarean birth. An increased reading in blood pressure may result from the excessive use of the vasopressor or oxytocic medication. Because gestational hypertension can persist into or occur first in the postpartum period, routine evaluation of blood pressure is necessary.)

The physiologic changes that occur during the reversal of the processes of pregnancy are distinctive; however, they are normal. To provide care during this recovery period the nurse must synthesize knowledge regarding anticipated maternal changes and deviations from normal. Hypovolemia resulting from hemorrhage is related to: a. Elevated temperature within the first 24 hours b. Rapid pulse c. Elevated temperature at 36 hours postpartum d. Hypertension e. Hypoventilation

A (NOT: During the first 24 hours postpartum, temperature may increase to 38¦ C as a result of the dehydrating effects of labor. After 24 hours the woman should be afebrile. Other causes of fever include mastitis, endometritis, urinary tract infection, and other systemic infections. Pulse, along with stroke volume and cardiac output, remains elevated for the first hour or so after childbirth. A rapid pulse, or one that is increasing, may indicate hypovolemia as a result of hemorrhage. Hypoventilation may occur after an unusually high subarachnoid block or epidural narcotic after a cesarean birth. An increased reading in blood pressure may result from the excessive use of the vasopressor or oxytocic medication. Because gestational hypertension can persist into or occur first in the postpartum period, routine evaluation of blood pressure is necessary.)

The physiologic changes that occur during the reversal of the processes of pregnancy are distinctive; however, they are normal. To provide care during this recovery period the nurse must synthesize knowledge regarding anticipated maternal changes and deviations from normal. Dehydrating effects of labor is related to: a. Elevated temperature within the first 24 hours b. Rapid pulse c. Elevated temperature at 36 hours postpartum d. Hypertension e. Hypoventilation

E (NOT: During the first 24 hours postpartum, temperature may increase to 38¦ C as a result of the dehydrating effects of labor. After 24 hours the woman should be afebrile. Other causes of fever include mastitis, endometritis, urinary tract infection, and other systemic infections. Pulse, along with stroke volume and cardiac output, remains elevated for the first hour or so after childbirth. A rapid pulse, or one that is increasing, may indicate hypovolemia as a result of hemorrhage. Hypoventilation may occur after an unusually high subarachnoid block or epidural narcotic after a cesarean birth. An increased reading in blood pressure may result from the excessive use of the vasopressor or oxytocic medication. Because gestational hypertension can persist into or occur first in the postpartum period, routine evaluation of blood pressure is necessary.)

The physiologic changes that occur during the reversal of the processes of pregnancy are distinctive; however, they are normal. To provide care during this recovery period the nurse must synthesize knowledge regarding anticipated maternal changes and deviations from normal. Unusually high epidural or spinal block is related to: a. Elevated temperature within the first 24 hours b. Rapid pulse c. Elevated temperature at 36 hours postpartum d. Hypertension e. Hypoventilation

C (NOT: During the first 24 hours postpartum, temperature may increase to 38¦ C as a result of the dehydrating effects of labor. After 24 hours the woman should be afebrile. Other causes of fever include mastitis, endometritis, urinary tract infection, and other systemic infections. Pulse, along with stroke volume and cardiac output, remains elevated for the first hour or so after childbirth. A rapid pulse, or one that is increasing, may indicate hypovolemia as a result of hemorrhage. Hypoventilation may occur after an unusually high subarachnoid block or epidural narcotic after a cesarean birth. An increased reading in blood pressure may result from the excessive use of the vasopressor or oxytocic medication. Because gestational hypertension can persist into or occur first in the postpartum period, routine evaluation of blood pressure is necessary.)

The physiologic changes that occur during the reversal of the processes of pregnancy are distinctive; however, they are normal. To provide care during this recovery period the nurse must synthesize knowledge regarding anticipated maternal changes and deviations from normal. Puerperal sepsis is related to: a. Elevated temperature within the first 24 hours b. Rapid pulse c. Elevated temperature at 36 hours postpartum d. Hypertension e. Hypoventilation

B (The intensity of afterpains depends on the lactating status of the patient, the number of times a patient has been pregnant, and the type of gestation whether single or multiple. Breastfeeding stimulates uterine contractions (UCs), which increase afterpains. First-time mothers may have only mild uterine cramping, so nulliparous women may have mild afterpains compared with the multiparous women. An overdistended uterus caused by multiple gestation and polyhydramnios makes the afterpains more noticeable. Therefore a patient who is lactating, is multiparous, and had multiple gestation would have more afterpains. If a patient is lactating but is nulliparous and had single gestation, the intensity of afterpains would be less. A patient who is nonlactating, is nulliparous, and has oligohydramnios may have less afterpains. The patient who had multiple gestation but is nonlactating and nulliparous may have less intense afterpains.)

The postpartum patient reports to the nurse, "I am having intolerable pain after the delivery." Which conditions would cause the patient's afterpains? A. Lactating, nulliparous, single gestation B. Lactating, multiparous, multiple gestation C. Nonlactating, nulliparous, oligohydramnios D. Nonlactating, nulliparous, multiple gestation

B (The presence of bilirubin in the amniotic fluid indicates the possibility of hemolytic anemia in the fetus. The degree of hemolytic anemia can be determined by using Doppler blood flow analysis. The presence of the placental hormone inhibin-A in the quad screen indicates Down syndrome. The amniotic fluid index values are used to detect Potter syndrome. Fetal hydrops is caused by polyhydramnios, which can be assessed by ultrasound scanning.)

The primary health care provider advises a pregnant woman to undergo a Doppler blood flow analysis after reviewing the amniocentesis reports. What clinical condition in the fetus could be the reason for this referral? A. Down syndrome B. Hemolytic anemia C. Potter syndrome D. Fetal hydrops

A (Autolysis is caused by a decrease in hormone levels. Subinvolution is failure of the uterus to return to a nonpregnant state. Afterpain is caused by uterine cramps 2 to 3 days after birth. Diastasis refers to the separation of muscles.)

The self-destruction of excess hypertrophied tissue in the uterus is called: a. Autolysis. b. Subinvolution. c. Afterpain. d. Diastasis.

B (Changes begin right after birth; the cutoff time when the transition is considered over (although the baby keeps changing) is 28 days. The transition period has three phases: first reactivity, decreased response, and second reactivity. All newborns experience this transition regardless of age or type of birth. Although stress can cause variation in the phases, the mother's age and wealth do not disturb the pattern.)

The transition period between intrauterine and extrauterine existence for the newborn: a. Consists of four phases, two reactive and two of decreased responses. b. Lasts from birth to day 28 of life. c. Applies to full-term births only. d. Varies by socioeconomic status and the mother's age.

B, C, D, E (Postpartum hemorrhage often results in anemia, and iron therapy may need to be initiated. Exhaustion is common after hemorrhage. It may take the new mother weeks to feel like herself again. Fatigue may interfere with normal parent-infant bonding and attachment processes. The mother is likely to require assistance with housework and infant care. Excessive blood loss increases the risk for infection.)

The visiting nurse must be aware that women who have had a postpartum hemorrhage are subject to a variety of complications after discharge from the hospital. These include: (Choose those that apply.) a. dehydration. b. anemia. c. exhaustion. d. failure to attach to her infant. e. postpartum infection.

C (The second trimester is best for dental treatment because that is when the woman will be able to sit most comfortably in the dental chair. Dental care such as brushing with fluoride toothpaste is especially important during pregnancy because nausea during pregnancy may lead to poor oral hygiene. Emergency dental surgery is permissible, but the mother must clearly understand the risks and benefits. Conscious relaxation is useful, and it may even help the woman get through any dental appointments; it is not a reason to avoid them.)

To provide the patient with accurate information about dental care during pregnancy, maternity nurses should be aware that: A. Dental care can be dropped from the priority list because the woman has enough to worry about and is getting a lot of calcium anyway. B. Dental surgery, in particular, is contraindicated because of the psychologic stress it engenders. C. If dental treatment is necessary, the woman will be most comfortable with it in the second trimester. D. Dental care interferes with the expectant mother's need to practice conscious relaxation.

A, B, C, E (Transvaginal ultrasound is useful in obese women whose thick abdominal layers cannot be penetrated with traditional abdominal ultrasound. This procedure is also used for identifying multifetal gestation, ectopic pregnancy, estimating gestational age, confirming fetal viability, and identifying fetal abnormalities. Amniotic fluid volume is assessed during the second and third trimester. Conventional ultrasound would be used.)

Transvaginal ultrasonography is often performed during the first trimester. While preparing your 6-week gestation patient for this procedure, she expresses concerns over the necessity for this test. The nurse should explain that this diagnostic test may be indicated for a number of situations (Select all that apply). A. Multifetal gestation B. Obesity C. Fetal abnormalities D. Amniotic fluid volume E. Ectopic pregnancy

D (Puerperal sepsis is a condition in which a woman's genital tract becomes infected due to low immunity caused by long labor, severe bleeding, or dehydration. Therefore the nurse should assess the patient for puerperal sepsis if the temperature of the woman after childbirth is raised to 100.4° F. Blood pressure is routinely assessed in postpartum patients to detect hemorrhage. A rapid pulse rate indicates the presence of hypovolemia as a result of hemorrhage. The respiratory rate is measured because hypoventilation can occur after a high subarachnoid block or epidural narcotic following a cesarean birth.)

Twenty-four hours after childbirth, a patient developed a high temperature of 100.4° F. Which monitoring action is most important for the nurse? A. Pulse rate B. Blood pressure C. Respiratory rate D. Assess for puerperal sepsis

C (Within 12 hours of birth women begin to lose the excess tissue fluid that has accumulated during pregnancy. One mechanism for reducing these retained fluids is the profuse diaphoresis that often occurs, especially at night, for the first 2 or 3 days after childbirth. Postpartal diuresis is another mechanism by which the body rids itself of excess fluid. An elevated temperature would cause chills and may cause dehydration, not diaphoresis and diuresis. Diaphoresis and diuresis sometimes are referred to as reversal of the water metabolism of pregnancy, not as the basal metabolic rate. Postpartal diuresis may be caused by the removal of increased venous pressure in the lower extremities.)

Two days ago a woman gave birth to a full-term infant. Last night she awakened several times to urinate and noted that her gown and bedding were wet from profuse diaphoresis. One mechanism for the diaphoresis and diuresis that this woman is experiencing during the early postpartum period is: A. Elevated temperature caused by postpartum infection. B. Increased basal metabolic rate after giving birth. C. Loss of increased blood volume associated with pregnancy. D. Increased venous pressure in the lower extremities.

A (Inversion of the uterus and hypovolemic shock are considered medical emergencies.)

What PPH conditions are considered medical emergencies that require immediate treatment? A. Inversion of the uterus and hypovolemic shock B. Hypotonic uterus and coagulopathies C. Subinvolution of the uterus and idiopathic thrombocytopenic purpura D. Uterine atony and disseminated intravascular coagulation

B, C, D (Convection, radiation, evaporation, and conduction are the four modes of heat loss in the newborn. Perspiration and urination are not modes of heat loss in newborns.)

What are modes of heat loss in the newborn (Select all that apply)? a. Perspiration b. Convection c. Radiation d. Conduction e. Urination

A (The fundus can rise to approximately 1 cm above the umbilicus within 12 hours after childbirth. By the sixth postpartum day, the fundus is normally located halfway between the umbilicus and the symphysis pubis. At the end of the third stage of labor, the uterus is in the midline, approximately 2 cm below the level of the umbilicus with the fundus resting on the sacral promontory. The uterus is about the same size as it was at 20 weeks' gestation at 24 hours afterbirth, not 12 hours.)

What assessment does the nurse expect to find in a postpartum patient 12 hours after childbirth? A. The fundus is approximately 1 cm above the umbilicus. B. The palpation of the uterus is not possible abdominally. C. The uterus is about the same size as it was at 20 weeks' gestation. D. The fundus is located midway between the umbilicus and the symphysis pubis.

D (The newborn's flexed position guards against heat loss because it reduces the amount of body surface exposed to the environment. The newborn's body is able to constrict the peripheral blood vessels to reduce heat loss. Burning brown fat generates heat. The respiratory rate may rise to stimulate muscular activity, which generates heat.)

What infant response to cool environmental conditions is either not effective or not available to them? a. Constriction of peripheral blood vessels b. Metabolism of brown fat c. Increased respiratory rates d. Unflexing from the normal position

B (Notify the physician of any increase in the amount of lochia or a return to bright red bleeding.)

What instructions should be included in the discharge teaching plan to assist the client in recognizing early signs of complications? a. Palpate the fundus daily to ensure that it is soft. b. Notify the physician of any increase in the amount of lochia or a return to bright red bleeding. c. Report any decrease in the amount of brownish red lochia. d. The passage of clots as large as an orange can be expected.

B (Rationale: Decreased fetal movement is an indicator for performing a contraction stress test; the size (small for gestational age) is not an indicator. Maternal diabetes mellitus and postmaturity are two indications for performing a contraction stress test. Although adolescent pregnancy and poor prenatal care are risk factors of poor fetal outcomes, they are not indicators for performing a contraction stress test. Intrauterine growth restriction is an indicator; but history of a previous stillbirth, not preterm labor, is the other indicator.)

What is an indicator for performing a contraction stress test? A. Increased fetal movement and small for gestational age B. Maternal diabetes mellitus and postmaturity C. Adolescent pregnancy and poor prenatal care D. History of preterm labor and intrauterine growth restriction

B (Degradation of fibrin leads to the accumulation of fibrin split products in the blood.)

What laboratory marker is indicative of disseminated intravascular coagulation (DIC)? A. Bleeding time of 10 minutes B. Presence of fibrin split products C. Thrombocytopenia D. Hyperfibrinogenemia

D (This is a normal laboratory value in the pregnant woman. This is a normal laboratory value in the pregnant woman. This is a normal laboratory value in the pregnant woman. A rubella titer of less than 1:10 indicates a lack of immunity to rubella, a viral infection that has the potential to cause teratogenic effects on fetal development. Arrangements should be made to administer the rubella vaccine after birth during the postpartum period since administration of rubella, a live vaccine, would be contraindicated during pregnancy. Women receiving the vaccine during the postpartum period should be cautioned to avoid pregnancy for 3 months.)

What laboratory results would be a cause for concern if exhibited by a woman at her first prenatal visit during the second month of her pregnancy? A. Hematocrit 38%, hemoglobin 13 g/dL B. White blood cell count 6000/mm3 C. Platelets 300,000/mm3 D. Rubella titer 1:6

C (Petechiae (bruises) scattered over the infant's body should be reported to the pediatrician because they may indicate underlying problems. Mongolian spots are bluish-black spots that resemble bruises but fade gradually over months and have no clinical significance. Telangiectatic nevi (stork bites, angel kisses) fade by the second year and have no clinical significance. Erythema toxicum is an appalling-looking rash, but it has no clinical significance and requires no treatment.)

What marks on a baby's skin may indicate an underlying problem that requires notification of a physician? a. Mongolian spots on the back b. Telangiectatic nevi on the nose or nape of the neck c. Petechiae scattered over the infant's body d. Erythema toxicum anywhere on the body

B (The L/S ratio indicates fetal lung maturity. AFP is assessed to check for the presence of neural defects. Presence of creatinine in the amniotic fluid indicates that the patient's gestational age is more than 36 weeks. The antibody titer is used to determine Rh incompatibility in the fetus.)

What parameter does the nurse check in the amniocentesis report of a pregnant patient to assess fetal lung growth? A. Alfa-fetoprotein (AFP) levels B. Lecithin-to-sphingomyelin (L/S) ratio C. Creatinine levels in the blood D. Antibody titer in the blood

D (All systems of the female body undergo adaptation during pregnancy and childbirth, including the musculoskeletal system. All the joints are completely stabilized 6 to 8 weeks after childbirth, but the joints of the foot do not recover completely to the prepregnant state. Hypotension, bradycardia, and pelvic relaxation are temporary changes and do not last for a long time after childbirth.)

What permanent change does nurse expect the patient to have after childbirth? A Bradycardia B. Hypotension C. Pelvic relaxation D. Increased shoe size

D (The woman first centers on herself as pregnant, then on the baby as an entity separate from herself, and then on her responsibilities as a mother. The expressions, "I am pregnant," "I am going to have a baby," and "I am going to be a mother" sum up the progression through the three phases.)

What represents a typical progression through the phases of a woman's establishing a relationship with the fetus? A. Accepts the fetus as distinct from herself—accepts the biologic fact of pregnancy—has a feeling of caring and responsibility B. Fantasizes about the child's gender and personality—views the child as part of herself—becomes introspective C. Views the child as part of herself—has feelings of well-being—accepts the biologic fact of pregnancy D. "I am pregnant."—"I am going to have a baby."—"I am going to be a mother.

B (Most women experience a heavier than normal flow during the first menstrual cycle, which occurs by 3 months after childbirth. She can expect her first menstrual cycle to be heavier than normal, and the volume of her subsequent cycles to return to prepregnant levels within three or four cycles.)

What statement by a woman who just gave birth indicates that she knows what to expect about her menstrual activity after childbirth? A. "My first menstrual cycle will be lighter than normal and then will get heavier every month thereafter." B. "My first menstrual cycle will be heavier than normal and will return to my prepregnant volume within three or four cycles." C. "I will not have a menstrual cycle for 6 months after childbirth." D. "My first menstrual cycle will be heavier than normal and then will be light for several months after."

B (Magnesium sulfate administration during labor poses a risk for PPH. Magnesium acts as a smooth muscle relaxant, thereby contributing to uterine relaxation and atony.)

What woman is at greatest risk for early postpartum hemorrhage? A. A primiparous woman (G 2 P 1 0 0 1) being prepared for an emergency cesarean birth for fetal distress B. A woman with severe preeclampsia on magnesium sulfate whose labor is being induced C. A multiparous woman (G 3 P 2 0 0 2) with an 8-hour labor D. A primigravida in spontaneous labor with preterm twins

D (Hemorrhage may result in hemorrhagic shock. Shock is an emergency situation in which the perfusion of body organs may become severely compromised, and death may occur. The presence of adequate urinary output indicates adequate tissue perfusion.)

When caring for a postpartum woman experiencing hemorrhagic shock, the nurse recognizes that the most objective and least invasive assessment of adequate organ perfusion and oxygenation is: A. Absence of cyanosis in the buccal mucosa B. Cool, dry skin C. Diminished restlessness D. Urinary output of at least 30 ml/hr

D (Periodic walking helps prevent thrombophlebitis. Pregnant women should avoid sitting or standing for long periods and crossing the legs at the knees. Pregnant women must wear lap belts and shoulder restraints. The most common injury to the fetus comes from injury to the mother. Metal detectors at airport security checkpoints do not harm fetuses.)

When discussing work and travel during pregnancy with a pregnant patient, nurses should instruct them that: A. Women should sit for as long as possible and cross their legs at the knees from time to time for exercise. B. Women should avoid seat belts and shoulder restraints in the car because they press on the fetus. C. Metal detectors at airport security checkpoints can harm the fetus if the woman passes through them a number of times. D. While working or traveling in a car or on a plane, women should arrange to walk around at least every hour or so

B, C, E (Suggestions for coping with postpartum blues include: (1) Remember that the "blues" are normal and that both the mother and the father or partner may experience them. (2) Get plenty of rest; nap when the baby does if possible. Go to bed early, and let friends and family know when to visit and how they can help. (Remember, you are not "Supermom.") (3) Use relaxation techniques learned in childbirth classes (or ask the nurse to teach you and your partner some techniques). (4) Do something for yourself. Take advantage of the time your partner or family members care for the baby—soak in the tub (a 20-minute soak can be the equivalent of a 2-hour nap), or go for a walk. (5) Plan a day out of the house—go to the mall with the baby, being sure to take a stroller or carriage, or go out to eat with friends without the baby. Many communities have churches or other agencies that provide child care programs such as Mothers' Morning Out. (6) Talk to your partner about the way you feel—for example, about feeling tied down, how the birth met your expectations, and things that will help you (do not be afraid to ask for specifics). (7) If you are breastfeeding, give yourself and your baby time to learn. (8) Seek out and use community resources such as La Leche League or community mental health centers.)

When helping a woman cope with postpartum blues, what suggestions should the nurse offer? Select all that apply. A. Have the father take over care of the baby, because postpartum blues are exclusively a female problem. B. Get plenty of rest. C. Plan to get out of the house occasionally. D. Do not ask for help because this will not foster independence. e. Use La Leche League or community mental health centers.

C (Vacillating between the desire to have her own nurturing needs met and the need to take charge of her own care and that of her newborn is characteristic of the taking-in stage, which lasts for the first few days after birth. Expressing a strong need to review events and her behavior during the process of labor and birth is characteristic of the taking-in stage, which lasts for the first few days after birth. One week after birth, the woman should exhibit behaviors characteristic of the taking-hold stage. This stage lasts for as long as 4 to 5 weeks after birth. Reestablishing her role as a spouse/partner reflects the letting-go stage, which indicates that psychosocial recovery is complete. Test-Taking Tip: Read the question carefully before looking at the answers: (1) Determine what the question is really asking; look for key words; (2) Read each answer thoroughly and see if it completely covers the material asked by the question; (3) Narrow the choices by immediately eliminating answers you know are incorrect.)

When making a visit to the home of a postpartum woman 1 week after birth, the nurse should recognize that characteristically the woman would: A. Express a strong need to review events and her behavior during the process of labor and birth. B. Exhibit a reduced attention span, limiting readiness to learn. C. Facilitate between the desire to have her own nurturing needs met and the need to take charge of her own care and that of her newborn. D. Have reestablished her role as a spouse/partner.

C (No movement in a 12-hour period is cause for investigation and possibly intervention. Alcohol and cigarette smoke temporarily reduce fetal movement. The mother should count fetal activity ("kick counts") two or three times daily for 60 minutes each time. Obese women have a harder time assessing fetal movement.)

When nurses help their expectant mothers assess the daily fetal movement counts, they should be aware that: A. Alcohol or cigarette smoke can irritate the fetus into greater activity. B. "Kick counts" should be taken every half hour and averaged every 6 hours, with every other 6-hour stretch off. C. The fetal alarm signal should go off when fetal movements stop entirely for 12 hours. D. Obese mothers familiar with their bodies can assess fetal movement as well as average-size women.

B (Visually impaired mothers cannot overcome the infant's need for eye-to-eye contact)

When working with parents who have some form of sensory impairment, nurses should realize that all of these statements are true except: a) One of the major difficulties visually impaired parents experience is the skepticism of health care professionals b) Visually impaired mothers cannot overcome the infant's need for eye-to-eye contact c) The best approach for the nurse is to assess the parents' capabilities rather than focusing on their disabilities d) Technologic advances, including the Internet, can provide deaf parents with a full range of parenting activities and information

A, B, C, D (The skepticism, open or hidden, of health care professionals presents an additional and unneeded hurdle for the parents. After the parents' capabilities have been assessed (including some the nurse may not have expected), the nurse can help find ways to assist the parents that focus on their strengths. The Internet affords an extra teaching tool for the deaf, as do videos with subtitles or nurses signing. A number of electronic devices can turn sound into light flashes to help pick up a child's cry. Sign language is acquired readily by young children. Childbirth education and other materials are available in Braille. Other sensory output can be provided by the parent, other people can participate, and other coping devices can be used.)

When working with parents who have some form of sensory impairment, what information should nurses consider when writing a plan of care? Select all that apply. A. One of the major difficulties visually impaired parents experience is the skepticism of health care professionals. B. The best approach for the nurse is to assess the parents' capabilities rather than focusing on their disabilities. C. Technologic advances, including the Internet, can provide deaf parents with a full range of parenting activities and information D. Childbirth education and other materials are available in Braille.

A (The standard days variation on the calendar method has a failure rate of 12%. The periodic abstinence method has a failure rate of 25% or higher. The postovulation method has a failure rate of 25% or higher. The coitus interruptus method has a failure rate of 27% or higher.)

Which contraceptive method has a failure rate of less than 25%? a. Standard days b. Periodic abstinence c. Postovulation d. Coitus interruptus

B, C (Jordanian mothers have a 40-day lying-in after birth, during which their mothers or sisters care for the baby. Japanese mothers rest for the first 2 months after childbirth. Asian mothers must remain at home with the baby up to 30 days after birth and are not supposed to engage in household chores, including care of the baby. Hispanic practice involves many food restrictions after childbirth, such as avoiding fish, pork, and citrus foods. Vietnamese mothers may give minimal care to their babies and refuse to cuddle or further interact with the baby to ward off "evil" spirits.)

Which culturally appropriate beliefs should the maternity nurse use to incorporate parental-infant attachment into the plan of care? Select all that apply. A. Asian mothers are encouraged to return to work as soon as possible. B. Jordanian mothers have a 40-day lying-in after birth. C. Japanese mothers rest for the first 2 months after childbirth. D. Encourage Hispanics to eat plenty of fish and pork to increase vitamin intake. E. Encourage Vietnamese mothers to cuddle with the newborn.

B (Vaginal rugae reappear by 3 weeks postpartum; however, they are never as prominent as in nulliparous women. The cervix regains its form within days; the cervical os may take longer. Most episiotomies take 2 to 3 weeks to heal. Hemorrhoids can take 6 weeks to decrease in size.)

Which description of postpartum restoration or healing times is accurate? A. The cervix shortens, becomes firm, and returns to form within a month postpartum. B. Vaginal rugae reappear by 3 weeks postpartum. C. Most episiotomies heal within a week. D. Hemorrhoids usually decrease in size within 2 weeks of childbirth.

C (The fundus descends 1 cm/day, so by postpartum day 14 it is no longer palpable. The lochia should be changed by this day to serosa. Breasts are not part of the involution process. The episiotomy should not be red or puffy at this stage.)

Which documentation on a woman's chart on postpartum day 14 indicates a normal involution process? A. Moderate bright red lochial flow B. Breasts firm and tender C. Fundus below the symphysis and not palpable D. Episiotomy slightly red and puffy

A (The fundus rises to the umbilicus after delivery and remains there for about 24 hours. A fundus that is above the umbilicus may indicate uterine atony or urinary retention. A fundus that is palpable at or below the level of the umbilicus is a normal finding for a patient who is 12 hours postpartum. Palpation of the fundus 2 fingerbreadths below the umbilicus is an unusual finding for 12 hours postpartum; however, it is still appropriate.)

Which finding 12 hours after birth requires further assessment? A. The fundus is palpable two fingerbreadths above the umbilicus. B. The fundus is palpable at the level of the umbilicus. C. The fundus is palpable one fingerbreadth below the umbilicus. D. The fundus is palpable two fingerbreadths below the umbilicus.

C (Prolactin levels in the blood increase progressively throughout pregnancy. In women who breastfeed, prolactin levels remain elevated into the sixth week after birth. Estrogen and progesterone levels decrease markedly after expulsion of the placenta and reach their lowest levels 1 week into the postpartum period. Human placental lactogen levels decrease dramatically after expulsion of the placenta.)

Which hormone remains elevated in the immediate postpartum period of the breastfeeding woman? A. Estrogen B. Progesterone C. Prolactin D. Human placental lactogen

D (For the first 3 days after childbirth, lochia is termed rubra. Lochia serosa follows, and then at about 11 days, the discharge becomes clear, colorless, or white. Diuresis and diaphoresis are the methods by which the body rids itself of increased plasma volume. Urine output of 3000 mL/day is common for the first few days after delivery and is facilitated by hormonal changes in the mother. Bowel tone remains sluggish for days. Many women anticipate pain during defecation and are unwilling to exert pressure on the perineum. The new mother is hungry because of energy used in labor and thirsty because of fluid restrictions during labor.)

Which maternal event is abnormal in the early postpartum period? A. Diuresis and diaphoresis B. Flatulence and constipation C. Extreme hunger and thirst D. Lochial color changes from rubra to alba

A (Early and frequent feedings prevent stasis of milk, which contributes to engorgement and mastitis.)

Which measure may prevent mastitis in the breastfeeding mother? a. Initiating early and frequent feedings b. Nursing the infant for 5 minutes on each breast c. Wearing a tight-fitting bra d. Applying ice packs before feeding

A (The Babinski reflex causes the toes to flare outward and the big toe to dorsiflex. The tonic neck reflex (also called the fencing reflex) refers to the posture assumed by newborns when in a supine position. The stepping reflex occurs when infants are held upright with their heel touching a solid surface and the infant appears to be walking. Plantar grasp reflex is similar to the palmar grasp reflex: when the area below the toes is touched, the infant's toes curl over the nurse's finger.)

Which newborn reflex is elicited by stroking the lateral sole of the infant's foot from the heel to the ball of the foot? a. Babinski b. Tonic neck c. Stepping d. Plantar grasp

B (When the uterus is still in the pelvis, visualization may be difficult. It is necessary to perform the test when the woman has a full bladder, which provides a "window" through which the uterus and its contents can be viewed. The woman needs a full bladder to elevate the uterus; therefore being NPO is not appropriate. Neither an enema nor an abdominal preparation is necessary for this procedure.)

Which nursing intervention is necessary before a second-trimester transabdominal ultrasound? A. Place the woman NPO for 12 hours. B. Instruct the woman to drink 1 to 2 quarts of water. C. Administer an enema. D. Perform an abdominal preparation.

C (Leg exercises promote venous blood flow and prevent venous stasis while the client is still on bed rest.)

Which nursing measure would be appropriate to prevent thrombophlebitis in the recovery period after a cesarean birth? a. Roll a bath blanket and place it firmly behind the knees. b. Limit oral intake of fluids for the first 24 hours. c. Assist the client in performing leg exercises every 2 hours. d. Ambulate the client as soon as her vital signs are stable.

D (Medical abortions are performed through the use of medications (rather than surgical procedures). They are mostly done in the first trimester, and they can be either elective (the woman's choice) or therapeutic (for reasons of maternal or fetal health).)

Which statement is the most complete and accurate description of medical abortions? a. They are performed only for maternal health. b. They can be achieved through surgical procedures or with drugs. c. They are mostly performed in the second trimester. d. They can be either elective or therapeutic.

D (Breastfeeding is associated with an increased incidence of jaundice. Neonatal jaundice occurs in 60% of newborns; the complication called kernicterus is rare. Jaundice in the first 24 hours or that persists past day 7 is cause for medical concern. Parents need to know how to assess for jaundice in their newborn.)

Which statement describing physiologic jaundice is incorrect? a. Neonatal jaundice is common, but kernicterus is rare. b. The appearance of jaundice during the first 24 hours or beyond day 7 indicates a pathologic process. c. Because jaundice may not appear before discharge, parents need instruction on how to assess it and when to call for medical help. d. Breastfed babies have a lower incidence of jaundice.

A, B, C (Assessment of physiologic parameters such as AFV, FBMs, and limb and head movements of the fetus by ultrasonography gives a reliable picture of fetal well-being. Abnormalities in the amniotic fluid volume are frequently associated with fetal disorders. Fetal breathing and limb and head movements reflect the status of the central nervous system. Daily fetal movement count is the most common method used to assess fetal activity. Ultrasound is not used to assess the daily fetal movement count. The fluid volume in the nape of the fetal neck is measured to assess structural abnormalities in the fetus.)

Which physiologic parameters does the nurse check in the ultrasound report to assess fetal well-being? Select all that apply. A. Amniotic fluid volume (AFV) B. Fetal breathing movements (FBMs) C. Fetal limb and head movements D. Daily count of fetal movements E. Fluid volume in the nape of the fetal neck

D (The first phase is an active phase in which the baby is alert. Decreased activity and sleep mark the second phase. The first phase is the shortest, lasting less than 30 minutes. Such exploratory behaviors include spontaneous startle reactions. In the first phase the newborn also produces saliva.)

Which statement describing the first phase of the transition period is inaccurate? a. It lasts no longer than 30 minutes. b. It is marked by spontaneous tremors, crying, and head movements. c. It includes the passage of meconium. d. It may involve the infant's suddenly sleeping briefly.

C (A high pulse rate of 129 beats per minute in a postpartum patient immediately after childbirth may be indicative of hypovolemia caused by blood loss during labor. This is an abnormal assessment finding postdelivery. Labor may cause dehydration, and this may result in a slight increase in body temperature of up to 100.4° F. This is a normal finding associated with labor. Blood pressure may be slightly altered after childbirth. A blood pressure of 126/80 mm Hg would be a normal finding in this patient. The respiratory rate increases during labor and then slowly comes back to normal after labor. Normal respiratory rate is 12 to 14 breaths per minute. Thus, 15 breaths per minute is a normal finding.)

Which postpartum patient finding would the nurse consider abnormal when assessing the patient's vital signs immediately after childbirth? A. Temperature 100.4° F B. Blood pressure 126/80 mm Hg C. Pulse rate 129 beats per minute D. Respiratory rate 15 breaths per minute

B (Twin pregnancies often end in prematurity. Serious efforts should be made to bring the pregnancy to term. A woman with a multifetal pregnancy often develops anemia, suffers more or worse backache, and needs to gain more weight. Counseling is needed to help her adjust to these conditions.)

Which statement about multifetal pregnancy is inaccurate? A. The expectant mother often develops anemia because the fetuses have a greater demand for iron. B. Twin pregnancies come to term with the same frequency as single pregnancies. C. The mother should be counseled to increase her nutritional intake and gain more weight. D. Backache and varicose veins often are more pronounced.

A (A lunar month lasts 28 days, or 4 weeks. Pregnancy spans 9 calendar months but 10 lunar months. A trimester is one third of a normal pregnancy, or about 13 to 14 weeks. The prenatal period covers the full course of pregnancy (prenatal means before birth). The EDC is now called the EDB, or estimated date of birth. It has nothing to do with the duration of bed rest.)

Which statement about pregnancy is accurate? A. A normal pregnancy lasts about 10 lunar months. B. A trimester is one third of a year. C. The prenatal period extends from fertilization to conception. D. The estimated date of confinement (EDC) is how long the mother will have to be bedridden after birth.

D (Contraceptive effectiveness varies from couple to couple, depending on how well a contraceptive method is used and how well it suits the couple. The contraceptive failure rate measures the likelihood of accidental pregnancy in the first year only. Failure rates decline over time because users gain experience.)

Which statement is true about the term contraceptive failure rate? a. It refers to the percentage of users expected to have an accidental pregnancy over a 5-year span. b. It refers to the minimum level that must be achieved to receive a government license. c. It increases over time as couples become more careless. d. It varies from couple to couple, depending on the method and the users.

A, C, D (A woman with a multifetal pregnancy often develops anemia due to the increased demands of two fetuses. This should be monitored closely throughout her pregnancy. Twin pregnancies often end in prematurity. Serious efforts should be made to bring the pregnancy to term. The client may need nutrition counseling to ensure that she gains more weight than what is needed for a singleton birth. The considerable uterine distention is likely to cause backache and leg varicosities. Maternal support hose should be recommended. Spontaneous rupture of membranes before term is common.)

Which statements about multifetal pregnancy are most appropriate? (Select all that apply.) A. The expectant mother often develops anemia because the fetuses have a greater demand for iron. B. Twin pregnancies come to term with the same frequency as single pregnancies. C. The mother should be counseled to increase her nutritional intake and gain more weight. D. Backache and varicose veins are often more pronounced. E. Spontaneous rupture of membranes before term is uncommon.

D (Signs and symptoms that must be reported include severe vomiting, fever and chills, burning on urination, diarrhea, abdominal cramping, and vaginal bleeding. These symptoms may be signs of potential complications of the pregnancy. Nausea with occasional vomiting, fatigue, and urinary frequency are normal first-trimester complaints. Although they may be worrisome or annoying to the mother, they usually are not indications of pregnancy problems.)

Which symptom is considered a first-trimester warning sign and should be reported immediately by the pregnant woman to her health care provider? A. Nausea with occasional vomiting B. Urinary frequency C. Fatigue D. Vaginal bleeding

D (A temperature elevation to greater than 100.4° F on 2 postpartum days not including the first 24 hours indicates infection.)

Which temperature indicates the presence of postpartum infection? a. 99.6° F in the first 48 hours b. 100° F for 2 days postpartum c. 100.4° F in the first 24 hours d. 100.8° F on the second and third postpartum days

C (The NT ultrasound screening technique is used to measure fluid in the nape of the fetal neck between 10 and 14 weeks' gestation. Fluid volume greater than 3 mm is considered abnormal. NT is used mostly to identify possible fetal genetic abnormalities. AFV, fetal body movements, and fetal heart activity are measured to assess fetal well-being.)

Which test does the nurse recommend for the patient to help assess fetal genetic abnormalities? A. Amniotic fluid volume (AFV) B. Fetal body movements C. Nuchal translucency (NT) D. Fetal heart activity

B (Endometrial biopsy is scheduled after ovulation, during the luteal phase of the menstrual cycle. A hysterosalpingogram is scheduled 2 to 5 days after menstruation to avoid flushing potentially fertilized ovum out through a uterine tube into the peritoneal cavity. Laparoscopy usually is scheduled early in the menstrual cycle. Hormone analysis is performed to assess endocrine function of the hypothalamic-pituitary-ovarian axis when menstrual cycles are absent or irregular.)

Which test used to diagnose the basis of infertility is done during the luteal or secretory phase of the menstrual cycle? a. Hysterosalpingogram b. Endometrial biopsy c. Laparoscopy d. Follicle-stimulating hormone (FSH) level

B (Afterpains are more common in multiparous women. Afterpains are more noticeable with births in which the uterus was greatly distended, as in a woman who experienced polyhydramnios or a woman who delivered a large infant. Breastfeeding may cause afterpains to intensify.)

Which woman is most likely to experience strong afterpains? A. A woman who experienced oligohydramnios B. A woman who is a gravida 4, para 4-0-0-4 C. A woman who is bottle-feeding her infant D. A woman whose infant weighed 5 pounds, 3 ounces

C (Erythema toxicum (or erythema neonatorum) is a newborn rash that resembles flea bites. This is a normal finding that does not require notification of the physician, isolation of the newborn, or any additional interventions.)

While assessing the integument of a 24-hour-old newborn, the nurse notes a pink, papular rash with vesicles superimposed on the thorax, back, and abdomen. The nurse should: a. Notify the physician immediately. b. Move the newborn to an isolation nursery. c. Document the finding as erythema toxicum. d. Take the newborn's temperature and obtain a culture of one of the vesicles.

C (The average infant heart rate while awake is 120 to 160 beats/min. The newborn's heart rate may be about 85 to 100 beats/min while sleeping. The infant's heart rate typically is a bit higher when alert but quiet. A heart rate of 150 to 180 beats/min is typical when the infant cries.)

While assessing the newborn, the nurse should be aware that the average expected apical pulse range of a full-term, quiet, alert newborn is: a. 80 to 100 beats/min. b. 100 to 120 beats/min. c. 120 to 160 beats/min. d. 150 to 180 beats/min.

D (According to traditional Hispanic customs, fathers do not see their wives or infants until they are cleaned and dressed. It is believed that this custom incorporates the individuals into the family and integrates the family. It does not imply that the father is too scared to hold the baby. In this situation, the father is following the traditional Hispanic custom by not seeing the baby after birth. It does not implicate that the father has no time to see the baby. The father would want to see the baby as soon as it is cleaned and dressed.)

While caring for a Hispanic patient, the nurse asks the infant's father to hold the baby after its birth. The father is unwilling to see the baby. What reason would the nurse attribute to this unwillingness? The father: A. Is too scared to hold the baby. B. Has no time to visit the baby. C. Is not following the traditional Hispanic custom. D. Is doing it to integrate the individuals into the family

C (Algerian mothers tightly wrap the infant in swaddling clothes to protect them physically and physiologically, a custom followed by Algerians as a part of the acquaintance process. Algerians follow this custom to give physical and psychological protection but not to protect the infant from evil spirits. Vietnamese mothers interact minimally with the infants to protect them from evil spirits. The mother has to accept the assistance from others, as she alone cannot take care of the baby. Therefore she cannot avoid others touching the infant. Asian and Jordanian mothers hand over the baby to the grandparents immediately after birth. They do this to get some rest after childbirth.)

While caring for an Algerian patient, the nurse suggests that the patient give kangaroo care to the infant. The mother is unwilling to follow the suggestion of the nurse. What is the reason for this unwillingness? The patient is attempting to: A. Protect the infant from evil spirits. B. Prevent others from touching the baby. C. Protect the infant physically and psychologically. D. Take adequate rest after the birth of the child.

D (The characteristics displayed by the infant are associated with a positive Moro reflex. The tonic neck reflex occurs when the infant extends the leg on the side to which the infant's head simultaneously turns. The glabellar reflex is elicited by tapping on the infant's head while the eyes are open. A characteristic response is blinking for the first few taps. The Babinski reflex occurs when the sole of the foot is stroked upward along the lateral aspect of the sole and then across the ball of the foot. A positive response occurs when all the toes hyperextend, with dorsiflexion of the big toe.)

While evaluating the reflexes of a newborn, the nurse notes that with a loud noise the newborn symmetrically abducts and extends his arms, his fingers fan out and form a "C" with the thumb and forefinger, and he has a slight tremor. The nurse would document this finding as a positive: a. Tonic neck reflex. b. Glabellar (Myerson) reflex. c.Babinski reflex. d. Moro reflex.

C (The Ortolani maneuver is used to detect the presence of hip dysplasia. Polydactyly is the presence of extra digits. Clubfoot (talipes equinovarus) is a deformity in which the foot turns inward and is fixed in a plantar-flexion position. Webbing, or syndactyly, is a fusing of the fingers or toes.)

While examining a newborn, the nurse notes uneven skin folds on the buttocks and a click when performing the Ortolani maneuver. The nurse recognizes these findings as a sign that the newborn probably has: a. Polydactyly. b. Clubfoot. c. Hip dysplasia. d. Webbing

C (Natural family planning is the only contraceptive practice acceptable to the Roman Catholic church. "Pulling out" is not the same as periodic abstinence, another name for natural family planning. The phases of the moon are not part of the calendar method or any method. Natural family planning is another name for periodic abstinence, which is the accepted way to pass safely through the fertility phases without relying on chemical or physical barriers.)

While instructing a couple regarding birth control, the nurse should be aware that the method called natural family planning: a. Is the same as coitus interruptus, or "pulling out." b. Uses the calendar method to align the woman's cycle with the natural phases of the moon. c. Is the only contraceptive practice acceptable to the Roman Catholic church. d. Relies on barrier methods during fertility phases.

B (Lack of response after 3 minutes of FAST indicates that the fetus has low activity levels. In this situation, to accurately assess fetal activity, the nurse should recommend a BPP of the fetus. Amniocentesis helps detect genetic abnormalities in the fetus. Fetal activity cannot be determined using this technique. In cordocentesis, the umbilical blood is tested for Rh incompatibility and hemolytic anemia in the fetus. Coombs' test is used to determine the presence of antibody incompatibilities in the fetus and the mother.)

While performing the fetal acoustic stimulation test (FAST) in a patient, the nurse observes that there is no fetal response even after 3 minutes of testing. Which test does the nurse suggest? A. Amniocentesis B. Biophysical profile (BPP) C. Cordocentesis D. Coombs' test

C, D (A floating fetus is seen in cases of elevated amniotic fluid volume, or polyhydramnios. Polyhydramnios is associated with neural tube defects and gastrointestinal obstruction. Renal agenesis and severe intrauterine growth restriction are associated with oligohydramnios, or low amniotic fluid volume. A low amount of fluid may not result in a floating fetus in the scanned image. The amniotic fluid level is unrelated to cardiac disease in the fetus.)

While reviewing the ultrasound reports of a patient, the nurse notices a floating fetus in the scanned image. What potential fetal risks should the nurse interpret from this finding? Select all that apply: A. Renal agenesis B. Growth restriction C. Neural tube defects D. Gastrointestinal obstruction E. Cardiac disease.

B (Postpartum women spend most of their time caring for the infant, especially those mothers who breastfeed and may not easily be able to share child care duties with their partners. As a result, the infant's father may feel depressed, bored, and jealous of the infant. The infant's father should express these feelings so that the mother can make more time for her partner. It is not appropriate to tell the father to spend more time with his friends, as this does not solve the problem in the home. Even if the father comes home early from work, the mother may not be able to spend time with the father, so this suggestion is not necessarily helpful. Engaging in other activities may relieve the infant's father from boredom but not from depression and jealousy. The father would get relieved of these feelings by having the mother's attention and care.)

While speaking to the newborn's father in a well child visit, the nurse finds out that the newborn's father is feeling depressed and bored, as the mother spends most of her time with the infant. What instruction should the nurse give to the father in this situation? A. "Spend more time with your friends." B. "Share your feelings with your partner." C. "Work fewer hours and come home early." D. "Try to distract yourself with other activities."

A (The main danger from taking baths is falling in the tub. The perineum should be wiped from front to back. Bubble baths and bath oils should be avoided because they may irritate the urethra. Soap, alcohol, ointments, and tinctures should not be used to cleanse the nipples because they remove protective oils. Warm water is sufficient.)

While teaching the expectant mother about personal hygiene during pregnancy, maternity nurses should be aware that: A. Tub bathing is permitted even in late pregnancy unless membranes have ruptured. B. The perineum should be wiped from back to front. C. Bubble bath and bath oils are permissible because they add an extra soothing and cleansing action to the bath. D. Expectant mothers should use specially treated soap to cleanse the nipples.

C (Patting or gently rubbing the infant's back provides comfort to the infant after feeding. Gentle stroking on the infant's back helps to soothe and quiet the infant. Making eye contact with the infant helps to develop a bond and enhances the trust between the mother and infant. Touching the infant with the mother's fingertips and caressing the infant's trunk help to develop attachment with the infant.)

While teaching the patient about breastfeeding, the nurse instructs the patient to rub the infant's back after feeding. What is the reason for this instruction? A. To develop trust in the infant B. To quiet and soothe the infant C. To provide comfort to the infant D. To promote attachment with the infant

C (CVS can be performed in the first or second trimester, ideally between 10 and 13 weeks of gestation. During this procedure, a small piece of tissue is removed from the fetal portion of the placenta. If performed after 9 completed weeks of gestation, the risk of limb reduction is no greater than in the general population.)

While working with the pregnant woman in her first trimester, the nurse is aware that chorionic villus sampling (CVS) can be performed during pregnancy at: A. 4 weeks B. 8 weeks C. 10 weeks D. 14 weeks

D (Blood pressure is affected by maternal position during pregnancy. The supine position may cause occlusion of the vena cava and descending aorta. Turning the pregnant woman to a lateral recumbent position alleviates pressure on the blood vessels and quickly corrects supine hypotension. Pressures are significantly higher when the patient is standing. This option causes an increase in systolic and diastolic pressures. The arm should be supported at the same level of the heart. The supine position may cause occlusion of the vena cava and descending aorta, creating hypotension.)

While you are assessing the vital signs of a pregnant woman in her third trimester, the patient complains of feeling faint, dizzy, and agitated. Which nursing intervention is appropriate? A. Have the patient stand up and retake her blood pressure. B. Have the patient sit down and hold her arm in a dependent position. C. Have the patient lie supine for 5 minutes and recheck her blood pressure on both arms. D. Have the patient turn to her left side and recheck her blood pressure in 5 minutes.

C (Love and support help a woman feel better about her pregnancy. The most important person to the pregnant woman is usually the father. Nurses can facilitate communication between partners about sexual matters if, as is common, they are nervous about expressing their worries and feelings. The second trimester is the time when a woman's sense of well-being, along with certain physical changes, increases her desire for sex. Desire is decreased in the first and third trimesters.)

With regard to a woman's reordering of personal relationships during pregnancy, the maternity nurse should understand that: A. Because of the special motherhood bond, a woman's relationship with her mother is even more important than with the father of the child. B. Nurses need not get involved in any sexual issues the couple has during pregnancy, particularly if they have trouble communicating them to each other. C. Women usually express two major relationship needs during pregnancy: feeling loved and valued and having the child accepted by the father. D. The woman's sexual desire is likely to be highest in the first trimester because of the excitement and because intercourse is physically easier.

C (A large baby or multiple babies overdistend the uterus. The cramping that causes afterbirth pains arises from periodic, vigorous contractions and relaxations, which persist through the first part of the postpartum period. Afterbirth pains are more common in multiparous women because first-time mothers have better uterine tone. Breastfeeding intensifies afterbirth pain because it stimulates contractions.)

With regard to afterbirth pains, nurses should be aware that these pains are: A. Caused by mild, continuous contractions for the duration of the postpartum period. B. More common in first-time mothers. C. More noticeable in births in which the uterus was overdistended. D. Alleviated somewhat when the mother breastfeeds.

C (Diluted fluid is mixed with ethanol and shaken. After 15 minutes, the bubbles tell the story. Amniocentesis is possible after the fourteenth week of pregnancy when the uterus becomes an abdominal organ. Complications occur in less than 1% of cases; many have been minimized or eliminated through the use of ultrasound. Meconium in the amniotic fluid before the beginning of labor is not usually a problem.)

With regard to amniocentesis, nurses should be aware that: A. Because of new imaging techniques, amniocentesis is now possible in the first trimester. B. Despite the use of ultrasound, complications still occur in the mother or infant in 5% to 10% of cases. C. The shake test, or bubble stability test, is a quick means of determining fetal maturity. D. The presence of meconium in the amniotic fluid is always cause for concern.

C (The woman lies on her back during the abdominal examination, possibly compressing the vena cava and aorta, which can cause a decrease in blood pressure and a feeling of faintness. The interview portion of follow-up examinations is less extensive than in the initial prenatal visits, during which so much new information must be gathered. Monthly visits are routinely scheduled for the first and second trimesters; visits increase to every 2 weeks at week 28 and to once a week at week 36. For pregnant women hypertension is defined as a systolic BP of 140 or greater and a diastolic BP of 90 or greater.)

With regard to follow-up visits for women receiving prenatal care, nurses should be aware that: A. The interview portions become more intensive as the visits become more frequent over the course of the pregnancy. B. Monthly visits are scheduled for the first trimester, every 2 weeks for the second trimester, and weekly for the third trimester. C. During the abdominal examination, the nurse should be alert for supine hypotension. D. For pregnant women, a systolic blood pressure (BP) of 130 and a diastolic BP of 80 is sufficient to be considered hypertensive.

A (Both prescription and OTC drugs that otherwise are harmless can be made hazardous by metabolic deficiencies of the fetus. This is especially true for new medications and combinations of drugs. The greatest danger of drug-caused developmental defects exists in the interval from fertilization through the first trimester, when a woman may not realize that she is pregnant. Live-virus vaccines should be part of postpartum care; killed-virus vaccines may be administered during pregnancy. Secondhand smoke is associated with fetal growth restriction and increases in infant mortality.)

With regard to medications, herbs, shots, and other substances normally encountered by pregnant women, the maternity nurse should be aware that: A. Both prescription and over-the-counter (OTC) drugs that otherwise are harmless can be made hazardous by metabolic deficiencies of the fetus. B. The greatest danger of drug-caused developmental deficits in the fetus is seen in the final trimester. C. Killed-virus vaccines (e.g., tetanus) should not be given during pregnancy, but live-virus vaccines (e.g., measles) are permissible. D. No convincing evidence exists that secondhand smoke is potentially dangerous to the fetus.

A (This is especially true for new medications and combinations of drugs. The greatest danger of drug-caused developmental defects exists in the interval from fertilization through the first trimester, when a woman may not realize that she is pregnant. Live-virus vaccines should be part of postpartum care; killed-virus vaccines may be administered during pregnancy. Secondhand smoke is associated with fetal growth restriction and increases in infant mortality.)

With regard to medications, herbs, shots, and other substances normally encountered, the maternity nurse should be aware that: A. Prescription and over-the-counter (OTC) drugs that otherwise are harmless can be made hazardous by metabolic deficiencies of the fetus. B. The greatest danger of drug-caused developmental deficits in the fetus is seen in the final trimester. C. Killed-virus vaccines (e.g., tetanus) should not be given during pregnancy, but live-virus vaccines (e.g., measles) are permissible. D. No convincing evidence exists that secondhand smoke is potentially dangerous to the fetus.

D (The first flow is heavier, but within three or four cycles, it is back to normal. Ovulation can occur within the first month, but for 70% of nonlactating women, it returns within 12 weeks after birth. Breastfeeding women take longer to resume ovulation. Because many women ovulate before their first postpartum menstrual period, contraceptive options need to be discussed early in the puerperium.)

With regard to postpartum ovarian function, nurses should be aware that: A. Almost 75% of women who do not breastfeed resume menstruating within a month after birth. B. Ovulation occurs slightly earlier for breastfeeding women. C. Because of menstruation/ovulation schedules, contraception considerations can be postponed until after the puerperium. D. The first menstrual flow after childbirth usually is heavier than normal.

B (Participation in preparation classes helps both siblings and grandparents.)

With regard to the adaptation of other family members, mainly siblings and grandparents, to the newborn, nurses should be aware that: A. Sibling rivalry cannot be dismissed as overblown psychobabble; negative feelings and behaviors can take a long time to blow over. B. Participation in preparation classes helps both siblings and grandparents. C. In the United States, paternal and maternal grandparents consider themselves of equal importance and status. D. Since 1990, the number of grandparents providing permanent care to their grandchildren has been declining.

B (Fertility assessment and diagnosis take time, money, and commitment from the couple. Religious, cultural, and ethnic-bred attitudes about fertility and related issues always have an impact on diagnosis and assessment. Both partners are assessed systematically and simultaneously, as individuals and as a couple. Semen analysis is for men, but the postcoital test is for the couple.)

With regard to the assessment of female, male, and couple infertility, nurses should be aware that: a. The couple's religious, cultural, and ethnic backgrounds provide emotional clutter that does not affect the clinical scientific diagnosis. b. The investigation takes 3 to 4 months and a lot of money. c. The woman is assessed first; if she is not the problem, the male partner is analyzed. d. Semen analysis is for men; the postcoital test is for women.

C (Fluid loss through perspiration and increased urinary output accoun for a weight loss of more than 2kg during the puerperium.)

With regard to the condition and reconditioning of the urinary system after childbirth, nurses should be aware that: a) Kidney function returns to normal a few days after birth b) Diastasis recti abdominis is a common condition that alters the voiding reflex c) Fluid loss through perspiration and increased urinary output accoun for a weight loss of more than 2kg during the puerperium d) With adequate emptying of the bladder, bladder tone usually is restored 2 to 3 weeks after childbirth

C (Avoiding overfeeding can also reduce regurgitation. The newborn's cheeks are full because of well-developed sucking pads. Teeth do develop in utero, but the nipple is placed deep because the baby cannot move food from the lips to the pharynx. Bacteria are not present at birth, but they soon enter through various orifices.)

With regard to the gastrointestinal (GI) system of the newborn, nurses should be aware that: a. The newborn's cheeks are full because of normal fluid retention. b. The nipple of the bottle or breast must be placed well inside the baby's mouth because teeth have been developing in utero, and one or more may even be through. c. Regurgitation during the first day or two can be reduced by burping the infant and slightly elevating the baby's head. d. Bacteria are already present in the infant's GI tract at birth because they traveled through the placenta.

B (An empty bladder facilitates the examination; this is also an opportunity to get a urine sample easily for a number of tests. All women should be assessed for a history of physical abuse, particularly because the likelihood of abuse increases during pregnancy. Noting body hair is important because body hair reflects nutritional status, endocrine function, and hygiene. Particular attention is paid to the size of the uterus because it is an indication of the duration of gestation.)

With regard to the initial physical examination of a woman beginning prenatal care, maternity nurses should be cognizant of: A. Only women who show physical signs or meet the sociologic profile should be assessed for physical abuse. B. The woman should empty her bladder before the pelvic examination is performed. C. The distribution, amount, and quality of body hair are of no particular importance. D. The size of the uterus is discounted in the initial examination.

C (Besides these potential problems, nurses need to be alert to the woman's attitude toward health care. The initial interview needs to be planned, purposeful, and focused on specific content. A lot of ground must be covered. Nurses must be sensitive to special problems, but they do need to inquire because discovering individual needs is important. People with chronic or handicapping conditions forget to mention them because they have adapted to them. Getting information on drug use is important and can be done confidentially. Actual testing for drug use requires the client's consent.)

With regard to the initial visit with a client who is beginning prenatal care, nurses should be aware that: A. The first interview is a relaxed, get-acquainted affair in which nurses gather some general impressions. B. If nurses observe handicapping conditions, they should be sensitive and not inquire about them because the client will do that in her own time. C. Nurses should be alert to the appearance of potential parenting problems, such as depression or lack of family support. D. Because of legal complications, nurses should not ask about illegal drug use; that is left to physicians

C (The newborn's thin chest wall often allows the PMI to be seen. The normal heart rate for infants who are not sleeping is 120 to 160 beats/min. However, a crying infant temporarily could have a heart rate of 180 beats/min. Heart murmurs during the first few days of life have no pathologic significance; an irregular heart rate past the first few hours should be evaluated further. Persistent tachycardia may indicate RDS; bradycardia may be a sign of congenital heart blockage.)

With regard to the newborn's developing cardiovascular system, nurses should be aware that: a. The heart rate of a crying infant may rise to 120 beats/min. b. Heart murmurs heard after the first few hours are cause for concern. c. The point of maximal impulse (PMI) often is visible on the chest wall. d. Persistent bradycardia may indicate respiratory distress syndrome (RDS).

D (The effectiveness of COCs can be altered by some over-the-counter medications and herbal supplements. Toxic shock syndrome can occur in some diaphragm users, but it is not a consequence of taking oral contraceptive pills. Hormonal withdrawal bleeding usually is lighter than in normal menstruation and lasts a couple of days. Oral contraceptive pills offer protection against the risk of endometrial and ovarian cancers.)

With regard to the noncontraceptive medical effects of combined oral contraceptive pills (COCs), nurses should be aware that: a. COCs can cause toxic shock syndrome if the prescription is wrong. b. Hormonal withdrawal bleeding usually is a bit more profuse than in normal menstruation and lasts a week. c. COCs increase the risk of endometrial and ovarian cancer. d. The effectiveness of COCs can be altered by some over-the-counter medications and herbal supplements.

B (Respirations should decrease to within the woman's normal prepregnancy range by 6 to 8 weeks after birth. Stroke volume increases, and cardiac output remains high for a couple of days. However, the heart rate and blood pressure return to normal quickly. Leukocytosis increases 10 to 12 days after childbirth and can obscure the diagnosis of acute infections (false-negative results). The hypercoagulable state increases the risk of thromboembolism, especially after a cesarean birth)

With regard to the postpartum changes and developments in a woman's cardiovascular system, nurses should be aware that: A. Cardiac output, the pulse rate, and stroke volume all return to prepregnancy normal values within a few hours of childbirth. B. Respiratory function returns to nonpregnant levels by 6 to 8 weeks after birth. C. The lowered white blood cell count after pregnancy can lead to false-positive results on tests for infections. D. A hypercoagulable state protects the new mother from thromboembolism, especially after a cesarean birth.

B (After 2 weeks postpartum, the uterus should not be palpable abdominally; however, it has not yet returned to its original size. At the end of the third stage of labor, the uterus weighs approximately 1000 g. It takes 6 full weeks for the uterus to return to its original size. After 2 weeks postpartum the uterus weighs about 350 g, not its original size. The normal self-destruction of excess hypertrophied tissue accounts for the slight increase in uterine size after each pregnancy.)

With regard to the postpartum uterus, nurses should be aware that: A. At the end of the third stage of labor it weighs approximately 500 g. B. After 2 weeks postpartum it should not be palpable abdominally. C. After 2 weeks postpartum it weighs 100 g. D. It returns to its original (prepregnancy) size by 6 weeks postpartum.

A (The first breath produces a cry. Newborns continue to expel fluid for the first hour of life. Newborns are natural nose breathers; they may not have the mouth-breathing response to nasal blockage for 3 weeks. Seesaw respirations instead of normal abdominal respirations are not normal and should be reported.)

With regard to the respiratory development of the newborn, nurses should be aware that: a. The first gasping breath is an exaggerated respiratory reaction within 1 minute of birth. b. Newborns must expel the fluid from the respiratory system within a few minutes of birth. c. Newborns are instinctive mouth breathers. d. Seesaw respirations are no cause for concern in the first hour after birth.

B (The woman has up to 8 days to insert the IUD after unprotected sex. Return to fertility is immediate after removal of the IUD. IUDs offer no protection for sexually transmitted infections. A consent form is required for insertion, as is a negative pregnancy test.)

With regard to the use of intrauterine devices (IUDs), nurses should be aware that: a. Return to fertility can take several weeks after the device is removed. b. IUDs containing copper can provide an emergency contraception option if inserted within a few days of unprotected intercourse. c. IUDs offer the same protection against sexually transmitted infections as the diaphragm. d. Consent forms are not needed for IUD insertion.

3. Which symptom described by a patient is characteristic of premenstrual syndrome (PMS)? a. "I feel irritable and moody a week before my period is supposed to start." b. "I have lower abdominal pain beginning the third day of my menstrual period." c. "I have nausea and headaches after my period starts, and they last 2 to 3 days." d. "I have abdominal bloating and breast pain after a couple days of my period."

a. "I feel irritable and moody a week before my period is supposed to start."

21. A pregnant woman's biophysical profile score is 8. She asks the nurse to explain the results. The nurse's best response is: a. "The test results are within normal limits." b. "Immediate delivery by cesarean birth is being considered." c. "Further testing will be performed to determine the meaning of this score." d. "An obstetric specialist will evaluate the results of this profile and, within the next week, will inform you of your options regarding delivery."

a. "The test results are within normal limits."

8. A client asks her nurse, "My doctor told me that he is concerned with the grade of my placenta because I am overdue. What does that mean?" The best response by the nurse is: a. "Your placenta changes as your pregnancy progresses, and it is given a score that indicates the amount of calcium deposits it has. The more calcium deposits, the higher the grade, or number, that is assigned to the placenta. It also means that less blood and oxygen can be delivered to your baby." b. "Your placenta isn't working properly, and your baby is in danger." c. "This means that we will need to perform an amniocentesis to detect if you have any placental damage." d. "Don't worry about it. Everything is fine."

a. "Your placenta changes as your pregnancy progresses, and it is given a score that indicates the amount of calcium deposits it has. The more calcium deposits, the higher the grade, or number, that is assigned to the placenta. It also means that less blood and oxygen can be delivered to your baby."

A key finding from the Human Genome Project is: a. Approximately 20,000 to 25,000 genes make up the genome. b. All human beings are 80.99% identical at the DNA level. c. Human genes produce only one protein per gene; other mammals produce three proteins per gene. d. Single gene testing will become a standardized test for all pregnant clients in the future.

a. Approximately 20,000 to 25,000 genes make up the genome. Approximately 20,000 to 25,000 genes make up the human genome; this is only twice as many as make up the genomes of roundworms and flies. Human beings are 99.9% identical at the DNA level. Most human genes produce at least three proteins. Single gene testing (e.g., alpha-fetoprotein) is already standardized for prenatal care.

10. After delivery, excess hypertrophied tissue in the uterus undergoes a period of self-destruction. What is the correct term for this process? a. Autolysis b. Subinvolution c. Afterpains d. Diastasis

a. Autolysis

23. On vaginal examination of a 30-year-old woman, the nurse documents the following findings: profuse, thin, grayish white vaginal discharge with a "fishy" odor; complaint of pruritus. On the basis of these findings, the nurse suspects that this woman has: a. Bacterial vaginosis (BV). b. Candidiasis. c. Trichomoniasis. d. Gonorrhea.

a. Bacterial vaginosis (BV)

3. The nurse sees a woman for the first time when she is 30 weeks pregnant. The woman has smoked throughout the pregnancy, and fundal height measurements now are suggestive of growth restriction in the fetus. In addition to ultrasound to measure fetal size, what other tool would be useful in confirming the diagnosis? a. Doppler blood flow analysis b. Contraction stress test (CST) c. Amniocentesis d. Daily fetal movement counts

a. Doppler blood flow analysis

4. A woman complains of severe abdominal and pelvic pain around the time of menstruation that has gotten worse over the last 5 years. She also complains of pain during intercourse and has tried unsuccessfully to get pregnant for the past 18 months. These symptoms are most likely related to: a. Endometriosis. b. PMS. c. Primary dysmenorrhea. d. Secondary dysmenorrhea.

a. Endometriosis.

The nurse caring for the laboring woman should know that meconium is produced by: a. Fetal intestines. b. Fetal kidneys. c. Amniotic fluid. d. The placenta.

a. Fetal intestines. As the fetus nears term, fetal waste products accumulate in the intestines as dark green-to-black, tarry meconium.

22. Which analysis of maternal serum may predict chromosomal abnormalities in the fetus? a. Multiple-marker screening b. Lecithin/sphingomyelin (L/S) ratio c. Biophysical profile d. Type and crossmatch of maternal and fetal serum

a. Multiple-marker screening

The nurse caring for a pregnant client knows that her health teaching regarding fetal circulation has been effective when the client reports that she has been sleeping: a. In a side-lying position. b. On her back with a pillow under her knees. c. With the head of the bed elevated. d. On her abdomen.

a. In a side-lying position. Optimal circulation is achieved when the woman is lying at rest on her side. Decreased uterine circulation may lead to intrauterine growth restriction. Previously it was believed that the left lateral position promoted maternal cardiac output, thereby enhancing blood flow to the fetus. However, it is now known that either side-lying position enhances uteroplacental blood flow. If a woman lies on her back with the pressure of the uterus compressing the vena cava, blood return to the right atrium will be diminished. Although this position is recommended and ideal for later in pregnancy, the woman must still maintain a lateral tilt to the pelvis to avoid compression of the vena cava. Many women will find this position uncomfortable as pregnancy advances. Side-lying is the ideal position to promote blood flow to the fetus.

13. Nurses should be aware that the biophysical profile (BPP): a. Is an accurate indicator of impending fetal well being. b. Is a compilation of health risk factors of the mother during the later stages of pregnancy. c. Consists of a Doppler blood flow analysis and an amniotic fluid index. d. Involves an invasive form of ultrasound examination.

a. Is an accurate indicator of impending fetal well being.

26. A first-time mother at 18 weeks of gestation comes for her regularly scheduled prenatal visit. The client tells the nurse that she is afraid that she is going into premature labor because she is beginning to have regular contractions. The nurse explains that this is the Braxton Hicks sign and teaches the client that this type of contraction: a. Is painless. b. Increases with walking. c. Causes cervical dilation. d. Impedes oxygen flow to the fetus.

a. Is painless.

14. As relates to dysfunctional uterine bleeding (DUB), the nurse should be aware that: a. It is most commonly caused by anovulation. b. It most often occurs in middle age. c. The diagnosis of DUB should be the first considered for abnormal menstrual bleeding. d. The most effective medical treatment is steroids.

a. It is most commonly caused by anovulation.

A maternity nurse should be aware of which fact about the amniotic fluid? a. It serves as a source of oral fluid and a repository for waste from the fetus. b. The volume remains about the same throughout the term of a healthy pregnancy. c. A volume of less than 300 ml is associated with gastrointestinal malformations. d. A volume of more than 2 L is associated with fetal renal abnormalities.

a. It serves as a source of oral fluid and a repository for waste from the fetus. Amniotic fluid serves as a source of oral fluid, a repository for waste from the fetus, and also cushions the fetus and helps maintain a constant body temperature. The volume of amniotic fluid changes constantly. Too little amniotic fluid (oligohydramnios) is associated with renal abnormalities. Too much amniotic fluid (hydramnios) is associated with gastrointestinal and other abnormalities.

Many parents-to-be have questions about multiple births. Maternity nurses should be able to tell them that: a. Twinning and other multiple births are increasing because of the use of fertility drugs and delayed childbearing. b. Dizygotic twins (two fertilized ova) have the potential to be conjoined twins. c. Identical twins are more common in Caucasian families. d. Fraternal twins are same gender, usually male.

a. Twinning and other multiple births are increasing because of the use of fertility drugs and delayed childbearing. If the parents-to-be are older and have taken fertility drugs, they would be very interested to know about twinning and other multiple births. Conjoined twins are monozygotic; they are from a single fertilized ovum in which division occurred very late. Identical twins show no racial or ethnic preference; fraternal twins are more common among African-American women. Fraternal twins can be different genders or the same gender. Identical twins are the same gender.

2. A 39-year-old primigravida thinks that she is about 8 weeks pregnant, although she has had irregular menstrual periods all her life. She has a history of smoking approximately one pack of cigarettes a day, but she tells you that she is trying to cut down. Her laboratory data are within normal limits. What diagnostic technique could be used with this pregnant woman at this time? a. Ultrasound examination b. Maternal serum alpha-fetoprotein (MSAFP) screening c. Amniocentesis d. Nonstress test (NST)

a. Ultrasound examination

27. A 25-year-old single woman comes to the gynecologist's office for a follow-up visit related to her abnormal Papanicolaou (Pap) smear. The test revealed that the patient has human papillomavirus (HPV). The client asks, "What is that? Can you get rid of it?" Your best response is: a. "It's just a little lump on your cervix. We can freeze it off." b. "HPV stands for 'human papillomavirus.' It is a sexually transmitted infection (STI) that may lead to cervical cancer." c. "HPV is a type of early human immunodeficiency virus (HIV). You will die from this." d. "You probably caught this from your current boyfriend. He should get tested for this."

b. "HPV stands for 'human papillomavirus.' It is a sexually transmitted infection (STI) that may lead to cervical cancer."

17. Which statement by a newly delivered woman indicates that she knows what to expect regarding her menstrual activity after childbirth? a. "My first menstrual cycle will be lighter than normal and then will get heavier every month thereafter." b. "My first menstrual cycle will be heavier than normal and will return to my prepregnant volume within three or four cycles." c. "I will not have a menstrual cycle for 6 months after childbirth." d. "My first menstrual cycle will be heavier than normal and then will be light for several months after."

b. "My first menstrual cycle will be heavier than normal and will return to my prepregnant volume within three or four cycles."

With regard to chromosome abnormalities, nurses should be aware that: a. They occur in approximately 10% of newborns. b. Abnormalities of number are the leading cause of pregnancy loss. c. Down syndrome is a result of an abnormal chromosome structure. d. Unbalanced translocation results in a mild abnormality that the child will outgrow.

b. Abnormalities of number are the leading cause of pregnancy loss. Aneuploidy is an abnormality of number that also is the leading genetic cause of mental retardation. Chromosome abnormalities occur in fewer than 1% of newborns. Down syndrome is the most common form of trisomal abnormality, an abnormality of chromosome number (47 chromosomes). Unbalanced translocation is an abnormality of chromosome structure that often has serious clinical effects.

11. Which statement regarding the postpartum uterus is correct? a. At the end of the third stage of labor, the postpartum uterus weighs approximately 500 g. b. After 2 weeks postpartum, it should be abdominally nonpalpable. c. After 2 weeks postpartum, it weighs 100 g. d. Postpartum uterus returns to its original (prepregnancy) size by 6 weeks postpartum.

b. After 2 weeks postpartum, it should be abdominally nonpalpable.

With regard to the structure and function of the placenta, the maternity nurse should be aware that: a. As the placenta widens, it gradually thins to allow easier passage of air and nutrients. b. As one of its early functions, the placenta acts as an endocrine gland. c. The placenta is able to keep out most potentially toxic substances such as cigarette smoke to which the mother is exposed. d. Optimal blood circulation is achieved through the placenta when the woman is lying on her back or standing.

b. As one of its early functions, the placenta acts as an endocrine gland. The placenta produces four hormones necessary to maintain the pregnancy. The placenta widens until week 20 and continues to grow thicker. Toxic substances such as nicotine and carbon monoxide readily cross the placenta into the fetus. Optimal circulation occurs when the woman is lying on her side.

22. Care management of a woman diagnosed with acute pelvic inflammatory disease (PID) most likely would include: a. Oral antiviral therapy. b. Bed rest in a semi-Fowler position. c. Antibiotic regimen continued until symptoms subside. d. Frequent pelvic examination to monitor the progress of healing.

b. Bed rest in a semi-Fowler position.

21. To reassure and educate pregnant clients about changes in their cardiovascular system, maternity nurses should be aware that: a. A pregnant woman experiencing disturbed cardiac rhythm, such as sinus arrhythmia requires close medical and obstetric observation, no matter how healthy she otherwise may appear. b. Changes in heart size and position and increases in blood volume create auditory changes from 20 weeks to term. c. Palpitations are twice as likely to occur in twin gestations. d. All of the above changes likely will occur.

b. Changes in heart size and position and increases in blood volume create auditory changes from 20 weeks to term.

In presenting to obstetric nurses interested in genetics, the genetic nurse identifies the primary risk(s) associated with genetic testing as: a. Anxiety and altered family relationships. b. Denial of insurance benefits. c. High false positives associated with genetic testing. d. Ethnic and socioeconomic disparity associated with genetic testing.

b. Denial of insurance benefits. Decisions about genetic testing are shaped by socioeconomic status and the ability to pay for the testing. Some types of genetic testing are expensive and are not covered by insurance benefits. Anxiety and altered family relationships, high false positives, and ethnic and socioeconomic disparity are factors that may be difficulties associated with genetic testing, but they are not risks associated with testing.

18. A woman has been diagnosed with a high risk pregnancy. She and her husband come into the office in a very anxious state. She seems to be coping by withdrawing from the discussion, showing declining interest. The nurse can best help the couple by: a. Telling her that the physician will isolate the problem with more tests. b. Encouraging her and urging her to continue with childbirth classes. c. Becoming assertive and laying out the decisions the couple needs to make. d. Downplaying her risks by citing success rate studies.

b. Encouraging her and urging her to continue with childbirth classes.

27. Which finding in the urine analysis of a pregnant woman is considered a variation of normal? a. Proteinuria b. Glycosuria c. Bacteria in the urine. d. Ketones in the urine.

b. Glycosuria

6. Cardiovascular system changes occur during pregnancy. Which finding would be considered normal for a woman in her second trimester? a. Less audible heart sounds (S1, S2) b. Increased pulse rate c. Increased blood pressure d. Decreased red blood cell (RBC) production

b. Increased pulse rate

24. Which nursing intervention is necessary before a second-trimester transabdominal ultrasound? a. Place the woman NPO for 12 hours. b. Instruct the woman to drink 1 to 2 quarts of water. c. Administer an enema. d. Perform an abdominal preparation.

b. Instruct the woman to drink 1 to 2 quarts of water.

13. The nurse should be cognizant of which postpartum physiologic alteration? a. Cardiac output, pulse rate, and stroke volume all return to prepregnancy normal values within a few hours of childbirth. b. Respiratory function returns to nonpregnant levels by 6 to 8 weeks after childbirth. c. Lowered white blood cell count after pregnancy can lead to false-positive results on tests for infections. d. Hypercoagulable state protects the new mother from thromboembolism, especially after a cesarean birth.

b. Respiratory function returns to nonpregnant levels by 6 to 8 weeks after childbirth.

15. Which time-based description of a stage of development in pregnancy is accurate? a. Viability—22 to 37 weeks since the last menstrual period (LMP) (assuming a fetal weight >500 g) b. Term—pregnancy from the beginning of week 38 of gestation to the end of week 42 c. Preterm—pregnancy from 20 to 28 weeks d. Postdate—pregnancy that extends beyond 38 weeks

b. Term—pregnancy from the beginning of week 38 of gestation to the end of week 42

A pregnant woman at 25 weeks' gestation tells the nurse that she dropped a pan last week and her baby jumped at the noise. Which response by the nurse is most accurate? a. "That must have been a coincidence; babies can't respond like that." b. "The fetus is demonstrating the aural reflex." c. "Babies respond to sound starting at about 24 weeks of gestation." d. "Let me know if it happens again; we need to report that to your midwife."

c. "Babies respond to sound starting at about 24 weeks of gestation." "Babies respond to sound starting at about 24 weeks of gestation" is an accurate statement. "That must have been a coincidence; babies can't respond like that" is inaccurate. Fetuses respond to sound by 24 weeks. Acoustic stimulations can evoke a fetal heart rate response. There is no such thing as an aural reflex. The statement, "Let me know if it happens again; we need to report that to your midwife" is not appropriate; it gives the impression that something is wrong.

Sally comes in for her first prenatal examination. This is her first child. She asks you (the nurse), "How does my baby get air inside my uterus?" The correct response is: a. "The baby's lungs work in utero to exchange oxygen and carbon dioxide." b. "The baby absorbs oxygen from your blood system." c. "The placenta provides oxygen to the baby and excretes carbon dioxide into your bloodstream." d. "The placenta delivers oxygen-rich blood through the umbilical artery to the baby's abdomen."

c. "The placenta provides oxygen to the baby and excretes carbon dioxide into your bloodstream." The placenta functions by supplying oxygen and excreting carbon dioxide to the maternal bloodstream. The fetal lungs do not function for respiratory gas exchange in utero. The baby does not simply absorb oxygen from a woman's blood system. Blood and gas transport occur through the placenta. The placenta delivers oxygen-rich blood through the umbilical vein and not the artery.

A woman's cousin gave birth to an infant with a congenital heart anomaly. The woman asks the nurse when such anomalies occur during development. Which response by the nurse is most accurate? a. "We don't really know when such defects occur." b. "It depends on what caused the defect." c. "They occur between the third and fifth weeks of development." d. "They usually occur in the first 2 weeks of development."

c. "They occur between the third and fifth weeks of development." The cardiovascular system is the first organ system to function in the developing human. Blood vessel and blood formation begins in the third week, and the heart is developmentally complete in the fifth week. "We don't really know when such defects occur" is an inaccurate statement. Regardless of the cause, the heart is vulnerable during its period of development, the third to fifth weeks. "They usually occur in the first 2 weeks of development" is an inaccurate statement.

A father and mother are carriers of phenylketonuria (PKU). Their 2-year-old daughter has PKU. The couple tells the nurse that they are planning to have a second baby. Because their daughter has PKU, they are sure that their next baby won't be affected. What response by the nurse is most accurate? a. "Good planning; you need to take advantage of the odds in your favor." b. "I think you'd better check with your doctor first." c. "You are both carriers, so each baby has a 25% chance of being affected." d. "The ultrasound indicates a boy, and boys are not affected by PKU."

c. "You are both carriers, so each baby has a 25% chance of being affected." The chance is one in four that each child produced by this couple will be affected by PKU disorder. This couple still has an increased likelihood of having a child with PKU. Having one child already with PKU does not guarantee that they will not have another. These parents need to discuss their options with their physician. However, an opportune time has presented itself for the couple to receive correct teaching about inherited genetic risks. No correlation exists between gender and inheritance of the disorder, because PKU is an autosomal recessive disorder.

The nurse is providing genetic counseling for an expectant couple who already have a child with trisomy 18. The nurse should: a. Tell the couple they need to have an abortion within 2 to 3 weeks. b. Explain that the fetus has a 50% chance of having the disorder. c. Discuss options with the couple, including amniocentesis to determine whether the fetus is affected. d. Refer the couple to a psychologist for emotional support.

c. Discuss options with the couple, including amniocentesis to determine whether the fetus is affected. Genetic testing, including amniocentesis, would need to be performed to determine whether the fetus is affected. The couple should be given information about the likelihood of having another baby with this disorder so that they can make an informed decision. A genetic counselor is the best source for determining genetic probability ratios. The couple eventually may need emotional support, but the status of the pregnancy must be determined first.

20. Which statement, related to the reconditioning of the urinary system after childbirth, should the nurse understand? a. Kidney function returns to normal a few days after birth. b. Diastasis recti abdominis is a common condition that alters the voiding reflex. c. Fluid loss through perspiration and increased urinary output accounts for a weight loss of more than 2 kg during the puerperium. d. With adequate emptying of the bladder, bladder tone is usually restored 2 to 3 weeks after childbirth.

c. Fluid loss through perspiration and increased urinary output accounts for a weight loss of more than 2 kg during the puerperium.

38. A woman has a breast mass that is not well delineated and is nonpalpable, immobile, and nontender. This is most likely: a. Fibroadenoma. b. Lipoma. c. Intraductal papilloma. d. Mammary duct ectasia.

c. Intraductal papilloma.

17. The nurse providing care for the antepartum woman should understand that contraction stress test (CST): a. Sometimes uses vibroacoustic stimulation. b. Is an invasive test; however, contractions are stimulated. c. Is considered negative if no late decelerations are observed with the contractions. d. Is more effective than nonstress test (NST) if the membranes have already been ruptured.

c. Is considered negative if no late decelerations are observed with the contractions.

With regard to the development of the respiratory system, maternity nurses should be aware that: a. The respiratory system does not begin developing until after the embryonic stage. b. The infant's lungs are considered mature when the lecithin/sphingomyelin (L/S) ratio is 1:1, at about 32 weeks. c. Maternal hypertension can reduce maternal-placental blood flow, accelerating lung maturity. d. Fetal respiratory movements are not visible on ultrasound scans until at least 16 weeks.

c. Maternal hypertension can reduce maternal-placental blood flow, accelerating lung maturity. A reduction in placental blood flow stresses the fetus, increases blood levels of corticosteroids, and thus accelerates lung maturity. Development of the respiratory system begins during the embryonic phase and continues into childhood. The infant's lungs are mature when the L/S ratio is 2:1, at about 35 weeks. Lung movements have been seen on ultrasound scans at 11 weeks.

15. Management of primary dysmenorrhea often requires a multifaceted approach. The nurse who provides care for a client with this condition should be aware that the optimal pharmacologic therapy for pain relief is: a. Acetaminophen. b. Oral contraceptives (OCPs). c. Nonsteroidal antiinflammatory drugs (NSAIDs). d. Aspirin.

c. Nonsteroidal antiinflammatory drugs (NSAIDs).

21. What should the nurse's next action be if the client's white blood cell (WBC) count is 25,000/mm3 on her second postpartum day? a. Immediately inform the physician. b. Have the laboratory draw blood for reanalysis. c. Recognize that this count is an acceptable range at this point postpartum. d. Immediately begin antibiotic therapy.

c. Recognize that this count is an acceptable range at this point postpartum.

In practical terms regarding genetic health care, nurses should be aware that: a. Genetic disorders affect equally people of all socioeconomic backgrounds, races, and ethnic groups. b. Genetic health care is more concerned with populations than individuals. c. The most important of all nursing functions is providing emotional support to the family during counseling. d. Taking genetic histories is the province of large universities and medical centers.

c. The most important of all nursing functions is providing emotional support to the family during counseling. Nurses should be prepared to help with a variety of stress reactions from a couple facing the possibility of a genetic disorder. Although anyone may have a genetic disorder, certain disorders appear more often in certain ethnic and racial groups. Genetic health care is highly individualized because treatments are based on the phenotypic responses of the individual. Individual nurses at any facility can take a genetic history, although larger facilities may have better support services.

35. After a mastectomy a woman should be instructed to perform all of the following except: a. Emptying surgical drains twice a day and as needed. b. Avoiding lifting more than 4.5 kg (10 lb) or reaching above her head until given permission by her surgeon. c. Wearing clothing with snug sleeves to support the tissue of the arm on the operative side. d. Reporting immediately if inflammation develops at the incision site or in the affected arm.

c. Wearing clothing with snug sleeves to support the tissue of the arm on the operative side.

16. Pelvic floor exercises, also known as Kegel exercises, will help to strengthen the perineal muscles and encourage healing after childbirth. The nurse requests the client to repeat back instructions for this exercise. Which response by the client indicates successful learning? a. "I contract my thighs, buttocks, and abdomen." b. "I perform 10 of these exercises every day." c. "I stand while practicing this new exercise routine." d. "I pretend that I am trying to stop the flow of urine in midstream."

d. "I pretend that I am trying to stop the flow of urine in midstream."

32. A healthy 60-year-old African-American woman regularly receives her health care at the clinic in her neighborhood. She is due for a mammogram. At her previous clinic visit, her physician, concerned about the 3-week wait at the neighborhood clinic, made an appointment for her to have a mammogram at a teaching hospital across town. She did not keep her appointment and returned to the clinic today to have the nurse check her blood pressure. What would be the most appropriate statement for the nurse to make to this patient? a. "Do you have transportation to the teaching hospital so that you can get your mammogram?" b. "I'm concerned that you missed your appointment; let me make another one for you." c. "It's very dangerous to skip your mammograms; your breasts need to be checked." d. "Would you like me to make an appointment for you to have your mammogram here?"

d. "Would you like me to make an appointment for you to have your mammogram here?"

A woman who is 8 months pregnant asks the nurse, "Does my baby have any antibodies to fight infection?" The most appropriate response by the nurse is: a. "Your baby has all the immune globulins (Ig) necessary: IgG, IgM, and IgA." b. "Your baby won't receive any antibodies until he is born and you breastfeed him." c. "Your baby does not have any antibodies to fight infection." d. "Your baby has IgG and IgM."

d. "Your baby has IgG and IgM." During the third trimester the only Ig that crosses the placenta, IgG, provides passive acquired immunity to specific bacterial toxins. The fetus produces IgM by the end of the first trimester. IgAs are not produced by the baby. By the third trimester the fetus has IgG and IgM. Breastfeeding supplies the baby with IgA. "Your baby does not have any antibodies to fight infection" is not an accurate statement.

16. Human chorionic gonadotropin (hCG) is an important biochemical marker for pregnancy and the basis for many tests. A maternity nurse should be aware that: a. hCG can be detected 2.5 weeks after conception. b. The hCG level increases gradually and uniformly throughout pregnancy. c. Much lower than normal increases in the level of hCG may indicate a postdate pregnancy. d. A higher than normal level of hCG may indicate an ectopic pregnancy or Down syndrome.

d. A higher than normal level of hCG may indicate an ectopic pregnancy or Down syndrome.

36. A nurse practitioner performs a clinical breast examination on a woman diagnosed with fibroadenoma. The nurse knows that fibroadenoma is characterized by: a. Inflammation of the milk ducts and glands behind the nipples. b. Thick, sticky discharge from the nipple of the affected breast. c. Lumpiness in both breasts that develops 1 week before menstruation. d. A single lump in one breast that can be expected to shrink as the woman ages.

d. A single lump in one breast that can be expected to shrink as the woman ages.

The _____ is/are responsible for oxygen and carbon dioxide transport to and from the maternal bloodstream. a. Decidua basalis b. Blastocyst c. Germ layer d. Chorionic villi

d. Chorionic villi Chorionic villi are fingerlike projections that develop out of the trophoblast and extend into the blood-filled spaces of the endometrium. The villi obtain oxygen and nutrients from the maternal bloodstream and dispose of carbon dioxide and waste products into the maternal blood. The decidua basalis is the portion of the decidua (endometrium) under the blastocyst where the villi attach. The blastocyst is the embryonic development stage after the morula. Implantation occurs at this stage. The germ layer is a layer of the blastocyst.

22. To reassure and educate their pregnant clients about changes in their blood pressure, maternity nurses should be aware that: a. A blood pressure cuff that is too small produces a reading that is too low; a cuff that is too large produces a reading that is too high. b. Shifting the client's position and changing from arm to arm for different measurements produces the most accurate composite blood pressure reading at each visit. c. The systolic blood pressure increases slightly as pregnancy advances; the diastolic pressure remains constant. d. Compression of the iliac veins and inferior vena cava by the uterus contributes to hemorrhoids in the later stage of term pregnancy.

d. Compression of the iliac veins and inferior vena cava by the uterus contributes to hemorrhoids in the later stage of term pregnancy.

33. The nurse must watch for what common complications in a patient who has undergone a transverse rectus abdominis myocutaneous (TRAM) flap? a. Axillary edema and tissue necrosis b. Delayed wound healing and muscle contractions c. Delayed wound healing and axillary edema d. Delayed wound healing and hematoma

d. Delayed wound healing and hematoma

1. A woman arrives at the clinic seeking confirmation that she is pregnant. The following information is obtained: She is 24 years old with a body mass index (BMI) of 17.5. She admits to having used cocaine "several times" during the past year and drinks alcohol occasionally. Her blood pressure (BP) is 108/70 mm Hg, her pulse rate is 72 beats/min, and her respiratory rate is 16 breaths/min. The family history is positive for diabetes mellitus and cancer. Her sister recently gave birth to an infant with a neural tube defect (NTD). Which characteristics place the woman in a high risk category? a. Blood pressure, age, BMI b. Drug/alcohol use, age, family history c. Family history, blood pressure, BMI d. Family history, BMI, drug/alcohol abuse

d. Family history, BMI, drug/alcohol abuse

The most basic information a maternity nurse should have concerning conception is that: a. Ova are considered fertile 48 to 72 hours after ovulation. b. Sperm remain viable in the woman's reproductive system for an average of 12 to 24 hours. c. Conception is achieved when a sperm successfully penetrates the membrane surrounding the ovum. d. Implantation in the endometrium occurs 6 to 10 days after conception.

d. Implantation in the endometrium occurs 6 to 10 days after conception. After implantation, the endometrium is called the decidua. Ova are considered fertile for about 24 hours after ovulation. Sperm remain viable in the woman's reproductive system for an average of 2 to 3 days. Penetration of the ovum by the sperm is called fertilization. Conception occurs when the zygote, the first cell of the new individual, is formed.

18. To reassure and educate pregnant clients about changes in the cervix, vagina, and position of the fetus, nurses should be aware that: a. Because of a number of changes in the cervix, abnormal Papanicolaou (Pap) tests are much easier to evaluate. b. Quickening is a technique of palpating the fetus to engage it in passive movement. c. The deepening color of the vaginal mucosa and cervix (Chadwick's sign) usually appears in the second trimester or later as the vagina prepares to stretch during labor. d. Increased vascularity of the vagina increases sensitivity and may lead to a high degree of arousal, especially in the second trimester.

d. Increased vascularity of the vagina increases sensitivity and may lead to a high degree of arousal, especially in the second trimester.

15. Nurses should be aware of the strengths and limitations of various biochemical assessments during pregnancy, including that: a. Chorionic villus sampling (CVS) is becoming more popular because it provides early diagnosis. b. Maternal serum alpha-fetoprotein (MSAFP) screening is recommended only for women at risk for neural tube defects. c. Percutaneous umbilical blood sampling (PUBS) is one of the triple-marker tests for Down syndrome. d. MSAFP is a screening tool only; it identifies candidates for more definitive procedures.

d. MSAFP is a screening tool only; it identifies candidates for more definitive procedures.

25. Which statement about a condition of pregnancy is accurate? a. Insufficient salivation (ptyalism) is caused by increases in estrogen. b. Acid indigestion (pyrosis) begins early but declines throughout pregnancy. c. Hyperthyroidism often develops (temporarily) because hormone production increases. d. Nausea and vomiting rarely have harmful effects on the fetus and may be beneficial.

d. Nausea and vomiting rarely have harmful effects on the fetus and may be beneficial.

11. The nurse caring for the pregnant client must understand that the hormone essential for maintaining pregnancy is: a. Estrogen. b. Human chorionic gonadotropin (hCG). c. Oxytocin. d. Progesterone.

d. Progesterone.

7. Which term best describes the interval between the birth of the newborn and the return of the reproductive organs to their normal nonpregnant state? a. Involutionary period because of what happens to the uterus b. Lochia period because of the nature of the vaginal discharge c. Mini-tri period because it lasts only 3 to 6 weeks d. Puerperium, or fourth trimester of pregnancy

d. Puerperium, or fourth trimester of pregnancy

25. The nurse recognizes that a nonstress test (NST) in which two or more fetal heart rate (FHR) accelerations of 15 beats/min or more occur with fetal movement in a 20-minute period is: a. Nonreactive b. Positive c. Negative d. Reactive

d. Reactive

2. What are the most common causes for subinvolution of the uterus? a. Postpartum hemorrhage and infection b. Multiple gestation and postpartum hemorrhage c. Uterine tetany and overproduction of oxytocin d. Retained placental fragments and infection

d. Retained placental fragments and infection

16. The two primary areas of risk for sexually transmitted infections (STIs) are: a. Sexual orientation and socioeconomic status. b. Age and educational level. c. Large number of sexual partners and race. d. Risky sexual behaviors and inadequate preventive health behaviors.

d. Risky sexual behaviors and inadequate preventive health behaviors.

30. The nurse who is teaching a group of women about breast cancer would tell the women that: a. Risk factors identify more than 50% of women who will develop breast cancer. b. Nearly 90% of lumps found by women are malignant. c. One in 10 women in the United States will develop breast cancer in her lifetime. d. The exact cause of breast cancer is unknown.

d. The exact cause of breast cancer is unknown.

12. A client asks the nurse when her ovaries will begin working again. Which explanation by the nurse is most accurate? a. Almost 75% of women who do not breastfeed resume menstruating within 1 month after birth. b. Ovulation occurs slightly earlier for breastfeeding women. c. Because of menstruation and ovulation schedules, contraception considerations can be postponed until after the puerperium. d. The first menstrual flow after childbirth usually is heavier than normal.

d. The first menstrual flow after childbirth usually is heavier than normal.

11. In comparing the abdominal and transvaginal methods of ultrasound examination, nurses should explain to their clients that: a. Both require the woman to have a full bladder. b. The abdominal examination is more useful in the first trimester. c. Initially the transvaginal examination can be painful. d. The transvaginal examination allows pelvic anatomy to be evaluated in greater detail.

d. The transvaginal examination allows pelvic anatomy to be evaluated in greater detail.

The nurse caring for a newborn checks the record to note clinical findings that occurred before her shift. Which finding related to the renal system would be of increased significance and require further action? a.The pediatrician should be notified if the newborn has not voided in 24 hours. b.Breastfed infants will likely void more often during the first days after birth. c.Brick dust or blood on a diaper is always cause to notify the physician. d.Weight loss from fluid loss and other normal factors should be made up in 4 to 7 days.

ANS: A A newborn who has not voided in 24 hours may have any of a number of problems, some of which deserve the attention of the pediatrician. Formula-fed infants tend to void more frequently in the first 3 days; breastfed infants will void less during this time because the mother's breast milk has not yet come in. Brick dust may be uric acid crystals; blood spotting could be attributable to the withdrawal of maternal hormones (pseudomenstruation) or a circumcision. The physician must be notified only if the cause of bleeding is not apparent. Weight loss from fluid loss might take 14 days to regain.

Under which circumstance should the nurse immediately alert the pediatric provider? a.Infant is dusky and turns cyanotic when crying. b.Acrocyanosis is present 1 hour after childbirth. c.The infant's blood glucose level is 45 mg/dl. d.The infant goes into a deep sleep 1 hour after childbirth.

ANS: A An infant who is dusky and becomes cyanotic when crying is showing poor adaptation to extrauterine life. Acrocyanosis is an expected finding during the early neonatal life and is within the normal range for a newborn.Infants enter the period of deep sleep when they are approximately 1 hour old.

A client is warm and asks for a fan in her room for her comfort. The nurse enters the room to assess the mother and her infant and finds the infant unwrapped in his crib with the fan blowing over him on high. The nurse instructs the mother that the fan should not be directed toward the newborn and that the newborn should be wrapped in a blanket. The mother asks why. How would the nurse respond? a."Your baby may lose heat by convection, which means that he will lose heat from his body to the cooler ambient air. You should keep him wrapped, and should prevent cool air from blowing on him." b."Your baby may lose heat by conduction, which means that he will lose heat from his body to the cooler ambient air. You should keep him wrapped, and should prevent cool air from blowing on him." c."Your baby may lose heat by evaporation, which means that he will lose heat from his body to the cooler ambient air. You should keep him wrapped, and should prevent cool air from blowing on him." d."Your baby will easily get cold stressed and needs to be bundled up at all times."

ANS: A Convection is the flow of heat from the body surface to cooler ambient air. Because of heat loss by convection, all newborns in open bassinets should be wrapped to protect them from the cold. Conduction is the loss of heat from the body surface to cooler surfaces, not air, in direct contact with the newborn. Evaporation is a loss of heat that occurs when a liquid is converted into a vapor. In the newborn, heat loss by evaporation occurs as a result of vaporization of moisture from the skin. Cold stress may occur from excessive heat loss; however, this does not imply that the infant will become stressed if not bundled at all times. Furthermore, excessive bundling may result in a rise in the infant's temperature.

5. A primiparous woman is to be discharged from the hospital the following day with her infant girl. Which behavior indicates a need for further intervention by the nurse before the woman can be discharged? a. The woman is disinterested in learning about infant care. b. The woman continues to hold and cuddle her infant after she has fed her. c. The woman reads a magazine while her infant sleeps. d. The woman changes her infants diaper and then shows the nurse the contents of the diaper.

ANS: A The client should be excited, happy, and interested or involved in infant care. A woman who is sad, tearful, or disinterested in caring for her infant may be exhibiting signs of depression or postpartum blues and may require further intervention. Holding and cuddling her infant after feeding is an appropriate parent-infant interaction. Taking time for herself while the infant is sleeping is an appropriate maternal action. Showing the nurse the contents of the diaper is appropriate because the mother is seeking approval from the nurse and notifying the nurse of the infants elimination patterns.

How would the nurse optimally reassure the parents of an infant who develops a cephalhematoma? a.A cephalhematoma may occur with a spontaneous vaginal birth. b.A cephalhematoma only happens as a result of a forceps- or vacuum-assisted delivery. c.It is present immediately after birth. d.The blood will gradually absorb over the first few months of life.

ANS: A The nurse should explain that bleeding between the skull and the periosteum of a newborn may occur during a spontaneous vaginal delivery as a result of the pressure against the maternal bony pelvis. Low forceps and other difficult extractions may result in bleeding. However, a cephalhematoma can also spontaneously occur. Swelling may appear unilaterally or bilaterally, is usually minimal or absent at birth, and increases over the first 2 to 3 days of life. Cephalhematomas gradually disappear over 2 to 3 weeks. A less common condition results in the calcification of the hematoma, which may persist for months.

During life in utero, oxygenation of the fetus occurs through transplacental gas exchange. When birth occurs, four factors combine to stimulate the respiratory center in the medulla. The initiation of respiration then follows. What are these four essential factors? a.Chemical b.Mechanical c.Thermal d.Psychologic e.Sensory

ANS: A, B, C, E Chemical factors are essential to initiate breathing. During labor, decreased levels of oxygen and increased levels of carbon dioxide seem to have a cumulative effect that is involved in the initiation of breathing. Clamping of the cord may also contribute to the start of respirations and results in a drop in the level of prostaglandins, which are known to inhibit breathing. Mechanical factors are also necessary to initiate respirations. As the infant passes through the birth canal, the chest is compressed. After the birth, the chest is relaxed, which allows for negative intrathoracic pressure that encourages air to flow into the lungs. The profound change in temperature between intrauterine and extrauterine life stimulates receptors in the skin to communicate with the receptors in the medulla. The stimulation of these receptors also contributes to the initiation of breathing. Sensory factors include handling by the health care provider, drying by the nurse, lights, smells, and sounds. Psychologic factors do not contribute to the initiation of respirations.

Which statements describe the first stage of the neonatal transition period? (Select all that apply.) a.The neonatal transition period lasts no longer than 30 minutes. b.It is marked by spontaneous tremors, crying, and head movements. c.Passage of the meconium occurs during the neonatal transition period. d.This period may involve the infant suddenly and briefly sleeping. e.Audible grunting and nasal flaring may be present during this time

ANS: A, B, C, E The first stage is an active phase during which the baby is alert; this stage is referred to as the first period of reactivity. Decreased activity and sleep mark the second stage, the period of decreased responsiveness. The first stage is the shortest, lasting less than 30 minutes. Such exploratory behaviors include spontaneous startle reactions. Audible grunting, nasal flaring, and chest retractions may be present; however, these behaviors usually resolve within 1 hour of life.

3. Which physiologic factors are reliable indicators of impending shock from postpartum hemorrhage? (Select all that apply.) a. Respirations b. Skin condition c. Blood pressure d. Level of consciousness e. Urinary output

ANS: A, B, D, E Blood pressure is not a reliable indicator; several more sensitive signs are available. Blood pressure does not drop until 30% to 40% of blood volume is lost. Respirations, pulse, skin condition, urinary output, and level of consciousness are more sensitive means of identifying hypovolemic shock.

2. Which practices contribute to the prevention of postpartum infection? (Select all that apply.) a. Not allowing the mother to walk barefoot at the hospital b. Educating the client to wipe from back to front after voiding c. Having staff members with conditions such as strep throat, conjunctivitis, and diarrhea stay home d. Instructing the mother to change her perineal pad from front to back each time she voids or defecates e. Not permitting visitors with cough or colds to enter the postpartum unit

ANS: A, C, D Proper perineal care helps prevent infection and aids in the healing process. Educating the woman to wipe from front to back (urethra to anus) after voiding or defecating is a simple first step. Walking barefoot and getting back into bed can contaminate the linens. Clients should wear shoes or slippers. Staff members with infections need to stay home until they are no longer contagious. The client should also wash her hands before and after these functions. Visitors with any signs of illness should not be allowed entry to the postpartum unit.

1. Many new mothers experience some type of nipple pain during the first weeks of initiating breastfeeding. Should this pain be severe or persistent, it may discourage or inhibit breastfeeding altogether. Which factors might contribute to this pain? (Select all that apply.) a. Improper feeding position b. Large-for-gestational age infant c. Fair skin d. Progesterone deficiency e. Flat or retracted nipples

ANS: A, C, E Nipple lesions may manifest as chapped, cracked, bleeding, sore, erythematous, edematous, or blistered nipples. Factors that contribute to nipple pain include improper positioning or a failure to break suction before removing the baby from the breast. Flat or retracted nipples along with the use of nipple shields, breast shells, or plastic breast pads also contribute to nipple pain. Women with fair skin are more likely to develop sore and cracked nipples. Preventing nipple soreness is preferable to treating soreness after it appears. Vigorous feeding may be a contributing factor, which may be the case with any size infant, not just infants who are large for gestational age. Estrogen or dietary deficiencies can contribute to nipple soreness.

6. Nurses play a critical role in educating parents regarding measures to prevent infant abduction. Which instructions contribute to infant safety and security? (Select all that apply.) a. The mother should check the photo identification (ID) of any person who comes to her room. b. The baby should be carried in the parents arms from the room to the nursery. c. Because of infant security systems, the baby can be left unattended in the clients room. d. Parents should use caution when posting photographs of their infant on the Internet. e. The mom should request that a second staff member verify the identity of any questionable person.

ANS: A, D, E Nurses must discuss infant security precautions with the mother and her family because infant abduction continues to be a concern. The mother should be taught to check the identity of any person who comes to remove the baby from her room. Hospital personnel usually wear picture identification patches. On some units, staff members also wear matching scrubs or special badges that are unique to the perinatal unit. As a rule, the baby is never carried in arms between the mothers room and the nursery, but rather the infant is always wheeled in a bassinet. The infant should never be left unattended, even if the facility has an infant security system. Parents should be instructed to use caution when posting photographs of their new baby on the Internet and on other public forums.

3. A woman gave birth 48 hours ago to a healthy infant girl. She has decided to bottle feed. During the assessment, the nurse notices that both breasts are swollen, warm, and tender on palpation. Which guidance should the nurse provide to the client at this time? a. Run warm water on her breasts during a shower. b. Apply ice to the breasts for comfort. c. Express small amounts of milk from the breasts to relieve the pressure. d. Wearing a loose-fitting bra to prevent nipple irritation.

ANS: B Applying ice packs and cabbage leaves to the breasts for comfort is an appropriate intervention for treating engorgement in a mother who is bottle feeding. The ice packs should be applied for 15 minutes on and 45 minutes off to avoid rebound engorgement. A bottle-feeding mother should avoid any breast stimulation, including pumping or expressing milk. A bottle-feeding mother should continuously wear a well-fitted support bra or breast binder for at least the first 72 hours after giving birth. A loose-fitting bra will not aid lactation suppression. Furthermore, the shifting of the bra against the breasts may stimulate the nipples and thereby stimulate lactation.

The parents of a newborn ask the nurse how much the newborn can see. The parents specifically want to know what type of visual stimuli they should provide for their newborn. What information provided by the nurse would be most useful to these new parents? a."Infants can see very little until approximately 3 months of age." b."Infants can track their parents' eyes and can distinguish patterns; they prefer complex patterns." c."The infant's eyes must be protected. Infants enjoy looking at brightly colored stripes." d."It's important to shield the newborn's eyes. Overhead lights help them see better."

ANS: B Telling the parents that infants can track their parents' eyes and can distinguish patterns is an accurate statement. Development of the visual system continues for the first 6 months of life. Visual acuity is difficult to determine, but the clearest visual distance for the newborn appears to be 19 cm. Infants prefer to look at complex patterns, regardless of the color. They prefer low illumination and withdraw from bright lights.

A primiparous woman is watching her newborn sleep. She wants him to wake up and respond to her. The mother asks the nurse how much he will sleep every day. What is an appropriate response by the nurse? a."He will only wake up to be fed, and you should not bother him between feedings." b."The newborn sleeps approximately 17 hours a day, with periods of wakefulness gradually increasing." c."He will probably follow your same sleep and wake patterns, and you can expect him to be awake soon." d."He is being stubborn by not waking up when you want him to. You should try to keep him awake during the daytime so that he will sleep through the night."

ANS: B Telling the woman that the newborn sleeps approximately 17 hours a day with periods of wakefulness that gradually increase is both accurate and the most appropriate response by the nurse. Periods of wakefulness are dictated by hunger, but the need for socializing also appears. Telling the woman that her infant is stubborn and should be kept awake during the daytime is an inappropriate nursing response.

Which information about variations in the infant's blood counts is important for the nurse to explain to the new parents? a.A somewhat lower-than-expected red blood cell count could be the result of a delay in clamping the umbilical cord. b.An early high white blood cell (WBC) count is normal at birth and should rapidly decrease. c.Platelet counts are higher in the newborn than in adults for the first few months. d.Even a modest vitamin K deficiency means a problem with the blood's ability to properly clot.

ANS: B The WBC count is normally high on the first day of birth and then rapidly declines. Delayed cord clamping results in an increase in hemoglobin and the red blood cell count. The platelet count is essentially the same for newborns and adults. Clotting is sufficient to prevent hemorrhage unless the deficiency of vitamin K is significant.

A woman gave birth to a healthy 7-pound, 13-ounce infant girl. The nurse suggests that the client place the infant to her breast within 15 minutes after birth. The nurse is aware that the initiation of breastfeeding is most effective during the first 30 minutes after birth. What is the correct term for this phase of alertness? a.Transition period b.First period of reactivity c.Organizational stage d.Second period of reactivity

ANS: B The first period of reactivity is the first phase of transition and lasts up to 30 minutes after birth. The infant is highly alert during this phase. The transition period is the phase between intrauterine and extrauterine existence. An organizational stage is not a valid stage. The second period of reactivity occurs approximately between 4 and 8 hours after birth, after a period of sleep.

The nurse should be cognizant of which important information regarding the gastrointestinal (GI) system of the newborn? a.The newborn's cheeks are full because of normal fluid retention. b.The nipple of the bottle or breast must be placed well inside the baby's mouth because teeth have been developing in utero, and one or more may even be through. c.Regurgitation during the first day or two can be reduced by burping the infant and slightly elevating the baby's head. d.Bacteria are already present in the infant's GI tract at birth because they traveled through the placenta.

ANS: C Avoiding overfeeding can also reduce regurgitation. The newborn's cheeks are full because of well-developed sucking pads. Teeth do develop in utero, but the nipple is placed deep because the baby cannot move food from the lips to the pharynx. Bacteria are not present at birth, but they soon enter through various orifices.

11. A recently delivered mother and her baby are at the clinic for a 6-week postpartum checkup. Which response by the client alerts the nurse that psychosocial outcomes have not been met? a. The woman excessively discusses her labor and birth experience. b. The woman feels that her baby is more attractive and clever than any others. c. The woman has not given the baby a name. d. The woman has a partner or family members who react very positively about the baby.

ANS: C If the mother is having difficulty naming her new infant, it may be a signal that she is not adapting well to parenthood. Other red flags include a refusal to hold or feed the baby, a lack of interaction with the infant, and becoming upset when the baby vomits or needs a diaper change. A new mother who is having difficulty is unwilling to discuss her labor and birth experience. An appropriate nursing diagnosis might be Impaired parenting, related to a long, difficult labor or unmet expectations of birth. A mother who is willing to discuss her birth experience is making a healthy personal adjustment. The mother who is not coping well finds her baby unattractive and messy. She may also be overly disappointed in the babys sex. The client might voice concern that the baby reminds her of a family member whom she does not like. Having a partner and/or other family members react positively is an indication that this new mother has a good support system in place. This support system helps reduce anxiety related to her new role as a mother.

The nurse is cognizant of which information related to the administration of vitamin K? a.Vitamin K is important in the production of red blood cells. b.Vitamin K is necessary in the production of platelets. c.Vitamin K is not initially synthesized because of a sterile bowel at birth. d.Vitamin K is responsible for the breakdown of bilirubin and the prevention of jaundice.

ANS: C The bowel is initially sterile in the newborn, and vitamin K cannot be synthesized until food is introduced into the bowel. Vitamin K is necessary to activate blood-clotting factors. The platelet count in term newborns is near adult levels. Vitamin K is necessary to activate prothrombin and other blood-clotting factors.

A newborn is placed under a radiant heat warmer. The nurse understands that thermoregulation presents a problem for the newborn. What is the rationale for this difficulty? a.The renal function of a newborn is not fully developed, and heat is lost in the urine. b.The small body surface area of a newborn favors more rapid heat loss than does an adult's body surface area. c.Newborns have a relatively thin layer of subcutaneous fat that provides poor insulation. d.Their normal flexed posture favors heat loss through perspiration.

ANS: C The newborn has little thermal insulation. Furthermore, the blood vessels are closer to the surface of the skin. Changes in environmental temperature alter the temperature of the blood, thereby influencing temperature regulation centers in the hypothalamus. Heat loss does not occur through urination. Newborns have a higher body surface-to-weight ratio than adults. The flexed position of the newborn helps guard against heat loss, because it diminishes the amount of body surface exposed to the environment.

Which information related to the newborn's developing cardiovascular system should the nurse fully comprehend? a.The heart rate of a crying infant may rise to 120 beats per minute. b.Heart murmurs heard after the first few hours are a cause for concern. c.The point of maximal impulse (PMI) is often visible on the chest wall. d.Persistent bradycardia may indicate respiratory distress syndrome (RDS).

ANS: C The newborn's thin chest wall often allows the PMI to be observed. The normal heart rate for infants who are not sleeping is 120 to 160 beats per minute. However, a crying infant could temporarily have a heart rate of 180 beats per minute. Heart murmurs during the first few days of life have no pathologic significance; however, an irregular heart rate beyond the first few hours should be further evaluated. Persistent tachycardia may indicate RDS; bradycardia may be a sign of congenital heart blockage.

2. Changes in blood volume after childbirth depend on several factors such as blood loss during childbirth and the amount of extravascular water (physiologic edema) mobilized and excreted. What amount of blood loss does the postpartum nurse anticipate? (Select all that apply.) a. 100 ml b. 250 ml or less c. 300 to 500 ml d. 500 to 1000 ml e. 1500 ml or greater

ANS: C,D

4. A 25-year-old multiparous woman gave birth to an infant boy 1 day ago. Today her husband brings a large container of brown seaweed soup to the hospital. When the nurse enters the room, the husband asks for help with warming the soup so that his wife can eat it. What is the nurses most appropriate response? a. Didnt you like your lunch? b. Does your physician know that you are planning to eat that? c. What is that anyway? d. I'll warm the soup in the microwave for you.

ANS: D Offering to warm the food shows cultural sensitivity to the dietary preferences of the woman and is the most appropriate response. Cultural dietary preferences must be respected. Women may request that family members bring favorite or culturally appropriate foods to the hospital. Asking the woman to identify her food does not show cultural sensitivity.

What is the most critical physiologic change required of the newborn after birth? a.Closure of fetal shunts in the circulatory system b.Full function of the immune defense system c.Maintenance of a stable temperature d.Initiation and maintenance of respirations

ANS: D The most critical adjustment of a newborn at birth is the establishment of respirations. The cardiovascular system changes significantly after birth as a result of fetal respirations, which reduce pulmonary vascular resistance to the pulmonary blood flow and initiate a chain of cardiac changes that support the cardiovascular system. After the establishment of respirations, heat regulation is critical to newborn survival. The infant relies on passive immunity received from the mother for the first 3 months of life.

Which infant response to cool environmental conditions is either not effective or not available to them? a.Constriction of peripheral blood vessels b.Metabolism of brown fat c.Increased respiratory rates d.Unflexing from the normal position

ANS: D The newborn's flexed position guards against heat loss, because it reduces the amount of body surface exposed to the environment. The newborn's body is able to constrict the peripheral blood vessels to reduce heat loss. Burning brown fat generates heat. The respiratory rate may rise to stimulate muscular activity, which generates heat.

1. A woman gave birth vaginally to a 9-pound, 12-ounce girl yesterday. Her primary health care provider has written orders for perineal ice packs, use of a sitz bath three times daily, and a stool softener. Which information regarding the clients condition is most closely correlated with these orders? a. Woman is a gravida 2, para 2. b.Woman had a vacuum-assisted birth. c.Woman received epidural anesthesia. d.Woman has an episiotomy.

ANS: D These orders are typical interventions for a woman who has had an episiotomy, lacerations, and hemorrhoids. A multiparous classification is not an indication for these orders. A vacuum-assisted birth may be used in conjunction with an episiotomy, which would indicate these interventions. The use of an epidural anesthesia has no correlation with these orders.

15. When should discharge instruction, or the teaching plan that tells the woman what she needs to know to care for herself and her newborn, officially begin? a. At the time of admission to the nurses unit b. When the infant is presented to the mother at birth c. During the first visit with the physician in the unit d. When the take-home information packet is given to the couple

ANS: A Discharge planning, the teaching of maternal and newborn care, begins on the womans admission to the unit, continues throughout her stay, and actually never ends as long as she has contact with medical personnel.

14. In many hospitals, new mothers are routinely presented with gift bags containing samples of infant formula. This practice is inconsistent with what? a. Baby Friendly Hospital Initiative b. Promotion of longer periods of breastfeeding c. Perception of being supportive to both bottle feeding and breastfeeding mothers d. Association with earlier cessation of breastfeeding

ANS: A Infant formula should not be given to mothers who are breastfeeding. Such gifts are associated with early cessation of breastfeeding. Baby Friendly USA prohibits the distribution of any gift bags or formula to new mothers.

8. A hospital has a number of different perineal pads available for use. A nurse is observed soaking several of them and writing down what she sees. What goal is the nurse attempting to achieve by performing this practice? a. To improve the accuracy of blood loss estimation, which usually is a subjective assessment b. To determine which pad is best c. To demonstrate that other nurses usually underestimate blood loss d. To reveal to the nurse supervisor that one of them needs some time off

ANS: A Saturation of perineal pads is a critical indicator of excessive blood loss; anything done to help in the assessment is valuable. The nurse is noting the saturation volumes and soaking appearances. Instead of determining which pad is best, the nurse is more likely noting saturation volumes and soaking appearances to improve the accuracy of estimated blood loss. Nurses usually overestimate blood loss. Soaking perineal pads and writing down the results does not indicate the need for time off of work.

10. What information should the nurse understand fully regarding rubella and Rh status? a. Breastfeeding mothers cannot be vaccinated with the live attenuated rubella virus. b. Women should be warned that the rubella vaccination is teratogenic and that they must avoid pregnancy for at least 1 month after vaccination. c. Rh immunoglobulin is safely administered intravenously because it cannot harm a nursing infant. d. Rh immunoglobulin boosts the immune system and thereby enhances the effectiveness of vaccinations.

ANS: B Women should understand that they must practice contraception for at least 1 month after being vaccinated. Because the live attenuated rubella virus is not communicable in breast milk, breastfeeding mothers can be vaccinated. Rh immunoglobulin is administered intramuscular (IM); it should never be administered to an infant. Rh immunoglobulin suppresses the immune system and therefore might thwart the rubella vaccination.

9. A 31-year-old woman believes that she may be pregnant. She took an OTC pregnancy test 1 week ago after missing her period; the test was positive. During her assessment interview, the nurse inquires about the woman's last menstrual period and asks whether she is taking any medications. The woman states that she takes medicine for epilepsy. She has been under considerable stress lately at work and has not been sleeping well. She also has a history of irregular periods. Her physical examination does not indicate that she is pregnant. She has an ultrasound scan, which reveals that she is not pregnant. What is the most likely cause of the false-positive pregnancy test result? a. She took the pregnancy test too early. b. She takes anticonvulsants. c. She has a fibroid tumor. d. She has been under considerable stress and has a hormone imbalance.

b. She takes anticonvulsants.


संबंधित स्टडी सेट्स

CSC-325: Quiz 8 Review Questions

View Set

Varcarolis: Chapter 13 - Bipolar and Related Disorders

View Set

Windows Command Line (examcomppass)

View Set

Jewish and Early Christian Art (AoW Midterm II)

View Set

6th Grade SS Chapter 30 Alexander the Great and his Empire

View Set